Nothing Special   »   [go: up one dir, main page]

Ortho Pedia Complete Merge

Download as pdf or txt
Download as pdf or txt
You are on page 1of 383

lOMoARcPSD|19937673

Compilation for dentistry Ortho pedo

Dentistry (Centro Escolar University)

Studocu is not sponsored or endorsed by any college or university


Downloaded by John Erickson Bangayan (bangayan1903064@ceu.edu.ph)
lOMoARcPSD|19937673

Downloaded by John Erickson Bangayan (bangayan1903064@ceu.edu.ph)


lOMoARcPSD|19937673

Pedodonrics r\ocl Or:hodoniics Set I

I The r;.tosl laworable


-quence ol err.rpticn of the pernanen(- teerh ia Lhe mandibubr arch rs.
!-
A_ 6- 1-2-4-5 -3 7 c- 6-t-?-3-&-5-7
. a_ 6-L-?-4-5-3-7 D- 6_L_2;3_4-7 -S
2 Th'e bone reaction adlacent to the hyalinized Usiue during the Lr:rial periocic! tooth rnovernenr r5
A- calcification r -^^.
a. resorprion 3, l;ffi=t""
3- After binh wtrat rocrth buds-are generartyt.initiaiea?
A - l'and Z--preqotars C- Perm.'Denririon
B- . D- \e. t€ and 3. motars
4- rhe the ctraoge-trcm geneqalized.cilts or tissues to a,inoielp.t"ielo *r* ,=,
4'' C- difterentiadon
D- grqil{h
5 otogic evenrs tetal,g/Owth sf,urts are:
r in_wiigt1r
4- 'ad$trctor-sesarnciirJ
.A...t
-l-3.4 c- t.?
.,Qt., L.-2:3 D- i,?.a
6- A clistdnce b.egnreen lhe maXillaCf centi'aI..incbors'ajr agle a;dou_td be-treared by:
A- lea:.ring-it alotte D- T-aking a radiograph ro rule ou[ fibrous '
q- surgica! exciSro-n oQrhe,labial lrenum lrenum
C. Ha\dql appt-ince
D 7- A nre(hod ol p(eqicting rlre Size ot ihe SrjecgeOane6us teelh ild the sp3ce avaitabte in the aich 6:
A- Bohon aoat)jsis C- Tanaka and -.rohnsoo anarysis
B- Mixed de(itiaiim anatysis o- peck aod peck anabsis
I; 8- Checi( biriog miy euse c6asrrktim ol the daxittao/ irch a.a p,o*..Ar.-sJ;a* ,1a *r, besr be resotvii(,
by:
r1- o.atsc.een C Uleb&cck
_ ' .4, Iongue g{rara, ,dw}rg
D- croqp,glasri.
B9 Oisrurbiaces
.
lhe appGiliond sqgg cir t6orh'
A, aretogeoEsi;.,mperfeai .'a' .' c-
A- enamel'hyFbpl?:ia .Ot irioded enar:et
I ro-,which blrihe ro[g!4i:no are Oro$4hFi(es:
1- sur.u.es ' . 2- Liuitiary.iribe.(o-Sry 3- s,y{lctlTdroses 4, cond),,Ie
.4: .1.2.3,4 tt. C" 1,2 dd 3
' _.,8-. I -.1 and 3
D, rlr--,laiiiat o ."JJA" i(ima6/-den(;r;dns?
' '.lA''
'i
. F3:'m 6
G-- uoin mu. aa minO'itnrJaitittrlarateot lhe jalne,qiafi.M:O
D. mand- 2?". molaii are widei. M.b tiah iheir.ro.countelpars
B 12. r"rato".lrt;on ia rtlprimarygi*lr'ir"'*';""Jt;;i.i;id;il-lrr'inremeoegree.aie iruary@'.ecred mi,
age-
A,- as soon as, pcjsible c-4
8.3 D-5
tt r:- A lingualt/ erup(ioo maxillary id(erat incisor:ir bge I canibe intereF,(ed:b)4
1: using a (ongue
2. using an. iaclined plane
3. iostrucrioO the child to use ll.re Oturib to push lhe.laleral iociio,
4- lhe use or . xi-rey apjtiaote vrirh an ririraedfinger sphng ar.rhe liogu5l.'surrace lateral inciso,
A- t.2.3 .C- all o( the three
a. L-Z.c D- ?..3'4
4 ia Haod.rrurGts radiographs afe taken as ofhodoirtfc diagnostic a-rds to
' A- dererrnio€ rhe aino.un( of grovrth of .rlie'chitd
B fincl out rfle qggree o[ tlre bon€ calc-ificatio.n
C- g513blish rhe age ofi'the child
! D. dere_rrnine if r-he age of:rhg.chitd hqs no sys(arnatic condifion
; tS The rnost retnrded.and postbriorposition,-of thecenUyles is:
A- centric re[arion . C- cehiiic position
B- all of these C- cei-ltriC occlusion
16- \^/ha( trrrcro,orgarisms are mosl frecuen(ty associated rwith Sctiwg cariaus lesioos?
A- Aerobic C - Spirochetes
- B Monilta alt icas O. Lactobacilft aild st,ep(ocrccus mu(ans
S I z. fne bng rae syndrome is mostalrva)rs assciaEd trith
A. nad briring C- loogrre rhrusi.,g
3 n'auth breathing O- depressioo
L-

Downloaded by John Erickson Bangayan (bangayan1903064@ceu.edu.ph)


lOMoARcPSD|19937673

L
i8, The ryEE of cldp used in most remval sthodonoc appliares vrhbh engag€sthe ild distobuccal undeicuts
L A
o{ the rosrerb{ lee(h
Ad'anns
ad gives exellent retentive propenies is
C- Ball
B. A1.rovyhead D. Circumferenrial
C 19. The malocclus:on characterized by a narr@iiag ot the ach. protrudihg iocisoG accomOanied bpr aboo'rmal
f.unction oI the,lips is
l A Cl l! c,ivis.ion 2 C- Cl H division 1
L B. ,Cl lll..divisbn 2 D. Cl t division 2
* 20- Aoetaior managementwherein,each siep mdlor inslrumeot is explaioed to the rsullgol Ore procedure
done is known as
I) 'A-- tetl-shovr-do C- .voice con'irol
I B- all cf rhese D- .l-Irand-over-rriouth
L, D zr- fhe classification o! a matocclusion presemiog 1'perrd-rent jn di-itoccltrsion eacessive lingual
molatrs
ioclindtion of lh€ maxillary cenral incisors dnd excessive.labial ore marllary lateralinciso6 '6 a:
A Cl ll division 1 C- Ct-l division I
B- Cl ll ctivision 2 'O- Cl..l division 2
22- Tfie trearnent alterna\ives for deep pit and.l"rs\ures include th-e ,allou/ing: ..-- .
l- observarion
L 2- Ptt and feature sealan.6
3- preventiv-e resin restoration
4- Arnalgain restoratbn

A- 2.3 and 4 C- 2 ontyl


B- ali of rhese D- 2andS
23. An irnportant rnediator of cellular response which has-the capaciry to stirnulate both osteo osteoblastic
L activity are the:
A- cyrokinins C- prostagtandin
B- , interleukins D- piezoelectriciry
D 2,r Thc fqrst teeth to ije extracred irr a >erial exracticn in a crowded arch as the permanent iociso.s erupt:
A. primary lateral incisors C- fir-sr primHry,rtrirlars
t]- permartpnt latelal incisors D- 'Primary rcaRines
L D 25. The'rnaric.lible onginateS from the follcnaring branchiat arch:
A- 3d c- 2* D- 4"
l) 26. Removahlo .rppliffrcce zuch a^g l.llwlcy.tlipli,rrr:r. ;frrrrtrrr:r<.1h; folloWlng,ntoV*rrrri;rr[
A rotational intdrminentC-
) B: bodily (pping O.
! 0 27 l'herhcory oI bone ,o,rmation tllat srdlBs tlEt in'ihe presence oI load suffrcienr lo bend the surlace or.the bone
on tke crinvex sioc irimutaies ostdoucric acriviry whiliron irre,rultice osteoUtaStE acriviry 's stimuLateu is
lhe
A. larv of osteogenesis C- V-principal of bone erow'th
['i 0 28.
B: long bone grow(h theory
qerm
D- Layy,,-of elecl.rogenesis
: The stage ol toolh
result is the
development affecied vrhpn a eys(.an odootoma or a.supernufrrerary rooth mily
stage
A. rnorphqdifferentiation C- .-protiferarion
A- histodifferentiarion D. initiatiiin
0 29. The best sedative for oVerty anxious child patient,is
A. sodiurn pentothat C- clrloral hydrate
B. nitrous oxide-oxygen D- dormiurn
D :o. The fi. is caused by a Jailure of rhe peiiodonrat lag- To devetop:
A. impaction C. prematureresorption
A- .. ectopic eruption D- anky4asis
C 3t an iodex used to anajlze the |roponiomliy ol tooth sizes in lhe mdillary and mandibula, arches is
A- Mixed dentirion apalysis C- Boltoo's analysis
B. Peck anU Peck's index D- Holdwe.y ratio
32. Hovr is transiucenqr, of the teeth restored alter bteaching?
A i th of ttre bleaching agent C- cold ap.plication
B l D- prolong rhe bleaching lime
I ft 33. The tr .vital y.oung permanent teeth with open apices
b A apexification C- apexogenesis
B. apexogenesis and apexification D- 'calcium hydroxide pulpotorny
C 3a. Ttre ideal rnanagerneni [or anterior open bite due ro mou(h breathing caused by enlarged adenoids is
A tongue crib C- r-efer to a pediatrician regartlinE the
i
t I .rn oral screen adenoids
i
O. habi(-b.eakingappliances
t- C 35. irauma (o lhe p,imary incisors may affed the developing permanent dentitioo and may cause (he
develbprnent o{

I
.: A. ncne of these
B- odontoma-like rnallorrrration ard.enamel
C- enarnel hypoplasia
D. odontsrna-like malforrnation
i !'rypoplasia
L C 36 \A/har represent the deciduous set in the Federacion Dentaire lnternational?
A lasr live terrers of the alphabet a- Roman Numerals

i'I
I
b

Downloaded by John Erickson Bangayan (bangayan1903064@ceu.edu.ph)


lOMoARcPSD|19937673

A.
B. =
arci circumterene
C 52, An
arch rensrh
apptiece tor l'he rea{ing deep bite ces ,o, a lO -- - - -- S,
l3ff"1"t""rtn*"t
A aterior bire ptane )ear chi;
B- anrerior bire,pl&e and cewical heailgear C_ ceru.ica, headgear
A 53- The first perrEnent tooth in the or"r ["ity?ni.rt
o. hagh-gul rreaigeqr
accedes into ihe oral caviry dt a chitd rh6 primary ssmd
,|

I molars are
A- 1- molar pemanent C- l. motiis
B. 2- permanent motar D. canines
A 54- The prccess ot bone grovvth by addition ot btine r'ssue
on one siaJe md resorprion in the orher n{1oF! *
g. deposit on c- rraNlocrion : -' ': -- --
-

D ss. Gtra.-s-J.]lJ-J oort, oeveropmenr numbe,6 (stage q)is


o' displacemenl
A "l l ol crom is complered C. Z.rJ o, sNo is completed ,
B. cr*rn is almqstcornple.led O. croM-6, is compteled
r
D s6- The rerm Ged in ontrooorit'rci ritricfi meurs res'tsrence ro d-isDtacement
-c. - . -.
A t.actim
B- drift . ""trr"ry -
O 57- Midline diastema after the permaneDt anines have-erupteE o- anir,oiage
maybe caused b)r
A resiodens
a. rib.ous rabia, rr€num 3, i,tli?:irTe(dry
rooth
D 58' Ttte grwth heory-that daitrrs that the bone and caru-lage growrtt is deremined by luncdona! matqices is
A- Sicher's bone-growtli theojr c. v-theory.of grNh
r-

- '-{-59'-Ihe-enrt=ro.eodrela6oGhip.otJtre-9&mer-@etaFis4haaged t6 a ,avorabterone by whal Mdirion?


A- negadve treewa)aspae C, posirive teeway spae
a. negarive te*ay space o. p"ri,i". tri";li-ri"i.
D 60, \^/ha( is rhe equivaleirt ot pt
lAofogic onhodoniic tgrce?
A, Nomal \renous pressure c. Ncirina, anehal presure
87,
B- combination o, artel.iat ild ven6us btood o; -rvorrnJ,;"pidriU,ooa.pr.r=rr"
ptessu'e
C 6l- skeleEl rdh(ionship and grorth directioncan be best atsessed
by a arE!)6is ol tandmarks in a
A piloramtc x-ray c- cepharoireBic Ediograph
B_ dental cast
A 62' rhe vnl J u""9 rcrmado-nthai sres rhroueh ianirag. r"r-.ri*
A endochondral bone lorrnhdori #;-rq3;HJ}fJr, by osreojenic tissu.e is
B. perichondral [omat-bn c- intrmembraneous bone lorrorion
-D 63_ Rerained E). pericteaj bone tormauon
intuiite swa-flowing riBy bad rb
A- prtrline maxillary incisors C, a.te.io. open bire
B. _all ot lh?se
6 s'r- n ocnt.t potte.lcr.rcssrrite or tne l- permnenr motars vrirhou?mj:;:t'"iJ}il'::^
tollowing be correcred by-rhe
A-
expansion of the arch with a W-arch appliahce C:, quadihelix
B-
c.ossbi(e eldtic D. occlusa, equilibrarion
D 65- comp,ehensive prwenrrye denial program is apolied Uesr :.-or,i
A- Dom.S q/ndrbme c "rrr"ai:;;ffi;
Paialytk
B. Menrally-ill handi'apped
, 66. An acrive rerirovabte apptiaoce shourd be adlusred.every
__" "lfJ;"tlv
A- 4:6 c' 2'3
l a- 4-s
)-
\- D 67' The totlowiog can be (he sequelae:of unresbred or imp(dperry
A_ crovrding o, the postHio( segment in rhe
r;,.if-*,-.., carib.s ol rhe deciduous motar
perrnanent dendrion
C. none o( the choies
D- borh choices
B- a high and labial cuspids in manci.flary arch
t fi68 e(uptim is an inxpnant facto. in widening a6d ctfan'ging rhe shape ol mailary denEt arch
A biosplls c rnolar
C- ffib,
L- B. canines D. incisq[E
Bus I )ental age eleven is Eenerally characterized by rhe €ruptiorr *ri-irro ;.-i;i:i
: A- maxillary io prerno,lar
I B. mandibular 1, prernolar
I
Cto The index used to detc'rmine the availability cf space for rhe incominj
l- A Hotdway rauo C Mi'i
B. Peck and Peck index
A 7L rhe tactor/factors that deterrnine the gravicy of a permicious h.aoit ,r l;".; ; ;J#bsion
1- rjuratiun Z-lntens,D/ 3. Frequenqf 4. Age o{ rhe
A" 1.2.3 c. afl of rhese
chatd

B ?3.a D. 4 onlv
c 72- The norrnar retarionship of the prirnary rnorar is

Downloaded by John Erickson Bangayan (bangayan1903064@ceu.edu.ph)


lOMoARcPSD|19937673

A- rrre.sid.strep C. Ilush terminal Plan'e


ts- disrat srep D- Class ll
A 73. When do*es ,abscess lfiocrrna,tion.develop and at the sarne.-tirne diagnostic procedure can
es I abli sh,ad arrtong c.rri:ldr',en's ree.t h ?
A- Non surppurative cases C- 'Vita, carious reertt
A- Su,ppuraUve cases D. Fractured
t
A 7a- Denal a-rch lenglh of- the rna.xi,lla and..rnandible ii always shorreneb curing
A- transitional deorition c- mix.ed dentidon
B- perrnanent d.entiir'on D_. pnmary deprhion
A 75. What kiod o[ speech sound is produced wirh nasopharyrngeat incornpelence?
A. hypernasal C. consonan'6
I
I
B- sibitants D- hyporasal
I
I
q 76- A goocl spaee maintainer should fuffill rhe following req3riremenr.s:
1. lt Should improved estherics
2- lt shotrtd--irnprove'the function pl rhe posrerior.s,egrnent'
3: lt should prov{-de enou!.n space for the eruption Jl n.'"rceeedaneous'teeth

t
4- trsh.ould be cleans:abte,
A- 2.3 and 4 'C- 1.2 and 3
B. 3 only D- 2,and 3
$. 77 -The arch perimeter is rnaintained onty ii: rhe foltowing conditions are preseor
A- there is no loss.of arch perimeter D- there is no loss ol one or more primary

t
B- all of rhese teeth
C- (here is lavo,rable mixed-dentirion anatysis.
b 7.8- Accidental intrusion of primary-.incboi arage 5-c.tn'be rnanaged ry
1."giving analgesic. coritroltinj'bteeding and obse'rve for S'rnonth to atlow reeruprion
2- observe the toorh,i[ srarelling or pus formaton occurs
3. extraction
4- orthodon,Lic exrusion
A. 2and3 C- .rll of l,hesc
L g- t.z and 3
0 79. TtE+reserce ot a didrtsmat beturetiir the.cenrat-iocisos beniveen ages ?-g should.be
D. I andz
A. trealed at ones ahd ,;conEneirded toronhod.ontic C_ ctSsgd ri:ingi a Hawley.ajq{ive dpptiare
€bsure jr. D, Gtoied widi tighiore resins
I B-cfb€€rysd because it will riSotve tiy itseil
I d ao. rhe Eargrial used tor.iMirbcr putp rrejiriiqnt which is places direcrly over tre deep c'aviq/ some ca;itus
i- cile,ntin is-[efi ]s
A. ca|,cirurn hydroxide C- zinc oxide-eugeno,l
'B. cavity varnish D- lormocresol zinc-oxide etrge'i-lo[ mixur,rre
0 8f- The'epyphyseal plares of long bones are exa.mple of
) A. grgwth sites C g'rowth fields
L DB2'-
B- deposinon
one o[ th-e follorrying causes internal resorp(ion vrhen
D- growth. cen(e6
used as directpulp.capping.material
a
A- zint-eugenot cemenr -c- zinc plibqdtrare
B- fornocresol D. calciurn $6roxioe
C as- The Structure tesponsible-for the downward disptacerhent of ,the maxillia b'lhe
A- frontom_axillary s.uture C. nasal cartilage
B- frontozygornatic'suture D. nasornaxillarY suture
-,q. D(awntsaaeues painted modets
C.
ptdrer models srudy cdr D-
-B-
|
I
t-
-- D 85. The clcses cortection
are beureen
beMreen chronologic age and the number ot perrnanen( teeth, r@a age and,bone a'ge

A. :root age and bone age C- d root age


A. the number-of erupted teeth and bone age O. d boqe Ege
D 86. is the condi0on among primary rnolats that causes shrits froat.scclusion
I level
\ A-
B-
irnpaction
resorp$on
C
D-
recession
anlgrlosis
87- What position of the iaurs is preseot during tdtanus?
A Norrrral s C-- Lockjaw
I
B. Proslru D Retrussive
88. A plrenorrenon iobserVed during rooth ,rnvernenl ig ft66nam as
I A. eleCtro-rnsqnetic C. piezoelectric
A. UoeEcrr.it D). eiecrropoterfrat
C Ag rhe porfu-orr_ir;OIc rnaodihft. dlhh rtartraJm htCaEuipalloo ot rhe rnaxilary. and riar.ralilrutar &rerh ts'
+ c. G€nl,h ocdtltiirn cenl'h 'Plarhn
I s- .dB'
i C 30- Ari arrusioo o{.+diBa'.ry incisor b besr rnanaged D'l
L l. reuievirB ale:rcdh wirhin X n*rut€s and cfearihg wlh iunning r./arar f r
,etmGllarnlitlton

Downloaded by John Erickson Bangayan (bangayan1903064@ceu.edu.ph)


lOMoARcPSD|19937673

2- s,p#nim rijlira ruosth ror


3- EivfrrrQ,amd@resirc
l- rc needm r'gfr#+fl 'Gle bo(h
A f.2sr{3 C, 3and4
a- Zzd3 p- tard3
o 91- The gEf,}id! rtFor/ aha{ mpli€t3s rh3 rcl€ o, ihe nsat in rhE grcwth ol the mdilla and mandibte is
'drtry,
) A- M6s C- peirwicls
I
' a- Scorrs. D, sthers
A 92. The rbspor'e & the perieilonta, ligiamenr ro ileavy ,orces my laad:to
A. hyatini*aaD[ C- pedodfrtal necrosE
a- n A. undermining resorp(pn
D, 93. Naii is'eeu-vdeor rdi
A rerha
B.
L_ c.
o. ed'G+R;E&id{+S.€
D 9a- wtte apsrlituilt gE ieraqr done?

t+ tl 9s.
lltol,lqsdclGer
::: i, .q itn
ittrte mong o' living substarce ad
oe preserued in clistrlled water 48

o-F @
tlhanges is mecured in unirs oI
utl
anq.easa per u,
A- growrh c-- differEnriarioh
B- develbpreot D- mahJration
L
.
C 90- do rtot.alr€rl ioo|lh-eruption?
-Vrrtrigh
A- sysremic dbeas.esi c. ph)6ical growrh
B- Herediv
D g7 a cnitd ono is attowec lo do home-wirhoutjkeatment urhen he D- Loializaipatlotogy (he-dental
thrours a temdertat uin in ollice
prior to rreatmeAt is an example bt a behavior.nranagement knov& s:
1- beh*viocatrpunishment C. crEsical conditioning
B j negaiive rekifocemenr D- pdsitve ,4iture
, * {eqrended appricadoi'rime torrhe gel aciduraed pnosphaie n""rialappra",i""
'^*
A- 4 Einsaes 'i,. ,,1 is
B- 3 rrirxjles -inut. '
tr 99' O- 2 rninutes
Ir9 flat'reqJire--oflhplontic maoagemenr I rhe primary denrnioh wtrich mqy alfsti rhe pemnen(
den ulrqeated are i
A eEdq cmssbires C- pemicious bral habiLs
B. dlo&theSe '^.^--^.-;.-r;--,-:
t"taihad primary inciso^
o.rm.-;n;t#=i=retlnique exrcrndr (esorpiion or rhe.roo.' , ,,i" ,.il?
Prrpolmiized- teeth c- Repositiooed ,racrrrre roots
, 1
u. s{rargnrenec, ilkytosec, roors
L D- Replanted awulsiOn

Ortho , Pedo Set t

L l- c
2.b
2t-
22- d,
b 41.
42'.
C

a_
61- C Br. d
3- d' 62- a 42. d
23- b: 43. a '63-
cb 83. c
4- c 24- d, 44. u_
i
5 d-
a b 44. b.
25. b
/-
I
45_ b 65. d 85- d
\ 6.a
l- 26. d 46. a 66- d. 86_ d-
7. b - 27. d 47_ a. -67
B. d. d _ 87. c
28- d 44. d 68. b BB- ---
I 9.b 29d 49 d 69. b
10. a' 89- c
30- d.
I

\ 50. d. 70_ c 90- c


I t- (l .
31. c 5t_ a 7t_ a
12. b- 91_ b
3?-.--- 52 co 72. C 92- c ar
1.3- cb 33- a. 'd
|t -'
t
l.t
15.
a 3a. c
53 a
5e. a
73.
74 bq
a e3-
94b
a 35- c 5s. d 75_ a 95.
t_ 16 d a
36- c. s6. d 76. a 96c
t7b 37_ d. c-7
d 77. b
IB a 97. b
38. b 58- d' 78. b og- a
19. c 39b 59. C 79_ b 99- b
:--
I 20-. a 4.0 a ,60_ d BO. a 100- d

)
II
\,

Downloaded by John Erickson Bangayan (bangayan1903064@ceu.edu.ph)


lOMoARcPSD|19937673

P edo.Cormtrires,Acud @r$,pdoo tiics

a.
increase density ol laoiina dura radiographiealty incomplete'eruption .
\ b.
dull s,o.und cv giercussion change in color of enamel
b 2- What aE1e'giroup is a space rnainr^a.in et.dttllcr.ttr to rnanagie?
a- 6 year old wirh loss o{ a primary mandi.bular Jirst motar
b- 5 year old chikl with loss of prirnarj, mandibular second molir
.c- 5 yeir old. ctrilJ with loss of prirnary rnaxillary centralincisor
\ d- nine year old with la:s of a, perrnanent maxillary. iirst molar
A S- For the retention form ls class ill arnong maxillary teerh
a- lingual dovetail c. all of these
b- pinpoint forrn is recommended O. ldbial dovetail
D'4" Wlhen is the bst timti ro ueat distema beMreen 2 mdilanf an(ericia-teerh. it the result's-a lotv and-fa

t
shape frenum?
Q. qruption of. perrnanent laterat inci:sors
b- 'complete penpanent dentirion
0s5- Formcresol.pulpotomytorheatthychildrm,isoota'IEalrhhiiardto&cm'butcaiiiionihould.beobserueror
children who have-
a- chicken pox q- pneurnonra
b-
rnumps d. .theurriatic fever or 4ephritis
-6" An anatomic crossbite. conirasteo wi0r a lunctional crossbite usually
L D
d- deviared cjosure r.o centrig occlusion
,b- smoorh'ilosure. to cent_ric occlusion
deoronstrate:
c- iyminetrical.individual denial arChes
d- marked urear facets
A l - Which o[ these refers to rhe atisence of rhe dental lamina:
a- congenita.l absence of the rhard molars d- ; continuous growth o[ the anterior crarritrl
I b- tongue buccinator mechanisnr uasd,
! c- tong'ue-thrusting
D g. The. -.it reasonahle.res.toration for rhe grossty broken down anterior-prirnary roorh is:
a- resn crovJn
-steeLcrovnis c- portehin crowns.
t)
b. staitules d. gdld 6owc
\ 0 9. lYnich o{ the followini iahibit norml.grow6 and developrirent:
I ,a- environmenuj'factor traumar
rb- funct'rOnal devjation .,. d- dysplasia
D XO. -Ab a Qer.reralis( you can.perform the phasesbf orrhodonti'cs tike:
a- rnu'lcibanded technique c- comprehensive.phase '
.b- Begg technique d. prop hylactic -o rth odo n tics
t
)
I 1l- Maxinary widor increses is greater uran rhe manqibutar. be_iarrse:
L- a- o( the disral dpping ol primary cuspids. inro rhe prirtrate; space
b- ot growlh aad developrieAr
c- _because ot'greater skeleral width-increas.
'e
ia Ole-mdilla
d. the mnillary alveola, p.oces.direrge v.vtiite ,rrandibular p{ceses s ?re p>rallel
I I,^ 12-Which of the Fl. changes.in soil tissue protile ile aif'ected orihcogntic rreatEieAi?
I a soft tissue thicknes over n6im c. 'ihickness ol Ote lmer lip
t- . b. solt tissr (hickrEss wer pogsnion d. thicknesfot upper fip
D. 13. tlprighting of-toorh is ditficutt il oot sl@ in prog.ess. because:
a- there is,lack of mchorage c- ove(exle-nded beds
I
b- Occlusion is not relieved d. overcontoured springs,
C 14- An op-.menI ol human behavio( ,aquires a kn cohcept ot:
)- a-
b'
dence c. matu
d. gene ion
C 15- Act la are:
a-
ronto-nasal,process
f ail of these
b-
maxillac/ tuberosiry midpatatal and tianspalatal suture
O 16- Crossbite of (he llwillary permanenr iociss E caused by:
a- thumbsucking ldge c- labial trenum
b. ovet ,etention ot a maillary pvrmary rmrso, d. premalure los or maiilaqr'brirhar), inciso(
C 17- Whar level of sies ,educrion is associared wih gptimat ttuoridalian )f cornmsiry w;te-r StpiliEs"
I a- 3006 c 55%
I
b- 809,6 d l5%
l- C fB. Premalrrre exfoliatlon of rhe.prinraqy carrine may indicare .; i

a- excess is arch lengrh c_ arch length-cleficienclr. :

b- crowd,ing. toss of space d. a skeleul defect gfvi{rg- rise'to rnalocclusion


C L9. Aviraminosis A & D:
a- is coosidered .borh locat and systemic in b gmbrffinid ctlefuncdon

L nature C- is a sycternic cause of malocclusion

Downloaded by John Erickson Bangayan (bangayan1903064@ceu.edu.ph)


lOMoARcPSD|19937673

L
I

t_ d- is a bcat cils€ of malocclusion


O 20. Ecrogc o_r delayed eruprion ot pemaoent'teeth in the anlerior segment is caused b1':
a. teeth rigtratioo c. presence ot supernumeraryleeth
b-' aI of rhese ar. pieihatirre loss ol primary teerl
A 21. whichor.l,tEt
I a- criEtemil sorirhs together
b. diasrema 6
c. inrra arch elastics
d. exrraoral lorce
fr ?-?- -tn or.der to resrore.the original.brdad contacl, ln priinary rnolars. the class 11 matrix rnaterial
Should.Ue pt5ced wirh:
a-. g:realer cuivature that^in the pe,rinantrht
D.- irregular..curvature than win the i>errnanent
c.
d..'
b zz.

L 3:'
b.r
c-
d;
QTcterig.des.rnelaninoogenicus
staptrylo'coccus. apreus
0 zq. The chhnismiis ttre:

D 2s.
b.
a.

-Gro e Jace re+rtts in:


i.E
L a- . {ilter,ti-'poqterior
o(,the'[apg
reladon of c. d
d: a
bones
t-dimension
b.
an increa<e in width of'the'rlace
A 26. \A1har is rhe advantage in the use of..extraorar.anchorag.e?
a- pelarits poste'nior rhovement of teeth in one arch without,disturbing. the opposite arch
b. increase overbite
c- more'f orce. can be apptied
d- ir has direcr reciprocal dition on.rhe opposite.arch

t a- ,or;bha[ plan'e
i
I
b..
ro.. syrnrnerry plane
syrnrnerq/ plane d- fatjal.'plane.
D z8- A' year bld patieot.deroonstrarcqufilaeral.ptEcal irop_shite in-eer*rt ircdusioh-
.r7
btcdus devia tio n o,l t h e nrar,rdri,bu,[a,r
ru'rrid'[,in, e . 6qate.ralcoistrft{bniot
rr.idtirc. b !l ate.ra [. co nstdetbn i of- ntsoG;tilrl*i{udrlJa
in:x-nhryiir.tt] arri larural oirti6" tiomlcenric retarisr ro ceabi
oc€sim-
oe'casio'n- What whar is as rhb ,rris,t,lilaetv l'* rlrErt
(hb rrrlst'|ilcerytre: i'rrEnt br frts ca<ez "
rhlF case?
a- rqassess the malocctusion in. the permanont'den$iion
) b- unitareral cross-elibtcs. torviden'the constricted side
c- bilareral arch wlre expansibii
L 4
d- bilateral pataral'expansidn'
?9- Ihe giroes.srcf resqrption of rhe roaLs ot lhe primary teerh is_conrned. fojlhe: .
a- any,panand/orsurraie.:ol.ih6 r-00rs c. . mioiilg.trrird onty
_.- b- .ceMcatihirds d. ,dpieifthird odty
^
D 30. A primary second mcilar-is ininfraoclusion and ils occlus?it surtacells lgvetwith'ihe ,ingivat margin or{he
permanent'rirst molhr rvhich is bbgihnirigro rip m"=anyleiai"gr"lts i"Ji""t. i"""roping permanenr
secdnd premota'r is present. The detldst shorildr "
a, mainrain al\rree(up(
b. extract th intain-er
\I c, main(ain
f-. .d- extract the, prima.ry'rnolar.,:-._
t- D 31- 'simple arilerior crotruciing rnaybe corrected b;c
a- expansion of the,wire ' extraction o.f one anSerior toorh
b- extraction of one anterior tooth and disking ,or strippingi rhe mediat or dGtat
eipansion of thd wire surfaces of the anterior
D 3'2. A modellcast:woutd,g,ive the.following inforrharion excepr:
a- arch. shape and symme{ry of rhe.rnaxiflaqy , depth of'the palalal vauir
mandible ;':':.ioblination of the r(>ots
b-
an individual tocth rnalp=sition
C 33. Cleidocranial dy=ostosis.is of inteiest ot the dentist because of:
I a, high inciderrce of clef6 c- multiple supernurcrary leeth
b, micrognaahia
L q, h.rgh caries'inrlex
C 34. Whar appliance should ool be used lo upri?t|t a permmenr o,ta d,Srtar second mhr balore cmstruuring a
tixcc, paniat delture?
a- banded segment with
b. bandedsegment wi6r
c. acrylic removable seg
L d- acrytic ramovabt€ wiih a helical fiooer,spririg

Downloaded by John Erickson Bangayan (bangayan1903064@ceu.edu.ph)


lOMoARcPSD|19937673

4 35- Recrangular wire is used in onhodontics applinces in order to:


a p{wenr buer and lingual tcbth mvemeirr c. inc{easi st mgth,o, appliancb
b- red6e the suess.m the preffigtar d- prewent irritatioo d rhe.g.iogiv:rt
--
\ C 36- These are. usualty obseryed as histotogic changes afier putpotorny. arrmge them in ordeJ
1. n*r odontobtastjc laye, at wouhd sire
I 2- bridge ol denim ,ormed
3- necrosb o, superiicia, pafl. ct, pulp nea.esr,catciuin,fi)dioxiOe
4- acute intlammatory chaoges ,(om imhedia(a iis$e beneath
a_ l. 2.3.4 c_ 3. 4. 1.2
I
I
b. 2.4,3,1 d- 3.2.4.1
)
A 37- The rpst preva-leni gtoup ot matocctusbn Angte ctassitibbrion is
a- Gtas*l c- Clss lt
b. Cls llt i- prevalence,ot rtraloccJugioh:
A' 3& FoIwing'premature loss. oa'lhe primar!, second' motars unerupted, first mf€tr win rqrJ6 ina cause space
toss in thei
.., a- lhe maxilla crr the rnandibte c- neither the- mandible no? lhe in3;6;;3..-''r :
b. mandibte _o.,1rry d- mhxilla only
D :s- ln absotuie nece;iiqr. rhe rote is:
a- disdisi the patierf tor mother app{iinrment
b: rake fiine in erplainirig (tte.proccdure ot dedtal rredrmem he needs-
c_ reaer lhe parient to a-pediatrictdentisr
d- to ,orce d child to submit to a trearmedr
,b A 40. GrNrh may resul( in rhe toINing.EXCEPT:
a. contiro'ousiincreise in.size c- change ln tgKrrrq/aa in quaririv-
b- cbange in foim,or propodion d- iiicre;ae.o;deci*Ase in slie
A 4r- what is your treatrilent lor a child with open -bi(e. The poslerigr teerh are the only one'i:ttrarcooacr rte
antagooist-
a- reler lhe child to an onhodonttst
t' remd'e lhe poste'ior'reeth in'brjth abil
b. re.move the arerbr reeth irLeach quadran,
A a2- olrins rhe primary denririon. pseudo crass il is manaseri
o, :

a. Manual movement "1n"Xl?lifftlt&?Eileil.


c- tonEde blade tgrapy
Headcap and.Chincap d. Stfit;+n
^ I - and Chin*p
B €. winci par( ot the priiury mqlarwill tracrure is-cts I
L a. tingqat nlargin ot flie ocalusal.acpect
resrol..aion:
b. isthmus
c. aacialrld.gin c, rhe inreproirinefl aspecr
) d- gingiral rirargin
A 44, The last piima.lf reerh lo uodergp reso(prion ol the .oots slrnine ycariago are:
g a:- @nines c- cenr'l ircisors
b. raterar d. prim'arymolaE
O 4S:'A 14 yeat old ,ernale ha deep venical pockes ivirh bone.lcss on me meiiat';:rect ol alt iirst motas bnd
s\omqdrifting ol mililaiy ilciso( hEwever. onty minimat irfllamritatiqn an<t.irliniriia'i plaque ade presenr. Whar
is Oe probalile diagn6s's?
ar oseomi/aris c- eenodonri[s
o b. giagircsis d. j!fienie periodonritis
--
o 46- A .nothet brings her dqughter lo the denial otlice ao^deotal tr"i'b,ii..t- ilin-rcat examinattoo t€l€al norml
occtrsaoa.afrer'rosingapr.imarymandibularsmondmcilar.wuat,mristoedi"rr.rodi ''---''--l--"-'-'-'
a. plac€ a linouat arch space mainhiriet
b. e\raluate lhe radiogreph drld bse his lr€aameGt with high lindings
c- obserye any dritliog d( rFelh
ct- use a lunctbnal sDace maintainer
p 47- Amilng rhe tblbwing, which is rhe'besr space mainEiner?
a. rernovatile acrylic applaanc€ c- {ixed lingual erch
b. band ancl loop a5rpliance d- a pulpotomized'piimary molars
C 48- The prrrpose of Snyde'r re9 is to:
a-
determin€ Lhe. exact nature of a specific organasm related to caraes prevalerrce
b.
predict rhe rare ot'saliv6qy flow
c-
predict, the narure of..the combined acidogenic organasrns in the orat caviry
d.
esdrnare the sativaqf dissotring caFaciry ol enamel
A qs- The ff- are essential crirCria lor ochbdonric iiagnosis exctuding
a- case history c- cilst ana\6is
b. radiographic ar,aly:SiS d- clinicalexamination
c 50- ln a class tl Division 2 rnatocclusion urhich or rhe ff- condrticn rs present?
a- perrlanent tir\r rnolar in mesiO-occlusion c lingual incjination of rnaxillaqT cenrral
b. open bfue laclsor

Downloaded by John Erickson Bangayan (bangayan1903064@ceu.edu.ph)


lOMoARcPSD|19937673

t
I

d.
A 5l- Los ar Pbns
toltouring the
i ;ffil Prernature exracdon of
which
q t
. 52
_ D^^ rnolar
r 5-
9f rhe prirryry r€,erh?
c- 'iirarillio r:-.'--."
ffiffi:,5:S*;-,
XA;:ffi;'"rn'ffiltnridona,euearedwirh:
1 cxt acdon d'e uealed with: ' -'srs'.rus secohd rnolat

o lt*?ffiffisrl**ur.-- - c' s,tinrins


' ,J* *,, _" rrea! a chib wirh a st,ghr oow:;ng L,f#k , r a'derior and extdliarion
o!
ve comprerery erupted

ffi"T.fttrfi#i#fffe.-'.{^,s
__..y. Fr. ano.rnuftipb
cdii)us
-
Th. reeo!.
i\-' bert
._, c. percussion
the both
premqrrre exuacrion
o! *",* o"l;#t?iTn
ola,
in*.r,o....",temanasedby,a,iria,izej:n#*u.#;#ril_,

,H?:r,ff .r*,marsurf aces,.rn.rr,r., jrrlrT.HJ,":ff "


Iarry rnolars. stoul(,.
restore
L- f,fl':tr, <t. prope, burk or
l9.s,h minimized alrJeslotarivc matcriat .o
dirrer in behavio,
io rhe denhl ofrice?
W..hich o, ;,)Uowing is ,ehred
lo rhis
teaming anct marudry
:.
rlffilt:"'.il **rior crqssbire
- *-"ic. **TJ#rn"*
blem
bo o{ rtie prepararbn,o
rorrj"
I
ar- r-gas by rofl@rirrg
tihe1,-
-
ae osp
L
m the .=A.:: .

' "t #"H;g,gX;;f,i]3'=,."o,,n. i rercandne,p"". rr;iu


##[;#F,-""*
._., r.r€rrrar
, central
intiso. urlh hcisai
b.
c-
d_
A 64. iln
ace
a-
b- before €'uPtion of perrnanenr
--' a
- Fr'rrdnenl first
rlr$ rnorar
losl
losr molar
r eruptron o, pe.manenl
a tirst fircrla,
9 anleao. crossOite ir:

o movc ree$t h thei. p.oper relanionship


wiarr lhe o(he6
ave
c. cofttEX
usatfy when ir reaches
srage o{ *,r"., ."a[n,"]",r3.*,

Downloaded by John Erickson Bangayan (bangayan1903064@ceu.edu.ph)


lOMoARcPSD|19937673

a- stage 2 c- g.rage 5
b- srage 6 d- 's69e 7
68- whar crireria will difterentiate betvreeh a trqe class lll a.rd a pseuito c{asslll mlocclusion?
a- the presence ol a lorward shift of the c- presence of bilatetal crossbhe
mandible during closure d- ocglusal relalion$rip between mairillaly
b- anterior crossbite arch rnandibular fkst molar
69. Shell teeth is associated with development disturbdnce id:
a- eruptton c- lunction
b- structural formation d- edoliation
70. The.rnost imponant denral tissue lo an.orthodontist is the:
a- pulpa, tissug c- Ddnodontdl ligarnent
b- cementum d- enamef
71. intercepdve orthodontics'is prescribed to'colleet crowding br inc'sors which is rdone by serial extracllon ol
<'
rtrhat teedr:
a deciduous second molars
b- per(Bnent canines and secohi, molars dH..
_,.
-'*
c- permnent li6t mlat
d- dsidmus.cilines, deciduous ti6t rilolars, ircluding iiisi'premot;|is
72. A modified dass lll prepeariod.lor an artraQam idsroiaiion in a iiJimary enine-is chJacletized by it's:
a- elimination ,or the need olane$hesia
b- lacial or lingual iroEtal
c. lack o, estheut
d. elimination of,the need ol a mitrix
7l- \vlry urt trur(! luU;r rpuru rrruil[uitl.r rrrq$ritit.llt!rt. ,i'r D.rilmlll't lorr ol.ptimoryAccood mnlir'
L spacc loss occurs rollowingl.emat(fe €rti0{'y)n or eronf,pterpolarfanrl reiiwal ol,Ihe-baxd and loop
b- a lingual nch will maiqiain rhe.arch'len9thI1!!-rd therdrdre Greate dtslema
c no.mal erupuion sequence is distirrbeu tolldfling exractio,n
d- norrol eruption sequenoe.: OisnrrbeO tblowi6b.jex(raCden andspace loss o@urs lollowing PrematlJre
eruprioo o[ seond prattnol4rs ainEieinq\ratoifh';'.6and aira rcop
7q.'whar is rhe chaEcreristics'dtr.iwen'ile.diabe(es rirellitus:
a- Blindne-ss j c- athxia
G
D- rnotorparalysis d- .,deafness
75. An archal dinnension completed d-arlidr.iarriong girls-at age.1o then in b'6Js:
a. intercanine distarce c- rntercanioe depth
b. intcrcaninc width 6; intercaninc fcngrh
76- Prognosis following. fracture of'the root-of a permaqent 66:qillary, inciso'r is rnost favorable if the'lracture rs in
the:
a- middle lhird of the root c" aRicat,third o( rhe roct
b. coronat third of the robt d- cervical'thirds
D ll. A poor man's cephalogram'refeq.to-tacid evdiuation because of:'EXCEP-'
a- skeletal iavr 4elatior,r-:hip caq be obsd'rVed c- iew to iaw rslatibn can be identified
b- 'posterior crossbites are revedled d. md,ndibular protrusion can be detected
C 78. Loops and helices are used in arth wire6 prirr.rarily to:
a- align tooth c. holds soft tissues away ifrom orthodontics
b- lncreacc lorcc o( thc v"irc. Dt rruhu(S
d- increses.'llexibility ot the ryire
( 79- guccat coil spring hay€ it undesirabl€i-side iffects srch as:
a. initiate iooth movement ' c- Endenry of plemolar to ]b(ate
b- -gingivalirritation ,d- interrninent pain
4' BO- Co o[ deay on a c:lss I cayity ol a decidu:q.us mCila(. the pulp was exposed- Follming
pul om hydiori(e.ro rdatlhe remaining pul6,.dentiD-witi fbrm:
a. 'ampurated puti c: aboi/e the levgl.af the ampurated.,pulp
t,. ar the level ol the amprrtated pJlp d: at tHe apical alga

U *'n.n awlse looDr is rcpln(ed the.,totlor/\ring ate cbrrectexcdpr \


"r. a- hardserubbing should 6e avoidgd lo.B(es€rve pefiodontal membraoe
b- curene lie atueolar socket belore insenioo
c- 9oq6 succcasfiil il 3wltion o@oE les than 30 minutes.
o. oon1 rel:rwc loo5c tlssuc
B2- All body rissues. except toolh el$Dd is r.dplaced about:
a- 6-7 yeaE q, 3-4 Years
b- 5-6 years d- l-2 years
D S3- Which ot-the fotlowing changes in sqft rissue'profile age effes_-ted by onhodonilc treatrtenP
z- soft tissue thickness-over pogonion c- thbkness ol lower lip
b. soft tissue thickness nasion d- thickneqs of upper lip

Downloaded by John Erickson Bangayan (bangayan1903064@ceu.edu.ph)


lOMoARcPSD|19937673

t
I go-
-a-
'b-
c_
|-
I
d.
t
I
b gt- Whicn.
Wh,ch-g(3Jre'follqruing primary tc.eth dre rnost severetlr involved wirh nufsiog Brigs?
a- s8&tiit'motar c mandibular
c- inci<oi
rnanrli!=rrrtlr i5rcisprs
d- topical-application
?

L A 98: Syslern'ic fluoridbtion rnay be accomplished-by a:


a.
b.
c- h fluoride
d-

L 'D gs- One of Fg$e-bel?ar b the cause or lailure in cta<s


a- rectirient caries
il arri3lgam re.storations in, primaryr m.olars:

b. wide p.ffp chamb'er


c- po.or pri:paration,of thg gjngirr€deoral rnargins
d- -narrow pthmus rntricli.evennJany chn fracture
L

,()rttr,o,. tredo Set 2

l- a
L ?h
a
a
41. a
42_ A
61- b
62- c
81. c
,82"
3-a b 43. b 6:3' rt 03, d
4-d d 4d. a 64. A &t. r;
.5-d d 4-5- d
'65- c &5- c-
6-b a 46. b 66- c fu.a
7- a c 47. d 67- b 87- d
B- b. d 48- c 68- a 88. b
l, 9.d
lp- d
a
b
49. a
50- c
69. b
70" c
89. d
90- b
L l1 d d 5la 7t- d 91_ d
12d a 52- a 72- b 92- c
I
13 b c 5J- d n.4 9e- d.
I 14. c c 54- b 74" a 94- d
I 95. d,
:
15. c a 55. d 75" b
16. b c 56. b. 7'6- c 96- b
17. c a 57- c 77. b 97. b
r8. c 'a 58" a 78- c 98- a
I
I r9. c b 59- c 79- c 99- c,
t
L- ?o. b a COa 8O-' a r oo-d

\ I

I
t-

Downloaded by John Erickson Bangayan (bangayan1903064@ceu.edu.ph)


lOMoARcPSD|19937673

Pedodontics And Orffrodon ucs


't-s
A 1. what technique best applied u,tren gaining cooperation of,dentai patient vrho is a the countqy?
A- tell. show and do C- dental education
A- orientation D- sedation
C 2- What is a mGl when doing surgicar'Drcectures amng young pade with chronic inlections ol the teefi?
A asepsis C- all of the lenered answers
B- antibiotics D- x-ray
Ds. what is the mst imponanl clinicat lactor in the correlatim of lhe majiillary atveob( growth Md rhe denbl
arch widrh?
A. no trea(mcnt can be accomplished in the mdilary
B- milillary Gnl be ediv attered in areaimenr
C. all o, rhe givm opriore
O. milillary an bePsily artered in tieatment
, 4- The clssiliation oilracture olilhe deciduous roottlwere lhere is an extensive traction sown s{ih pulp
exposure, ts
A. rlpe lV c' tYPe ll
- S- e B- type I O- type It
D Oevetopihg mandibutar prognarhiim uJirh mdiltary'deticienry cmlbe irirercepred by
- A
B-
Hawley appliane
Use ol chin cap
C. Iongue rrustin'g
D. Musde exercise
4 e. tne roengenograph exam'inalioo lhat shows best incipienr inrerprotimal iavities is
A bitewing C- no-O pedo
a , t";.3r"*llilliecory*rricrr.slaresirhar borie dd cedrase ,".. n"&fr;*iTlffit"o. e,Gvrh is
deteimined by luncrionat matties- The skeletal riiiues sUppon and protect lu'ncrional matriis- Ihis theory
is knMn s hrDort esis.
A_ sicher's c- Mosjs
B- Petrovic's D Srnn'q
6 8- The diagnostic too to onfm rhe skele6l retarionship oa rhe mailla ro rhe riandible wth ot its landmarks is
A panoramic radiograph C- dental casts
a --
cepliaJonrerric radrqgraph D. periaoial .adiooraph
The requirement ol.a 9oritd space maintaihdr includes: 1. lt shoutct. maini: in surlicie-nt att for rhe eruption ol the
permanen( sucesor 2. tt should not iolcrrere with the develgprient of lhF toot r and tlE denial arches,3- lt
should providd luncrion idrlhe sptrcti. a. lr. shoutd improve the:ii:pearifneE otthe child
A. 1.2.9.4 c. 3 and 4
B.
E. r.2,3
t.z,J D
D- 1ah.t.3
1and.3
4 1o: Prir4ary leeih with gross pirip exposures and dqgeneradve pritp andperiapicd lesiohs: l- be atway! be
cxttuctcd. z.-May dleg"tt succeerla4euus lcdth.causlng:hypoptritic 3. May cause rhe:tormtbn oI
" rhe dnamel surfbce. a- unaninded. espeeiafy tihe child is unmanageabte
dtsrective or irregular piLs on
A. l-2.3 onv c. I only
1 ad 2 onty O. 4 onty
-8. rlifference bettyqetr llre riFgatiye r+inlo,ccmcnt._and pujishmen( is
f- I't The
A- F,unlshme-nl weSkdns thdiespoirse qr- producreg ani'iilaive durcome,whire reinfoiLement
s(rengtheng it by a€movirig. an aiersive Ouicorhe
U- puur3ltrn:r.,t -.,r'{}Iutlteil5 tlte res-pffisevl)oae ltun Utc rri:gaWe rernlorceinent strength
C. punishrrjent weakens lhe .espoosc mbre thdn the neqaaive ralntoicerrreart rcakens
_ O pugishmept strengthens (he response by.remwing ao aversive oulcome
J 12. The aeerh rhar trequently presenl dnomalies in inOrpnongy ot rhe crirrm in lhe are
A. f.,laxittary 1'bi@spids and mandibula( 2d bicuspids
B- tr,tuittary 2:s bicuspids and.mandibutar tarera, inciso(
C- Maxiuary bicuspirs ild militrary bicuspids
D- ndibutar Zd bicuspids
D rl- Ttre i ta, caiies iNotuement of rhe mandibutar primary trrst motar is

a
C 14. The anab6is used to evaluate the amount ol space ava;table io the arch ior
3 3lil,i:JJ!,1?Jl,l,lfr'
rhe suceedaneous leeth is
calld analysis
A. Peck arrd Peck's C- Mixed,dentition
B. Nance's D. Boiton's
15- The hyDeractive men(alis muscle Gn 6use gros malcciusion tO the anterior t@th which is correcred by
A, Hawley appliance C_ Musle exe.cise
B- Oral screen D. t-ip plumper
D 16' A child.T years o( age has a la?ge sptre besveen the permanent cenrat incisors- A supprnumerary (ooth is
suspecled: The choiee of the lilm lo, roentgenog.aphic exam ro detect its paesence is
A peria{rical tilr C. panoramic ailm
B- occlusa, lilm O- _ay ot ltle choice
B -l-7 Sequecce and time ol er.Jprion ol perrileneol ieeth are targey dete(mined by
A race C- habits
B genes D. nutrition
D 18. The following grovrh rheo.], srates rhar the groMh of mailta. mandible and rhe cranidj base is influenced by
th- rho -6-r..-
^rdil.^o ^, ^'crt

Downloaded by John Erickson Bangayan (bangayan1903064@ceu.edu.ph)


lOMoARcPSD|19937673

4.. MoS S C- Sicher's


L a. Petrovt's
E fS- Cheek hriting b corrected by the use of
D- Scorfs
A- blte bloek C- cross elastic
A- oral screen O- Io.ngue gudrd
A 20- Retained inlantile swalloraring into the childhood W resuEing in any of the follorruing:
1- Anterior open bite 3. labially inclined rnaxiitary and rnandibutar inciscr
t
2. Tongue trusting 4- Mandibular prognathGm

A. 1.2.3 C- land2
B- L only D- 1.2.3.a
C 2l.TheconttaiMictionlor'.emovalolpdmarytee-.thirclude:].-lpreqehcEoracute,oral.inleclion,2-pIesenceol
eute R6temadc intection, 3- children wirh unconirolled:diabetes rnellirrrs.,4-,leuk'emia
A I only c- 1.2,3;4
B- f.2 onty D_ 1.2.3
A 22-Bonesg'rowbytheaddition-olbsetissue.on:oneside:olabonyco{texandEkingaway.lrorn,theotherside.
Ihis procss.s is kncnarn as
A drift C- deposition
A- resorption D- ,rranslocaiion
D 23- A space maiitainer an,be placed in rhe deoial arqh uqder ihg lollowirio,coirdirions: l- Lgs ol og'e or more
primary reeih. 2- No loss ot arch pdrimetei.3. Faircralile mixeO OaOrition dnatysis. 4-'Eary eruptbnlol I'
peri"nanent molar which h-as mesially rnoved
A- I only c. t.2
B. 1.2.4 only o. 1.2.3
b Za- Abberations-during the ini(iation stage of tooih devetbprnent qlay lead ro
A- none ol the above G- anodontia
B. both supemumerary'qeeth.and anodontia D. supernumerary.teeth
0 25. Ao eroption ma:<il3ary lateral incisor which !s lingually locked cdn ,be corricted eaily by
A- referring to an orthodontist C- 'leaVlng it alone
B- extracting the rnaxillarv deciduoqs cuspi{ D- ; tong.ue blide/d€pressor therapy
c 26- lnclus-ron q/st along the median raplre in neurborn infanB-are kno\Arn as
A Bohn's nodqles- C. Epsteinis p.earls
A- I'J.rtaJ teetl r ft- Mrrcour,letentiOoqTsr
I C 27. The sourcs ol rhe torces. ihat mairilai(he sdcood ,nanuibuF irciar in place inclpde: 1. mesiaLlorqehom
(he {irst pemmenr molar.2: Fj;si'pr&nlrry,:moladgrertind anbppdsif r,Oisrat tbrce. S- Tonque ma, c6eeks,4
Alveolar process and periodoriial tissues
A 1.and 3 oohy c- 1.2.3-4
R '1,.?.4
'D- 1 onty,
C 28. Orher narnes of rnilk.teeth are
A_ firsr C. allrthe lenered ansurers
B. deciduous D- baby's
4 29- The behavio. m.inagerent"lhat increses lhe'likelihood ol a particular behavior,,to than attempting to repfess
rI ts
A- positivereintorcernen[ . C- punishment-
B- borh of rhe choices O-
none oI rhe choices
8 :O- Among the etiologic lactors in matocclusion in a chitd the follcm.ring is the least likely rnodilied
A- gongenital defecr C- nutritionaltdbficiency
B-- heredity D- abnorrnal oral
, 31. The leogrh o, the dental arch ,rom the riesial ot one mandibrrlar,tirsi permanerir molaa mesial surtace of the
corrssponding toorh on' rhe opposi(e Side in alwa)6 shonened during
A rnixed C- perrnaneqt denUrion
B- primary dentition D- transition
n 32- During extracdon of prirnary teeth rhe loilowing should be considere.d
A- botlr choices
B. the psychological prepaqedness of rhe child
C- none of the choices
D- the curva(ure of the roots of the teeih such as the rnolars
n 33. The musculan lorce that. influence the shape of the. dental lrches are
A- Orbhularis oris C- Buccinator
A. Masseter D. Tongue
I 4 34- The place d.awn lrotrr tire langenl ol the upper external adrditory mealus fo!the jnrerk)r rybit is the
{-
I-

!
A Frmk un pjane
B- Cranial plane
C- Palatal.ptarie
D. Furcrional plane
A 35- The management ot (he iarge disance berween the milillary incisors ar rhe age
A- recomreod clcsure with fixed appharces
8- takc an x-ray ro find-oul the presGne of ,a mcsiodens
I
i
C- all ol the.choices
I D- rule our fibrous labial
t_ I36- A c-year old child visited a dendst for lhe lirst 6me aod received propyljla:,S quire well, Lerr the dendsr yr6 so
glad rha, he gaw rhe child a toy This kiod ol geStrrre iS Callerl
A classicaj ccnditioning C. bribery
'A- positivereinforcernent D. public

Downloaded by John Erickson Bangayan (bangayan1903064@ceu.edu.ph)


lOMoARcPSD|19937673

5-

A remains rhe sare in bo(h arches


B reroins the silE in rhe mdiltary dch whita it deseass in rhe,mandibijlar arch
C. decreEes io both &ches
D, insea€es in rhe.,mai:i[ary arch while t decreEes in the..na
O 34. The tolchri€ is the diFfereoce ih size belwain lhe primary teeth nent in rhe poslerior segment
A. posreiiq liatritiry C_ discepancy
a- Nance's leewav Space D- latb rneiial shift
k 39. avoid while t eadog a ycung uniobirerative
, af|d becoming arlgry C- a
D-b
O 40- ( vira, putp tissue trom the rool cahal and co ary
A apexifietioh C. root.ean:{ tnerapy
B purp6romy D- purpecrordy
^ ,r l- tniury to rhe pr.ima.D. ddmido, (nay euse disrurbeces
D in Ore dwetLdnsilerrorient sorne
of the
cooseQuencg ro,tiauma incbde
A nooe ol the.choices C_ odontcima-fke mafforinarion
B- -borh,ihoic€s 6- enarhet hy?dptasia
b 42- A^ over-reaainedlooth should be e)cracted
A as sooh asrthe suceedaneous tolih sta,ts to erupt
B- as soori as.it is observed rhairhe succeedaheouswill be matpced when ir erup(s
C-as a geneEl rule
o-wah and se unril rhe succeeoaneous roiith ,ias;enipted so that the root d, rlre primary toorh wilt be
,esorbed
zL3. Disrat steFis iscaused by

E-
A_ mandibutarprognathism
B_ a,bsence ot primary Second molE s
C. unueared carious iesion on the distal sudees ol pdmary Zd molaE
D. mandibutar.hypeftropqy
4 44- Topical fluoride applicarion shoutd be rdpeated every

A_ retrudfed @nract C. intercuspal.


B- cent ic D. postu'i.ai
n 52. Achild anxiery during secondvisit.lo the dentat onice ciin bes-t be moirifi& by
A- bo(h choies
A. explaining'!o rhe child prmedures ro be dom step by step
c. doing exactp what he was rold md shdwn
O- mne ol the choices
4 S3. Tne mosr caries Suscepdble surrace on rhe tinst primary maxiilary .not& is the
A- rnesaal pir. C- eentral groove
a. bucca.l pit D- distal surJace
54 The apdieation and mechanical bonding oa resign material to qci, i:tched enaniel'surlace on non-eriou3 pits
and lissure ol ttre p.imary molars is klr@n r
A- occlusal seala^t G- proflhylactic a.nti caries prevention
B- pn re,vcnt;or) O. p
o" 55- one his tee(h versus permamint leelh thai is tlre
A..de c- i,
B. en hird rlrar rul Lr- b r
imisalty cir mcjusalty
C 56- Ihe besr way ro inrer&fropen bite dua to murh lreathiog is lo
A. refer rcr.a odhodontist for a lixed C- put an oral $reen
applieEe D- rerer to a pediarrician lor rearmenr ot
€} - instdt a nxed crib apptimc€ enlarged adenbids

Downloaded by John Erickson Bangayan (bangayan1903064@ceu.edu.ph)


lOMoARcPSD|19937673

t
C 57 - The primaae space iD a mrrnal prirury dendrim i;e l()@ted'l- -Ihe canine ild laterai the maiiltary..atch-2-
The canhe and laterat incisor in rf,e mndibular ych. 3-
.ihe lirst,molar and cahine ndie Daillaqy archi 4
L' trirst pnmary molar,arid canine in mah.dibular arch-
A 2and3 C- 1and4
B- 2 ar.d j D. Jand2
0 58- lnrnat position of lhe dentist is best sulted for treating tearful children?
Akneeting C- bending
B-
sitting doram to the patienr's leyel D- standing
A 59- The fcillowing are abnormal rnuscJe_and pressure habi.ts. except
A. hypenrophy ot the rnandible C- finger subking
B- rnouth breathing D- tongue thru-s-ting
b AO- A functionat shift ol the rnandible during closure maybe gause by
A cuspal pternaturities C- none of the choices
B. bo$r choices D- condyrldr
, 61- Wha( inorphologic pqculiaiiry a dentist must recognized'd'mong deciduous m6lars?
A- bell-shaped crow.n C- tgreater- jM-D diameter of,,rooLs
B- -lesser M-Ddiam.eter of roots 'D- bell shaped rooG
A 62- The-.permanenl teeth'-*rrich ace.edes behind the primary secood rholars are
L A. 1o molars
B. 3d rnolars
. C- wisdom-too(h
'D- Zd rnolars
D 63, The space rnaintainer of choic€'lor a trnlateral.toss of the mandibular Lo rnolar is. the
A lingual hotding arch C- Nance holdin$ arch
B- 'distal stioe D'. 'Badd
0'6a. The term to denote the change from generalized cells or tissues to-a rnore specialized kind
A- grovr,th ' C- maRrration
B. differentiaticin D- developrrrnnr
4 OS- Tlre tirsr reeth to erupt rn the decrduous dentition are (he:
A- mandibrrlar centraf incis0r:; C- rrr.xillury cenLr.at incisors
B- mandibular.second molars D- mandibular lirsr rnolars
C 66- A cemeot that is used as a luring as well aS re-srnrariwe rnaterial
A polucarboxy{ate C- gla,ss ionorner
B- zinc phosphare. O- nonE of these choices
A 6l - The long face sylrrlrunre'is alrnosr alwaji ,.*oc.iate with:
A rDouth breathing C- torlgue thrusring
B. nail-biting D- depression
L 0 68- What is usualtyr a"retlection of the an(ero-.i)osterior skeletal:,relarionshlp in tllb dentition?
n- .rll uf 0rc ghrcn trn3wcrs C- a*lal lnctrnarron
O- overbite D- ov€r jel.
c 69. The lerm used to describe th'e compressea and degeneraied periodontal raemb(ane onhodon(ic rcvemeill
IS:
A. compression C- hyalipiz.ation
B. atrophy D- resorption
C 70- The-toltowog-g.rg,w(h (heory s(ates._lhat the sutuces and the synchondroses dre qlr.e expanSion of the
cranial base and in the difteient areas o[ the'f acial skeleton-
A- Moss c. sichers
B- Scon's D- P.etrovic's
( 71- The most retruded and.unresraiiried pcirion o, theinBnd,iDle in which rhe antesuperior rhe cohdyles arsin
contacr with-ihe c5ncavirics ol the,ericular disk is koowi-bs rbe:
A- physiologic rest C- , cenric relation
B- centric occlusion D- terminal
bTz- The bone resporse on the tension srde is:
A- borh chcices C- i forrnation
B- depo5ition D- none of rhe choices
A zl- The grovrnh movernent of the mandible is cornpleinented by the growthof the mbxitta
A- [onnrard and clownward C- dovrnward and backward
a- forwar-d only D- lorward and upward
D 74- A child with ectoderrnal dysplasra wrll manifest the foltorving:
A. oligodontia C- anodoniia
B- enamel hypoplcia D- all ol rhe choices
6 75. ceriain Frpnaral ,iltn.s thal can cootribute ro gross coog€.li(.dl dctormiries wilh malocrlusion ue:
A- gernrarr rnea=les (:. troth choices
A- nredir:atijrn:; l;rkr-'n {t-lnng pregnancy D- rtu(rc o( the choices
6 76- All index ro der€rming wfle{her rhe lc^aer incrsor (eefii are excessivelywbe mesiodistalty utd to indicaic ttrar
rhe good ratio beween the ,aciolingual dimension ol tt{e mandibular ircisor and ir.s mesiodislal dimension
:lruultl br: approxima.te;t l:t ls ana.lysis:
A- Bslron"s C. Mixed dentition
e- Peck and"Peck's O- iJaoce's
C 77-- What is the rndjor cauSe oi sr;-nple singte Vpes of ant.erior crosgbites?
t_ A- unctional
f C. deotal
B- all of the afls\ivers D. skeletal
L7A- The besi tir.ne of the day ro rreat a child patient and to tdrich the- child responds positively is:
A- in the afternoon E- late in the rnorning

Downloaded by John Erickson Bangayan (bangayan1903064@ceu.edu.ph)


lOMoARcPSD|19937673

^ c. depencrent r the
cmpc{adon c,; child
&79-
i ; r";-;;;nt'upon p-d;;;;;fiu'l'tl""nc *n.? l-l"lETHH*" rn-ary reerh ii
rocore,,
sesuenceo-,",j"Y
- tB- hagh lactobaciilus inda "HlX
nursino C. rampant caaies
tt'ao. An aojuncde D. iwenile carbs
pa[em arid can' t'eatment ot'"11e-tJt^ g"1.oo*rd.rr,"i iL-intormadon on possibre grouah
e bone ag€. o, the padenl is:
A hand Misr r
. -- -A- cephatomernlcrdiograph C- panorarnierabogqph
" "t ':[ "'*;;sio. -he,e ,n. -.yo:I,:-l- *:;#ffiT b in habiruar mesioccrusion
A- E;lSi"" "1 and the rhandibular incisors are ,requentty in crosslrite is
ct I olrlrio..s,or
wirh
.8. Ct trr C- Cl xt airlsio. r
^
" " 'SI;}[:T awlsed primaq, incisor wrrere;n accioenr?oof;:tiE"*"t ,*
'ora^ be,ore rhe pa(.ienr w5
the looth in runni.ng
water then soak.in dGrilted
^ L*,1 warer do a pulpeclrorryr Oerore ,eimplbnring
the
B- there is no need:tolejmpla.il the
both becasse
I$rt?"'ff*,t *nini*"r", iililJ;"r?-Jic+
c. ..
s:. f..
purp. reimprant and sprinr
rheinalati;#;;:r?ffi"",J:=il*,LJil-,i,::""1fJ he
;,,fi'
none
1 physior-lhe choices
a- C. me.ttalty retaided
c- p.ema(ure
84_ o. borh a ine ctroi&s
A as so replaced try a spac€ maanlainer
B- sbme use a change in the arch tengrh
C- may be liyt unr
- ss. tnD-. atway-s uecal caos€ anygross changes in rhe arch
P i ot lhe child
A. "r*r ^"iJr-t
mesiobuccaj crusp o' the maritlary f molar
morar occluc3s'in rhe mesioringual-fossa
ot thi: ma.idibrirar l,
'8. rhe mesiobucq al
cusp oI the ma)dlldry ln
morar 'nola' oetudes io mesiobuccar groove or rhe mandi.gqhl
1-
C. mesiobuccal ct

' uu'
?;:"ff"n'i"mg,f lt*:l#il;;iJ:f:,1^e denrar,,d;;tu,";:i;; *,*,o 6ri remnans o,
A Epsrein.s pearts
o, _:..^
,r u1- tnrJs-orrsreiantioncysct 9. Eoho'soodules
o Thc rnoximrrm ir4ercuspatior o- Natal cysils
reladon'ship is the: ol rhe teelh vlith rhe Ir __ -__.
mpotoinandibular
'rre strucn res in a balanced unre$rained
usron
C. cenuic rEt-ation

A- 3 onry n- 4-
E 1.2.3 c. t onV
I- 89- T.he cla-isitication
--'' -' lrach,.. ol
ot "acture o. 1,3,4
den6n is: ^, the deciduous tooth r he.e ,rrarl'it I'Ir.-.rrre o, rhe withod irwolving
the
A typ€ tV
B. ype 1 C. type il
4 90- wha( ptacrice constiture o- aype lll
te n.p.-^,i,-
A generat .dent' rrv Pteventive and ;nterceprive odhodoirlies?
€ndodonricoeniisrry c- ProSthelicdenris:ry
-
u ^- _?-
91. The filling.materiat use( o. ofihodondcs
A- - - ot ptacement is:
B-
C- gam
o.
O gz. i a, :lT "rg. eugenot.
ry dentition
amargam
will ,esuli in
^ v relarimshrp
- t'!JfiHjiif,[i*=##
^
".ir ,fl,llF;iJ5,H#
a bi,a=rar and rurip,e

.* ,ij:!iF^:.i:E!Tr"1t";;,* o, p,imary mo,ar, *,*


urrur' 5*
permanent l' nrolar is xrmfl as: @
> rs talionship the
.q- nega(ave lr€eway : pace
_B- posidve leenray space

Downloaded by John Erickson Bangayan (bangayan1903064@ceu.edu.ph)


lOMoARcPSD|19937673

c
o ;,9
',. .oa (J U.c, U.o.o.o rOED O.o D O.o O Dr)

f
t-
f q,
U
U
o f, E B [i 3 B E B E ts 8 ; g B Y. B S ; A A E
c E
t
q)
o' 8. E

j
r!
o
ag
qJE
OC
x6
ia
.Ei
l!
q)
/o(!
cx a=
I

ut'E eu
0', q )3,
EL CI
I
i-0' g D (lcp D.G U .!D U !J 9.o OO U G U UD O
€g ,9 EE E
o Ec
Eg cui1.. N
ujr..ci
I
-d
F
Jo'
r^l-vOgUru?Eedd-$Jr"j
(D (D (D (o iD (D (D tO (o,.. N N F. N oi o
i\ t\ F. 6
dJc
F
r{
L oIJ E Fc'
b
qc 3 -I
Ed E=E
ro
E9
;c
0., 'F
l!
€'a
6 ",9
8-08 . :
o
o ots
qg .-,0 d ruv
o|
c-, qQ d; UOE.; E

B:E
o E
,vl

o
e9
c
0r
I]
ci -^. id
E
\
0,
--u
EE
I

t)S
L. O
.9 6 u (!lcElf
o rt.Q I
d I ! \, o o-G o o o.o oo
'6
D
c
c
0)
tr,
tc
)
g
'Ea
gE
;E

.[
?
".Ni ri
\,

t v or..tri
v rf v i,rj v lf v oi CjJ$lrj
\' rn
.er, toF.trjoiej
L1 n n t/) u)'tn tn Ul tn 16
'0) cq,
iE
0, .B
,x
s
tE
c ,8,
(,, I
.
t, c _.U 5.
d ,,o
6E
ut
o G' "3 fl o
E ot $l c
0)
c d €B 0,
]f 0 a
0)
trO
!o :!u o,
e.
€p B. i o p(!GG9.o$.c)mD
E E
0t 9l, (! uNlro!u u
U'
f
E
xq)
l) vl
sF
,8d
E
.o. IIE
4'bE-
A
a : o nunxnnrndsicBssEHEse
o
0)
o| icl r-
(U (u
ob
,s
qJ
.o. v)f
,g '6
co
6 EB 3 fi Est fi.;t
3i c 'fip
E '6c
0, ,g'
,o
h(U q,o o.
d
uf, o 5E .4

OJ oE Fg
Eu
-C l- E
o.J

c
ox o
o .,
,$ggiillgEtgt
q,
o xo
C'
o
C
8B OJ
o u\, l) .c, i\l q D l! o urc D uE \r.o E D c
dU Eg<o'oc,9 c0t E a t,f, o , 6 , , ,O-cvr.j v6ro'n.ccj oo
fo,.,..
ci < ai o uo 6 -Crl(.)!t!n(1Jl\@[1- ? ?F F F - F ;N
(Dt-
0to
d
ol
o
O4
E
0.
q{; cC\ qa

_-) ..-- I
-) -) ----) ,-) ) - )
-.-_J

Downloaded by John Erickson Bangayan (bangayan1903064@ceu.edu.ph)


lOMoARcPSD|19937673

t,

t_
Pedornoruttrcs a,npd,Grtlhno,Cont iqs

6 1- An rldex to@iermine whethe. rhe r6wer iicsor lee(t, are excessivelyw*:te,rEsiip$siagr'incfii=rb'thaf {he
good rdE between the ,acio liDgoal dimension ef (he mandibuln i cborirddcFf,stat dirnrinsion shouldbe
L aPproxirnately 1:1 is
A mixed d,entirion
analysas:
,Nahce's
'4. Peck and Peck's Bol.ton's
02. ln an evsrl/ anxious child [he sedative of choice is:
A sodi0rn penlo(hal chloral hybrate
iB- nitrous oxide o)q/gen dor.mictim
D3. The lollowing branclr;al arch is rhe orig.in cf the rnandibte:
A- ?ro C- 4o.
'B- 3'o D- 1.
b 4- Arnong-rhe follosring rvhich offers the rnost success in rhe treatrnent ot'the tipping during the abe
range ol 5 to 8rt€ar"S
A supr'aversion C- infraversion.
B. simple single matposed teerh
o5- Gross m;ilocGlusion of the anterior reeth caused by hlperactive rnentatis rnusqle,is.
A oral'screen C- Hawley apgiliance
B, musclg exercise : :':i r D- Lip
D 6. The analysis used to cvaluatCthe amounr ot space availabte in rte irch for thesricceettftlg reihh is catleo
:+- anal'ysis:
A Bolron'i
A- Nance's
Dl" The rnanagernent ol deep pit and fiss.ures include t he.: I ol lowin grtre atme nt altematives
L 1- obseruation
Z. pit and.fissures sealanrs
3. preVentive resin restorations
4: arnalg.arn' reslora tion
A- all of chsices C. 2 only
8. 2.3 and a D- 2and3
L D 8- The,capaciry to stimulate boih,the osteobtastic
cellular response:
ancJ osteoclastic activity is brought following imporranrof
A- igrrernleukins C- piezoelectriiry
,B- cytokinins D- prosraglandJas
.l_ f '9- -,UVh,art ris us,ualiV a rellecrion sf the antero-posterior skeleral relatiooship in.the ffitiUonZ
-A. axial=,incliriation C- overiet
ttO. o.vt:r'[liitr: D: ', all,ol these
O l,G- ttrne .U,est rirrne ol the day ro rreat a ihild pa(ien( and to. which,.the ctiild pgiidyefy respqod5
A- llate irr [lre rnlrrrtirrg . D. depepdent upon th'e e'oorper:iliilffirr d{ EAe
B, 'afternooo child
-€1 -
earl/ in the morning
f f f - lie mosa sl,sreprihle sr.,rrace on (hc tirsr.primary muillary 66161 tu devebp eries is [he-
G7L ceoual grmve C- mesial.grove
.6. distar grcove o, buccdl ili
O f2 Retained iniantilir swailwingin chitdhocid may resu[(.in any of rhe follosinq
1,- F.n(erior crossbite
2- tortgue (hrus'ting
3- iabialty inctined maxillary and mandibutar incisors
4-'mandibular prognarhism
A_ 1.2.3.4 C. 1and2
B- I only D_ 1.2.3
O13- ,n the primary dentitiarn an imponan( preven(ive onhodontic measure is:
L A placiag sdace mainr2rinem
B- proper resroration oi the primuy rcta.s especiaily the iilefproxiFEl sprc6
C. placiog space .egainers
D- all ol {he choices
I & 14, In rhe preseoce ot'a load su'Ilicientto rhe b6e. the surrace ol (he bone oo iDe stimulard oqeclas(c activily
I whil€ on lhe concave surlace Gteob,asric arivity is this is knom as the:
A. long bong growth rtreory C. V-principte ol bone grcwth
B- law ol eledrogenesis D- lawof osleboenesis
p I5- tn a serial exiractiofrin a crov+ded a(ch d lhe peJmaent ceDtral ircisoE erupq.the tirsr reerh ri be extrrred
.
$ -,ta are the.
t A- prinrary lateral rncisors
I C- perrnanen( lateral incisors
A- lirst prrrnaqy incisors D. prirnary canines
A 16- Primary teeth rvith gr.oss pulp exposure and degenera.tive pu.[p arrd periapical'lesions:
1- should alvrays be e>ctracted
2- may-.affecr tlle succeedaneous t.eeth causrng hy.poptastic defects.
i
3. rnay cause torrnation of dc-tecnve or irreJg.ular pits on the enamel surface
! 4- maybe lef{ unanended. especiaily if the chitd is unmanageable
A- I.2 and 3 C- 4 oniy
-f
B- I only D- 1 and 2.only
B L7. Prernaturely lost prirnory incisors shoultJ l-re relrtaced by ds lrace rnaintain,er
t
I
I
f-

Downloaded by John Erickson Bangayan (bangayan1903064@ceu.edu.ph)


lOMoARcPSD|19937673

A- as sooo as posslble because h ca,n a change in the arch length


B. maybe left unueated Decause it will not cause a grow change in the arclr of lengrh
C. sometirnes because it iS'unesthetic
D. alleys because it wilt affbct the speech of the chilcl
C 78.Pit and fissure sealants are,indicated in
1- deep. retendve p-rS and Frssures
2- well-coalesced pits and ftssures
3- patiencs with many interprcixirnal caries
4. padents r,.rho have received pteventive treatr.nent such as'systematic and topical
fluoride application
A- 1and3 C. 1and4
B- 2and4 D. 2and3
C 19. The contraindicatlons for the removal of the primaQ/ teeth included
1. presence ol a.ute oral inlection
2- presence of acute systertatic infection
3- children with control diabee.s rnellitus
4. leukernia
A L.2 c- 1-2.3.4
EI,. 1 or,ty D- 1.2.3
b-ZO- The size of the succeedaneoirs teeth',and the spage availabte in the arch caR be:predicted by:
A Tanaka;and Johnson anallrsis C- Bohon analyaG
A. Mixed dentition analysis D- Peck an'd Peck analysis
a 2L- The trearnent of the deep bi.te cases for a 1o-year old chilcl is the lotloiring appliance:
A high-pull headgear C. ceruicat headgear
B- anterior bite plane D- anter.ior bite plane arld cervical headgear
O 22- lf the mandibutar primary cuspids are to be e8racted gecause ol iaries:
-A the space will provide eidugh Space for.the alignment of lhe madibular lateial inciso.s
-B- it an be left una(endediustbsause-it is'in the briterioi segmenl
C- the space shoutd be mainE-iried wiih,a bilatdfal,bed Md lodp
O. rhe sna.--e 5hould bc maioFined wiih a tirigu-al holding arch
A,23. The ,illing mdterial lo( rhe indi[ect capplng in rhe oider o, placement is:
b,
/r- calcium hydroxide. zinc oxide-eugeno!. amdlgam
A. zinc oxide-eugenot, zinc phosphari:. amalg:im
C- zinc oxide-eugenol, calcium hydrc'Xide. llni pi.rosphate. -amalgam
D. zioc oxide-eugenol, calciuin try.droiXirle. Fmargam
( 24- A distaj step in lhe pdrrEor deolition wiil result in:
A. Cl llt rnolar relatipnship C- Cl lt rnolar relationship
3. End-to-end rnolar reli:tionShip D- Cl I rilo[1r relatioii:hip
il 25. fhe failure o{the perio<lontat tighrnerll to rJevulop resutr in:
A prernaiurE resorption C ectopic eruption
B. impaction D- aol<y'osis
's
,' 26- The followiog 3 expl,anadon given fo an occlusel cAviqr prepararioh in a deciduous molar more shajlqv/
than in a permanentiloo(h:
A children,rhavelessa(ention,span
B. .tllere.are lcsi caamel rods ii.rithe p.irua.y.teg-lh '
C- the crowns of.rhe primary reeth arqsmallel b.ut more btilbous
D. rhe rhickness ot the enarEl is')approxioBrebl rhat dr (he pir'manent teeh
I
D 27. The lunctional shiftof the madibule duridg closuie maybb caused:bl4
A- condytar a5lrnmetries C- cuspalprematurities
-z
B. both choices D. ngne-of. rhe choices
1) 28- e,nterior cross bite in ihe primary denlition when letl unatenoed can resuh in permanenr
dentition:
A- no effect C- Cl ll rnalocclusion
A- rnidtine shift D. C! ltl rnalocctusion
S-
C 29- The ideal res(oidtion lcra resial and distal cilies involvement of lhe mandibular piimary molar is:
A. preventive resin-restoration C- Staioless steel crov/n
B- Cl lt-MOD composire nlling D- Cl tl.MOO -- arnalg4m lilling
( 3O- The shcrtening o{ the dental arch tength of the rnaxilla and rnahdible atwayrs occurs during
A- transitional dentition C. permanent dentitlon
B- mixed d.entition D. Primary dentition
0lt During the transition from the primary to the permaneYrt dentition the arch length general\r
A- increases in the maxillaqy arch while it decreaSes in the rnandibular'arch
B- rernain the sarne in the maxillary arch while it decrea.ses in the mandibular arc.h
C- rernains the sarne in the both arches
D- decreases in the both arches
D 32- Retained infantile srValloun-ng may.lead to-
A- proclined maxillary :ocisors C- anterior cpren bite
B. tongue-thrusting D, atl of the chcices
A 33- The diflerence between negatve reinlorcemeot and p\rnishrnenr is:
A frun'shmeot reakens the response ,irorE than the negative ,eintorce.inenr r^.€akens it
B- punisihmeot weakens the respoEe more than':ttle negative reinforcement slrenglhens
C- punishment strengthens the ,espmse by reri@ir€ an averswe outoome
O, punishment weak€ns the respoGe by producing an aversive oulcome
D 34- The tollovriflg is dre stage c.{ tmth dgvelopment when odontoma or a supernrrmerary roolh resulti

Downloaded by John Erickson Bangayan (bangayan1903064@ceu.edu.ph)


lOMoARcPSD|19937673

A- hisrodiff erentiaj G- prclileralion


B. morphodilferenuation D- initiation
A 35- Cheek Dning is correc.ted by-
A- oral screen C- cross elastic
A- bite block D- tongue guard
f 36- The term used lo derribe the compres-d and degenerated periodorital merbrane onhodontrc movemenl
ts:
A. resorption C- hyalinization
B- compression 'D- atro-phy
D37- A space maintainet can be placed in tne-dental arbh under the foltorving condirions:
I toss o[ one or more primary teeth
2. no loss of arch perimeier
3. favorable mixed dentition ana\rsis
4- early eruption ol the first permanent molar which has rnesiatty moved
A L.2.4 C. land2
B- I only D- 1.2.3
D 38- The general characteristics of the dental age tl ls the eruption of
A- mandibular canine C., maxilla"ly 1T prernolar
t '-
B. mandubular I' prernotar ._ ' D. maxillhiy canine'-
4 39- The'rnost restruded and posGrbr positiciri-o'FGL dondites is rhlie:
A- centric relation C- centric occlusion
B- centric position D- alt of'the choices
C 40- The anrerior opery biie euied by mouth bredrhing as a ,esull ofi enlarged adenoids is b€s( managed bi:
A- tongue crib C. relerring to a pediatrician regarding the
B_ habit br:e,aking appliances adenoids
D- an oral screen
C 41- The first teerh to erupi in the deciduous are the:
A. rnaodibular second molars C. mahdibular cen(ral incisors
B. m.rxillary central incisors D. tnandibula/'1- molars
A 42- The following rnar result from <J-rsturb.ance during [he appositional srage of tooth'development.
A- mcttled enamel C. de,ptinogenesis
B. arnelogenesis impertecta D- eharnel hypoplasia
6 qS. The epiphpeal plates ol long bones are exarnple of:
A- depoSirion centers C. grofl(h sites
B. groMh enrers O. grwttlietds
D44. Consricrioas of lhe ,lEillary arch an4 posterior crosi bi(es cairsed by ch'eek biting can-best be maraged Dy:
A bite lock C- .Gross elastics
B. tongue guard D. orai streen
C45- VJhat'spaces are found distal to the prirnary canioe and ire'prominent in the rnaxitlary arch?
A- diasterna C. grow-th
B- pr imare I); develuprnent
0 4G. lrr urrho0ontxs resistance to displacqrnent refers to:
A auxillary C. Ur;ft
B- anchcrage D- 'tractaon
0 47 " A.child wi(h ectoderrnal dyplasia wil['manifCst the fc,llowing:
A enarnel hypoplasia C- oligpndonria
B- anoclontia D- all of the'choices
b aA- what practice constitules preventand interceptive onhodontics?
A- endodontics C- orthodontics
B- general dentistry D- ptosthodootics
C 49,rwhich incorrect procedur:e causes external resorption of the roots of'the teeth:
A straighiened anlg1losed rools C. replanted avutsion
8.. pulporomized teeth D- repositioned fractured ro<.rts
\- C 50- Among the lollovring rhe growrh sited are:
1- sutures
- 2- maxillary ruberosiry
:i; 3- synchondroses
+: a. conoyle
A- land3 C. 1.2.3 and 4
B- 1,2* arrd 3 -O- 1 onty
C 51- A cement used as luting as.arell as a restorative rnaterial is:
A zinc phosphate C- gtass,ionomer
A- polycarborylate D- nofte o( these choices
C 52-Themostretrudedaodunstrainedpositimoathemandiqleinyrhbhtfe.anterosuperiorsudaceolthe
condyJes a(e in conract with rhe cohcavi(ies ol the atutufai disk is kriivr as (he:
A- centric occlusion C- cencric relation
B physiologic resr D- le,rminal lringe posrtton
5:lr llE ueathenl cl arr,awltgd Ftrimary incisc! ilrcrElG the acotlent took place 45 firioutes berore (he palrpn!
ws seen bythi: Oenrist
A wash in running waier then sobk in disiilled waer; $re ieimptam and,sptin(
B- lhere is no need o reimplant th6 lm)th beeuse of ioor pognosrs
c. was the tooth -n running $rater, lhen soak i.i distilldd watef. rernove oie pulp. reimplanr and'splior

Downloaded by John Erickson Bangayan (bangayan1903064@ceu.edu.ph)


lOMoARcPSD|19937673

O, "rcn rhe tcoth in running wate, hen soak in disrilled water then do a pulpectomy berore reimplanling
the lrc(h
D 54 The bng ,ae syndrone is assEiated yrith:
A nail bhing C deptession
B. tongue hrusrir)g D- mouth brearhing
D 55. The lollowing tocirh buds are iniriared aFrer birth:
A, perroneat set pdmary set C-
B- t'mo 2d premotars D-
1". z- and 3d molars
D 56. A[er the p€rmanent canines have erupted. midiine di8tem maybe caused b)t
A mesiodens C- -ribrous labial ,renum
B. a supernumeiary rooth O, all ot these
D sz Ouriog lhe transitim p€.iod trom the'early mixed dendtion to rhe pe(manent dirnensional chilges occur
in rhe orerbite and o\/e(iel These aie:
A. overbite and bveijer'incrbded
B- ovelbire and overjet decreEea,
C- ,overbile decrease while ov€riet remairis.ahe
O- overbite remairis rhe sam while bwriat dacreased
C 58- tn gaining-;Oie c.ooperat'ron o( a denhl gadent wbo is a tareigne-r which ot-rhe totlowirigaagptied?
A orie.el,arido C. teiil. shw arid do i: :' , ,'
8: sedation D. dental educa$Oh , i.' .{1.r :;'
O 59- A denbt'po5leriof, cross bire ot the,lr peihaneot moja, wi(hour maodibular shift c r.b6 by the lollffJing:
A- expansioh ot rhe a.ch with a W-arch C_ occlusal €quilib- ration
appliilce D. cross bte elastics
B- qqacr'helix
D 60. ln children afrecred by rhe lotlqwing. comprdhensive'prwenrive denaat program isrbest
A- Oown's syndrome C- mentalD, ill
_ B- physicarly.handiGpped D- ,p4ralytic
A 6f . whar posirion-o, rhe denilsr is besr suiied tor treating tearfut chilclren?
A sining dom to the pal;enr's tevel C- sfilding
B- krre:lirrg
- 52 Booe OroE by theaddition ot bone tissues on one side D- bendiog
D o!, a t ody conex a-nd taking away the other side.
Thas proces Ls k@rn as:
A- translocalion C- deposition
B- ree(prion D- drift
D 63- The rerm todeidterhe change lrom qeneralized cells or tissuEs to a milre specialiTptJ kin.l
A ma(urat'ion ' C- growth
B. diflcren(iation c)- dcvetopmpnt
4 64 Nalh'' rtnee of dwRlondeet n.urr)bHr 6 (6 stab€) is:
A lhe crNn is carhplered C. Zr3 ol the croiln is compiered
B, the crowh en.irhel is compteted O. 1/:i or.the ctdwn is comOleted
4 65. hn over (etained robt shoutil be e;<iraqed:
A 6 soon 6 it is obser.wed trlat the sucjif;ec,aneous top(h will be malposed when n erupt
g. as.goon 3s lhe sucleedanequs tootlr s(a(!s,to erupt
C- wa( and see un(il:the succe.edarieous tooth-hE eruptd so that the roots ot the primaryy/ill be restrbed
O. shoenelalrule
6 sn. The primaG lpaces in a no.mat primry irentition tre:leatcd in:
l-r(he canine ahd laqer.Iliinciso. in\rhe rhdiltary brch
2. the canine md laeral iicisor;in the mandjbular arch
3. the frst p.inltry jmlar and the caoine in th.e muilary,arch
4- afie ftrst D.imary mlar and the ciline in tlie maf,dibular arch
A. ?.a C- 2 and 3
B- r.4 o. 1 and z
B 67- The best ol dsessmtsnt ot the skele{al-relarionship and grtr^h directions is mde try hndmarks in a:
A, cdirs
qenral C. lateial skufl x-ray
8- cephajomerric ,adiograph D- panoramic xlray
c 68. ln the Federadon Oentaiie.'lntemational rorcnelatu(e which of the tollowing deciduogs set
A. Roman numerals C- Arabic nureals
B- Last fMe leners in rlrc alphaber D- Lst rive letteE of the alphabet
D 69- The downward displacemenr ot rhe mdifla Ls alfected by rhe:
A. lronromaxillary surure C- frontozygomtic suture
B. ndal enilaoe D_ nasoMillary suture
0 70- The toltMifig is the dilterence in size between the primary rceolmd their pemanent buccessors in th€
poster0r segment:
A. late mesial shifr C- posterio( liabiliry
a. posterior size discrepancy O: Nance's leeway spae
A 7l The tolt@ing can be lhe resutts o, un,estored or improperly lilled proximal Brries ot ttEdeciiluous f,lolar
A. boih choices
B- crediog ol rhe posGrior segmen( in {he pe(manenl dedition
C- hich labial cuspids in ihe mdiltary arch
O. rione ol the c}oices
D 72- The fi61 petmanent toodh in the o(al avity which accedes into (he oral cavity behind lhe primry second
molars are lhe:
A- 3- molars B- canines

Downloaded by John Erickson Bangayan (bangayan1903064@ceu.edu.ph)


lOMoARcPSD|19937673

C. 2- p,ermanenl rnolars D- 1o perrnanent rnola{s


D 73- The ,ol@ing is used to anat)zed rhe proponionalit).ot rhe tobrh sizes in rhe mdila,y and mandibula,
I arches:
I A mixed dentition anatysis C- Holdway rati)
L. 8- Psk and Peck.s anal)6is D.. golton.s ilalysis
D 74.'A4-yearokf d)ildvistedthebenEstrorlhetirnrimeandre(iivedprojtry'uGquhewefl.Betoreheletr.rhe
Cendsr was so glad lhar he gar/e lhe chitd a tqr, This khd ot liesture is caflbd:
A pubtic relatioo . Q- bribery

8. classical coo(titioning D- .,posirive r-einrorcement


D 75. The space maintainer of cloice lq a bilireral.and muttipte ross of ;toras #rne muittary arch is:
A- lingual holding arch C. band ancl loop
A- rernoyable space rnainhiner D- Nance's holding. arch
76. The lollowing increase; the rnaxillary arch perlrneter
A- increase in lenglh and depth cit rhg ma(ill:i4y base
B- adiustrnent o[ the ihaxitlary in reliation wirh the cranial base
C- angul.ation of the m-iiitlary perrnarient incisors and increase in'the;gvid(h o, rhe rnaxitla
D. all of rhe choices
, 7?- One hisrObgic tearure o! pdmry;fee$ VCrSfos perrlt*.it Eerh b.Jhai gfeaq( at bcts cavi$pprerH.atioo,6 the:
A dehdnal tubrrles are lacs regtdar ,C. bahds ol hecius are.less in number
B. eonirel in rhe ginEivalrhirdttatrun : t'D. dendn is tess dense
incisaJly o( occtusatly
n 78- lncip'renl interproximal ca\rities are bist by the tolo$ririg .oentEenograph'E examination:
A. bite-wing C- panoramic
B- # o pedo fitm .D. periqpicat
D 79- The rnanagernent of a large disrance 'between the-maxillary inciiqrs at age 12 is:
A- rule out tibrous labial lrenum "er,uaiUeking,and x-ray to find out the presence of
B. recomrnend closure With,fixed appliarrces -,C. a mesiodens
D. all of the choices
is defrned as the norinal chang€s in the arnount ol living substance and is rneasure din
unit o[ increased per unit of time:
A- developrnerrl C., growth
B- dil'f erentiarion D- maturation
E 81- The mand molar usually @rnes rheFially ripped-iod very likety shes a lihgual ripping
when rhe motars ae tostr earty. This is tlG,esiJtradf:
A, Stron kS i'/c. abslnce.ot space.maintainef
i B. absence of fie priim.Jy se(ild drciduous
molar
*b: a.bsencd oJ (he,tinguat roo(
I 5 82. The.material u*d fot indirrct pulp l(barmenr s,irich is piaced diiPdl}/ wer rhe deep caviry some c*bus
I
denrin is left is:
A- cavity varnish C- forrnocresol ziftc-oxide eugenol rnixture
B. calciurn hydroxide D- iinc oxide eugenol
C 83. Children look at rhe foilgrvling as too.rh Sralues:
A. painred rnodels C. plaster models
B- study casls D. 'dental sta(ues
e 84- The bone response when
A- tension C- forination
B- resorp(ion D-, all'ofrthe choices
B 85- Functional rnaiocctusion and duhl bites are diagnosed by.the follo_-wing positionrcf rhe
A- intercuspat registrarion C. prorrusMe
B. retrudded conract D- retrussive
D 86- Th'e' 1" perrnanent-rnolar achieves a Cl t relarionshjp by rneans of the following excepr:
A the Nance leevray space
B. a late mesial shifr
C. a greater forward growth o[ the rnandible rhan rhe ma:<illa
D. an ear\r rnesial sh'ift
c 87- T-he Rcng lace syndrome is alrnost atrnrays associared with:
A nail biting C- moutlr breathinq
B- torrgue thrusting D- depress'ron
I D 84. The diagnosric root to confirm rhe sketebl reta(ionship ol rhe rouiUa to lhe ,ne^Cibutar analysis ot rls
I
I landmarks is:
A- penaf ical radiograph C- panoramicradiograph
a- dental ca<ts D- cephalomeitric radiograph
C 89- An adiunclive diagoostic tool for trealmcnt paahnidg in onhqdonrics ihat g.ives into(ma(lon possrble gro!^ah
)
I panero and 6n givc Jccuratc borie age piuturu uI the patlent is
t
i, A cephalornetric C- hand-rvrist radiograph
A- tornograrn D- panoramic radiograph
I 90. The equilibrium cf physiologlc orihodonric force is:
A- cornbrnarion of aneraai aoct venous btood C- normal venous pressure
-Pressure t . norTnal aner;al pressur-e
I B- normal capitlaqy blood p.ressure
\ /rt 91- The sequence and (ime of eruption of the peimanen( ree(h are taige\r dererrnined by=
A- genes C- nutrition
B- habis D- race

Downloaded by John Erickson Bangayan (bangayan1903064@ceu.edu.ph)


lOMoARcPSD|19937673

A- sp?,c:B r"egain,ing c- Spa-C? maintenance


.8. s.pac,e .c.kcsrure . O- space.supervision
0 g: Tlr'e sournces of. f.o,r,ce that rnaintaio r,he secoad rnrandi.bular mola.r in place includes:
1 rnesial force lirom rhe 1r perrhanent r.nolar
2 lq p.rrirnacf rnotar exerting-an opFrosite- distal lorce
\ 3- lqngu€ and cheeks
4. alveola4- p,rocess and peri'odon(a, tisstres
I
A- ;1 and 3 C. r onl'y
e. -1.3 and 4 D. 1.2,3 and 4
D gq. The.toltowirlg are ci,aracrerizedby-rhe prirnaqy reerh
EXCEpT:
A trle cro,wns of;the primar! taeth are srnaller.but rnore belt shaped cervicaHy
lhe, primary'rbtffi are wideithelr permadena sirccssors
?
c. the pridary iriolaE ile wid.e. thm their f€rrrraoeni succes$.s and rhe cr@lns9l rhe teeh are smaler
birt mo(.e be[ shaped cervbdlly
D- nooe'of the choies-
The lonoibing.are abriqmal mrrscte and-presre- habitr Excepi:
A fmger-srid*,ing'
B- l1@nroifry-of Oe rnanOibl,e.
/1. 96 S,kelerd qlffi,iplrrs are predici.o.by ine fottowing developrritint
1 in,crehse.in.height
2- rr,rendrctre
3- increase in weight
4- ossification o[ the adductor'sesarnoid
A 1'.2 and 4 C- landZ
-fl.: -1.3.4
D- 1.2'ancl 3
At age'ts'a linguallyierupting maxiuary lateral,incisor can be intercq,pt ed by;
1. using a torigue ptane
Z. using an inclined plane
3- instructinE,the ctrikJ'-in used'the thumb'to pushed the,lalera[,incisor
4 tfEiusc of a Ha'rley appliance urirh an activated-tinger strinEal the linguqt sudace of. the ta(eia! inciso,
,A- afl dt the choidds c- 2,3:and a
B. 1.2 and 3 D. l.z ad a
9B- ciss ecirring h tire buc:il'6-nd linguat areas ol t}le dental .idges itrinlans consid€red;{o bermnans or
I '/FirxurcoLfs:
g.tand tilssue are called:
,4. Aohn.s'rbdt,les
I\ q- Mr.rcoos. relgntiord g$ct D-
'99. ,GemaLr prertatd-ractorsrf,ut can coniribure to gross.congeniral d
.n\ r.rrr,ed-rcario,n.'ta.king. dirring pre$nanq1
a- Gerirruanrn&aslgs :

I '1OO- ln rr)ost,rgmqrilble aopliances the qrpe of clasp.used is the


L A- .ball'
B- arrowhead
G. Aciam
D- . circumterential

L l:
'9'
Or.dro- Peda.
2l- c
?2- b .d
Set 4
al
42- q
c 6t_
62_ d
a 81.',b
82. b
3.d 2f- a a3- b 63_ b 83. c.
4-b 2a- c 44 d- 6a- a u.b
5-d 25,. d 45b 65. a Bs- b
46- b 86- d.
L 5.d
7_ d
8.d
26. d
27. b
2s-ba
a7- d
4,8. b.
56- b
67. b
6g- c
87. c
88- d.
9.c 29- s a9. c ' 59-b 89- c
lo d. 30- c 50. c 70. d .90 b.
li 11. c 3r. -d 5lc vl. a 9fa
i2. d 32_ d 52- c' 72. d 97 c.
rf. d 33- a s3. b 73. d 93d
tab 3a. d 5a_ d 74.d- 94 d.
15d 35- a 55. a.cl 75. b 95- ,h.r -
t
)
I

I
- L6a
t7h
36-
17
c
rI
56_ d
57 i,
7.S_ c
7] bt
96 bo
97 ,a td
l8- a C 38. d 58- c 74. a 9&r
1.9. c l9- a 59. d 79. d sl,- (
z$. b nc_ c 50.ab BO_ c NGO- c
I
!
\

I
II
l-

Downloaded by John Erickson Bangayan (bangayan1903064@ceu.edu.ph)


lOMoARcPSD|19937673

Pedodontics And Orthodontics


Set 5

Dr rn rryiewine . r"""."-'. ,"o-,"nJ;;;;;;;;il;;;;;;;;;-n,* .ro*,^n *u,


nor be hetptut in estabtishing the denat age ol.lhe individiral "i,*
a rhe degree ot root Oevetrcpment of{he premIanent Eeth
b. rhe seguence ol erupu-on of tha dermanent teeth
c. rhe gingival teeth
\ d- (he numbe(
I n 2. whal is rhe possibre rmilei{ muillary reft cenua, incisor. the order
di$repancjes noted.fie t
a prohged,fetentioo ol Lhe primilq,. maai[ary lelt centat incGor
b. lacio - rersioo upon e;uprion ol lh.e p€rmrientmandibulal'.leh cenual inciso,
. c- sbr enrral wiih spee tos
d, gtn ce?ral incisor
(3. Mandibu at
a child be reated using a
a.
b- unitaieralJingjualiarih elpaniler
c. unilateral cross elasriG
d- ,biLateral lingual arch exp&atatr
\ D4_ Normal occlusion ah 6e achieve44irolftled'inolarregulation is present. The eruption sequence in
'b the maxillary ach is:
a- cantne. second.premolar. fi6( Dremolar
b- .canine. list preniolar. second p?enEtar
c. second premofar. enine, tist premolar
d. liEr premoiar. Seqmd premlar. mine
\- 05. which drug is nd( indicated ,or-premedic:iling'a seven year old with severe appiehereibn?
a. alphaprodirre c- hydroxyrine
b- meperidine d- naloxone
A removabte space mqinraine( is noLadvisable for children bbcaUse.
a- difficrrlt ro clean d- will riot rnaintain vertical
b. o{ no esthetic value dimension
L? they rnay not u5e it
Why does a primary rnolar rernain.uneruDred o1-supmergiecl-?
a- rhe;roorh is anliylosets.to its bone crypt
b- roo( resorp,trol is not evident 3";.-

c- rhic\.conicat bone
d. rhere is no.perrnaridgit suc'cesSor i
Myofunction;il theraphy, an adjunet lo rnechanbrheraphy was introduced by
a- Er- Allan Brodie c- Andrevrs
b. De,a41 and Anderspns d_ Df. htfred Rogers
L9 What is rhe b,est'rime to tre.at malocclusion?
o. berweenB-fZyears c- at arD/ age.dependigg upon rhe
L- D- becween 12 - 14 years problern invol.ve.d
d- b,erween 11- 12 years
c 10. The greatest pe?iod of-craniat growrh occurs beNveen'
a- 1O and 12 yrs- of age c- birth and 5,yea6:of age
b. 7 days d. 6 and B years-bf bge
A tt. A 2-year ofd chiict fell face ciorvn traumbtrT,ioE the upper [e[t cenrral incisor intruding the_said tooth
apervyise- What.imrnediate treatment. will you do?
a. carelul removal o( the incisor
b- make the patienl'cornfoitable.without disturbing rhe roorh
c. apposirioning o(-inrrgrded teerh
d- reposirioning ahd splinting of-intruded roorh
nt2
I

w[31: are the rypicat taciat profile changes that occur during the rransition lrorh childhood to
- adulrhood?
a- flanering.of the protile r;ry'rcal.occurs,with rnatura(ion
b- as (he l3ce enlarges.'there ls a general acce.ntuation of lips relative (o nose and chin
sttucture-
c rhe face e.longates narrow and, thins with time
d. rhe p;ofile remains esser,rtitllyconstant
DL3 Radioqraph reve;il absence of pulpchamber of the second rnotar. This is parhognornonic of-
a. ileidocraniat dysostosi c- hutchinoson's dentition
b. amelogenesis irnper{ecta d- denrinogenesis irnperfecta
L r'{' A 10 year old child met at ac€ideot wirh lost of milillary pe(maen( incisr leeol rhar are in normal
occlus;on- As a denlifl what will be y@r rreamed of cho;ce?
a' delay the lrea(menl lot a;tix b.idge appliarce unril rtie putp Ch:mber ot the laterat incjeG woutd
have become smalle,
b' a space should be meNured and obse(yed e.rery3 months il the space rs sBning ro ciose
c- constructs a spce ma,inainer d soon as possitile
(r. E soon as rhesolr dssue hears- atix b.bge should be done
C rs. The primary tactor thd.t inltuence $rovrth ard deVelopment are:

Downloaded by John Erickson Bangayan (bangayan1903064@ceu.edu.ph)


lOMoARcPSD|19937673

a-,errrdocrineg;lands c. endocrine gland.s and heredity


b. acquired during childhood d- heredity
I 9 l.s. lWhy d'o :boys and girls d.iffer in behavircr in the d,erttat offiCe?
t
a_ matuflry c-.rnaturiry and mental health
b- rrrental teeth d.. learning
4 L7. The final enlargement of bone occurs by.
a.- 'drift and displacemeot c- dr ift=displace.inent+resorpti,on
b- d,isp.lacement cr- drift
e 18- What lreatment can be done for geographic toftg.ue no(ed.in late childnobd?
a- excision of the discrete lesion
.b- no treatraent is iod.icated
c- the use of nystatin rubbed against-the surface
I -d- anribiotic.Iorelieveinlectio.n
A Is. What as the besr approac-h to manag{ng a healrtDr six year old who hd io betorciblv Serparared,fi'm
his rhothqr and caried to the operatory?
a use voice control
b. .prerediete
c- dismiss rhe ctrild
and appainr when cciopera$on an be bipeteii
d-
use nitrou.s oxide and anal(jesia r -l:j
t_-
::
:

L
::r ''
0 zo. The r.rse of headgeair should be prescr.ibed in- 'i" ..j l
' '_ - Lj-

a,; all of these


b- Slight'restrainllrom forward rnovement of maxillary teeifr
c- po.sreric.r rnovement of mandibr.rlar reeth
d- lacial asymrnetry
8, ?1. A 4 Vl year old child has falten within.the past 2 hours and trau.mati.e'd both ma,xill4ty- central
L incisor'to ttre ppinl that they are-extremety- ri"lo'U;1..' The treatrhent'of chbice taould be to:
a_ immobilize thern with a splint and do nothing-rnore
b_ check radiographicalty lor r.cot f ractures. arid-if none pro.ceed wi(h,a,Sptiot and pe,riodic ':'
exarnination
c- do nothinq and ob5erve penodic'aity
d- exract them because they worrld be lost soon +ny.r ray
L A 22. lVhat shotrld the irnmediate treatrnent of a toorh that ha a susralned a fiacture-to lhe middle third. of
{t[-ue,roo[ .[n,Clu O e:
a- S.plinting
rb- Pblpeclorny to both,.ponions df'rhe tooth
e.- to the coronal ponion and apicoectorny_ of the root'pon;on

& 23_ !d
frn .patien6'3ketetat'g'rowth par'tern. the nrdst important lactor is:
&-, genedc heredigr c- diet
b" dental occlusion 'd- habils
D 2a. w/c of,these rnay reqpire a rernovahle acrylic-rype spice reatntarner
a. bilateral loss.of the rnandibular deciiluous l-irstrrnolars
b- :premature loss ot.right deciduous secoqd rnolar,rvirh mesial.drift of .the Jirst p'q,rrrranernt n'rl,dlar
a.bout 1 mm
c- '-premature ko6s of neandibqlar right'deciduotrs-fir.st and.Second.rnolars
d- 'bila'terat loss of manCibu[ar decidrors lirst a4d +."rlO,molar
D 25. The development of the pgsti'rio,r occtusion is depenJ.* upt":
rl
lj a- rnaintenance'of a f'avoratjle'sequence of eruptron
'b. anainrnent of a norma! mO'lar,relationship,wirh rninimal'decre'dse of Space avaitable lor the
i cuspids and mola{s
c- satistadory looth size space available ralio
d- all ot ttEse
P 26. Nance:s leeway space rerers (o:
a- the:roNard growth ol muilia and. mandible
b- the wrdth ot he primate space + median criasriina
c- a left over space ,or termihat plane adiustment v)h.ich equals O,9 mm on edch rendibular
quadranl
d- the dirlerence beaween cad+e and 3+415
C 21. la space closure aher ixemature exfraction tlE-t@th showiog the grehtesf closure is:
a- mandibulBr second decidudus molar moiar
b- upper larst'decidmus mla.
c- maxillary. second deciduous
I d- maodibular first deciduoG motar
I I 28. The conepi ol mandibular growth is observed in:
l_ a- easal se.plum c- median rapha€
b- alreolar.bm€ d, mniribulirr syrhFtq6b
D 29- Cal,cification ol{eeih occurring at bi(h ue:
t
a primary anre.io( teeth upper and lorei. caine arid tist primarv rnolas
l b- .only rhe primary teerh
L c- all-primary teeth.anii {irst fermarem motai
d. all{he Brinrary teelh and Oe(manent

i-
I
L

Downloaded by John Erickson Bangayan (bangayan1903064@ceu.edu.ph)


lOMoARcPSD|19937673

8 3A. Whuern t',ile ontrlodontic force agifiied-is.,lig.h(-resorption will-probab! be-:


a- inClrect c- apacal
b, dlrec.t .d.'underrnining
I 'nr.
thre pressure siCe of the'aMeolar crest when orthodontic forces
\-
I D 31- Forc'es
exceed phys iologic -lirnits causes:
a- bone:assault c- direct resorption
b- osteoblasticactivity d- undermining resorprion
A 32- Which of these orthodontic procedures..b most easity retained-
a- anterior cr'ossbite c - gen-e r alized,spacing

L b- diastema d. expansion
D 33. Diastema bewveen perman.ent maxillaqf centrat incisor of a nine.year old chitd
with ontf 23 of the crou.rn.seen clinically is us'tralty the result of:
a- abnormal labial lrenum
i b" a norrnal eruption pattern.of these teeth.
c- a fiick rnidline septa,of macilljry'b6ne
i_ d- failure ol premaxitiae to tuse
b Za- with the premarure.rn<ilar. u*rat other te€.th can be'Osed to evaluaqe occlusion (cla-ss l. r)
a-
rnolar reldtion c- inc'rsor relarion

t b-
canine relaulon
& 35. Malocclusion is cornmonty caus'e by:
a- hypothyroidisrn
b- ear,V lost of primary teerh
c- panial anodontia
d,- midline relation

d- delay.ed eruptiort of succeedaneous teeth-du.e to growt[r of alveolar process


A 36. Purulent le.sion in'the [acial vestibule of a 1O year old boy is rnoSt like]y
L a- r odontogenic'(isnrla
b- -pyogenic granuloma
c- an apthous Ulqe
d- herpetic lesion
D 37 " .Head.gear should not be prescribed in whichiol theJf- siluation
a-,posterior rnovelnent'of rnand.ibtilar.teeth is necessary
b. 'the ma.ndibular-plane angle is triqh'
c. f acial asyrnrnetry is severe
d. th,ere is no chance tha!'the headgear will 'be worn
D 38. loarnixeddentition,thelastpriniarytoothio_-bereirlaced'bythepermaoenttoothisthe:
a- .s'randibular 1- molar

c.- npxil,lary snQl molar-(decidu6usli


d- de,ciduous canine
39. i[.he -\f principle of grovrt]rlls best illusrriited by the:
a.
:n-n,andibular symphysis c- nasal seprLrm
lrnedian raphe
b. d. Sphenb,occipital syrucih,ondnosis
C 40. Which dI tE ff- should be done in the first-visit of a.chikl ro the denrai office
,Premedication'
a-
b..
Arralgesic
c-
Nor-mal procedgre-oral examination
d-
l-irrn approach.
A 41- 'Loss of sJrace i': mn;t rapid-tcillowing a prcriru(ure extraotiori o[ wtriulriplurraly-reerh
a- mocillary second rnolar c- maxillary first mcilar
L b- mandibular tirst rno"lar d- mandibular second qolar
,{ ,42- The-use of.vrhich ol.rhe ff- acfjunctive 4ids is riecesary to iheck on rhe elficacy ol an eral proph)4dis for
a 1rcung patient?
a- disclosihg solution c- snyder test
b- unwa*€d lloss d. sandpaper srip
C 43. lf there wse no:rollow urt'o, orthodohtic treairnent. serial exlxtitin rnay resqlt rn:
L a- devdlopment cil c?os bite
b- increase overbite
c- canine tipping distarv whib second pr€molars ripping. mesralty
d. increse owrjei
4 44. When doEs a,bimuifiary piosmjsioft.oEur?
a- when both dental dches are toMard with respect ol basal bong
b- a large horiz'ontal o\rer{aii with Dildieraf cross bite.exist
c- interfereni:c in growot and devetoprnent resuttiag trom itlcess
d- the maxillary teeth is forward with ,espcct to baeal bone
I
I I 45 The dental age reters to the:
I ;i, period of .Fotcnrial Gruption
t_ b, stage oa dental maturalion
L- ldpse ln rhe humbef Of'years gven lor a @oth to erupt
d- age of toorh erup(iod
) B 46- Whai is the choice oI appliacne whqrs the.missing te€rh aie Ue primary second rfuai. ,iqt[ ana Neri
I mesial migration ot rhe mand: 1'trDlar rklht and left measqiring 2-3lm: overbire 2 mm?
L a- headgear wilh cervical strap

i
L

Downloaded by John Erickson Bangayan (bangayan1903064@ceu.edu.ph)


lOMoARcPSD|19937673

b- spjir saddle acrylic space rnainrainer biiareral sand bops


c_ lingual holcring arch
, 47. y.i/hich ,etainer [s indicated on a 5 yea, old chill wtren extaction ol the primry,milaribularteft s*ood
rDlar is necessary
a-
Iunctional remryable acrylic c- lingua, aich
b-
crown-with distal boire d- baj}d-
and bop
0.48- Whal caries incidence is lound in patienb wirh Down syhdrome?
a- higher thiljin norma, childien
b- none o[ lhe aborve ate cFjfied nor correladon has be-en rt]und between caries incidence the
d@n +ndrome
c, same 6 in normat children
d- tou€r tha in norman children
C 49, 'f hc m6t common nuritional problem in children is:
a intake o, too much ftuid
b. inabiliryio chew hard tood
c- malnuBirion due to poof intake ol mitk, truit ild vegetable
d-. poor prbteih in6ke
C 50. Growrh of I'ierhead of Oie condyle stops at age:
€r- 12-15 yrs c-'25;yrs
b- 2.0.23 yrs d- 1Od5 yrs
C 51. Uprightihg.fie n:iolar js,difficult if not slorv in progress because
a. overcontoured spring c- occlusion is',not relieved
b. there is lack of anchorage control d! overextenbed bands
A 52 ln=rease trie anterq:posterior dirnension of the rnandibte surgicalbf,
a. ir is for class ll occlusion the treatment stated betow
b- it is indicated for,-chin-up-
c- it,is tor class, I malocclusion the rreatment plhn state below
d- it is the uearceenG plan foi class lrt rnalocclusion
D 53- Two rnorphological q4pes of primary denral arches are:
a- closed or tight c. spaced onty
b. open nor spaged d: smalt oi targe tee(h
n 5a Endochondral bone differs.frorn intramernbranousrbcine ih ttrac '
a- the former gro!t6;-rFV
9ot, inters$tial and.oppOritionat gqowth; theJaner by oppositio-n onty
b-
b. rhe lormer resor6.g with'diffict
the with.diffict lV under pressure: the fih"r *irr,
the,laiter wirh oifficutcy
difficulw
c. the'lormer r-esorbs'ea-si$z'by prd\sure3 rhe tat(ef with diffigulty .

d- rhe forrner gro\ rs byr appgsitioi onl14 ih. 1.o", Uy interstliat ind apposirional growlh
4 55. \^/hen does bimaxillary protrusio_o sccui?
a when rrExillary and mandibutil dental€rchB a're tort4Erd with r€spect to bsal booe
b. mdiltary rAerh .ire foNd.id lrom rhe babat bone
c the presence o, trilateral cross bi(cs
d. hn.zontal:werlcp is too,lirge
56- when is{he erupdon period ot mandibular hst molar if norrhat @velopmehr should @orinues
a- at the age ot 6 c 6-7 )€:ir bld
b 5-6 and 6"/ year old d--S-6 year old
57- whal is rhe $fi lissue piofile-(line frcm torehead-lip-chin) of..a childwiah a matocluslon caused by
persistent thumb suckino?
a, rouhd c- cronvex
b- srraagh( d. concave
58- lt )ou weralo prepare a mandibular tingual il,ch ai:plimce to maintain space, lhe design shot td contacr:
' a. linguar:6pect below lhe cingula.crf the mos-t anterior teeth
b- incisatJrhirds on the lingat surfaie of the anterio, tee(h
c 2 cm betow rhe cinqula ot louer ante i6r re-eth aLrhe gingivat .tissue
d. lhe middlb lhirds ol the lorer anterior teeth
859. whatrYpe,'ofanchorageisptescribed(ocorrectbilateralcrossaueilyourere.tbuseanacrivelingual
arch
a.reciprocal c- sirriple
b.
contralateral d- coneplex
0 6c f rank fort-hprizontat is a ref erence ptane construc{.ed by pining the [f- landmarks
a porion and sella c- nasion and sella
b- po.ion afld orbita;e d- porioo and nasion
, 6l what e/ill be your procedure of chdice on a child patiect srispeded to hdve leukemiawi(h'badty ialecred
primary roorh?
a- p(wide pafliative treatmenl
h- re,e( rhe patierit ro a hermalologis(-belore prescribing tra:atment
c- adminisrer antibiotic and refer.iratienr to a hermatotogbt
d- request ,or complete btood rount (CAC) !o detmine hemoglotlin
O 62 Pzin on thc oral mucosa wilh aprhous lesions in the giogival margin o{ rhe palare. eeMGt.
Vmphadenr:,s
Paiienr is 6 years otd and ,eb(ale- wha( ,s yos diagnosis?
a. Koplik's spots
.b- Necro(izingulceralivegiogivftis
c. Eq4hemamuttit'ornre

Downloaded by John Erickson Bangayan (bangayan1903064@ceu.edu.ph)


lOMoARcPSD|19937673

L
d. F+erpetic gingivo_stornariris
I er. !n rwhi'ch con6lr'or'r is there dryness of skin.-briute
na'ils and congrenliaity niissing primary
l teeth ? an-ci p.errrr.dr:]e!-D(
It-; a_ cf,erindocranial dysostcgis c- pierre-robin syndrome
lb. ectoderrnal dysplasra d- Erynhema multiforrrre
ACo.64. Which o/ tire if_ sta,ternent islrue:
a. Surface thd't.face the direerion of grovrafi.arg afeas-Ot
deposition
: |1"j,-1^::t:T::fle.g =kdL;;i6;--;;;-ri-matro,,
i- 5 IH:":='ffi ,b,:,5 j:HY l*{:_""Eim=t*,t.",++""'i*"*, runcriqr arrd
i;;;;iri1rp,r., snorv.lr
c 6s' ro reduce,::y_r^,li o".i"s ;";;;#il;ffi['#1ffi."fii:Hrffi:ffi
*,,-1".1=:..:*111,:::1y"I,1'"-hebirecrr*;Gil;;f
usdd?
a- Pilocarpine c- atropine
b- MproQarnbre d. g.en-rian viotet
I 66- Which of these is oiagnostic ot ankykrsis
of primary ,i"r,ars?
a- changre in color of enarnel
i._ b- increa<e density of ramina dura radiographicarty
c- dull sourfd oo percussion
d. incomplete erupti<.rn
0'67. A'generajist can-ezrsily anil safely correct .,.
this o( space probiemst
, ,. probteins fypec- space ,[e"*i"i"n
L :t,- space
3:1t^t^1=^.tr"pancy
rnarntenance O_ spaCe regaii*ng
o 68. A primary left incisor in a S.'year old--patieot is auscasiea 'tt."rn b-e trealed .

a- Putporomy by
pulpectorny
:-- - -----d.
c-
b' Extraction
n 69- Devitarize with formocresol
ages of1 tooth development?
L rnorpftodiftere n ti.ition. m i n e raliz
iff e reri'riarion. m in€rifz ation
a tio n

mor phoUif f eren riirion, min era{iza t-ron


c7f) ifferenriilinr r. 76fno rahlJtror I
anagernent procedure include:
L C

t
,a v!. e
acterized by rafrich of tftertf- cephalo;netric

i 4 para4l€t ,elarionshrp of .dnreriq crmiil-basei, patatal. cielusion


b- chss I skeElat aod d;nrat oattem
c- shbrtbned idi:e height
and mandrbuta plane

d- palala, irtafiF rippgd uErartr h rhe dnrerior in coniuncrion u/irh


o 7z. Faaar pbne ij eitatihstreir.uv a steep.mandibutar dane
Jr*ilriiii"1;;,I1",
A lg
Poonninn q
L 3-
h
PL
men(on ro nn0.l,irur
, /t3- fhumbsueking Oil S yelrotO: n*,ra to pogonion air=-
pr. A ro,nd|on

a- e?hbryodicdystunclioo
h it is a lecal caGe of majcct6ion

t c- ia b consadered borh lo|cat ardnsternic io nature


d- it is a systemic Guse orma
fl t4. A tighi{nhodonric ,orc€ apptied
a- direcr
b_ iadirecl
on a rion vrifl be:
c. undermining
,) 7s- pMA,is .,=yiroo, ol . d- apical
1n)
a- caries increx d- pe.iodonral index:periodonta!
L b- materia, a,ba index
c. pulp capprne;mareria, targloal atbch gingivar
O 76: Excision ol the fienuir.when mrdtne (ria$ema
cur be{ween two mdillary incisq is.indicated:
a- betore onhodonric rreatment c- whlnlhe cnito is 1zyeac oU
" ffl}X.fft
incisors L,esin ro d. wr*rn a[ perr,=^a;ii,;;-;;,irily
l * 77 . ta
"nr,oo"iilo"--rir.ii oa rhe ff. tandrork is use !o evatrrare .,unror"Hffi
a- ,a,rrterio, crantal.base c- craDial vault
b. nasornariltary cornplex d. n8al flq)r
I O 78. The pe.manent irrcisors qrcukt normaly be crowded
wiere at no{,o( ahe lad rhat:
I a, $rere is an earty riSt pef manenunola. adiustment
t b- Ihe bwet prrmar! annes themselves into the pf,imale space upon the eruprion ol the
- permanent taterat incisor 'eloated
c. theyrerupt into a more aflerior pGilion rhan their predsesso.s
d. a[ oGttre chorces
) A 79- An orat ioiectr'on cause b),. a bad;y b,oken rmrh. prrp, pofyp.
a. rnseae ' herrnrology will r:oveat
it ,cuirophirs
L t. insese monocyles c- pred6minance or ;sinopt
d- |r.rer=" lymphocytes
irl-'

Downloaded by John Erickson Bangayan (bangayan1903064@ceu.edu.ph)


lOMoARcPSD|19937673

D 80- What.feAtinbni?rocedrrewill you sugqest on a€hild.rirh dcep sealed caries on a


\ EE{l]"ffertt moltr which-is Vita! and negdive-to percusston
)
I
-a- Puh€ctJmy ' . c- Fdpodomy
b- inclrect piilp capfinq d- direca pulp capping
C 41. lf one single.ractor could be desigoaied is.conr-rt uting mbr ro d.\€lopment ot sirpne
gingivitis in children, it would,rbe:
c- poor orat hygiene
b- high fever d. ph)Eicar.eaiure ol.food indigesiim
O 8.2- ln a parient,v_rith a su;pec^redbase'd leukerhia and a badly inreeted primhq,
tOolh. the prc€dpre. o, choice rculd.be to: : ,i.'.
a-- obtain a blood count end 6dmit tne.Lfr;tO ro a hospiral tor extracdgn
b.
c.
d- acrion
I

O 83- ln Oe lre 'rh€


complicarion &at will uisa is the:
a- rnesiolinguar,movement of Zd b- oraereBrrededinolaf
c- extnrSion of the buFca.l segfflen's
d- llare ol lorper
Ig="*
L O 94- When is a primary tooth. recornrne..nde.d'lOr .eXtractioa?:= ir,==.
&
loss of antagonist
b-
nonvital tooth
c- exposure of pulp.,
d- c&ri€s. involving ; furcation
-:-

&gS- EctoQerrnal dysplasia is a.lesion ia.rrolviag suugtures derrEloping{rom,,the ectoderm- tr can


ocEur in all ages and js rnanifesteO fyrar.roOotia- tt.is caused by:
a- a horrnonat defidenq/ c- deficienry.of..vitamin A and D
L C 86. Conon
b-
pe
d-'.caused.by rnalnutritign
onochtolophenbl is apptied, on rhe:
a-
b. hyperemic. pulp
c. prrlp chamber with.p{p:'potyp -
d.
L c 1ts7-
accidentally exposed,piilp-st0mp
Which srar.ernent is appropriate'for natal rbeifr
a. usualiiy norrnal pfimay'incisors c- qsuailyisupernumerary te.-eth
lb- exi'raction'is indicated .d- eruption is sooo afte.r binn
:' :
A 8E-,4 suiper,murne{acf tooth resu-tts-from,a.deviatio-nidpfing: ,,j,
I
I a- dii,f:lerrentiation .-"' ... c- catcif-rcation
I
b. appo'sirion d-'inidation ,..,i
p lG9. -4. spa.cre- r.ryaintenanc.p is no(-.indicatedYor premature-tossr.of a: :-:

a- finan
rP'e f
b. 'p,rirnary .rhanpibulai.central incisoi
c.- 'primary
d- primaql maxill3ry cenfal ingrp:or
* 90. LJprighting.o,f rooth ls diffictrlt.if not s,lory in prgg-fess.because:

b- overextendd<fbands.' d-:'there iS.a l-aek oi 'dncfrora'ge


C 91- ldentify the factcr resp.onsibte lbr alv.eolar. pr.ocess f&mation:
a- leogthening of condlb
b- nutrition
c- tooth'eruption
d. grow(h and develbpmen( ot'mandible and rnaxilla
E 92. Peg-shaped later:als
a- .it is anornalies in nun'tbgr c- it is an osseous dysplasia
b- it ls anomalies in size ancl sliape.. d- it C dgndl arpomiry
L D93. After approxirnately six years gf ige. the grlatesr increasb in siz€ 6f tne
mandible occur
a- Der.ween the canines c- at the symphysis
L. along the lor,.rer border, ti- distal to dle first molar
I
L a- normal lip posture
b- abnorrnal lip'posture
c- normal overbite and overj.et
d- prognathic raand'ible
ft 95. When does rnesial drift occu?
a- all of rhese c- prernature loss o{ adjacent teerh
I
I
b- rnterproximal attri(ion - interproximal caries
-d
I
t
C- 96- To deterrnine t-he presence of plaque.in children. vrrhat adjuncrrve aid.rnay be
t used:
a- snyder test c. .disclosing solutiort
b- rubbar cup d- super gloss
t
q / lnrlrrect pulp cclping ,proccdurcs on p,rirnary, molors are indicirted whe rr:
I
I
a- the carious lesion is suspeqed.prodlrcing.an exposure oi the.pr,llp
I tt. removal of decay h?" expgsed dre pulp
c. the carious lesion has just penetrated the dentinoenanrel iu.nction

t-

Downloaded by John Erickson Bangayan (bangayan1903064@ceu.edu.ph)


lOMoARcPSD|19937673

\ d- a tooth has a large. longtstanding lesibn with a histoqy ol contrnuous paln


I A 98- A nonvit'al gdmary incisor.-(a,Lscess-Ctie to trauma! in,a lour yeat old paiient
I can be effectively treated by:
a- Pulpectoql =.
7 4ay f orrrpc.rasol px.tlporocr/
b, Extraction d- S-rnin torrnocresol PulPo(omy
B 99- Herpetic lesion of rhe gingival occur rircsthy at what age:
a-
during puber,ly c. can occur'ln any aEe
b-
,--S y€ars old d. 6:12 years old
O 1oo- Radlogiaphiial\r.;, rhe.2-d p.emelars de not visibE in,a 4 yeil old child.
whar shoukiitDe denlist do?
a. reter [he child to a pediaUician to'rule qrl sysr9mic imbalance
b. advise rhe pai€ntsi,thar odhodonlic treat[rent=?i[H be ddne wheh the chitd is much oldet
c- the realh cogldbe congenitaly riissing
d- watchlpl obseruation on the lxesdnce o, leel}i.ane. 2 yeils

L
Orrho. Pedo Set S
rb 21. b 41. a 6r. 81. c
2-a 7t. a 42,- E 62- B7 {l
( 83
L -.l.
4tl
Sd
€ tJ.
?a-
2s- d
d
<t .1 {
4.1-
45- b
a
63_
b4
65. c
84.
,85-
rl
d
b
6. 2:6. d 46. b 66. d llb- (
I 7- a 27. c 47_ ,b 67_ b 87.
I
I 8_d 28- b 4B- d 68: b 88. d
i
9.c 29. d 49- C 69- a 89d
l0 c 30. b 50. c 70. c }Ci. a
tI. b 3r d sr. c 7l- d UI c
12. a 32. a 52- a T2'- d 92b
13. d 33. b 53. d 73- b 93. d
14. c 3q. b 54. a 7a_ a %a
15. c 35. b ar
JJ. a 75. d 95a
16. d 36. a 56. c 76_ d 96. c
L7. a 37. d sz- c '77 - a 97. a
rB. b 38- d 58. a 78_ d 98- a
f9.a 39- s9- 79. a 99- b
L 20b 40_ C 60. 80- b loo. d

Downloaded by John Erickson Bangayan (bangayan1903064@ceu.edu.ph)


lOMoARcPSD|19937673

MCQ - ORTHODONTICS

Operation Research (heuristic) (International University)

Studocu is not sponsored or endorsed by any college or university


Downloaded by John Erickson Bangayan (bangayan1903064@ceu.edu.ph)
lOMoARcPSD|19937673

Choose the Best Answer:


1. The floor of the orbit is made up of the bones:
A. frontal, lesser wing, sphenoid
B. maxillary, lacrimal, ethmoid
C. maxillary, zygomatic, palatine
D. greater wing of the sphenoid, zygomatic

2. The only extraocular muscle that does not originate from the annulus
of Zinn is the:
A. Medial rectus
B. Inferior rectus
C. Superior rectus
D. Inferior oblique
E. Superior oblique

3. Which of the following nerves does not enter the orbit through the superior fissure?
A. CN II
B. CN III
C. CN IV
D. CN VI

4. What means “poor outcome” after cataract surgery according to WHO definition:
A. Presenting Visual Acuity < 6/24 in the operated eye
B. Presenting Visual Acuity = 6/60 in the operated eye
C. Presenting Visual Acuity < 6/60 in the operated eye
D. Best Corrected Visual Acuity < 6/60 in the operated eye

5. Which of the following muscles does NOT originate from the annulus of
Zinn?
A. Medial rectus
B. Lateral rectus
C. Superior rectus
D. Superior oblique
E. Inferior rectus

6. Third cranial nerve innervate all except


A. SR
B. SO
C. IR
D. MR

7. Traumatic hyphema may cause of blind by:


A. Glaucoma and corneal blood staining

Downloaded by John Erickson Bangayan (bangayan1903064@ceu.edu.ph)


lOMoARcPSD|19937673

B. Later cataract
C. Iris atrophy post traumatic
D. Lens ligaments rupture
E. Pupil dilated post traumatic

8. which of the following viruses is transmissible even after medical instrument is cleaned
with alcohol?
A. HSV
B. Adenovirus
C. HIV
D. Epstein-Barr virus

9. which of the following statements regarding topical fluorescein is false?


A. It is a nontoxic, water-soluble dye
B. It is used to detect disruption of intercellular junctions.
C. It can be used to detect subtle epithelial irregularity even without epithelial defect.
D. It has antiviral properties

10. Which of the following statements regarding anatomy of posterior


corneal stroma compared to anterior stroma is correct?
A. More rigidity
B. Tighter strength
C. More lamellar orthogonal arrangement
D. Predominantly oblique lamellar orientation
E. Extensive vertical lamellar branches and interweaving

11. All of the following organisms can invade an intact corneal epithelium, except:
A. Neiseiaminigitidis
B. Corynebacteriumdiphteriae
C. Shigella
D. Pseudomonas aeruginosa

12. Which one is the most common of microbial keratitis in Cambodia?


A. Bacterial Keratitis
B. Fungal keratitis
C. Viral keratitis
D. Parasite keratitis
E. Mix organisms keratitis

13. Follicular conjunctivitis is associated with all of the following conditions, except:
A. Herpes simplex virus conjunctivitis
B. Adult inclusion conjunctivitis
C. Drug-induced conjunctivitis

Downloaded by John Erickson Bangayan (bangayan1903064@ceu.edu.ph)


lOMoARcPSD|19937673

D. Allergic conjunctivitis

14. which of the following association is incorrect?


A. Mesectoderm – endothelium
B. Mesectoderm – stroma
C. Type IV collagen – Bowman’s layer
D. Type IV collagen –Descemet’s membrane

15. Which of the following is a type of intraepithelial neoplasia of the conjunctival?


A. Avascular
B. Actiniform
C. Leukoplakic
D. Pigmentary
E. Ulcerative

16. Mutation in the TIGR/Myocilin gene are associated with which of the following
disorders?
A. Pigment dispersion syndrome
B. Pseudoexfoliation syndrome
C. Juvenile POAG
D. Nanophthalmos

17. A young male of 22 years present with gradual decrease of visual


acuity in both eyes and
change of refractive error on examination with retinoscope there was
high astigmatism. Give likely diagnosis?
A. Keratoglobus
B. Keratoconus
C. Megalocornea
D. Buphthalmos
E. keratitis

18. In normal eye, the average corneal thickness is:


A. 520 um
B. 540 um
C. 560 um
D. 580 um
E. 600 um

19. Which of the following is a cause of primary limbal stem cell deficiency
A. Pterygia

Downloaded by John Erickson Bangayan (bangayan1903064@ceu.edu.ph)


lOMoARcPSD|19937673

B. Sclerocornea
C. Contact lens wear
D. Cicatricial pemphigoid
E. Chemical injury

20. In marfan syndrome the lens is displaced


A. Into the anterior chamber
B. Downward and outward
C. Upward and outward
D. Downward and inward

21. Which of these is found in excess of aqueous humor compared to blood


A. Ascorbic acid
B. Lactid acid
C. Glutathione
D. Sorbitol

22. Which structure does not develop from Neural ectoderm


A. Tarsal gland
B. Retina
C. Smooth muscle of iris
D. Optic cup

23. The important cause of canaliculitis is:


A. Staphylococcus spp
B. Streptococcus spp
C. Actinomycetes
D. Haemophilus influenza
E. Chlamydia trachomatis

24. According to "The Global Initiative for the Elimination of Avoidable


Blindness (VISION 2020, the Right to Sight)", what is the most cost
effective intervention for prevention visual impairment?
A. Correction of refractive error
B. Screening program for DR
C. Screening examination of premature infants for ROP
D. Screening program for early detection in POAG
E. Screening for strabismus

25. A Jackson’s Cross Cylinder used during refraction


A. Is useful to refine the power of the sphere
B. Determines the axis of the cylinder
C. Corrects power and the axis of the astigmatism

Downloaded by John Erickson Bangayan (bangayan1903064@ceu.edu.ph)


lOMoARcPSD|19937673

D. Has power of sphere double the power of the cylinder


E. Has power of sphere and cylinder the same sign

26. An aphakic patient wears +14.00 D glasses at a vertex distance of 12


mm. He needs
hydrogel contact lens for cosmetic reason. What is the appropriate
power of the contact lens you should prescribed?
A. +12.50 D
B. +14.00 D
C. +15.50 D
D. +17.00 D
E. +18.50 D

27. Transposition: Identify the incorrect answer:


A. + 1.00 / + 3.00 x 90 = + 4.00 / - 3.00 x 180
B. + 5.00 / - 1.00 x 75 = + 4.00 / + 1.00 x 165
C. - 6.00 / + 2.50 x 125 = - 3.50 / -2.50 x 35
D. – 3.00 / - 1.25 x 180 = - 4.25 / + 1.25 x 180
E. +2.00/ -4.00 x 180 = -2.00/ +4.00 x 90

28. Persistent hyperplastic primary vitreous is characterized by: except


A. A dehiscence in the posterior lens capsule
B. Secondary cataract
C. Glaucoma
D. Spontaneous intraocular hemorrhage
E. Bilateral occurrence

29. Which of the following is not an ocular emergency?


A. Ocular trauma
B. Sympathetic Ophthalmitis
C. CRAO
D. CRVO
E. Endophthalmitis

30. The following diseases predispose to neovascularization of the iris and angle, except:
A. CRVO (Central Retinal Vein Occlusioin)
B. Chronic retinal detachment
C. Chandler syndrome
D. Diabetic retinopahty

31. Children of mothers with diabetes are at increased risk for the
development of:
A. Optic nerve hypoplasia

Downloaded by John Erickson Bangayan (bangayan1903064@ceu.edu.ph)


lOMoARcPSD|19937673

B. Pigmentary glaucoma
C. Nerve fiber layer hemorrhages
D. Optic disc swelling
E. Retinoblastoma

32. Which of the following is the cause of sensory exotropia?


A. Herpes simplex keratitis
B. Ectropion uvea
C. Optic disc drusen
D. PDR with TRD
E. Melanocytoma of optic disc

33. A patient is found to have A-pattern exotropia with a compensatory


head posture. The
most likely head posture is
A. chin-up
B. chin-down
C. right head tilt
D. left head tilt
E. right face turn

34. Earliest symptom of sympathetic ophthalmitis is:


A. Photophobia
B. Pain
C. Loss of near vision
D. Loss of distant vision
E. Loss of accommodation

35. A patient of 45 years old presented with facial palsy. The epiphoria in
this patient was due to
A. Ectropion
B. Entropion
C. Lagophthalmos
D. Lacrimal pump failure
E. Hyper secretion of tears

36. The most common sign of Graves’ ophthalmopathy is:


A. lid retraction.
B. Conjunctival injection over the horizontal rectus muscles.
C. Superficial punctate keratitis (SPK)
D. Proptosis
E. Diplopia in up gaz

Downloaded by John Erickson Bangayan (bangayan1903064@ceu.edu.ph)


lOMoARcPSD|19937673

37. Cigarette smoking has been most strongly associated with which manifestation of age-
related macular degeneration?
A. Hard drusen
B. Soft drusen
C. Focal retinal pigment epithelium (RPE) hyperplasia
D. Choroidal neovascularization
E. Geographic atrophy

38. Most sensitive part of eye is:


A. Fovea centralis
B. Mucula lutea
C. Blind spot
D. Temporal retina
E. Nasal retina

39. First sign in sympathetic ophthalmitis


A. Aqueous flare
B. Keratic precipitates
C. Retrolental flare
D. Constriction of the pupil
E. Delen fuch’s nodules

40. A 25-year-old man, after motor vehicle accident, is evaluated in the


emergency room
for trauma to the left orbit. He has marked proptosis and an intraocular
pressure of 45 mm Hg on the affected side. A CT scan shows
intraorbital hemorrhage.
Which of the following actions would be the LEAST effective in acutely
reducing
intraocular pressure?
A. Lateral canthotomy and cantholysis
B. Administration of topical aqueous suppressants
C. Administration of intravenous mannitol
D. Administration of high-dose oral corticosteroids
E. Anterior chamber paracenthesis

41. Primary objective of use of atropine in anterior uveitis is


A. Relaxation of ciliary muscle
B. Increase blood flow
C. Prevent posterior synechiae formation
D. Increase supply of antibodies
E. Reduce accommodation

Downloaded by John Erickson Bangayan (bangayan1903064@ceu.edu.ph)


lOMoARcPSD|19937673

42. A 65-year-old black woman has presenting symptoms of choroidal neovascularization


and multiple tiny lid nodules. The most likely diagnosis is:
A. Syphilis
B. Tuberculosis
C. Sarcoidosis
D. Toxoplamosis
E. Toxocariasis

43. Cystoid macular edema has been seen with all the following except:
A. Retinal capillary hemangioma
B. Topical epinephrine
C. Choroidal hemangioma
D. Oral tetracycline
E. Oral nicotinic acid

44. The mechanism of glaucoma in Sturge-Weber syndrome is


A. Increased episcleral venous pressure +
B. Angle-closure glaucoma
C. Neovascular glaucoma
D. Pupillary block glaucoma

45. A newborn infant with nystagmus is diagnosed with aniridia. Optic nerve examination is
most likely to reveal
A. Coloboma
B. Pollar
C. Hypoplasia
D. Swelling

46. Factor that may increase IOP include all of the following except:
A. Valsava maneuver
B. Aerobic exercise
C. Ketamine
D. Blepharospasm

47. Which of the following types of exodeviation is the most common?


A. Pseudoexotropia
B. Congenital exotropia
C. Duane syndrome type 2
D. Intermittent exotropia

48. A patient undergoes bilateral lateral rectus recession for intermittent exotropia. The
patient should be told that a stable postoperative result will be know in what period of
time?
A. Immediately after surgery

Downloaded by John Erickson Bangayan (bangayan1903064@ceu.edu.ph)


lOMoARcPSD|19937673

B. 48-72 hours after surgery


C. 6-8 weeks after surgery
D. 4-6 months after surgery

49. Which of the following is the best method to determine whether a patient is at risk of
angle closure?
A. Gonioscopy
B. Darkroom prone position test
C. Pharmacologial pupil dilation
D. Darkroom test

50. Which systemic medication is the most appropriate treatment for an infant with
ophthalmia neonatorum secondary to Chlamydia trachomatis?
A. Erythromycine
B. Doxyclocline
C. Ofloxacine
D. Azithromyine

51. A 45-year-old man presented with full-thickness left lower lid margin
avulsion about
1/3 of the lid length after trauma. Which of the following is the most
appropriate
management?
A. Direct closure if possible
B. Semilunar flap
C. Adjacent tarsoconjuctival flap and skin graft
D. Tarsoconjuctival flap from upper eyelid and skin graft
E. Skin graft using postauricular skin

52. According to the finding from the Early Treatment for Retinopathy of Prematurity Study
(ET-ROP), laser therapy should be initiated for patients with type of ROP?
A. Five contiguous clocks hours of Zone II, Stage 3.
B. Zone I, Stage 2.
C. Zone I, Stage 3 with Plus disease
D. Zone III, Stage I.

53. Potential late complications of ROP include all of the following except:
A. Strabismus
B. Myopia
C. Negative-angle-kappa
D. Glaucoma

54. That is the most significant risk factor for developing retinopathy of prematurity?

Downloaded by John Erickson Bangayan (bangayan1903064@ceu.edu.ph)


lOMoARcPSD|19937673

A. Male age
B. Gestation age
C. White race
D. Lund disease

55. The following HLAs are associated with an increased incidence of uveitis: except
A. HLA-B27
B. HLA-A29
C. HLA-B51
D. HLA-A1

56. What percentage of Ant uveitis has cystoid macula oedema as a complication?
A. 1%
B. 5%
C. 15%
D. 25%

57. The following conditions can give rise to both uveitis and erythema nodosum: except
A. Crohn’s disease
B. Behcet’s disease
C. Sarcoidosis
D. Rheumatoid arthritis

58. What is the most common intraocular infection in patients with AIDS?
A. Acute retinal necrosis
B. Toxoplasmosis
C. Candidiasis
D. CMV retinitis

59. Posner-Schlossman syndrome:


A. Is painless
B. Does not cause glaucomatous field loss
C. Is associated with hypopyon in the majority of cases
D. Is a self-limiting condition

60. Which form of bilaterally symmetric refractive error at a level of 3.5 diopters would
place a child at the greatest risk for isometric amblyopia?
A. Myopia
B. Hyperopia
C. Astigmatism
D. No risk of isometric amblyopia

61. Which one of the following statements regarding retinitis pigmentosa is true?

Downloaded by John Erickson Bangayan (bangayan1903064@ceu.edu.ph)


lOMoARcPSD|19937673

A. The X-linked form is least common but most disabling


B. Signs and symptoms typically precede ERG abnormalities
C. Retinal pigmentary changes in the midperiphery are always present
D. The initial visual field defect is a ring scotoma

62. Congenital nasolacrimal duct obstruction is most commonly caused by an obstruction of:
A. The upper canaliculus
B. The valve of Hasner
C. The lower punctum
D. The valve of Rosenmüller

63. Which method of clinical measurement of the accommodative convergence to


accommodation ratio (AC/A) employs the distance and near deviations and their
relationship?
A. Near point of accommodation
B. Near point of convergence
C. Gradient method
D. Heterophoria method

64. Anterior chamber depth:


A. Is less in women than in men
B. Increase with age
C. Increase by hyperopia
D. Decrease in very high myopia
E. Rarely connected with AC volume

65. ertinent clinical features used to confirm the diagnosis of a dacryocele in a 2-week-old
infant include which of the following?
A. The presence of a bluish swelling just below and nasal to the medial canthus
B. The presence of a bluish swelling just below and temporal to the medial canthus
C. The presence of a bluish swelling just above and temporal to the medial canthus
D. The presence of a bluish swelling just above and nasal to the medial canthus

66. In peripheral iridotomy with laser:


A. dark colour iris responds poorly to argon laser iridotomy
B. more energy is needed for argon laser than YAG laser for a given size iridotomy
C. Steroid pre-treatment is effective in decreasing the intraocular pressure
D. presence of red reflex during the procedure indicate the iridotomy is patent
E. significant cataract develops in 50% of cases following treatment

67. Which of the following is the most appropriate description of the cornea after birth?
A. It flattens, reaching adult power by the age of 12 years
B. It has a diameter of approximately 8 mm

Downloaded by John Erickson Bangayan (bangayan1903064@ceu.edu.ph)


lOMoARcPSD|19937673

C. It changes most in the first 5 years, reaching adult diameter by age 8 years
D. It is cloudy at birth, because of edema and endothelial immaturity

68. In PIC (primary idiopathic choriodopathy):


A. Myopia is an association
B. More male than female are affected
C. Vitritis is common
D. Viral prodrome is common
E. Both eyes are affected in 25% of cases

69. A 62-year-old woman with a 45-minute history of blunt trauma


presented with pain,
proptosis, lid swelling and ecchymosis. Because of tight eyelids, other
eye examinations
could not be evaluated. Which of the following is the first step
management?
A. CT scan of orbits
B. Anterior chamber tapping
C. Canthotomy and cantholysis
D. Tarsorrhaphy to prevent exposure keratitis
E. Explore globe

70. Specific sign of glaucomatous damage is


A. Baring of circumlinear blood vessels
B. Bayoneting
C. The laminar dot sign
D. Disc haemorrhages
E. Superior or inferior polar notching of the cup

71. Which of the following is true regarding acute angle closure?


A. There are no racial differences
B. Can be caused by pharmacological dilation
C. Myopia is a risk factor
D. Is more common in males
E. Is a uniocular disease

72. The inner plexiform layer contains the following cells except:
A. Photoreceptors
B. Müller cells
C. ganglion cells
D. Bipolar cells
E. Amacrine cells

Downloaded by John Erickson Bangayan (bangayan1903064@ceu.edu.ph)


lOMoARcPSD|19937673

73. Cataract associated with Down’s syndrome usually presents as


A. Anterior subcapsular cataract
B. Scattered punctate cataract
C. Nuclear cataract
D. Christmas tree cataract
E. Posterior subcapsular cataract

74. The following statements are true regarding dry age-related macular degeneration
(AMD):
A. it will typically have a visual acuity of 6/60 or worse within 2 years
B. the optimal management is with low visual aids
C. the contralateral eye will be simultaneously affected in all cases
D. the patient should be promptly referred for fluorescein angiography
E. dry AMD can be treated with photodynamic therapy without verteporfin

75. A 65-year-old welder comes to the emergency department directly from work and
complains of bilateral pain and photophobia. The most likely cause is:
A. infrared burn to the eyes
B. acute photic keratitis
C. ultraviolet absorption by the lens.
D. secondary anterior uveitis.
E. retinal toxicity and secondary choroiditis.

76. Which is the more common sign in normal-tension glaucoma?


A. Parapapillary atrophy
B. Disc hemorrhage
C. High intra-ocular pressure
D. Shallow cupping with optic nerve pit
E. Glaukomflecken

77. Which ocular condition is associated with an increased risk of complications with
cataract surgery?
A. Ocular hypertension
B. Exfoliation syndrome
C. Pigment dispersion syndrome
D. Angle recess

78. Laser trabeculoplasty is contraindicate in which of the following


conditions?
Previous laser trabeculoplasty
A. Pseudophakia
B. Fuchs endothelium dystrophy
C. Pigmentary glaucoma

Downloaded by John Erickson Bangayan (bangayan1903064@ceu.edu.ph)


lOMoARcPSD|19937673

D. Inflammatory glaucoma

79. In central retinal artery occlusion: Except:


A. the occlusion typically occurs near the optic nerve head
B. the most common cause is arteriosclerosis of the retinal artery
C. liquifactive infarction of the retinal nerve fibre layer is a feature
D. gliosis is a prominent repair mechanism
E. the photoreceptors undergo atrophy

80. According to OHTS, which one of the following is associated with an increased risk of
converting from ocular hypertension to POAG?
A. A history of diabetic mellitus
B. Decrease age
C. Small cup disc ratio
D. Lower central cornea thickness

81. In patients with VKH syndrome, the presence of diffuse choroiditis is most likely to be
found during which stage of the disease
A. Recurrent
B. Prodromal
C. Late (chronic)
D. Early (acute uveitic)

82. What is the triad of reactive arthritis syndrome?


A. urethritis, polyarthritis, and conjunctival inflammation
B. ulcerative colitis, polyarthritis, and conjunctival inflammation
C. genital ulcers, polyarthritis, and vasculitis
D. palmar rashes, pauciarticular arthritis, and fevers

83. In which of the following is peripheral iridotomy is the treatment of choice?


A. Secondary angle closure following dense Pan retinal Photocoagulation
B. Iridocorneal Endothethelial dystrophy
C. Phacoanaphylactic glaucoma
D. Phacomorphic glaucoma

84. A 14 years old boy with bilateral iris atrophy and corectopia is found to have elevated
IOPs. His father has a similar condition. Which of the following is the most likely
diagnosis?
A. ICE syndrome
B. Lowe Syndrome
C. Axenfeld-Reiger syndrome
D. Hallermann-streiff syndrome

Downloaded by John Erickson Bangayan (bangayan1903064@ceu.edu.ph)


lOMoARcPSD|19937673

85. As per the mechanical theory of glaucoma pathogenesis, deprivation of


which of the following factors may accelerate retinal ganglion cell loss?
A. heat-shock protein
B. calcium
C. neurotrophins
D. dopamine
E. glutamate

86. Patient with primary angle closure usually have:


A. Short axial length
B. An anterior chamber depth > 2.2 mm
C. Increased anterior curvature of the lens
D. Small cornea diameter and radius of curvature
E. All of above

87. What anatomical modification is found in increased frequency in primary congenital


glaucoma?
A. Increased axial Length
B. Hyperopia
C. Hyperplastic optic nerve
D. Decrease cornea diameter

88. Female with uncontrolled diabetes presents with painful red eye and
visual acuity is also
decreased. On examination there was raised Intraocular Pressure and
new blood vessels on the iris. The treatment includes all except.
A. Atropine
B. Beta blockers
C. Steroids
D. pain killers
E. Pilocarpine

89. Which of the following medication is contraindicated in the treatment of glaucoma in a


toddler?
A. Dorzolamide
B. Latanoprost
C. Brimonidine
D. Timolol

90. The preferred therapy for infantile glaucoma is:


A. Topical beta-blockers
B. Topical bromonidine

Downloaded by John Erickson Bangayan (bangayan1903064@ceu.edu.ph)


lOMoARcPSD|19937673

C. Trabeculotomy or goniotomy
D. Oral acetazolamide

91. Which of the following is the preferred initial surgical procedure for an infant with
primary congenital glaucoma and corneal clouding?
A. Goniotomy
B. Trabeculectomy
C. Cyclophotocoagulation
D. Trabeculotomy

92. The wavelength of ND-YAG laser is


A. 532 nm
B. 1064 nm
C. 840 nm
D. 647 nm

93. Which of the following conditions is Least likely to be complication of periocular


triamcinolone injection?
A. Ptosis
B. Raised IOP
C. Cataract formation
D. Iris atrophy
E. Vascular occlusion

94. Using a high-power slit beam 2 mm long and 1mm in height with maximal light intensity,
you observe approximately 90 cells in the anterior chamber. What grading of AC cells
will you give this eye?
A. 3+
B. 1+
C. 2+
D. 4+
E. 0.5+

95. A 45-year-old man with chronic angle closure glaucoma had undergone
uneventful
trabeculectomy with mitomycin C. One week after the operation, he
developed uniform
shallow anterior chamber. The IOP was 40 mmHg. The bleb was low.
Which is the most likely diagnosis in this patient?
A. Encapsulated bleb
B. Pupillary block
C. Expulsive suprachroidal hemorrhage
D. Malignant glaucoma

Downloaded by John Erickson Bangayan (bangayan1903064@ceu.edu.ph)


lOMoARcPSD|19937673

E. Steroid induced glaucoma

96. Which of the following systemic diseases is NOT associated with


ectopia lentis?
A. Homocystinuria
B. Ehlers-Danlos syndrome
C. Marfan syndrome
D. Myotonic dystrophy
E. Sulfite oxidase deficiency

97. The most common cause of non-infectious anterior uveitis is


A. Behcet’s disease
B. Leukemia
C. Ankylosing Spondylitis
D. VKH Syndrome
E. Fuch’s Heterochromic cyclitis

98. The most common cause of Uveitic glaucoma in adults is:


A. Sympathetic ophthalmia
B. Toxoplasmosis
C. Posner-Schlossman syndrome
D. Sarcoidosis
E. Herpetic uveitis

99. A 48 years old man with ulcerative colitis and chronic lower back pain complains of red
painful right eye for a few days. Which of the following is the most likely to be present
on eye examination?
A. Purulent conjunctivitis
B. Disciform keratitis
C. Inflammatory cells in anterior chamber with keratic precipitates
D. Anisocoria with dilate pupil in the right eye
E. Multiple retinal hemorrhages and cotton wool spots

100. Granulomatous Anterior Uveitis


A. Is rarely related to infections
B. Is associated with HLA B27
C. Does not response to topical steroid treatment
D. May follow herpes zoster ophthalmicus
E. Is usually treated with topical antibiotics

101. The most earliest sign of anterior uveitis is:


A. Aqueous flare
B. Aqueous cells

Downloaded by John Erickson Bangayan (bangayan1903064@ceu.edu.ph)


lOMoARcPSD|19937673

C. Keratic precipitates
D. Constriction of pupil
E. Raised intraocular pressure

102. Which is not an ocular feature of Down’s syndrome?


A. Mongoloid slant
B. Iris Hypoplasis
C. Cataract
D. Aniridia

103. The risk of postoperative endophthalmitis after cataract surgery is increased by:
A. Surgery complicated by vitreous loss
B. Topical use of iodine-containing antiseptic solution preoperatively
C. Use of topical steroid postoperatively
D. Planned aphakia
E. Use of topical antibiotic postoperatively

104. An infant with nystagmus is diagnosed with aniridia. Fundus examination is most
likely to reveal
A. Choroidal coloboma
B. Optic atopthy
C. Optic nerve hypoplasia
D. Papilledema
E. Hypertrophy of RPE

105. Prominent Corneal nerve are a feature of all except


A. Acanthamoeba keratitis
B. Keratoconus
C. Fuch Endothelial dystrophy
D. Mycotic keratitis

106. Which of the following types of exodeviations is the most common?


A. Pseudoexotropia
B. Congenital exotropia
C. Duane syndrome type 2
D. Intermitent exotropia
E. Consecutive exotropia

107. Latanaprost
A. Is the most effective of the prostaglandins
B. Rarely causes iris colour change
C. Increases trabecular and uveo-scleral outflow
D. Increases risk of cystoid macular oedema after cataract surgery

Downloaded by John Erickson Bangayan (bangayan1903064@ceu.edu.ph)


lOMoARcPSD|19937673

E. an be used in patients with a history of herpes keratitis provided there is no active


disease.

108. The following are seen in neurofibromatosis:


A. lisch nodules
B. cherry red spot
C. café-au-lait spots
D. roth’s spots
E. lisch nodules, café-au-lait spots

109. What is the MOST common systemic association with scleritis?


A. Systemi lupus erythematous
B. Polyarteritis nodosa
C. Rheumatoid arthritis
D. Inflammatory bowel disease
E. Wegener’s granulomatosis

110. Which one of the following is a systemic feature of rheumatoid arthritis?


A. Nodules
B. Asymmetric arthritis
C. Nail pits
D. Cerebral haemorrhage
E. Polycythaemia

111. A large pituitary tumor will usually cause which of the following
kinds of visual field defects?
A. Bitemporal hemianopsia
B. Left homonymous hemianopsia
C. Right homonymous hemianopsia
D. Right homonymous inferior quadrantanopsia
E. Left homonymous inferior quadrantanopsia

112. A 26 years old patient presented with severe ptosis in the right
side. He had
motorcycle accident 3 weeks ago. On examination he had right CN III
and frontal branch of facial nerve injuries. The levator function was
poor in the affected side.
How would you manage this patient?
A. Observation
B. Botulinum toxin injection
C. Mullerectomy
D. Levator advancement

Downloaded by John Erickson Bangayan (bangayan1903064@ceu.edu.ph)


lOMoARcPSD|19937673

E. Frontalis suspension

113. Which of the following features found in pathological specimen is most


suggestive of retinoblastoma?
A. Calcification
B. Rosettes formation
C. Touton giant cells
D. Intralesional macrophages
E. Intralesion necrosis

114. In Goldman applanation tonometry:


A. Thick mires cause over-estimation of the true IOP
B. Thin mires may be due to too low fluorescein concentration
C. Low fluorescein concentration may cause under-estimation of the true IOP
D. A corneal thickness of 480 microns will lead to under-estimation of the true IOP
E. All of the above

115. Nevus Flammeus is associated with:


A. Von Recklinghausen’s Disease
B. Tuberous Sclerosis
C. Sturge webber syndrome
D. Coat’s disease

116. Chronic progressive external ophthalmoplegia (CPEO) is a


disease with the mode of
transmission of:
A. Autosomal dominant
B. Autosomal recessive
C. X-link recessive
D. X-link dominant
E. Mitochondrial transmission

117. Following trabeculectomy a leaking bleb


A. If the sclera flap is less than one half sclera thickness
B. In myopia eyes
C. Following application of mitomycine C
D. In young patients
E. All of the above

118. The short posterior ciliary arteries are about in number:


A. 10
B. 20
C. 30

Downloaded by John Erickson Bangayan (bangayan1903064@ceu.edu.ph)


lOMoARcPSD|19937673

D. 40
E. 45

119. The white dot lesions of which disease are least apparent on fluorescein
angiography?
A. serpiginous choroiditis
B. punctate inner choroidopathy (PIC)
C. acute posterior multifocal placoid pigment epitheliopathy (APMPPE)
D. birdshot retinochoroidopathy
120. Patient with inactive uveitis about 1 year ago, which one of the following types of
cataract is most likely to alter a patient spectacle prescription from +3.25 / -0.50 x 75 to
+1.00 / -0.50 x 75?
A. Anterior cortical cataract
B. Posterior sub capsular cataract
C. Nucleus cataract
D. Polar cataract

121. A 58-year-old man suddenly loses all vision in one eye. It begins
to improve 30 minutes later, and a few hours later is back to normal.
The most likely diagnosis is:
A. Retinal arterial occlusion
B. Retinal vein occlusion
C. Retinal detachment
D. Vitreous haemorrhage
E. Acute optic neuropathy

122. Bilateral paralysis of accommodation can occur in patients with:


A. Diabetes
B. Syphilis
C. Diphtheria
D. All of the above
E. None of the above

123. Features that may help distinguish CRVO from carotid artery
occlusiv disease include all of the following except:
A. dilated retinal veins
B. tortuosity of retinal veins
C. ophthalmodynamometry
D. retinal artery pressure

124. of the following is CORRECT concerning drug-induced cataract?


A. Amiodarone causes stellate anterior axial pigment deposition.
B. Phenothiazine causes pigment deposition in the posterior lens capsule

Downloaded by John Erickson Bangayan (bangayan1903064@ceu.edu.ph)


lOMoARcPSD|19937673

C. Prolonged treatment of eyelid dermatitis with topical steroids does not cause
cataract.
D. Echothiophate use in children causes progressive cataract formation.

125. Frosted branch angiitis is MOST characteristic of infection with:


A. HIV
B. HSB
C. HBV
D. CMV
E. HZV

126. Techniques for entropion repair include all this followings except:
A. lid retractor reattachment.
B. botulinum toxin injection.
C. Quickert-Rathbun sutures
D. transverse tarsorrhaphy.
E. Kuhnt-Szymanowski procedure

127. Retinoschisis
A. Should be treated with laser to prevent progression
B. Causes absolute scotoma
C. Is associated with water mark
D. Is unilateral in the majority of cases
E. Does not cause retinal detachment

128. Fleischer ring is found in:


A. Keratoconus
B. Chalcosis
C. Argyrosis
D. Buphthalmos
E. None of the above

129. Which fat soluble vitamin is used in the treatment of xerophthalmia?


A. Vitamin A
B. Vitamin B6
C. Vitamin C
D. Vitamin D
E. Vitamin E

130. Ptosis in Horner's syndrome, is due to paralysis of:


A. Riolan's muscle
B. Horner's muscle
C. Muller's muscle
D. The levator palpebral muscle

Downloaded by John Erickson Bangayan (bangayan1903064@ceu.edu.ph)


lOMoARcPSD|19937673

E. Orbicularis oculi muscle

131. Vitamin A deficiency causes night blindness by


A. Inducing cataract
B. Impaired optic nerve function
C. Causing retinitis pigmentosa
D. Damage to the visual cortex
E. Depletion of rhodopsin in photoreceptors

132. The following conditions are associated with high risk of trabeculectomy failure
A. Pseudoexfoliation
B. Prolong steroid use
C. Primary angle closure glaucoma
D. Aphakia
E. All of the above

133. Retinal dialysis:


A. Is associated with posterior vitreous detachment
B. Is associated with myopia
C. Has a mobile posterior flap
D. Can be treated by cryotherapy, encirclement and a local scleral
buckle
E. Is commonest in the superior temporal quadrant

134. Which of the following is true regarding cataracts associated with systemic
diseases?
A. Atopic dermatitis causes posterior polar cataract
B. Myotonic dystrophy causes Christmas tree cataract
C. Steroids cause anterior subcapsular cataract
D. Wilson’s syndrome causes snowflake cataract
E. Diabetes causes sunflower cataracts

135. In manual small incision cataract surgery with an unsutured wound


A. There is usually flattening of the cornea in the axis of the incision
B. There is usually steepening of the cornea in the axis of the incision
C. The postoperative astigmatism is unpredictable
D. The risk of endophthalmitis is less than it is for a sutured wound
E. The risk of cystoid macular oedema is less than it is for a sutured wound

136. Steroid drop with the greatest anti inflammatory action is


A. Loteprednol 0.2%
B. Fluromethalone 0.1%
C. Medrysone 1.0%

Downloaded by John Erickson Bangayan (bangayan1903064@ceu.edu.ph)


lOMoARcPSD|19937673

D. Prednisolone acetate 1.0%

137. An inert intraocular Foreign body is


A. Copper
B. Iron
C. Nickel
D. Platinum

138. Acute retinal necrosis is caused mainly by


A. Varizella zoster
B. Cytomegalo virus
C. Syphilis
D. Leptospira

139. All statements hold true for retinal tamponade using silicon oil except
A. Produces secondary glaucoma
B. Expands with changing atmospheric pressure
C. To be removed after 8 -12 weeks
D. Reduces tendency for proliferative vitreoretinopathy

140. Which holds true for congenital retinoschisis?


A. Splitting of retina at outer plexiform layer
B. Sparing of fovea
C. Splitting of retina in the nerve fiber layer
D. Extends up to ora serrata

141. What does not hold true for central serous choriodopathy
A. Smoke stack appearance of FFA
B. Ink blot appearance on FFA
C. Flower petal appearance on FFA
D. Vision correctable by convex lenses

142. Depth perception in and infant develops by the age of


A. 3 months
B. 8 weeks
C. 12 months
D. 6 months

143. Which of these anti glaucoma drugs are contraindicated in patients with sulpha
allergy?
A. Bimatoprost
B. Brinzolamide
C. Timolol maleate
D. Travoprost

Downloaded by John Erickson Bangayan (bangayan1903064@ceu.edu.ph)


lOMoARcPSD|19937673

144. Which statement is true regard to the lacrimal gland?


A. The palpebral part is larger than the orbital part
B. Both palpebral and orbital portions have ductules that open separately into the
conjunctiva
C. The ductules open into the conjunctiva at the upper border of the upper tarsus
D. Biopsy of the orbital part is more likely to impair tear production than biopsy of
the palpebral part
E. Lacks of a true capsule

145. Which one of the following conditions is associated with formation of iris
granuloma?
A. Neurofibromatosis
B. Juvenile xanthogranuloma
C. Pseudoexfoliation syndrome
D. Pseudoxanthoma elasticum
E. Xanthelasma

146. Which one of the following causes non-arteritic anterior ischemic optic
neuropathy (NAION)?
A. Rheumatoid arthritis
B. Injury
C. Arterosclerosis
D. HIV
E. Thyroid eye disease

147. The instrument generally used to measure intraocular pressure in a scarred cornea
is
A. Pulse air
B. Airpuff
C. Tonopen
D. Perkins

148. Rate of formation of aqueous humor is


A. 3.2 ul/minute
B. 3.4 ul/minute
C. 2.9 ul/minute
D. 2.3 ul/minute

149. Which of the following causes raised IOP in uveitis?


A. Is rarely related to infections
B. Is associated with HLA B27
C. Does not respond to topical steroid treatment
D. May follow Herpes Zoster Ophthalmicus

Downloaded by John Erickson Bangayan (bangayan1903064@ceu.edu.ph)


lOMoARcPSD|19937673

E. Is usually treated with topical antibiotics

150. Malignant melanoma of the choroid


A. Rarely occurs in non-white Europeans
B. Presents commonly as field loss
C. Does not arise from pre-existing nevi
D. Metastasizes to the kidney
E. Usually requires exenteration of the globe and orbit

151. All of these diagnostic tests are useful in evaluating a patient


with a retained magnetic intraocular foreign body except:
A. Indirect ophthalmoscopy
B. Computed tomography
C. Electrophysiology
D. Magnetic resonance imaging (MRI)
E. Echography

152. Neuroblastoma that is metastatic to the orbit:except:


A. arises in the abdomen in 90% of cases
B. first appears as an orbital mass in 8% of cases
C. is the second most common malignant orbital tumor of childhood.
D. affects both orbits in 40% of children.
E. rarely advances to orbital bones.

153. Retinoblastoma
A. Is a benign tumour that can safely be observed
B. Occurs in middle-aged females
C. Is inherited when it occurs bilaterally
D. Is due to a mutation in chromosome 21
E. All of the above

154. The average corneal endothelial cell count is


A. 2000/mm2
B. 1800/mm2
C. 3500/mm2
D. 2800/mm2

155. Hydroxyapatite orbital implant after enucleation: except


A. is usually wrapped in donor sclera.
B. receives the four rectus muscles.
C. requires a peg to produce maximal movement of the implant.
D. may be rejected by the body’s immune system
E. generally undergoes drilling 6 to 12 months after placement.

Downloaded by John Erickson Bangayan (bangayan1903064@ceu.edu.ph)


lOMoARcPSD|19937673

156. What is the most crucial prognostic factor of idiopathic macular


hole?
A. Size of the hole
B. Causation of macular hole
C. Duration of patient’s symptom
D. Presence of posterior vitreous detachment
E. Presence of macular edema

157. Which is not true for sympathetic ophthalmitis:


A. Never seen within a week of injury
B. Retrolental flare is an early sign
C. Non granulomatous panuveitis
D. Auto immune aetiology

158. You have followed a 33 years old man for retinitis pigmentosa for 6 years. He
came to see you complained of reduced vision in the right eye for 2 months. His visual
acuity on last year’s visit was 20/20 both eyes. Today the visual acuity was 20/40 OD,
and 20/20 OS. Which of the following conditions is the most likely cause of central
vision deterioration in this patient?
A. Corneal edema
B. Lens dislocation
C. Secondary glaucoma
D. Vitreous hemorrhage
E. Cystoid macular edema

159. A 58 years old female has non-proliferative diabetic retinaopathy (NPDR) with
center involved macular edema. Which of the following treatments is most likely to result
in highest vision gain in this patient?
A. Conventional macular laser photocoagulation
B. Subthreshold macular laser
C. Intravitreous triamcinolone acetonide injection
D. Intravitreous anti-vascular endothelial growth factor injection
E. Photodynamic therapy

160. Refractive index of vitreous humour is:


A. 1.42
B. 1.37
C. 1.39
D. 1.33

Downloaded by John Erickson Bangayan (bangayan1903064@ceu.edu.ph)


lOMoARcPSD|19937673

161. In phakic asymptomatic patients, which of the following types of


retinal break is almost always treated, whereas the others rarely
treated?
A. operculated tears
B. lattice degeneration with or without hole
C. retinal dialysis
D. atrophic holes

162. In contact lens users the most common cause of infection is:
A. Pneumococcus
B. Streptococcus
C. Pseudomonas
D. Staphyloccus

163. Which of the following patient characteristics is most likely to be


associated with
macular hole?
A. Age 20 to 40 years
B. Male gender
C. Blunt trauma
D. Hyperopia
E. Congestive heart failure

164. A 30-year-old man came in for eye check up. The VA is 20/20 in
both eyes and he
has no history of flashing or floater. Dilated retinal examination
revealed 3-clockhour lattice degeneration with mild overlying vitreous
traction in his right eye.
What is the proper management?
A. Observation
B. Laser photocoagulation
C. Cryotherapy
D. Pneumatic retinopexy
E. Pars plana vitrectomy to remove vitreous traction and endolaser

165. Thinnest part of retina is


A. Macula
B. Around optic disc
C. Equator
D. Ora serrata

166. Which of these help in visualization of corneal cells?


A. Confocal microscope

Downloaded by John Erickson Bangayan (bangayan1903064@ceu.edu.ph)


lOMoARcPSD|19937673

B. Optic coherence tomography


C. Pachymetry
D. Slit lamp biomicroscope

167. Most common type of corneal dystrophy is


A. Meesman
B. Microcytic
C. Macular
D. Granular

168. Persistent hyperplastic primary vitreous:


A. Caused elongated ciliary muscles
B. Angle closure glaucoma is a recognised feature
C. Optic disc abnormality is seen
D. Always causes microphthalmic eye
E. Visual prognosis is usually good

169. The following are true about intravitreal gas injection:


A. It can cause secondary retinal tear
B. It usually causes cystoid macular edema
C. C3F8 has a shorter half life than SF6
D. SF6 is more expansile than C3F8
E. SF6 causes more increase in the intraocular pressure than C3F8

170. Mutations in the rhodopsin gene are associated with which


inherited ocular disease?
A. Juvenile glaucoma
B. Leber optic neuropathy
C. Retinitis pigmentosa
D. Stargardt disease
E. Optic nerve pit

171. In acute bacterial endophthalmitis following cataract operation:


A. staphylococcus epidermidis is the most common pathogen
B. presents within 48 hours of surgery in the majority of cases
C. Topical and systemic antibiotic are usually adequate to control
the infection
D. Steroid is contraindicated
E. Intravitreal anti-VEGF is the treatment of choice

172. Which of the following sign is useful in distinguishing between the congenital and
acquired form of Horner's syndrome?
A. Absent light reflex of the affected eye

Downloaded by John Erickson Bangayan (bangayan1903064@ceu.edu.ph)


lOMoARcPSD|19937673

B. Lighter iris color of the affected eye


C. Decreased facial sweating of the affected side
D. Dilatation of the affected pupil with 10% cocaine
E. Pupil sizes become equal after placing ice over the eye

173. All are branches of ophthalmic artery except


A. Central retinal artery
B. Anterior choroidal artery
C. Anteriorciliary artery
D. Short posterior ciliary artery

174. Causes of decreased corneal sensation are all except:


A. Herpes simplex keratitis
B. Acanthamoeba keratitis
C. Post keratoplasty
D. Acoustic neuroma

175. The following drugs can cause affect a patient’s visual acuity:
A. Ofloxacin
B. Ethambutol
C. Na Sulfacetamide
D. Gentamycin

176. Iridocyclitis is associated most frequently with which form of JIA


(Juvenile idiopathic
arthritis)?
A. Monoarticular
B. Pauciarticular (oligoarthritis)
C. Polyarticular, Rhemutoid factor negative
D. Polyarticular, Rhemutoid factor positive
E. Systemic

177. Approximate number of retinal photoreceptors cells?


A. 1 million
B. 25 million
C. 10 million
D. 125 million

178. A 60-year-old woman presents with red eye, OS. She was
diagnosed as secondary
glaucoma, OS and start anti-glaucoma medications for 3 months. Left
eye examination

Downloaded by John Erickson Bangayan (bangayan1903064@ceu.edu.ph)


lOMoARcPSD|19937673

reveals small cystic-like nodules all over the bulbar and tarsal
conjunctiva.
Which of the following medication is most likely to be the cause?
A. Brinzolamide
B. Bimatoprost
C. Brimonidine
D. Betaxolol
E. Timolol

179. Which statement is not true about cornea verticillata?


A. Seen in fabry disease
B. Caused by long term use of chloroquine
C. Involves corneal endothelium
D. Reversible condition

180. The first day following cataract surgery, high IOP is most likely due to the
following condition
A. Retinal detachment
B. Subconjunctival hemorrhage
C. Retained lens material
D. Retained viscoelastic material
E. Steroid induce glaucoma

181. True about adenoviral conjunctivitis


A. Genome is a linear double stranded DNA
B. Incubation period of 3 weeks
C. Formation of true membrane
D. Ganciclovir only modality of treatment

182. During phacoemulsification, you notice that the anterior chamber is shallow.
What is the management?
A. Decrease power
B. Increase Vacuum
C. Decrease bottle height
D. Decrease aspiration flow rate
E. Extend the corneal incision

183. A-30-year old man had severe pneumothorax after car accident 2 days ago and
multiple fractures. Today he notices of blurred vision in both eyes.The VA is 20/200 BE.
Anterior segment examination is within normal limit. Fundus exam revealed multiple
large cotton wool spots, retinal hemorrhages and retinal edema mostly around the optic
discs

Downloaded by John Erickson Bangayan (bangayan1903064@ceu.edu.ph)


lOMoARcPSD|19937673

A. Central retinal arterial occlusion


B. Central retinal vein occlusion
C. Ocular ischemic syndromes
D. Purtscher retinopathy
E. Retinitis sclopetalia

184. The average posterior chamber intraocular lens power in an adult is


A. 35D
B. 30D
C. 25D
D. 20D
E. 15D

185. A patient’s refractive error is -2.00 D -1.00 D x 180o .How should we classify the
refractive error of this patient?
A. Simple myopic astigmatism, with-the-rule
B. Simple myopic astigmatism, against-the-rule
C. Compound myopic astigmatism, with-the-rule
D. Compound myopic astigmatism, against-the-rule
E. Mixed astigmatism, with-the-rule

186. The best way to measure the patient’s strabismus with intermittent XT is:
A. Hirschberg test
B. Krimsky test
C. Maddox tod test
D. Prism and cover test
E. Bagolini lenses

187. Which of the following organisms causes the highest number of blindness
currently?
A. Chlamydia trachomatis
B. Cytomegalovirus
C. Herpes zoster virus
D. Neisseria gonorrhoeae
E. Staphylococcus aureus

188. The management of neovascular glaucoma includes


A. Peripheral laser trabeculoplsty
B. Miotics
C. Selective laser trabeculoplasty
D. Cyclodestruction
E. Unaugmented filtration surgery

Downloaded by John Erickson Bangayan (bangayan1903064@ceu.edu.ph)


lOMoARcPSD|19937673

189. A 45-year-old male presents with sudden visual loss in the left eye. Visual acuity
is 20/20 on the right and 20/70 on the left. There is a positive relative afferent papillary
defect on the left and an inferior altitudinal visual field defect in the same side. The
fundus examination is normal. What is the most likely diagnosis?
A. Anterior ischemic optic neuropathy
B. Retrobulbar optic neuritis
C. Compressive optic neuropathy
D. Branch retinal vein occlusion
E. CMV retinitis

190. During phacoemulsification, you notice that the anterior chamber


is shallow. What is
the management?
A. Decrease power
B. Increase vacuum
C. Increase bottle height
D. Increase aspiration flow rate
E. Extend the corneal incision

191. A 60 year-old women presents with eye injection and retained


lens fragments 3 mm in the vitreous cavity. after phacoemulsfication
for 1 month. What would be the best treatment option?
A. Continue observation
B. Aggressive topical anti-inflammation
C. Vitrectomy and removal of lens fragment
D. Intravitreal steroid injection
E. Re-operation and remove lens fragment with phacoemulsification
handpiece

192. Which of the following is true regarding cataracts associated with


systemic diseases?
A. Atopic dermatitis causes posterior polar cataract
B. Myotonic dystrophy causes Christmas tree cataract
C. Steroids cause anterior subcapsular cataract
D. Wilson’s syndrome causes snowflake cataract
E. Diabetes causes sunflower cataracts

193. Neurotrophic keratopathy is caused by


A. Symblepharon
B. Herpetic keratitis
C. Deep coma
D. Paralysis of orbicularis oculi

Downloaded by John Erickson Bangayan (bangayan1903064@ceu.edu.ph)


lOMoARcPSD|19937673

194. The normal resolution of visual acuity level of 6/6 is attained at the age of
A. 6 years
B. 4 years
C. 3 years
D. 8 years

195. Which is correct about the stereoscopic disc photography?


A. The ability to detect change is not related to image quality
B. The interobserver variability is one of the limitations of this instrument
C. Clinical examination can detect disc hemorrhage better than the stereoscopic disc
photography
D. The strereoscopic disc photography describes optic disc quantitative change
E. This technique is the gold standard for detecting generalize nerve fiber layer
defect

196. Which of the following statements about cataract surgery in patients with diabetes
is correct?
A. Patients with diabetes enrolled in the ETDRS who underwent cataract surgery did
not show an immediate improvement in visual acuity
B. Patients with diabetes with CSME should have cataract surgery performed prior to
focal laser
C. Patients with diabetes and high-risk proliferative changes visible through their
cataract should ideally have scatter laser immediately before cataract extraction
D. Patients with diabetes and high-risk proliferative change visible through their
cataract should have scatter laser 1 to 2 months prior to cataract extraction
E. Preoperative phenylephrine drops for dilation are contraindicated in patients with
diabetes undergoing cataract surgery

197. The first day following cataract surgery, high IOP is most likely
due to the following condition
A. Retinal detachment
B. Subconjunctival hemorrhage
C. Retained lens material
D. Retained viscoelastic material
E. Steroid induced glaucoma

198. Which is not a viscoelastic?


A. 1% sodium hyaluronate
B. 3% methyl cellulose
C. 5% acetyl cysteine
D. 2% hypermellose

Downloaded by John Erickson Bangayan (bangayan1903064@ceu.edu.ph)


lOMoARcPSD|19937673

199. According to epidemiology of primary congenital glaucoma, which one of the


following is TRUE?
A. Incidence: 1 in 2,000 births
B. Gender: female > male
C. Laterality: unilateral > bilateral
D. Heredity: autosomal recessive predominant
E. Most cases are diagnosed before 1 year

200. What is the mechanism of action of fluoroquinolones such as


ofloxacin, levofloxacin,ciprofloxacin, moxifloxacin, and gatifloxacin?
A. Alter bacterial cell membrane permeability
B. Inhibit cell wall synthesis
C. Inhibit bacterial DNA replication
D. Interfere with bacterial folic acid metabolism
E. Inhibit protein synthesis

Downloaded by John Erickson Bangayan (bangayan1903064@ceu.edu.ph)


lOMoARcPSD|19937673

Orthodontic Multiple Choice Question

Orthodontics (University of Medical Sciences, Ondo)

Studocu is not sponsored or endorsed by any college or university


Downloaded by John Erickson Bangayan (bangayan1903064@ceu.edu.ph)
lOMoARcPSD|19937673

Page 1 of 7
BDS FINAL PROFESSIONAL EXAMINATION 2007
ORTHONDONTICS
MODEL PAPER (MCQs)

Tot a l M a r k s: 4 5 Tim e Allow ed: 4 5 M inut es


Tot al N o. of M CQs: 4 5

1. Met hods of bit e opening includes:


a) Intrusion of posterior teeth and extrusion of anterior teeth.
b) Extrusion of posterior teeth and intrusion of anterior teeth.
c) Mesialisation of posterior teeth.
d) Retroclination of incisors.
e) Lower incisor extraction.
Ke y : b

2. Cort ical drift is a grow t h process involving:


a) Deposition of bone.
b) Resorption of bone.
c) Deposition and resorption.
d) Bone bending.
e) Apposition of bone.
Ke y : c

3. The foca l film dist a nce for a lat eral cephalogram is:
a) Four feet.
b) Three feet.
c) Five feet.
d) Six feet.
e) Two feet.
Ke y : c

4. M om ent is defined as:


a) Force x Distance from centre of rotation.
b) Force x Distance from centre of resistance.
c) Force x Range.
d) Force x Springback.
e) Force x Modulus of elasticity.
Ke y : a

5. M inim um anchorage is defined as:


a) 2/3 rd of extraction space is utilized by the movement of anchor
unit.
b) ½ of the extraction space is utilized by the movement of anchor
unit and the remaining ½ by the movement of the moving unit.
c) 1/3 rd of the extraction space is utilized by the movement of
anchor unit.
d) Absolutely no movement of anchor unit.
e) Absolutely no movement of anterior teeth.
Ke y : a

6. Classical pat t ern of ext ract ion in Class I I cam ouflage is:
a) Extraction of upper 1st premolars and lower canines.
b) Extraction of upper 1st premolars and lower 2 nd premolars.
c) Extraction of all 2 nd premolars.
d) Extraction of upper 2 nd premolars and lower 1st premolars.
e) Extraction of all 1 s t molars.
Ke y : b

7. Front al cephalogram is used t o:


a) Assess facial symmetry.
b) Overjet.
c) Dental compensation in sagital plane.
d) Deep bite.
e) Open bite.
Ke y : a

Downloaded by John Erickson Bangayan (bangayan1903064@ceu.edu.ph)


lOMoARcPSD|19937673

Page 2 of 7
BDS FINAL PROFESSIONAL EXAMINATION 2007
ORTHONDONTICS
MODEL PAPER (MCQs)

8. N at al t eet h are defined as:


a) Teeth present at the time of birth.
b) Teeth erupting in the 2 nd-3 rd month.
c) Teeth erupting between 6 months to 2 year.
d) Teeth erupting after 1 year of age.
e) Teeth erupting after 2 years of age.
Ke y : a

9. 1 st order bends are:


a) Tipping bends.
b) Toquing bends.
c) In and out bends.
d) Anchorage bends.
e) Up and down bends.
Ke y : c

10. Cleft lip and palat e pat ient oft en requires expansion. Appliance
of choice in such cases is:
a) Hyrax appliance.
b) Hass appliance.
c) Cap splint type of expansion appliance.
d) SARPE.
e) Spring jet.
Ke y : c

11. The m ost com m on sequence of erupt ion of perm a nent dent it ion
in upper arch is:
a) 6-1-2-3-4-5-7.
b) 6-1-2-4-3-5-7.
c) 6-1-2-5-4-3-7.
d) 6-1-3-2-4-5-7.
e) 6-2-3-4-5-1-7.
Ke y : a

12. Face m ask is prim arily used t o produce:


a) Dorsal effect.
b) AP effect.
c) Ventral effect.
d) Transverse effect.
e) Vertical effect.
Ke y : b

13. Relapse in rot at ed t eet h can be avoided:


a) By CSF.
b) By frenectomy.
c) By maintaining integrity of arch.
d) By extraction of retained teeth.
e) By scaling.
Ke y : a

14. Ext ract ion is m andat ory in t he t reat m ent of crow ding if
crow ding is:
a) Less than 4mm.
b) More than 4mm and less than 9mm.
c) More than 10mm.
d) More than 4mm but O/J more than 6mm.
e) More than 4mm with –ive O/J.
Ke y : c

Downloaded by John Erickson Bangayan (bangayan1903064@ceu.edu.ph)


lOMoARcPSD|19937673

Page 3 of 7
BDS FINAL PROFESSIONAL EXAMINATION 2007
ORTHONDONTICS
MODEL PAPER (MCQs)

15. Decom penstaion is do ne as part of:


a) Convetntional orthodontics.
b) Pre-surgical orthodontics.
c) Camouflage treatment.
d) Surgical camouflage.
e) Interceptive orthodontics.
Ke y : b

16. AN B a ngle is used t o a ssess:


a) Sagittal jaw discrepancy.
b) Vertical jaw discdrepancy.
c) Soft tissue profile.
d) Dental pattern.
e) Crowding.
Ke y : a

17. Ant erior Bolt an rat io is:


a) 91.3%.
b) 100%.
c) 61.7%.
d) 82.3%.
e) 77.2%.
Ke y : e

18. Flush Term inal Plane m eans:


a) Distal surface of upper ‘c’ flushes with distal surface of lower ‘c’.
b) Distal surface of upper ‘e’ flushes with distal surface of lo wer ‘e’.
c) Mesial surface of upper ‘e’ flushes with distal surface of lower
‘e’.
d) Distal surface of upper ‘e’ flushes with mesial surface of lower
‘e’.
e) Mesial surface of upper‘d’ flushes with medial surface of
lower‘d’.
Ke y : b

19. Sunday bit e is defined as:


a) Habitual forward posturing of mandible to Class I.
b) Posturing of mandible to Class III due to a premature contact.
c) Posturing the mandible laterally to establish maximum cuspation
in bilaterally narrow maxilla cases.
d) Posturing the mandible back to Class II, when actually in Class
I.
e) Under closure of jaw.
Ke y : a

20. Early loss of upper ‘e’ leads t o anchorage loss of upper 1 st


m olar as:
a) Mesial tipping, mesio -buccal rotation and distalisation.
b) Mesial tipping, mesio -palatal rotation and mesialization.
c) Distal tipping, mesio -palatal rotation and mesialization.
d) Distal tipping, mesio -palatal rotation and distalisation.
e) Mesial tipping, buccal flaring and mesialization.
Ke y : b

21. Ext ract ion pat t ern for Class I I surgical cases is:
a) Upper 4′s only.
b) Lower 5′s only.
c) Lower 4′s only.
d) Lower canine.
e) Upper 4′s and lower 5′s.
Ke y : c

Downloaded by John Erickson Bangayan (bangayan1903064@ceu.edu.ph)


lOMoARcPSD|19937673

Page 4 of 7
BDS FINAL PROFESSIONAL EXAMINATION 2007
ORTHONDONTICS
MODEL PAPER (MCQs)

22. Dent al com pensat ion in skelet al Class I I I cases is:


a) Proclined lower incisors only.
b) Retroclined lower incisors.
c) Proclined lower and retroclined upper incisors.
d) Retroclined lower and Proclined upper incisors.
e) Proclined lower and Proclined upper incisors.
Ke y : d

23. Opt im al force for Bodily m ovem ent is:


a) 50-75 gm force per tooth.
b) Less than 50 gm force per tooth.
c) 15-25 gm force per tooth.
d) 200-400 gm force per tooth.
e) 75-125 gm force per tooth.
Ke y : e

24. I n RPE act ivat ion is done:


a) Once weekly.
b) Twice weekly.
c) Twice daily.
d) Once monthly.
e) Twice monthly.
Ke y : c

25. M ost ect opically erupt ed t oot h is:


a) Upper 1s t molar.
b) Upper canine.
c) Lower 2nd molar.
d) Lower incisor.
e) Lower 1st premolar.
Ke y : a

26. Prognat hic profile is seen in pat ient s w ith:


a) Mandibular hypolasia.
b) Skeletal Class II malocclusion.
c) Skeletal Class III malocclusion.
d) Pier-Robbins syndrome.
e) Stickler’s syndrome.
Ke y : c

27. Unfavorable m ixed dent it ion analysis is an indicat ion for:


a) Space maintenance and space regaining.
b) Space maintenance and space supervision.
c) Space regaining and gross discrepancy cases.
d) Space supervision and gross discrepancy cases.
e) Space maintenance.
Ke y : d

28. Excess in low er ant erior Bolt on is an indicat ion for:


a) Build up in upper anteriors.
b) Stripping in lower molars.
c) Stripping in lower premolars.
d) Stripping in upper anteriors.
e) Stripping in lower incisors or lower incisor extraction.
Ke y : e

Downloaded by John Erickson Bangayan (bangayan1903064@ceu.edu.ph)


lOMoARcPSD|19937673

Page 5 of 7
BDS FINAL PROFESSIONAL EXAMINATION 2007
ORTHONDONTICS
MODEL PAPER (MCQs)

29. Absolut e anchorage:


a) Is provided by ankylosed teeth.
b) Is provided by implants.
c) Is also called Stationary Anchorage.
d) Means Zero Anchorage Loss.
e) Means Zero Anchorage Burn.
Ke y : d

30. Pseudo- Class I I I is defined as:


a) Habitual forward posturing of mandible to Class I.
b) Posturing of mandible to Class II due to a premature contact.
c) Posturing the mandible laterally to establish maximum cuspation
in bilaterally narrow maxilla cases.
d) Posturing the mandible back to Class II, when actually in Class
I.
e) Underclosure of mouth.
Ke y : b

31. Bit e of accom m odat ion m eans:


a) Habitual forward posturing of mandible to Class I.
b) Posturing of mandible to Class III due to a premature contact.
c) Posturing the mandible laterally to establish maximum cuspation
in bilaterally narrow maxilla cases.
d) Posturing the mandible back to Class II, when actually in Class
I.
e) Underclosure of mouth.
Ke y : c

32. Met hod of I nt ra- or a l a nchorage reinforcem ent is:


a) Headgear.
b) Face mask.
c) Chin cup.
d) Nance appliance.
e) Reverse Pull Headgear.
Ke y : d

33. Mesiodens is a supernum erary t oot h:


a) Between central incisor and lateral incisor.
b) Between central incisors.
c) Extra canine.
d) Extra pre-molar.
e) Extra-molar.
Ke y : b

34. Tissue borne RPE appliance is:


a) Hyrax appliance.
b) Hass appliance.
c) Quadhelix.
d) NiTi expander.
e) Spring jet.
Ke y : b

35. Aspirin is a t erat ogen for:


a) Cleft lip and palate.
b) Crouzen syndrome.
c) Apert’s syndrome.
d) Acromegaly.
e) Hemifacial microsomia.
Ke y : a

Downloaded by John Erickson Bangayan (bangayan1903064@ceu.edu.ph)


lOMoARcPSD|19937673

Page 6 of 7
BDS FINAL PROFESSIONAL EXAMINATION 2007
ORTHONDONTICS
MODEL PAPER (MCQs)

36. Pierre- Robb ins Syndrom e is charact erized by:


a) Large mandible and cleft palate.
b) Short maxilla and cleft palate.
c) Large maxilla and cleft palate.
d) Short mandible and cleft palate.
e) Short mid-facial region and cleft palate.
Ke y : d

37. M uscle involved in t ort icollis is:


a) Temporalis.
b) Lateral pterygoid.
c) Medial pterygoid.
d) Zygomaticus major.
e) Sternocleido-mastoid.
Ke y : e

38. Class I I elast ics are given from :


a) Upper canine to upper 1 s t molar.
b) Lower canine to lower 1 s t molar.
c) Upper canine to lower 1 s t molar.
d) Lower canine to upper 1s t molar.
e) Between upper and lower incisors.
Ke y : c

39. Co- Cr shift is norm al:


a) 1-2mm.
b) 2-3mm.
c) 3-4mm.
d) 3-5mm.
e) 4-5mm.
Ke y : a

40. Canine guided occlusion m eans:


a) Canine to canine contact on balancing side, no contact on
working side.
b) Contact on incisors.
c) Three point contact.
d) Canine to canine contact on working and balancing side.
e) Canine to canine contact on working side, no contact on
balancing side.
Ke y : a

41. RPE ( evidence of m id sa git a l sut ure brea k ) is a ssessed by:


a) OPG.
b) Lateral cephalogram.
c) Upper occlusal radiograph.
d) Lower occlusal.
e) Peri-apical.
Ke y : c

42. W ell aligned deciduous dent it ion is an indicat ion for:


a) Potential crowding.
b) Potential spacing.
c) Disto-occlusion.
d) Mesio-occlusion.
e) Well-aligned permanent teeth.
Ke y : a

Downloaded by John Erickson Bangayan (bangayan1903064@ceu.edu.ph)


lOMoARcPSD|19937673

Page 7 of 7
BDS FINAL PROFESSIONAL EXAMINATION 2007
ORTHONDONTICS
MODEL PAPER (MCQs)

43. Most com m only congenit ally m issing t oot h is:


a) Upper lateral incisors.
b) Upper 1s t premolar.
c) Upper central incisor.
d) Lower lateral incisor.
e) Lower canines.
Ke y : a

44. Est het ic bra ck et s a re:


a) Metal brackets.
b) Steel brackets.
c) Ceramic brackets.
d) NiTi brackets.
e) Co-Cr brackets.
Ke y : c

45. Ret ent ion in rem ovable appliances is obt ained by:
a) Z-spring.
b) Adams clasps.
c) Expansion screw.
d) Labial bow.
e) Retraction spring.
Ke y : b

Reference:
• M oyers Tex t Book .
• Cont em porary Ort hodont ics by W illiam Proffit .
• An int roduct ion t o ort hodont ics Laura M it chell.

Downloaded by John Erickson Bangayan (bangayan1903064@ceu.edu.ph)


lOMoARcPSD|19937673

Orthodontics AND Pediatric Revalida 2021

Dentistry (Centro Escolar University)

Studocu is not sponsored or endorsed by any college or university


Downloaded by John Erickson Bangayan (bangayan1903064@ceu.edu.ph)
lOMoARcPSD|19937673

REVALIDA
ORTHODONTICS AND PEDIATRIC ○ Crowding of anterior teeth
DENTISTRY ○ Spacing of anterior teeth
○ Labioversion of anterior teeth
1. Trident factor of oral habit which is now 9. Which of the following conditions is usually
considered as the most important present in a Class II, division 2
contributory factor to development of malocclusion?
malocclusion: ○ Open bite
○ Intensity ○ Mesiocclusion of permanent first
○ Frequency molars
○ Blank ○ Steep mandibular plane
○ duration ○ Lingual inclination of maxillary central
2. A linguoverted max. Premolar with respect incisors
to midsagittal plane is said to be in? 10. Increase in breadth means increase in
○ Protraction ○ Length
○ Retraction ○ Width
○ Attraction ○ Vertical dimension
○ Contraction ○ Depth
○ Abstraction ○ height
3. What are the growth dimensions in the 11. The action taken to preserve the integrity of
maxilla that are sex-linked? what appears to be normal occlusion at a
○ Height and depth specific time:
○ Height and width ○ Limited corrective orthodontics
○ Width and depth ○ Extensive corrective orthodontics
○ width ○ Preventive orthodontics
4. Generalized osteoclastic activity along the ○ Interceptive orthodontics
walls of the alveolar socket is the bone 12. A face that is broad and short is:
response to ○ Dolichofacial
○ Extrusion ○ Normofacial
○ Rotating force ○ Mesofacial
○ Elongating force ○ Brachyfacial
○ Depressing force 13. It is the synchondrosis whose activity stops
5. Late mesial shift is possible because of after age of 25.
○ Space differential bet. Deciduous ○ Sphenoethmoidal
and permanent teeth in the ○ Intersphenoidal
posterior segment ○ Spheno-occiptal
○ Primate spaces ○ Intraoccipital
○ None of the above 14. Anterior crossbite in the primary dentition
○ Terminal plane usually indicates a developing:
6. In adult patient with excessive overbite the ○ Class III malocclusion
lower facial height in relation to the upper ○ Class I malocclusion
and lower middle third is: ○ Class II malocclusion
○ Markedly short ○ Class IV malocclusion
○ Blank 15. Inclined plane should not be left in the
○ Markedly long mouth for more than 2 months to prevent
○ Normal creation of:
7. Orthodontic correction of which of the ○ Posterior cross bite
following is most easily retained? ○ Anterior open bite
○ Posterior crossbite ○ Anterior cross bite
○ Diastema ○ Posterior open bite
○ Expansion 16. Continued apposition of alveolar bone on
○ Rotation the free borders of the alveolar process as
○ Anterior crossbite the teeth erupt increases the __ of the
8. Crowding of permanent posterior teeth may maxilla.
be due to: ○ Blank
○ Mesial drifting of permanent first ○ Width
molar ○ Length
1

Downloaded by John Erickson Bangayan (bangayan1903064@ceu.edu.ph)


lOMoARcPSD|19937673

REVALIDA
○ Height 25. The major criterion to differentiate between
17. Which of the following cannot be assessed a true class III and a pseudo- class III
from cephalometric radiographs? malocclusion is:
○ Mandibular retrusion ○ Degree of anterior crossbite
○ Incisor inclination ○ Occlusal relationship maxillary and
○ Adequacy of dental arch perimeter mandibular first molar
○ Direction of mandibular growth ○ Existence of a forward shift of the
18. Cases of maxillary protrusion in a growing mandible during closure
child can be treated with ○ Presence of bilateral crossbite
○ Fixed braces only 26. A supra erupted mand. Canine with respect
○ Chin cap to Frankfort horizontal plane is said to be in?
○ Face mask ○ Attraction
○ Headgear ○ Protraction
19. The correct angulation of the inclined plane ○ Retraction
in relation to the tooth in crossbite is: ○ Contraction
○ 35° ○ Abstraction
○ 45° 27. Orthodontic correction of which of the
○ 55° following is most easily retained?
○ 15° ○ Expansion
○ 25° ○ Posterior crossbite
20. At age 4-5, what normal sign of primary ○ Anterior crossbite
dentition augers well for the erupting ○ Diastema
permanent incisors in terms of space ○ Rotation
availability: 28. The maxilla is displaced downward and
○ Class I cuspid relationship forward by growth in ___ parts of the bone:
○ Growth spaces, interdental spaces ○ Posterior and anterior
○ Flush terminal plane ○ Anterior and superior
○ Upright vertical incisor relationship ○ Anterior and inferior
21. The arrangement of 2 forces of equal ○ Posterior and superior
magnitude and opposite but non collinear 29. Interstitial growth is observed at which of
lines of action: the following sites?
○ Center of rotation 1. Spheno-occipital synchondrosis
○ Center of resistance 2. Maxillary tuberosity
○ Moment 3. Mandibular condyle
○ Couple 4. Zygomaticomaxillary suture
22. This will decrease lower arch perimeter 5. Apex of an erupting premolar
during transitional period: ○ 2&4
○ Distal tipping of lower cuspid ○ 2&5
○ Late mesial shift of first permanent ○ 1&2
molar ○ 1&4
○ Labial position of permanent incisors ○ 1&3
○ Blank 30. When a simple tipping force is applied to
23. Frankfort-horizontal is a reference plane the crown of a single-rooted tooth, the
constructed by joining which of the center of rotation is usually located:
following landmarks? ○ Two- third the root length from the
○ Porion and orbital apex
○ Porion and sella ○ 5mm beyond the apex
○ Porion and nasion ○ At the apex
○ Nasion and sella ○ One-third the root length from the
24. The following are factors that will apex
compensate incisor liability, except: ○ At the cervical line
○ Intercanine width growth 31. The sutures of the nasomaxillary complex
○ Favorable tooth size ratio are oblique and more or less parallel with
○ Upright position of primary incisors each other thus, growth in these areas
○ Labial positioning of permanent would serve to displace the maxilla:
incisors ○ Downward and forward
2

Downloaded by John Erickson Bangayan (bangayan1903064@ceu.edu.ph)


lOMoARcPSD|19937673

REVALIDA
○ Upward and backward ○ By environmental factors
○ Forward and upwards ○ Two of the choices
○ downward ○ By muscles
32. The “V” principle of growth is best 41. This aims to recognize and eliminate
illustrated by the: potential irregularities and malpositions in
○ Nasal septum the developing craniofacial complex:
○ Mandibular ramus ○ Extensive corrective orthodontics
○ Mandibular symphysis ○ Preventive Orthodontics
○ Spheno-occiptal synchondrosis ○ Limited Corrective orthodontics
33. Form birth to 5 years, the dominance of ○ Interceptive Orthodontics
growth is on the 42. The appliance to be used in correcting the
○ nasomaxillary structures anterior crossbite is determined by the:
○ Mandible ○ Amount of overbite
○ Cranial structures ○ All of these
○ Muscular structures ○ Age of the patient
○ Muscular components of the body ○ Cooperation of the patient
34. It is considered to be the best space 43. Chin cap Is used to redirect the growth of
maintainer the
○ A well restored deciduous teeth ○ Cranial base
○ Functional space maintainer ○ Mandible
○ Crown and crib ○ Maxilla
○ Fixed space maintainer ○ Cranial case
35. The profile of the patient with protruded 44. The arrangement of 2 forces of equal
mandible and retruded maxilla is: magnitude and opposite but non collinear
○ Convex lines of action:
○ Straight ○ Moment
○ Concave ○ Couple
○ Blank ○ Center of resistance
36. The best appliance for 7 years old child ○ Center of rotation
with Class 1 type 3 (inlocked maxillary 45. A supra erupted max. Canine with respect
central incisor) is to Frankfurt Horizontal plane is said to be
○ Cross bite elastics in?
○ Myofunctional appliance ○ Attraction
○ Mandibular acrylic inclined plane ○ Retraction
○ Band and crib ○ Abstraction
37. Proximal caries, if not restored, will __ the ○ Protraction
arch length ○ Contraction
○ Blank 46. Two or more teeth moving in opposite
○ Not affect directions and pitted against each other by
○ Decrease the appliance. Usually, the resistance to
○ increase each other is equal and opposite. The
38. Effect of abnormal eruptive path anchorage is:
○ Delayed eruption ○ Reciprocal
○ Elongation of adjacent teeth ○ Stationary
○ Space loss ○ Simple
○ Increase in arch length ○ extraoral
○ Deflection of eruption 47. Crowding of permanent posterior teeth may
39. Nasomaxillary complex is hafled to the be due to:
cranium by the following sutures except: ○ Crowding of anterior teeth
○ Zygomaticotemporal ○ Spacing of anterior teeth
○ Frontotemporal ○ Mesial drifting of permanent first
○ Pterygopalatine molar
○ Zygomaticomaxillary ○ Labioversion of anterior teeth
○ Frontomaxillary 48. Effect of abnormal eruptive path
40. The basic form of the arch is determined: ○ Space loss
○ In intrauterine life ○ Increase. In arch length
3

Downloaded by John Erickson Bangayan (bangayan1903064@ceu.edu.ph)


lOMoARcPSD|19937673

REVALIDA
○ Deflection of eruption 57. A Distal shoe space maintainer is indicated
○ Elongation of adjacent teeth when a primary:
○ Delayed eruption ○ Second molar is lost before
49. The order from the greatest to least change eruption of a permanent first molar
of the dimension of the cranium: ○ Second molar is lost after eruption of
○ Width, height, depth a permanent first molar
○ Depth, height, width ○ First molar is prematurely lost
○ Height, depth, width ○ Incisor is avulsed
○ Depth, width, height 58. Gingival margin of a tooth that will receive a
50. he order from the least to greatest change stainless stress crown should have a ___
of the dimension of the cranium: finish line
○ Width, height, depth ○ Blank
○ Height, depth, width ○ Shoulder
○ Depth, width, height ○ BOTH
○ Depth, height, width ○ Feather edge
51. Primary second molar usually erupt during 59. Poor personal experiences in the dental
ages: office:
○ 14-20 months ○ Hostile parents
○ 30-36 months ○ Manipulative parent
○ 20-30 months ○ Overprotective parent
○ 36-48 months ○ Neglectful parent
○ 8-14 months 60. Reluctant to accept treatment:
52. The purpose of pulp treatment and root ○ Positive
canal filling is to: ○ Negative
○ Enlarge the root canal ○ Definitely negative
○ Remove pulpal tissue ○ Definitely positive
○ Blank 61. In acute ingestion of fluoride, the following
○ Prolong the usefulness of a tooth to can be given to the patient to counteract its
function in mastication effect, Except:
53. Lack of anatomic detail of dental restoration ○ Milk of magnesia
procedures: ○ Alum
○ Elongation of opposing teeth ○ 2 tsp of ipecac syrup
○ Elongation of adjacent teeth ○ milk
○ Blank 62. The least desirable method used in child
○ Increased arch length management is:
54. A child in the “ugly ducking” stage is ○ Tell-show-do technique
characterized by ○ Hand-over-mouth technique
○ Absence of maxillary incisors ○ Voice control
○ A flat nasal bone ○ Gift before treatment
○ Presence of only one maxillary central 63. Uses the principles of learning theory:
incisor ○ Blank
○ Diastema between 2 central ○ Behavior management
incisors ○ Behavior shaping
55. Induce formation of apical closure of young ○ Behavior modification
permanent molar using CAOH 64. The maximum number of cartridges of 2%
○ Apoxogenesis lidocaine with 1:100,000 epinephrine for a
○ Apexification 40 pound child patient is:
○ Apeximation ○ 9
○ Apex formation ○ 5
56. The following irrigating solution can be used ○ 7
as irrigant in pulpectomy procedure except: ○ 3
○ Distilled water ○ 10
○ Sodium hypochlorite and water 65. The prognosis of a cervical third root
solution fracture:
○ Normal saline solution ○ Is favorable
○ Anesthetic solution
4

Downloaded by John Erickson Bangayan (bangayan1903064@ceu.edu.ph)


lOMoARcPSD|19937673

REVALIDA
○ Depends on whether tooth is 73. A mechanical aid for keeping the mouth
discolored open and operates on a reverse scissors
○ Is not favorable action:
○ Blank ○ Molt mouth prop
66. Transillumination of soft tissue is useful in ○ Mc kesson bite block
detecting which of the following problem in ○ Padded and wrapped tongue blade
a child? ○ Rubber bite block
○ Koplick’s spot ○ Finger guard
○ Sialolithiasis 74. Stabilization period for teeth with fractured
○ Sickle cell disease roots:
○ Abnormal frenum attachment ○ 2-3 days
○ Aortic stenosis ○ 7-14 days
67. The most important predisposing factor to ○ 2-3 months
injury of the anterior teeth is: ○ 2-3 weeks
○ Protruding anterior teeth 75. Quadrant used in the gluteal region when
○ Blank intramuscular sedation is used:
○ Crossbite of anterior teeth ○ Lower inner quadrant
○ Rotated anterior teeth ○ Upper inner quadrant
68. In young child, paradoxical excitement ○ Lower outer quadrant
occurs most frequently following ○ Upper outer quadrant
premedication with: 76. Sudden and firm commands used to get the
○ An amphetamine child’s attention:
○ Nitrous oxide ○ Tell, show and do
○ A barbiturate ○ HOME
○ A narcotic ○ Hypnodontics
69. Type of fear that is based on the feelings ○ Voice control
and attitude that have been suggested to 77. pulpotomy with formocresol of primary
the child by other without having had the tooth:
experience: ○ Result in higher success rate
○ Blank compared to calcium hydroxide
○ BOTH ○ Induces formation of a dentin bridge
○ Objective fear at site of amputation
○ Subjective fear ○ Should never be used
70. Indicated restoration on a primary anterior ○ Result in lower success rate
tooth with small mesial and distal caries and compared to calcium hydroxide
a cervical caries but without pulp 78. In pedodontic triangle the apex of the
involvement triangle is
○ Blank ○ Assistant
○ Strip-off crown ○ Child
○ Stainless steel crown ○ Parent
○ Composite resin ○ dentist
71. The most important predisposing factor to 79. The prognosis of a cervical third root
injury of the anterior teeth is: fracture:
○ Protruding anterior teeth ○ Is favorable
○ Rotated anterior teeth ○ Is not favorable
○ crossbite of anterior teeth ○ Depends on whether tooth is
○ blank discolored
72. An incorrigible 4 year old child who keeps ○ Blank
on kicking and throwing objects in the clinic 80. The dental procedure that produces the
can be manage by: greatest negative response in children:
○ General Anesthesia ○ All of these
○ Tell-show-do technique ○ Injection
○ All of these ○ Extraction
○ Physical restraint and conscious ○ Cavity preparation
sedation

Downloaded by John Erickson Bangayan (bangayan1903064@ceu.edu.ph)


lOMoARcPSD|19937673

REVALIDA
81. A mechanical aid for keeping the mouth 88. Radiographs of a 5 year old patient show
open and operates on a reverse scissor’s permanent maxillary first molars inclined
action: mesially with resulting resorption of the
○ Padded and wrapped tongue blade distal portions of the roots of primary
○ Finger guard second molars. The condition described is:
○ Rubber bite block ○ Premature eruption
○ Mc Kesson bite block ○ Intrafollicular resorption
○ Molt mouth prop ○ Internal resorption
82. They have minimal apprehensions and are ○ Ectopic eruption
reasonably relaxed: ○ Ankylosis
○ Defiant 89. In acute ingestion of fluoride, the following
○ Timid can be given to the patient to counteract its
○ Cooperative effect, except:
○ enthusiastic ○ 2 tsp of ipecac syrup
83. Dental fluorosis is most likely to occur on a : ○ Milk
○ None of the above ○ Alum
○ Two year old ingesting 1ppm of ○ Milk of magnesia
fluoride 90. Fluoride reduces the incidence of carious
○ On a 3 year old child ingesting lesions on the
0.50ppm of fluoride ○ Smooth surface
○ 17 year old patient ingesting 1ppm ○ Pit and fissure
of fluoride ○ Lingual surfaces of teeth
84. In managing abnormal oral habits, therapy ○ Occlusal surfaces
must be: 91. Type of fear which is produced by direct
○ Mechanical physical stimulation of sense organ
○ A means of punishment ○ Objective fear
○ Of physical control ○ Subjective fear
○ Of conditioning responses ○ Associative fear
85. The first dental appointment of a child ○ Acquired fear
patient should be: 92. In infant oral care, clean mouth with gauze
○ 6 years old after feedings and at bed time done during:
○ Blank ○ 0-6 mons
○ 2 1/2 to 3 years old ○ 24-36 mons
○ Before 1 year old ○ 12-24 mons
86. Pulpotomy with formocresol of primary: ○ 6-12 mons
○ Result in lower success rate 93. Which of the developmental space will
compered to calcium hydroxide cause a decrease in arch perimeter when
○ None of the above pre-empted?
○ Result in higher success rate ○ Primate space
compared to calcium hydroxide ○ Interocclusal space
○ Induces formation of a dentin bridge ○ Interdental space
at site of amputation ○ Leeway space
○ Should never be used 94. When comparing cementum to bone in their
87. How will extraction of primary maxillary responses to orthodontic forces, cementum
central incisor in a 5 year old child with resorbs:
incisal spacing affect the size of the ○ Less readily
intercanine space? ○ Not at all
○ The intercanine space will increase in ○ Under lighter loads
size 95. In inferior nerve block for a child patient, the
○ Blank injection must be made ___ than for an
○ No charge will occur in the size of adult patient.
the intercanine space ○ Slightly higher
○ The intercanine space will decrease in ○ Slightly lower and more posteriorly
size ○ Blank
○ The intercanine space will decrease in ○ In level
size
6

Downloaded by John Erickson Bangayan (bangayan1903064@ceu.edu.ph)


lOMoARcPSD|19937673

REVALIDA
96. The treatment option for patients who have ○ Result in necrosis of pulp
sustained extensive orofacial & or dental 99. To avoid vomiting and complications during
trauma treatment with sedation, no milk or solid
○ Conscious sedation food should be taken ___ before the
○ Nitrous oxide & oxygen inhalation scheduled procedure.
○ General anesthesia ○ After midnight
○ No treatment ○ 6 hrs
97. Device that assess the arterial hemoglobin ○ 8 hrs
oxygen saturation and pulse rate: ○ 4 hrs
○ Pretracheal stethoscope 100.In treating a pediatric patient, one must
○ Pulse oximeter establish good communication with the
○ Automated vital sign monitor ○ Both parent and child
○ none ○ Parents only, because they are the
98. Pulpotomy with calcium hydroxide in the ones who will pay the bill
primary teeth: ○ The caretaker or the Yaya who takes
○ Blank cares of the child
○ Result in acute inflammation of ○ Child only, since he is the one to be
pulp treated
○ May form calcific barrier on the pulp
stump

Downloaded by John Erickson Bangayan (bangayan1903064@ceu.edu.ph)


lOMoARcPSD|19937673

Orthodontics-and-Pediatrics

Dentistry (Centro Escolar University)

Studocu is not sponsored or endorsed by any college or university


Downloaded by John Erickson Bangayan (bangayan1903064@ceu.edu.ph)
lOMoARcPSD|19937673

CLUSTER VII: ORTHODONTICS AND PEDIATRIC DENTISTRY

Ortho Pedo

1. Which of the ff is the most essential factor related to correction of an anterior crossbite?
a) Shape of the tooth involved
b) Age of the patient
c) Depth of the crossbite
d) Space available mesiodistally
2. Appliance for correcting Class 1 deep bite in a growing child.
a) Inclined plane
b) Tongue depressor
c) Band and crib
d) Maxillary bite planes
3. A supra erupted max. canine with respect to Frankfurt Horizontal Plane is said to be in?
a) Retraction
b) Contraction
c) Abstraction
d) Protraction
e) Attraction
4. The ff are sources of extraoral anchorage except:
a) Facial
b) Cranial
c) Cervical
d) Occipital
e) Mucosa
5. Early loss of upper D will end up to what type of terminal plane?
a) Any of the above
b) Mesial step
c) Distal step
d) Flush terminal plane
6. The most common malocclusion in the mixed dentition period is:
a) Anterior open bite
b) Posterior open bite
c) Crowding
d) Class I type 2
e) Class II Div. 1
7. Cranial vault grows primarily by cartilage growth at:
a) Synchondrosis
b) Sutures
c) Fontanelles
d) Disappears at birth
8. It is the growth center of the mandible which is responsible for the major increase in length
a) Ramus of the mandible
b) Alveolar process
c) Condyle
d) Lingual tuberosity
9. The ff is sequelae of untreated crossbite, except:
a) Abrasion
b) Faceting on the labial surface of the tooth in crossbite
c) Periodontal involvement
d) Faceting of the labial surface of the tooth opposing the inlocked tooth
10. The basic form of the arch is determined:
a) By muscles
b) In intrauterine life
c) By environmental factors
d) Two of the choices
11. A mixed dentition analysis determines:
a) Size of permanent incisors
b) Space available versus space required
c) Discrepancies in jaw size
d) Skeletal growth pattern
e) Intercanine width
12. Bone ossification can be detected with the use of:
a) BMR
b) Hand and wrist x-ray
c) Electromyographic exam
d) Biostatic
13. The ff are sources of extraoral anchorage except:
a) Cervical
b) Cranial
c) Mucosa
d) Occipital
e) Facial
14. From birth to 5 years, the dominance of growth is on the
a) Nasomaxillary structures
b) Cranial structures
c) Muscular components of the body
d) Mandible
15. Which of the ff is not a space maintainer?
a) Palatal expansion appliance
b) Class III restoration
c) Nance holding arch
d) Lingual Arch
e) Stainless steel crown
16. Which of the ff orthodontic movements of the teeth are most difficult to accomplish?
1. Tipping Rotation
2. Intrusion extrusion
3. Translation
4. Rotation

Downloaded by John Erickson Bangayan (bangayan1903064@ceu.edu.ph)


lOMoARcPSD|19937673

CLUSTER VII: ORTHODONTICS AND PEDIATRIC DENTISTRY

5. Extrusion
a) 3 and 5
b) 1 and 2
c) 1 and 3
d) 3 and 4
e) 1 and 5
17. Upper arch width increases significantly more than that of lower arch due to
a) Distal tipping of cuspid
b) Labial positioning of permanent incisors
c) Vertical alveolar growth
d) Diverging alveolar growth
18. Which of the ff conditions is usually present in a Class II, Division 2 malocclusion?
a) Mesiocclusion of permanent frist molars
b) Lingual inclination of maxillary central incisors
c) Open bite
d) Steep mandibular plane
19. Orthodontic correction of which of the ff is most easily retained?
a) Diastema
b) Posterior crossbite
c) Expansion
d) Anterior crossbite
e) Rotation
20. At age 4-5, what normal sign of primary dentition augers well for the erupting permanent incisors in terms of space availability:
a) Growth spaces, interdental spaces
b) Flush terminal plane
c) Upright vertical incisor relationship
d) Class 1 cuspid relationship
21. Interstitial growth is observed at which of the ff sites?
1. Spheno-occipital synchondrosis
2. Maxillary tuberosity
3. Mandibular condyle
4. Zygomaticomaxillary suture
5. Apex of an erupting premolar
a) 1 and 2
b) 2 and 4
c) 1 and 4
d) 2 and 5
e) 1 and 3
22. As arch perimeter increase, arch length?
a) Increases
b) Decreases
c) No change
23. It is the synchondrosis whose activity stops at age of 25.
a) Intersphenoidal
b) Intraoccipital
c) Sphenoethmoidal
d) Spheno-occipital
24. The V principle of growth is found in the ff structures of the skull except
a) Alveolar process
b) Mandible
c) Palate
d) Orbits of the eyes
25. Lack of anatomic detail of dental restoration produces:
a) Increased arch length
b) Elongation of opposing teeth
c) Elongation of adjacent teeth
26. This will decrease lower arch perimeter during transitional period:
a) Late mesial shift of first permanent molar
b) Distal tipping of lower cuspid
c) Labial position of permanent incisor
27. It is the arch dimension that is occupied by all the succedaneous teeth
a) Arch length
b) Intercanine length
c) Arch perimeter
d) Arch width
28. The relationship of the upper and lower gum pads is such that
a) The upper is within the lower
b) The upper is posterior to the lower
c) The lower is anterior to the upper
d) It presents a convex profile
29. The ff are factors that will compensate incisor liability, except:
a) Upright position of primary incisors
b) Favorable tooth size ratio
c) Intercanine wdth growth
d) Labial positioning of permanent incisors
30. The space that allows an increase in the mandibular intercanine width is
a) Leeway space
b) Interdental space
c) Anterior intermaxillary space
d) Primate space
31. The arrangement of 2 forces of equal magnitude and opposite but noncollinear lines of action:
a) Moment 2
b) Couple
c) Center of resistance
d) Center of rotation
32. What are the growth dimensions in the maxilla that are sex-linked?
a) Width
b) Height and depth

Downloaded by John Erickson Bangayan (bangayan1903064@ceu.edu.ph)


lOMoARcPSD|19937673

CLUSTER VII: ORTHODONTICS AND PEDIATRIC DENTISTRY

c) Height and width


d) Width and depth
33. The ff are vertical planes, except:
a) Palatal
b) Facial plane
c) V-axis
d) Axis of 1
34. The correct angulation of the inclined plane in relation to the toot in crossbite is:
a) 55 degree
b) 45 degree
c) 25 degree
d) 35 degree
e) 15 degree
35. Orthodontic correction of which of the ff is most easily retained?
a) Anterior crossbite
b) Posterior crossbite
c) Diastema
d) Expansion
e) Rotation
36. The ff are sequelae of untreated crossbite, except:
a) Abrasion
b) Faceting on the labial surface of the tooth in crossbite
c) Periodontal involvement
d) Faceting of the labial surface of the tooth opposing the inlocked tooth
37. A supra erupted mandibular canine with respect to Frankfurt Horizontal plane is said to be in?
a) Retraction
b) Attraction
c) Contraction
d) Protraction
e) Abstraction
38. Anterior crossbite in the primary dentition usually indicates a developing:
a) Class IV malocclusion
b) Class I malocclusion
c) Class II malocclusion
d) Class III malocclusion
39. Two or more teeth moving in opposite directions and pitted against each other by the appliance. Usually, the resistance to each
other is equal and opposite. The anchorage is:
a) Simple
b) Reciprocal
c) Extraoral
d) Stationary
40. This will decrease low arch perimeter during transitional period:
a) Late mesial shift of first permanent molar
b) Labial position of permanent incisor
c) Distal tipping of lower cuspid
41. The major criterion to differentiate between a true Class III and a pseudo-Class III malocclusion is:
a) Presence of a bilateral crossbite
b) Occlusal relationship between maxillary and mandibular first molars
c) Existence of a forward shift of the mandible during closure
d) Degree of anterior crossbite
42. Two or more teeth moving in opposite directions and pitted against each other by the appliance. Usually, the resistance to each
other is equal and opposite. The anchorage is:
a) Stationary
b) Simple
c) Extraoral
d) Reciprocal
43. Which of the ff cannot be assessed from cephalometric radiographs?
a) Incisor inclination
b) Mandibular retrusion
c) Direction of mandibular growth
d) Adequacy of dental arch perimeter
44. Frankfort-horizontal if a reference plane constructed by joining which of the ff landmarks?
a) Porion and nasion
b) Porion and orbitale
c) Porion and sella
d) Nasion and sella
45. The appliance to be used in correcting the anterior crossbite is determined by the:
a) Cooperation of the patient
b) Age of the patient
c) All of these
d) Amount of overbite
46. If a 7-year old patient loses a primary mandibular canine about the same time the adjacent lateral incisor is erupting or shortly
thereafter, the dentist should be alert to the possibility of:
a) A developing crossbite
b) Ana early eruption of the permanent canine
c) A tongue habit
d) Lingual collapse of mandibular anterior teeth
47. In deep bite the postural vertical dimension is/has:
a) No relations with occlusal vertical dimension
b) Not in harmony with occlusal vertical dimension
c) In harmony with occlusal vertical dimension
48. Deep bite cases in young children can be3treated with
a) Space maintainer
b) Bite plane
c) Fixed braces
d) Extraoral force appliance
49. If a 7 year old patient loses a primary mandibular canine about the same time the adjacent lateral lateral incisor is erupting or
shortly thereafter, the dentist should be alert to the possibility of:

Downloaded by John Erickson Bangayan (bangayan1903064@ceu.edu.ph)


lOMoARcPSD|19937673

CLUSTER VII: ORTHODONTICS AND PEDIATRIC DENTISTRY

a) A developing crossbite
b) A tongue habit
c) An early eruption of the permanent canine
d) Lingual collapse of mandibular anterior teeth
50. Cases of maxillary protrusion in a growing child can be treated with
a) Headgear
b) Chin cap
c) Face mask
d) Fixed braces only
51. Treatment for crown fracture involving the pulp of a primary incisor:
a) DPC
b) Extraction
c) Formocresol pulpotomy
d) Pulpectomy
52. The least desirable method used in child management is:
a) Voice control
b) Hand-over-mouth technique
c) Tell-show do technique
d) Gift before treatment
53. The first dental appointment of a child patient should be:
a) Before 1 year old
b) 6 years old
c) 2 ½ to 3 years old
54. Device that assess the arterial haemoglobin oxygen saturation and pulse rate
a) Pretracheal stethoscope
b) Pulse oximeter
c) Automated vital sign monitor
d) None
55. Anodontia, diagnosed in a 5 year old child, primarily affects the growth of the:
a) Maxilla and mandible
b) Maxilla
c) Midface
d) Alveolar bone
e) Mandible
56. A child in long-term remission of acute leukemia has dental problems characterized susceptibility to:
a) Development of jaw deformities
b) Periodontal bone loss
c) Dental caries
d) Oral infection
57. In acute ingestion of fluoride, the ff can be given to the patient to counteract it effects, except:
a) Milk of magnesia
b) Milk
c) Alum
d) 2 tsp of ipecac syrup
58. Treatment for crown fracture involving the pulp of a primary incisor:
a) Extraction
b) Pulpectomy
c) DPC
d) Formocresol pulpotomy
59. The least desirable method used in child management is:
a) Hand-over-mouth technique
b) Voice control
c) Gift before treatment
d) Tell-show do technique
60. Mandibular teeth are best anesthetized with:
a) Infraorbital injection
b) An inferior alveolar and long buccal injection
c) Mental injection
61. A distal shoe space maintainer is indicated when a primary:
a) Second molar is lost before eruption of a permanent first molar
b) Second molar is lost after eruption of a permanent first molar
c) First molar is prematurely lost
62. The dental procedure that produces the greatest negative response in children:
a) Extraction
b) All of these
c) Injection
d) Cavity preparation
63. Periapical lesions, pulpitis an pulpotomy of a primary molar will ___ the eruption of successor premolar.
a) Delay
b) Have no effect on
c) Hasten
64. Child is 11 years old. Tooth #55 has been exfoliated but tooth #65 is still ver intact. What must be done?
a) Retain 65 and place a band and loop for 55
b) None, wait for 65 to exfoliate
c) Extract 65 at once
d) Do percussion test before extracting 65
65. The purpose of pulp treatment and root canal filling is to:
a) Prolong the usefulness of a tooth to function in mastication
b) Enlarge the root canal
c) Remove pulpal tissue
66. Transillumination of soft tissues is useful in detecting which of the ff problems in a child?
a) Abnormal frenum attachment 4
b) Sialolithiasis
c) Koplick’s spot
d) Aortic stenosis
e) Sickle cell disease
67. Mandible comes from ____ branchial arch
a) First

Downloaded by John Erickson Bangayan (bangayan1903064@ceu.edu.ph)


lOMoARcPSD|19937673

CLUSTER VII: ORTHODONTICS AND PEDIATRIC DENTISTRY

b) Third
c) Fourth
d) Second
68. Gingival margin of a tooth that will receive a stainless steel crown should have a ___ finish line:
a) Shoulder
b) BOTH
c) Feather edge
69. Radiographs of a 5 year old patient show permanent maxillary first molars inclined mesially with resulting resorption of the distal
portions of the roots of primary second molars. The condition described is:
a) Ankylosis
b) Intrafollicular resorption
c) Premature eruption
d) Internal resorption
e) Ectopic eruption
70. In managing abnormal oral habits, therapy must be:
a) A means of punishment
b) Of physical control
c) Of conditioning responses
d) Mechanical
71. The stage at which a tooth begins its eruptive movement:
a) Stage 0
b) Stage 3
c) Stage 6
d) Stage 2
72. The purpose of pulp treatment and root canal filling is to:
a) Prolong the usefulness of a tooth to function in mastication
b) Removal pulpal tissue
c) Enlarge the root canal
73. Induce formation of apical closure of young permanent molar using CAOH
a) Apexification
b) Apeximation
c) Apex formation
d) Apexogenesis
74. Which of the ff dental sequelae is likely In a child with a history of generalized growth failure (failure to thrive) in the first six
months of life?
a) Retrusive mandible
b) Small permanent teeth
c) Retrusive maxilla
d) Dentinogenesis imperfect
e) Enamel hypoplasia
75. Grinding of the sharp edges is sufficient
a) Enamel fracture
b) Lateral luxation
c) Extrusion
d) Dentin fracture
76. The dental procedure that produces the greatest negative response in children:
a) Cavity preparation
b) Injection
c) Extraction
d) All of these
77. To avoid ingestion of lethal dose of fluoride, it is safe dispense up to ___ of fluoride
a) 700 mg
b) 500 mg
c) 300 mg
d) 120 mg
78. Mechanical aid for keeping the mouth open and operates on a reserve scissors action:
a) Padded and wrapped tongue blade
b) Molt mouth prop
c) Finger guard
d) Mc Kesson bite block
e) Rubber bite block
79. Supervision of a child’s development of occlusion is most critical at ages:
a) 3-6 years
b) 7-10 years
c) 14-17 years
d) 11-14 years
80. Poor personal experience in the dental office:
a) Manipulative parents
b) Overprotective parents
c) Neglectful parents
d) Hostile parents
81. Indicated restoration on a primary anterior tooth with mesial and distal caries and a cervical caries but without pulp
involvement:
a) Strip-off crown
b) Composite resin
c) Stainless steel crown
82. Device that assess the arterial haemoglobin oxygen saturation and pulse rate
a) Pulse oximeter
b) Pretracheal stethoscope
c) None
d) Automated vita sign monitor
83. Type of fear which is produced by direct 5 physical stimulation of sense organ
a) Associative fear
b) Acquired fear
c) Objective fear
d) Subjective fear
84. Pulpotomy with formocresol of primary tooth:
a) Result in higher success rate compared to calcium hydroxide

Downloaded by John Erickson Bangayan (bangayan1903064@ceu.edu.ph)


lOMoARcPSD|19937673

CLUSTER VII: ORTHODONTICS AND PEDIATRIC DENTISTRY

b) None of the above


c) Should never be used
d) Induces formation of a dentin bridge at site of amputation
e) Result in lower success rate compared to calcium hydroxide
85. Stabilization period for teeth with fractured roots:
a) 2-3 weeks
b) 2-3 months
c) 7-14 days
d) 2-3 days
86. In inferior alveolar nerve blocked, the barrel of the syringe should be directed on the plane:
a) Between R and 6
b) On top of D
c) Between C and D
d) Between D and E
87. The type of fear that is based on the feelings and attitude that have been suggested to the child by others without having had
the experience
a) Objective fear
b) Subjective fear
c) BOTH
88. The most important predisposing factor to injury of anterior teeth is:
a) Crossbite of anterior teeth
b) Rotated anterior teeth
c) Protruding anterior teeth
89. Quadrant used in the glutal region when intramuscular sedation is used:
a) Lower inner quadrant
b) Upper outer quadrant
c) Upper inner quadrant
d) Lower outer quadrant
90. If a primary tooth has been extracted before succedaneous tooth has begun eruptive movement, its eruption will be ____
a) Hastened
b) Not affected
c) Delayed
91. Choice of nursing nipple to prevent oral habit formation:
a) Nonphysiologic nipple
b) Evenflo
c) Nuk Sauger nipple
d) Mimiflo
92. A 4 year old child visited a dentist for the first time and received prophylaxis quite well. Before he left, the dentist was glad that
he gave the child a toy. This kind of gesture is called:
a) Positive reinforcement
b) Classical conditioning
c) Bribery
d) Public relations
e) None of the above
93. The prognosis of a cervical third root fracture:
a) Depends on whether tooth is discoloured
b) Is favourable
c) Is not favourable
94. Uses the principles of learning theory:
a) Behaviour shaping
b) No answer text provided
c) Behaviour management
d) Behaviour modification
95. Type of fear which is produced by direct physical stimulation of sense organ
a) Associative fear
b) Subjective fear
c) Acquired fear
d) Objective fear
96. A light bluish, dome –shaped lesion on the inside lip of a 2 year old child is most likely a:
a) Melanoma
b) Hemangioma
c) Sucking callous
d) Hematoma
e) Mucocele
97. The ff irrigating solution can be used as irrigant in pulpectomy procedure except:
a) Anesthetic solution
b) Sodium hypochlorite and water solution
c) Distilled water
d) Normal saline solution
98. Transillumination of soft tissue is useful in detecting which of the ff problems in a child?
a) Koplick’s spot
b) Aortic stenosis
c) Sickle cell disease
d) Sialolithiasis
e) Abnormal frenum attachment
99. The least desirable method used in child management is:
a) Gift before treatment
b) Voice control
c) Hand-over mouth technique
d) Tell-show do technique

A. Aracon
  
B. Abstracon 
C. Protracon
D. Retracon

Downloaded by John Erickson Bangayan (bangayan1903064@ceu.edu.ph)


lOMoARcPSD|19937673

CLUSTER VII: ORTHODONTICS AND PEDIATRIC DENTISTRY

E. Contracon 
A. Aracon
  
B. Abstracon 
C. Protracon 
D. Retracon 
E. Contracon 
1. A supra erupted mand. canine with respect to Frankfurt Horizontal plane is said to be in?
Attraction
2. A supra erupted max. canine with respect to Frankfurt Horizontal plane is said to be in
Abstraction
3. A linguoverted max. premolar with respect to Midsaggital plane is said to be in?
A. Aracon
 
B. Abstracon 
C. Protracon 
D. Retracon 
E. Contracon 
4. A linguoverted max. incisor with respect to Orbital plane is said to be in?
A. Aracon  
B. Abstracon 
C. Protracon 
D. Retracon 
E. Contracon 
3.A linguoverted max. premolar with respect to Midsaggital plane is said to be in?
Contraction
4. A linguoverted max. incisor with respect to Orbital plane is said to be in?
Retraction
Neutroclusion with maxillary anteriors are lingual in relaon  to the lower anteriors and there is mesial
dri/ing of molars:***
A. Class 1 type 3 and 5
B. Class 1 type 3 and 4
C. Class 1 type 5 and 4
5. Neutroclusion with maxillary anteriors are lingual in relaton to the lower anteriors and there is mesial
drifing of molars
Class 1 Type 3and 5
6. The following are vertical planes, except
Palatal
7. Bone ossification can be detected with the use of
Hand and wrist x-ray
8. The least desirable method used in child management is:
Voice control
9. Spoiled children are with parents who are:
Overindulging
10. A mechanical aid for keeping the mouth open and operates on a reverse scissors action
Molt mouth prop
11. Child’s arms and legs can be immobilized with
Posey straps
To avoid voming  and complicaons
 during treatment with sedaon,
 no milk or solid food should be taken__
before the scheduled pro-
cedure.
To avoid voming  and complicaons
 during treatment with sedaon, no milk or solid food should be taken__
before the scheduled pro-
cedure.
12. To avoid vomiting and complications during treatment with sedation, no milk or solid food should be taken ____ before
scheduled procedure
After midnight
13. For sedation via intramuscular route, the __ of the gluteal region is the safest
Upper outer
14. Stabilization period for teeth with fractured roots
2-3 months (8-12 weeks)
15. The correct angulation of the inclined plane in relation to the tooth in crossbite is
45 degree
16. The following are sequelae of untreated crossbite, except
Faceting of the labial surface of the tooth opposing the inlocked tooth
17. The appliance to be used in correcting the anterior crossbite is determined by the
All of these: amount of overbite, age of patient, cooperation of the patient
18.Inclined planes should not be left in the mouth for more than 2 months to prevent creation of
Anterior open bite
19. The best appliance for 7 years old child with Class 1 Type 3 (inlocked maxillary central incisor) is
Mandibular acrylic inclined plane
20. Appliance for correcting Class 1 deep bite in a growing child
Maxillary bite planes
21. The overbite reduction achieved by the bite plane is due largely to
Overeruption of posterior teeth
22. In adult patient with excessive overbite the lower facial height in relation to the upper and lower
middle thirds is
Markedly short
23. In deep bite the postural vertical dimension is/has
Not in harmony with occlusal vertical dimension
24. Pulpotomy with formocresol of primary tooth
Induces formation of a dentin bridge at site of amputation
25. The prognosis of a cervical third root fracture
Is not favourable
26. The most important predisposing factor to injury of the anterior teeth is:
Protruding anterior teeth
27. The ultimate objective of pulp capping is
To preserve the pulp and odontoblast 7 so that secondary dentin will be deposited
28. The purpose of pulp treatment and root canal filling is to:
Prolong the usefulness of a tooth to function in mastication
29. Stage of Nolla’s clacification wherein tooth begins eruptive movement:
6
30. Which of the facial radiographs is best used in assessing whether the paent  is gummy or not?
Smiling photograph

Downloaded by John Erickson Bangayan (bangayan1903064@ceu.edu.ph)


lOMoARcPSD|19937673

CLUSTER VII: ORTHODONTICS AND PEDIATRIC DENTISTRY

31. Lip profile is influenced more by


Deposition of upper incisor
32. The profile of the patient with protruded mandible and retruded maxilla is
Concave
33. Which of the following cannot be assessed from cephalometric radiographs
Adequacy of dental arch perimeter
34. Treatment for crown fracture involving the pulp of primary incisor:
Formocresol pulpotomy
35. The dental procedure that produces the greatest negative response in children
Injection
36. In inferior nerve block for a child patient, the injection must be made __ than for an adult patient.
Slightly lower and more posteriorly
37. The maximum number of cartridges of 2% lidocaine with 1:100,000 epinephrine for a 40 pound child
paent
 is
3
38. In inferior alveolar nerve blocked, the barrel of the syringe should be directed in the plane:
Between C And D
39. Indicated restoration on primary anterior tooth with small mesial and distal caries and a cervical caries
but without pulp involvement
Strip off crown
40. In pulpectomy of primary teeth, filling of root canal should be:
2-3 mm above the radiographic apex
41. The type of fear that is based on the feelings and attitude that have been suggested to the child by
others without having had the experience
Subjective fear
42. On the 8rst dental visit, the basic fear of the children below 2 years old is concerned with.
Anxiety and being separated from parents
43. Giving gifts to children.
Should be considered as a token for friendship
44. The arrangement of 2 forces of equal magnitude and opposite but noncollinear lines of action
Couple
45. When one side of the arch is intact and there are several primary teeth missing on the other side, use
A transpalatal arch
46. Choice of nursing nipple to prevent oral habit formation:
Nuk Sauger nipple –mimics mom’s nipple
47. Mesial surface of E can be disked in case the 3 cannot erupt because of insuScient space. This is:
Space regaining
48 In managing abnormal oral habits, therapy must be
Of conditioning response
49. Sudden and firm commands used to get the child’s attention
Voice control
50. Uses the principles of learning theory
Behavior modification
51. Poor personal experience
Hostile parents
52. Reluctant to accept treatment
Negative
53. They have minimal apprehension and are reasonably relaxed:
Cooperative
54. In treating a pediatric patient, one must establish good communication with the
Both parents and child
55. The ff are factors that will cooperate incisor liability, except
Favorable tooth size ratio
56. Guiding of behavior through eye contact, posture and facial expression
Non-verbal
57. Partial or complete immobilization of the patient to protect him from injury while providing dental care
Physical restraint
58. The following are factors that will compensate incisor liability, except
Favorable tooth size ratio
59. Crowding of permanent posterior teeth may be due to
Mesial drifting of permanent first molar
60. As arch perimeter increase, arch length?
No change
61. This is measured form the distal of second primary molar to distal of second primary molar on the
other side following the contour of the arch
Arch perimeter
62. Which of the developmental space will cause a decrease in arch perimeter when pre-empted?
Leeway space
63. Upper arch width increases significantly more than that of lower arch due to
Diverging alveolar growth
64. Point of injection for mandibular blocking in pediatric patient is __ the occlusal plane
Below or at the level
65. An incorrigible 4-year-old who keeps on kicking and throwing objects in the clinic can managed by
Physical restraints and conscious sedation
66. If a child complains of pain on direct pulp capped tooth, the first treatment option should be
Pulpotomy
67. The following irrigating solution can be used as irrigant in pulpectomy procedure except:
Anesthetic solution
68. The first dental appointment of a child patient should be
Before 1 year old
69. Least reduction in tooth preparation for posterior SSC
Lingual
70. Occlusal reduction for SSC
1-1.5 mm
71. Grinding of the sharp edges is sufficient
Extrusion 8
72. Restore with bonding agent and composite
Enamel fracture
73. In acute ingestion of Luoride, the following can be given to the patient to counteract its effect, except
Alum
74. Gingival margin of a tooth that will receive a stainless steel crown should have a ___ finish line
Feather edge

Downloaded by John Erickson Bangayan (bangayan1903064@ceu.edu.ph)


lOMoARcPSD|19937673

CLUSTER VII: ORTHODONTICS AND PEDIATRIC DENTISTRY

75. Chewable fluoride tablets


Systemic
76. Mandibular teeth are best anesthetized with
An inferior alveolar and long buccal injection
77. Pulpotomy with calcium hydroxide in the primary teeth
May form calcific barrier on the pulp
78. Lack of anatomic detail of dental restoration produce
Elongation of opposing teeth
79. Child is 11 years old, tooth #55 has been exfoliated but tooth #65 is still very intact. What must be
done?
None, wait for 65 to exfoliate
80. Two or more teeth moving in opposite directions and pitted against each other by the appliance.
Usually, the resistance to each other is equal and opposite. The anchorage is:
Reciprocal
81. The following are sources of extraoral anchorage except
Mucosa
82. The order from the greatest to least change of the dimensions of the cranium
Height, depth, width
83. At age 4-5, what normal sign of primary dentition augers well for the erupting permanent incisors in
terms of space availability
Growth spaces, interdental spaces
84. The basic form of the arch is determined
In intrauterine life
85. The Pg are true regarding the deciduous maxillary central incisor, except:
Developmental lines are usually not evident
86. To avoid ingestion of lethal dose of fluoride, it is safe to dispense up to __ of fluoride
120 mg
87. Medium-size punch hole is use for
Primary molars
88. Effect of abnormal eruptive path
Delection of eruption
89. Proximal caries, if not restored, will: ... the arch length
Decrease
90. Lack of anatomic detail of dental restoration produces
Elongation of opposing teeth
91. Trident factor of oral habit which is now considered as the most impt. contributory factor to the dev’t
of malocclusion
Frequency
92. The P. are true regarding thumb sucking EXCEPT:
For the first 3 years, damage to occlusion is confined to the anterior segment and is usually temporary
93. Growth of maxilla follows the __ growth curve
Neural
94. At age 5-10, the mandible is __ completed
65%
95. Lack of anatomic detail of dental restoration produces
Elongation of opposing teeth
96. At age 4-5, what normal sign of primary dentition augers well for the erupting permanent incisors in
terms of space availability
Growth spaces, interdental spaces
97. The basic form of the arch is determined
In intrauterine life
98. This aims to recognize and eliminate potential irregularities and malpositions in the developing
craniofacial complex
Interceptive orthodontics
99. The action taken to preserve the integrity of what appears to be normal occlusion at a specific time:
Preventive Orthodontics

ORTHODONTICS AND PEDIATRIC DENTISTRY

Downloaded by John Erickson Bangayan (bangayan1903064@ceu.edu.ph)


lOMoARcPSD|19937673

CLUSTER VII: ORTHODONTICS AND PEDIATRIC DENTISTRY

1. Which of the following conditions is usually present in a Class II, Division 2 malocclusion?
a. Open bite
b. Steep mandibular plane
c. Mesio-occlusion of permanent molars
d. Lingual inclination of maxillary central incisors

2. Excessive orthodontic force used to move a tooth may:


1. Cause hyalinization
2. Cause undermining resorption
3. Crush the periodontal ligament
a. 1&2
b. 2&3
c. 1&3
d. 1,2&3
e. 3 only

3. Severely crowded mandibular incisors usually result from:


a. Mesial migration of teeth
b. Premature loss of primary teeth
c. Presence of supernumerary teeth
d. Tooth size-arch length discrepancy

4. How will extraction of a primary maxillary central incisor in a 5-year old child with incisal spacing affect the size of the intercanine
space?
a. The intercanine space will increase in size.
b. The intercanine space will decrease in size.
c. No change will occur in the size of the intercanine space.

5. The major criterion to differentiate between a true Class III and a pseudo-Class III malocclusion, is:
a. Degree of anterior crossbite
b. Presence of a bilateral crossbite
c. Existence of a forward shift of
the mandible during closure
d. Occlusal relationship between
maxillary and mandibular first
molars

6. Orthodontic correction of which of the following is most easily retained?


4. Diastema
5. Rotation
6. Expansion
7. Posterior crossbite
8. Anterior crossbite
7. A bimaxillary protrusion occurs when:
a. Large horizontal overlap with bilateral crossbite exits
b. The maxillary dentition is forward with respect to basal bone
c. Both dental arches are forward with respect to basal bone
d. None of the choices

8. The following are the conditions found on the area of pressure in the presence of heavy forces:
1. Area of hyalinization
2. Occlusion of blood vessels
3. Stretched periodontal fibers
4. Undermining resorption
a. 1,2&3
b. 1,2&4
c. 2,3&4
d. 3 only
e. 1&4 only

9. Generalized osteoclastic activity along the walls of the alveolar socket is the bone response to:
a. Depressing force
b. Elongating force
c. Extrusion
d. Rotating force
e. Both B&D

10. The major source of anchorage in a maxillary Hawley appliance is the:


a. Molar clasp
b. Labial bow
c. Acrylic portion

Downloaded by John Erickson Bangayan (bangayan1903064@ceu.edu.ph)


lOMoARcPSD|19937673

CLUSTER VII: ORTHODONTICS AND PEDIATRIC DENTISTRY

d. Interproximal clasp
e. Bracket of the band

11. A narrow maxillary arch with respect to midsagittal plane is said to be in:
a. Protraction
b. Retraction
c. Contraction
d. Distraction
e. Both C&D

12. A distal shoe space maintainer is indicated when a primary:


a. Incisor is avulsed
b. First molar is prematurely lost
c. Second molar is lost after eruption of a permanent first molar
d. Second molar is lost before eruption of a permanent molar

13. Complete unilateral lip-jaw palate cleft.


a. Class I
b. Class II
c. Class III
d. Class IV

14. Anodontia, diagnosed in a 5-year old child, primary affects the growth of the:
a. Midface
b. Maxilla
c. Mandible
d. Alveolar bone
e. Maxilla & mandible

15. Which of the following are likely to occur during orthodontic therapy?
1. Gingival irritation
2. Cementum resorption
3. Increased mobility of teeth
4. Demineralization of enamel adjacent to appliances in patients with poor oral hygiene
a. 1,3&4
b. 1&4 only
c. 2&4 only
d. 1,2,3&4
e. 3&4 only

16. The normal downward and forward direction of facial growth results from:
1. Upward and backward growth of maxillary sutures and the mandibular condyle.
2. Vertical eruption and mesial drift of the dentitions.
3. Interstitial growth in the maxilla and the mandible.
4. Epithelial induction at growth centers
a. 1&2
b. 1&3
c. 1&4
d. 2&3
e. 2&4

17. An 8 year old girl has 2mm diastema between permanent maxillary central incisors. Permanent maxillary lateral incisors are in
position. The diastema is probably the result of:
1. Thumb sucking
2. An abnormal frenum attachment
3. A normal developmental process
a. 1&2
b. 1&3
c. 2&3
d. 2 only
e. 3 only

18. When comparing cementum to bone in their response to orthodontic forces, cementum resorbs:

Downloaded by John Erickson Bangayan (bangayan1903064@ceu.edu.ph)


lOMoARcPSD|19937673

CLUSTER VII: ORTHODONTICS AND PEDIATRIC DENTISTRY

a. More readily
b. Less readily
c. Not at all
d. Under lighter loads
e. By indirect (undermining) resorption

19. The tooth in the mandibular arch most likely to be malposed in cases of arch space discrepancy, is the:
a. First molar
b. Second molar
c. First premolar
d. Second premolar

20. Slow progress in molar uprighting in an adult patient is usually due to:
a. Overextended bands
b. An overcontoured spring
c. Lack of anchorage control
d. The occlusion not being
relieved
e. The stabilizing wire not being
passive

21. The “V” principle of growth is best illustrated by the:


a. Nasal septum
b. Mandibular ramus
c. Mandibular symphysis
d. Spheno-occipital synchondrosis

22. Anterior crossbite in the primary dentition usually indicates a developing:


a. Class I malocclusion
b. Class II malocclusion
c. Class III malocclusion
d. Class IV malocclusion
e. None of the above

23. A mixed dentition analysis determines:


a. Intercanine width
b. Skeletal growth pattern
c. Discrepancies in jaw size
d. Size of permanent incisors
e. Space available versus space required

24. With flush terminal plane, permanent first molars will:


a. Initially be Class II
b. Initially be Class III
c. Immediately assume a normal relationship
d. Erupt immediately into an end-to-end relationship

25. Arch shape & symmetry are best evaluated from the:
a. Study cast
b. Panoramic x-ray
c. Frontal photograph
d. Lateral cephalogram
e. Intraoral photograph

26. Bodily force that moves the central incisor mesially produces:
a. Stretching of the periodontal fiber on the distal side
b. Compression of the periodontal fiber in the distal side
c. Osteoblastic activity on the mesial side
d. Osteoclastic activity on the distal side

27. A 9 year old patient has a slightly convex profile and a suspected tooth mass-arch length (circumference) discrepancy. Before
instituting space management procedures, the dentist should:
a. Complete a space analysis
b. Complete a cephalometric analysis
c. Seek orthodontic consultation
d. All of the above
e. Either a or c above

Downloaded by John Erickson Bangayan (bangayan1903064@ceu.edu.ph)


lOMoARcPSD|19937673

CLUSTER VII: ORTHODONTICS AND PEDIATRIC DENTISTRY

28. A primary maxillary anterior tooth in a 4 year old child was traumatically intruded into the tissues so that only half the tooth is
visible. The most appropriate treatment is to:
a. Extract the tooth
b. Perform a pulpotomy
c. Administer no treatment
d. Place orthodontic bands on adjacent teeth and draw the tooth down with elastics

29. Which of the following orthodontic movements of teeth are most difficult to accomplish?
1. Tipping
2. Rotation
3. Intrusion
4. Extrusion
5. Translation
a. 1&2
b. 1&3
c. 1&5
d. 3&4
e. 3&5

30. Interstitial growth is observed at which of the following sites?


1. Spheno-occipital syndrondrosis
2. Maxillary tuberosity
3. Mandibular condyle
4. Zygomaticomaxillary suture
5. Apex of an erupting premolar
a. 1&2
b. 1&3
c. 1&4
d. 2&4
e. 2&5

31. When force is applied, bone reflexes that produces deformation of crystalline materials and allows flow of electric current
necessary for tooth movement:
a. Deposition
b. Piezoelectricity
c. Hyalinization
d. Undermining resorption

32. Cephalometrics is useful in assessing which of the following relationships?


1. Tooth-to-tooth
2. Bone-to-bone
3. Tooth-to-bone
a. 1&2
b. 1&3
c. 2&3
d. 1,2&3
e. 3 only

33. Which of the following is the most essential factor related to correlation of an anterior crossbite?
a. Age of the patient
b. Depth of the crossbite
c. Shape of the tooth involved
d. Space available mesiodistally

34. Frankfort horizontal is a reference plane constructed, by joining which of the following landmarks?
9. Porion & Sella
10.Porion & Nasion

Downloaded by John Erickson Bangayan (bangayan1903064@ceu.edu.ph)


lOMoARcPSD|19937673

CLUSTER VII: ORTHODONTICS AND PEDIATRIC DENTISTRY

11.Porion & Orbitale


12.Nasion & Sella

35. Which of the following is not a space maintainer?


a. Lingual arch
b. Nance holding arch
c. Class III restoration
d. Stainless steel crown
e. Palatal expansion appliance

36. Space closure is least likely to occur after loss of which of the following teeth?
a. Primary mandibular canines
b. Primary mandibular second molars
c. Primary maxillary first molars
d. Primary maxillary central incisors
e. Permanent maxillary central incisors

37. Crowding that occurs with mandibular incisors after age 18 is most often the result of:
a. Orthodontic relapse
b. Periodontal disease
c. Impacted third molars
d. Physiologic maturational change

38. The undesirable side-effect most commonly associated with the use of a finger spring to tip the crown of a tooth is:
a. Pain
b. Gingival irritation
c. Tendency for the tooth to intrude
d. Severe mobility of the tooth
e. Tendency for the root apex to move in the direction opposite from the crown

39. After the age of 6, the greatest increase in size of the mandible occurs:
a. At the symphysis
b. Between the canines
c. Along the lower border
d. Distal to the first molars

40. A distal shoe space maintainer is indicated when a primary:


a. Canine is in crossbite
b. First molar is prematurely lost
c. Second molar is lost after eruption of a permanent first molar
d. Second molar is lost prior to eruption of a permanent first molar

41. Gonion, menton and pogonion are cephalometric landmarks located on the:
13.Midline
14.Mandible
15.Bony chin
16.Skeletal profile

42. A major disadvantage of treatment using cervical headgear is:


a. Impaction of maxillary canines
b. Extrusion of maxillary incisors
c. Extrusion of maxillary molars
d. Potential deformity of the neck
e. Psychologic trauma due to appearance

43. Asymmetrical anterior open bite with normal posterior occlusion is characteristics of:
a. Thumb sucking
b. Mouth breathing
c. Abnormal swallowing habits
d. Both a&b
e. Both b&c

44. If the norm for the cephalometric angle SNA is 82°, a patient’s reading of 90° for SNA most likely indicates:
17.Maxillary protrusion

Downloaded by John Erickson Bangayan (bangayan1903064@ceu.edu.ph)


lOMoARcPSD|19937673

CLUSTER VII: ORTHODONTICS AND PEDIATRIC DENTISTRY

18.The patient’s ethnic background


19.Protrusive maxillary incisors
20.Dysplasia of the anterior cranial base

45. Ideally, a malocclusion should be treated between the ages of:


a. 5 & 8 years
b. 8 & 10 years
c. 10 & 12 years
d. 12 & 14 years
e. None of the above. The age at which a malocclusion is treated should depend on the problem involved

46. Radiographs of a 5 year old patient show permanent maxillary first molars inclined mesially with resulting resorption of the distal
portions of the roots of primary second molars. The condition described as:
a. Ankylosis
b. Ectopic eruption
c. Premature eruption
d. Internal resorption
e. Intrafollicular resorption

47. Fractured maxillary anterior teeth generally occur most often in children with which of the following dental conditions?
a. Class I malocclusion
b. Class II, Division 1 malocclusion
c. Class II, Division 2 malocclusion
d. Class III malocclusion
e. Marked incisor crowding

48. When a simple tipping force is applied to the crown of a single-rooted tooth, the center of rotation is usually located:
a. At the apex
b. At the cervical line
c. 5mm beyond the apex
d. One-third the root length from the apex
e. Two-third the root length from the apex

49. Which of the following is most common orofacial malformation that produces malocclusion?
a. Cleft palate
b. Ectodermal dysplasia
c. Pierre Robin syndrome
d. Osteogenesis imperfecta
e. Cleidocranial dysostosis

50. The cranial vault increases rapidly in size the first few years postnatally and completes approximately 90% of its growth by 6 years
of age. This growth is typical of which of the following types of tissues?
a. Neural
b. Dental
c. Genital
d. Lymphoid
e. General (Somatic)

51. On the first dental visit, the basic fear of a child under the age of 2 is related to:
a. Fear of injection
b. Anxiety over being separated from a parent
c. Anxiety regarding the instruments used to perform dental treatment
d. Not understanding the reason for dental treatment

52. A child with Down’s syndrome is characterized as being:


a. Affectionate
b. Fearful of quick movements
c. Capable of learning dental procedures
d. Both a & b
e. All of these

53. A 3 year old child has an acute fever, diarrhea, oral vesicular lesions and gingival tenderness. The most likely diagnosis is:
21.Thrush
22.Drug allergy
23.Aphthous ulcerations
24.Acute herpetic stomatitis
25.Necrotizing ulcerative gingivitis

Downloaded by John Erickson Bangayan (bangayan1903064@ceu.edu.ph)


lOMoARcPSD|19937673

CLUSTER VII: ORTHODONTICS AND PEDIATRIC DENTISTRY

54. Transillumintaion of soft tissues is useful in detecting which of the following problems in a child?
a. Koplick’s spot
b. Sialolithiasis
c. Aortic stenosis
d. Sickle cell disease
e. Abnormal frenum attachment
55. Which of the following is least likely to result from persistent long-term
a. A deep overbite
b. Protrusion of maxillary incisor
c. Constriction of maxillary arch
d. Rotation of maxillary lateral incisor
e. Lingual inclination of mandibular incisors

56. The least desirable method used in child management is:


a. Tell-show do technique
b. Voice control
c. Gift before treatment
d. Hand-over-mouth technique

57. A child who is reluctant to accept treatment, exhibit negative attitude, but not so pronounced is classified under Frankl?
a. I
b. II
c. III
d. IV

58. The amount of pulp tissue to be removed in formocresol pulpotomy procedure.


a. Half of the coronal pulp
b. Coronal pup tissue down to the cervical constriction of each root canal
c. Infected portion of coronal pulp tissue
d. Total removal of pulp tissue including radicular pulp

59. Which of the following dental sequelae is likely in a child with a history of generalized growth failure (failure to thrive) in the first
six months of life?
a. Retrusive maxilla
b. Enamel hypoplasia
c. Retrusive mandible
d. Small permanent teeth
e. Dentinogenesis imperfecta

60. In festooning and trimming a stainless steel crown, greater length is necessary in the region of the mesiofacial bulge in a primary:
a. First molar
b. Second molar
c. Maxillary canine
d. Maxillary lateral incisor

61. Using a topical fluoride rinse before acid etch direct bonding of orthodontic brackets is contraindicated because fluoride:
a. Decreases the solubility of enamel
b. Increases the pH of the etching agent
c. Causes copious amounts of saliva
d. Directly reacts chemically with the bonding agent

62. If a 7-year old patient loses a primary mandibular canine about the same time the adjacent lateral incisor is erupting or shortly
thereafter, the dentist should be alert to the possibility of:
a. A tongue habit
b. A developing crossbite
c. An early eruption of the permanent canine
d. Lingual collapse of mandibular anterior teeth

63. The average age at which calcification of crowns of permanent central incisors is completed, is:
a. At birth
b. 2-3 years of age
c. 4-5 years of age
d. 6-7 years of age

64. Amalgam is most often the restorative material of choice for primary teeth. The most important modification in its use for
children is in:
a. Cavity preparation
b. Use of rubber dam
c. Use of a zinc-free alloy
d. Condensation of the alloy

Downloaded by John Erickson Bangayan (bangayan1903064@ceu.edu.ph)


lOMoARcPSD|19937673

CLUSTER VII: ORTHODONTICS AND PEDIATRIC DENTISTRY

e. Placement of matrix bands

65. The most frequent cause of fracture of a root tip during extraction of a primary molar is:
a. Ankylosis of the tooth
b. Improper use of cowhorn forceps
c. Presence of a supernumerary premolar
d. Root resorption between the apex and the bifurcation
e. Asymmetric root resorption in which only one root is completely resorbed

66. The most common cause of sinus tracts in gingival tissues of children is:
a. Pericementitis
b. Periapical cyst
c. Acute periapical abscess
d. Chronic periapical abscess

67. A light bluish, dome-shaped lesion on the inside lip of a 2-year old child is most likely a:
a. Mucocele
b. Melanoma
c. Hematoma
d. Hemangioma
e. Sucking callous

68. Primary second molars usually erupt during ages:


a. 8-14 months
b. 14-20 months
c. 20-30 months
d. 30-36 months
e. 36-48 months

69. A child in long-term remission of acute leukemia has dental problems characterized by unusual susceptibility to:
a. Dental caries
b. Oral infection
c. Periodontal bone loss
d. Development of jaw deformities
e. All of the above

70. Supervision of a child’s development of occlusion is most critical at ages:


a. 3-6 years
b. 7-10 years
c. 11-14 years
d. 14-17 years

71. A 4-year old child has a traumatized central incisor with a Class III (Ellis) fracture. The injury occurred about one month ago, and
examination indicates that the pulp is necrotic. There are no other pathologic findings. Treatment of choice is:
a. Watchful observation
b. Extraction and use of a space maintainer
c. Pulpectomy & root canal filling using gutta-percha points & cement
d. Endodontic treatment and root canal filling with a resorbable paste

72. A child with congenital heart disease requires special treatment planning for dental care because of potential problems with:
1. Bleeding
2. Local infection
3. Systemic infection
4. Enamel hypoplasia
a. 1,2&3
b. 1&3
c. 1 only
d. 3 only
e. 1,2,3&4

73. Which of the following approaches is best for a child suffering from cerebral palsy
a. HOM exercise
b. Towel method
c. Use of sedation
d. Physiological approaches

74. Type of fear which is produced by direct physical stimulation of sense organ
a. Associative fear

Downloaded by John Erickson Bangayan (bangayan1903064@ceu.edu.ph)


lOMoARcPSD|19937673

CLUSTER VII: ORTHODONTICS AND PEDIATRIC DENTISTRY

b. Objective fear
c. Subjective fear
d. Acquired fear

75. Agent used to reduce pain without affecting consciousness


26.Hypnotic drug
27.Analgesic
28.Barbiturate
29.General anesthesia

76. The drug that modifies fear and anxiety


a. Flanax
b. Ethamizid
c. Celestamine
d. Seconal

77. A general characteristic of a 12 year old child


a. Mother as the center of his world
b. Exhibits tantrums
c. Separation anxiety
d. Rejects parental authority

78. This is a psychological approach to manage a child


a. Nitrous oxide inhalation
b. Giving diazepam
c. Multisensory techniques
d. HOM

79. This is a contraindication of HOM


a. Spoiled child
b. Very young patient
c. Defiant child
d. Uncooperative behavior but capable of understanding

80. Induce formation of apical closure of young permanent molar using CAOH
30.Apexogenesis
31.Apeximation
32.Apexification
33.Apex formation

81. The growth movement of the mandible is complimented by the growth of the maxilla, which is:
34.Down and forward
35.Forward only
36.Downward and backward
37.Upward and backward

82. Direction of displacement of the mandible in an individual with developing Class II malocclusion:
a. Down and forward
b. Forward only
c. Down and backward
d. Forward and upward
e. Upward and backward

83. Child refuses to accept treatment or open the mount minimally:


a. Frankl 1
b. Frankl 2
c. Frankl 3
d. Frankl 4

84. Definitely negative behavior.


a. Frankl 1
b. Frankl 2
c. Frankl 3
d. Frankl 4

85. Child has good rapport with the dentist.


a. Frankl 1
b. Frankl 2
c. Frankl 3
d. Frankl 4

Downloaded by John Erickson Bangayan (bangayan1903064@ceu.edu.ph)


lOMoARcPSD|19937673

CLUSTER VII: ORTHODONTICS AND PEDIATRIC DENTISTRY

86. The treatment option contraindicated in patients who are not able to breathe nasally.
a. No treatment
b. Conscious sedation
c. Nitrous oxide & oxygen inhalation
d. General anesthesia
87. The treatment option for patients who have sustained extensive orofacial &/or dental trauma
a. No treatment
b. Conscious sedation
c. Nitrous oxide & oxygen inhalation
d. General anesthesia

88. In infant oral care, clean mouth with gauze after feedings at bed time done during:
a. 0-6 months
b. 6-12 months
c. 12-24 months
d. 24-36 months

89. Space differential between combine width of CDE and 345 is __


a. Positive
b. Negative
c. Zero
d. 2.2 mm
e. 2.4 mm

90. Which of the developmental space will cause a decrease in arch perimeter when pre-empted?
a. Primate space
b. Interdental space
c. Interocclusal space
d. Leeway space

91. This is an increase in size by expansion from within


a. Sutural apposition
b. Maturation
c. Interstitial growth
d. Development

92. This is growth movement which mean movement of whole bone as a unit
a. Drift
b. Displacement
c. Deposition
d. Relocation

93. Mandible comes from ___ branchial arch?


a. First
b. Second
c. Third
d. Fourth

94. Maxilla is formed from the maxillary process and _______?


a. Median nasal process
b. Lateral processes
c. Globular process
d. Tuberculum impar

95. In young child, paradoxical excitement occurs most frequently following premedication with:
a. A narcotic
b. A barbiturate
c. Nitrous oxide
d. An amphetamine

96. The maximum amount of anesthesia that can be given to a 36-pound child is:
a. 4 carpules
b. 3 carpules
c. 2 carpules
d. 1 carpule

97. Dental fluorosis is most likely to occur on a:


a. Two-year old child ingesting 1 ppm of fluoride

Downloaded by John Erickson Bangayan (bangayan1903064@ceu.edu.ph)


lOMoARcPSD|19937673

CLUSTER VII: ORTHODONTICS AND PEDIATRIC DENTISTRY

b. 17-year old patient ingesting 1 ppm of fluoride


c. On a 3-year old child ingesting 0.5- ppm of fluoride
d. None of the above

98. Pulpotomy with formocresol of primary tooth


a. Result in lower success rate compared to calcium hydroxide
b. Result in higher success rate compared to calcium hydroxide
c. Should never be used
d. Induces formation of a dentin bridge site of manipulation
e. None of the above

99. A 4-year old child visited a dentist for the first time and received prophylaxis quite well. Before he left, the dentist was glad that
he gave the child a toy. This kind of gesture, is called:
a. Classical conditioning
b. Positive reinforcement
c. Bribery
d. Public relations
e. None of the above

100. If a primary tooth has been extracted before succedaneous tooth has begun eruptive movement, its eruption will be _______.
a. Hastened
b. Delayed
c. Not affected

Judy
ORTHO-OPEDIA

1. At age 5-10, the mandible is __ completed.

a. 45%

b. 40%

c. 65%

d. 96%

2. Gonion, menton and pogonion are cephalometric landmarks located on the

a. Midline

b. bony chin

c. mandible

d. skeletal profile

3. A linguoverted max. incisor with respect to Orbital Plane is said to be in?

a. Attraction

b. Abstraction

c. Protraction

d. Retraction

e. Contraction.

Downloaded by John Erickson Bangayan (bangayan1903064@ceu.edu.ph)


lOMoARcPSD|19937673

CLUSTER VII: ORTHODONTICS AND PEDIATRIC DENTISTRY

4. Inclined planes should not be left in the mouth for more than 2 months to prevent creation of:

a. anterior open bite

b. posterior open bite

c. anterior cross bite

d. posterior cross bite.

5. Deep bite cases in young children can be treated with

a. fixed braces

b. bite plane

c. extraoral force appliance

d. space maintainer

6. . The correct angulation of the inclined plane in relation to the tooth in crossbite is:

a. 15º

b. 25º

c. 35º

d. 45º

e. 55º

7. The following are true regarding thumb sucking EXCEPT:

a. it is a learned habit, not a symptom of a deeper emotional disturbance

b. for the first 3 years, damage to occlusion is confined to the anterior segment and is usually temporary

c. normal from 1-1 1/2 then disappears at age 2 with proper attention to nursing

d. habit beyond age 4 can be controlled by badgering attempts of parents to break the habit

8. Bone ossification can be detected with the use of:

a. hand and wrist x-ray

b. BMR

c. electromyographic exam

d. biostatic

Downloaded by John Erickson Bangayan (bangayan1903064@ceu.edu.ph)


lOMoARcPSD|19937673

CLUSTER VII: ORTHODONTICS AND PEDIATRIC DENTISTRY

9. Bodily force that moves the central incisor mesially produces:

a. stretching of the periodontal fiber on the distal side

b. compression of the periodontal fiber in the distal side

c. osteoblastic activity on the mesial side

d. osteoclastic activity on the distal side

10. The basic form of the arch is determined:

a. by environmental factors

b. by muscles

c. in intrauterine life

d. two of the choices

e. all of these

11. . The following are sequelae of untreated crossbite, except:

a. faceting on the labial surface of the tooth in crossbite

b. faceting of the labial surface of the tooth opposing the inlocked tooth

c. abrasion

d. periodontal involvement.

12. Neutroclusion with maxillary anteriors are lingual in relation to the lower anteriors and there is mesial drifting of
molars.

a. Class 1 type 3 and 5

b. Class 1 type 3 and 4

c. Class 1 type 5 and 4.

13. Effect of abnormal eruptive path:

a. space loss

b. delayed eruption

c. deflection of eruption

d. increase in arch length

e. elongation of adjacent teeth

14. When a simple tipping force is applied to the crown of a single-rooted tooth, the center of rotation is usually located:

Downloaded by John Erickson Bangayan (bangayan1903064@ceu.edu.ph)


lOMoARcPSD|19937673

CLUSTER VII: ORTHODONTICS AND PEDIATRIC DENTISTRY

a. at the apex

b. at the cervical line

c. 5mm beyond the apex

d. one-third the root length from the apex

e. two-third the root length from the apex.

15. A dolichofacial face would likely to have ___ arch form.

a. Round

b. average

c. broad and square

d. long and tapering

16. Which of the following is the most common orofacial malformation that produces malocclusion?

a. Cleft palate

b. Ectodermal dysplasia

c. Pierre Robin syndrome

d. Osteogenesis imperfecta

e. Cleidocranial dysostosis

17. Upper arch width increases significantly more than that of lower arch due to?

a. diverging alveolar growth

b. vertical alveolar growth

c. labial positioning of permanent incisors

d. distal tipping of cuspid

18. At age 4-5, what normal sign of primary dentition augers well for the erupting permanent incisors in terms of space
availability:

a. Class I cuspid relationship

b. growth spaces, interdental spaces

c. upright vertical incisor relationship

d. flush terminal plane

19. Crowding of permanent posterior teeth may be due to:

Downloaded by John Erickson Bangayan (bangayan1903064@ceu.edu.ph)


lOMoARcPSD|19937673

CLUSTER VII: ORTHODONTICS AND PEDIATRIC DENTISTRY

a. mesial drifting of permanent first molar

b. crowding of anterior teeth

c. labioversion of anterior teeth

d. spacing of anterior teeth

20. This will decrease lower arch perimeter during transitional period:

a. late mesial shift of first permanent molar

b. distal tipping of lower cuspid

c. labial position of permanent incisors

21. . Late mesial shift is possible because of

a. primate spaces

b. terminal plane

c. space differential bet. Deciduous and permanent teeth in the posterior segment

d. none of the above

22. The cranial vault increases rapidly in size the first few years postnatally and completes approximately 90% of its
growth by 6 years of age. This growth is typical of which of the following types of tissues?

a. Neural

b. Dental

c. Genital

d. Lymphoid

e. general (somatic)

23. Orthodontic correction of which of the following is most easily retained?

a. Diastema

b. Expansion

c. Rotation

d. Posterior crossbite

e. Anterior crossbite

24. The following are vertical planes, except:

Downloaded by John Erickson Bangayan (bangayan1903064@ceu.edu.ph)


lOMoARcPSD|19937673

CLUSTER VII: ORTHODONTICS AND PEDIATRIC DENTISTRY

a. Axis of 1

b. v-axis

c. facial plane

d. palatal

25. The overbite reduction achieved by the bite plane is due largely to:

a. intrusion of posterior teeth

b. overeruption of posterior teeth

c. mesial drifting of posterior teeth

d. buccal and lingual movement of posterior teeth.

26. Two or more teeth moving in opposite directions and pitted against each other by the appliance. Usually, the
resistance to each other is equal and opposite. The anchorage is:

a. Simple

b. Stationary

c. Reciprocal

d. Extraoral

27. The arrangement of 2 forces of equal magnitude and opposite but noncollinear lines of action:

a. center of rotation

b. couple

c. moment

d. center of resistance

28. In adult patient with excessive overbite the lower facial height in relation to the upper and lower middle thirds is:

a. normal

b. markedly short

c. markedly long.

29. The basic form of the arch is determined:

a. by environmental factors

b. by muscles

c. in intrauterine life

d. two of the choices

Downloaded by John Erickson Bangayan (bangayan1903064@ceu.edu.ph)


lOMoARcPSD|19937673

CLUSTER VII: ORTHODONTICS AND PEDIATRIC DENTISTRY

e. all of these

30. Growth of maxilla follows the__ growth curve.

a. general / bodily

b. lymphoid

c. neural

d. genital

31. The following are vertical planes, except:

a. Axis of 1

b. v-axis

c. facial plane

d. palatal

32. Early loss of upper D will end up to what type of terminal plane?

a. mesial step

b. flush terminal plane

c. distal step

d. any of the above

33. The following are sources of extraoral anchorage except:

a. Cervical

b. Occipital

c. Cranial

d. Facial

e. Mucosa

34. Lack of anatomic detail of dental restoration produces:

a. increased arch length

b. elongation of adjacent teeth

c. elongation of opposing teeth

d. A and B

Downloaded by John Erickson Bangayan (bangayan1903064@ceu.edu.ph)


lOMoARcPSD|19937673

CLUSTER VII: ORTHODONTICS AND PEDIATRIC DENTISTRY

35. . Increase in the width of the maxilla involves palatal growth following the expanding V principle. The buccal
segments move:

a. downward and forward

b. posterior and superior

c. upward and posterior

d. upward

36. As arch perimeter increase, arch length?

a. Increases

b. Decreases

c. no change.

37. How will extraction of a primary maxillary central incisor in a 5-year old child with incisal spacing affect the size of the
intercanine space?

a. The intercanine space will increase in size.

b. The intercanine space will decrease in size.

c. No change will occur in the size of the intercanine space.

38. . The most common malocclusion in the mixed dentition period is:

a. Class II Div. 1

b. anterior open bite

c. posterior open bite

d. crowding

e. Class I type 2

39. From birth to 5 years, the dominance of growth is on the

a. nasomaxillary structures

b. cranial structures

c. mandible

d. muscular components of the body

40. Which of the following is the most essential factor related to correction of an anterior crossbite?

Downloaded by John Erickson Bangayan (bangayan1903064@ceu.edu.ph)


lOMoARcPSD|19937673

CLUSTER VII: ORTHODONTICS AND PEDIATRIC DENTISTRY

a. age of the patient

b. depth of the corssbite

c. shape of the tooth involved

d. space available mesiodistally

41. Generalized osteoclastic activity along the walls of the alveolar socket is the bone response to:

a. depressing force

b. extrusion

c. elongating force

d. rotating force

e. both B & D

42. Orthodontic correction of which of the following is most easily retained?

a. Diastema

b. Expansion

c. Rotation

d. Posterior crossbite

e. Anterior crossbite

43. In deep bite the postural vertical dimension is/has:

a. in harmony with occlusal vertical dimension

b. not in harmony with occlusal vertical dimension

c. no relations with occlusal vertical dimension

44. The following are sources of extraoral anchorage except:

a. Cervical

b. Occipital

c. Cranial

d. Facial

e. Mucosa

Downloaded by John Erickson Bangayan (bangayan1903064@ceu.edu.ph)


lOMoARcPSD|19937673

CLUSTER VII: ORTHODONTICS AND PEDIATRIC DENTISTRY

45. Which of the developmental space will cause a decrease in arch perimeter when pre-empted?

a. primate space

b. interdental space

c. leeway space

d. inter-occlusal space

46. If a 7-year old patient loses a primary mandibular canine about the same time the adjacent lateral incisor is erupting
or shortly thereafter, the dentist should be alert to the possibility of:

a. a tongue habit

b. a developing crossbite

c. an early eruption of the permanent canine

d. lingual collapse of mandibular anterior teeth

47. Maxilla is formed from the maxillary processes and ?

a. medial nasal process

b. globular process

c. lateral processes

d. tuberculum impar

48. Severely crowded mandibular incisors usually result from:

a. mesial migration of teeth

b. premature loss of primary teeth

c. presence of supernumerary teeth

d. tooth size-arch length discrepancy

49. The following are factors that will compensate incisor liability, except:

a. intercanine width growth

b. labial positioning of permanent incisors

c. upright position of primary incisors

d. favorable tooth size ratio

50. The basic form of the arch is determined:

Downloaded by John Erickson Bangayan (bangayan1903064@ceu.edu.ph)


lOMoARcPSD|19937673

CLUSTER VII: ORTHODONTICS AND PEDIATRIC DENTISTRY

a. by environmental factors

b. by muscles

c. in intrauterine life

d. two of the choices

e. all of these

51. Type of fear which is produced by direct physical stimulation of sense organ

a. associative fear

b. objective fear

c. subjective fear

d. acquired fear

52. The dental procedure that produces the greatest negative response in children:

a. Extraction

b. Injection

c. cavity preparation

d. all of these

53. In periodontic triangle the apex of the triangle is:

a. Child

b. Parent

c. Dentist

d. Assistant

54. Treatment for crown fracture involving the pulp of a primary incisor:

a. DPC

b. formocresol pulpotomy

c. pulpectomy

d. extraction

55. Giving gifts to children.

a. should promised to the child for every appointment to ensure cooperation.

Downloaded by John Erickson Bangayan (bangayan1903064@ceu.edu.ph)


lOMoARcPSD|19937673

CLUSTER VII: ORTHODONTICS AND PEDIATRIC DENTISTRY

b. should be considered as a token for friendship

c. both A&B

d. Both B&C.

56. The dental procedure that produces the greatest negative response in children:

a. Extraction

b. Injection

c. cavity preparation

d. all of these

57. Mandible comes from ___ branchial arch

a. First

b. Second

c. Third

d. Fourth

58. Grinding of the sharp edges is sufficient.

a. lateral luxation

b. dentin fracture

c. enamel fracture

d. extrusion.

59. Guiding of behavior through eye contact, posture and facial expression.

a. Distraction

b. positive reinforcement

c. non-verbal communication

d. HOM exercise

e. TSD

60. In acute ingestion of fluoride, the following can be given to the patient to counteract its effect, except:

a. 2 tsp of ipecac syrup

b. Milk

Downloaded by John Erickson Bangayan (bangayan1903064@ceu.edu.ph)


lOMoARcPSD|19937673

CLUSTER VII: ORTHODONTICS AND PEDIATRIC DENTISTRY

c. milk of magnesia

d. alum

61. Radiographs of a 5-year old patient show permanent maxillary first molars inclined mesially with resulting resoprtion
of the distal portions of the roots of primary second molars. The condition described is:

a. Ankyloses

b. ectopic eruption

c. premature eruption

d. internal resorption

e. intrafollicular resorption

62. Maxilla is formed from the maxillary processes and ___

a. medial nasal process

b. globular process

c. lateral processes

d. tuberculum impar

63. A 4-year old child has a traumatized central incisor with a Class III (Ellis) fracture. The injury occurred about one
month ago, and examination indicates that the pulp is necrotic. There are no other pathologic findings. Treatment of
choice is:

a. watchful observation

b. extraction and use of a space maintainer

c. pulpectomy and root canal filling using gutta-percha points and cement

d. endodontic treatment and root canal filling with a resorbable paste.

64. Indicated restoration on a primary anterior tooth with small mesial and distal caries and a cervical caries but without
pulp involvement:

a. stainless steel crown,

b. composite resin

c. strip-off crown,

65. Lack of anatomic detail of dental restoration produces:

a. increased arch length

Downloaded by John Erickson Bangayan (bangayan1903064@ceu.edu.ph)


lOMoARcPSD|19937673

CLUSTER VII: ORTHODONTICS AND PEDIATRIC DENTISTRY

b. elongation of adjacent teeth

c. elongation of opposing teeth

d. A and B

66. How will extraction of a primary maxillary central incisor in a 5-year old child with incisal spacing affect the size of the
intercanine space?

a. The intercanine space will increase in size.

b. The intercanine space will decrease in size.

c. No change will occur in the size of the intercanine space.

67. An 8-year-old girl has 2mm diastema between permanent maxillary central incisors. Permanent maxillary lateral
incisors are in position. The diastema is probably the result of:

1.thumb-sucking
2.an abnormal frenum attachment
3.a normal developmental process

a. 1&2

b. 1&3

c. 2&3

d. 2 only

e. 3 only

68. . A child in the “ugly duckling” stage is characterized by

a. a flat nasal bone

b. absence of maxillary incisors

c. diastema between 2 central incisors

d. presence of only one maxillary central incisor

69. The prognosis of a cervical third root fracture:

a. is not favorable,

b. is favorable,

c. depends on whether tooth is discolored

70. The most frequent cause of fracture of a root tip during extraction of a primary molar is:

a. ankylosis of the tooth

b. improper use of cowhorn forceps

Downloaded by John Erickson Bangayan (bangayan1903064@ceu.edu.ph)


lOMoARcPSD|19937673

CLUSTER VII: ORTHODONTICS AND PEDIATRIC DENTISTRY

c. presence of a supernumerary premolar

d. root resorption between the apex and the bifurcation

e. asymmetric root resorption in which only one root is completely resorbed.

71. Transillumination of soft tissues is useful in detecting which of the following problems in a child?

a. Koplick’s spot

b. Sialolithiasis

c. Aortic stenosis

d. Sickle cell disease

e. abnormal frenum attachment

72. Child is 11 years old. Tooth # 55 has been exfoliated but tooth # 65 is still very intact. What must be done?

a. none, wait for 65 to exfoliate

b. do percussion test before extracting 65

c. extract 65 at once

d. retain 65 and place a band and loop for 55

73. A child in long-term remission of acute leukemia has dental problems characterized by unusual susceptibility to:

a. dental caries

b. oral infection

c. periodontal bone loss

d. development of jaw deformities

e. all of the above.

74. . In pedodontic triangle the apex of the triangle is:

a. Child

b. Dentist

c. Parent

d. Assistant

75. The treatment option contraindicated in patients who are not able to breathe nasally.

a. no treatment

Downloaded by John Erickson Bangayan (bangayan1903064@ceu.edu.ph)


lOMoARcPSD|19937673

CLUSTER VII: ORTHODONTICS AND PEDIATRIC DENTISTRY

b. nitrous oxide & oxygen inhalation

c. conscious sedation

d. genera anesthesia

76. An incorrigible 4-year-old child who keeps on kicking and throwing objects in the clinic can be managed by:

a. tell-show-do technique,

b. physical restraint and conscious sedation

c. general anesthesia,

d. all of these

77. The first dental appointment of a child patient should be:

a. before 1 year old,

b. 2 1/2 to 3 years old,

c. 6 years old

78. The treatment option for patients who have sustained extensive orofacial & or dental trauma

a. no treatment

b. nitrous oxide & oxygen inhalation

c. conscious sedation

d. general anesthesia

79. . In managing abnormal oral habits, therapy must be:

a. Mechanical

b. of physical control

c. of conditioning responses

d. a means of punishment

80. A 4-year-old child visited a dentist for the first time and received prophylaxis quite well. Before he left, the dentist was
glad that he gave the child a toy. This kind of gesture is called:

a. classical conditioning

b. positive reinforcement,

c. bribery,

Downloaded by John Erickson Bangayan (bangayan1903064@ceu.edu.ph)


lOMoARcPSD|19937673

CLUSTER VII: ORTHODONTICS AND PEDIATRIC DENTISTRY

d. public relations

e. none of the above.

81. The purpose of pulp treatment and root canal filling is to:

a. remove pulpal tissue,

b. prolong the usefulness of a tooth to function in mastication,

c. enlarge the root canal,

82. A distal shoe space maintainer is indicated when a primary:

a. incisor is avulsed

b. first molar is prematurely lost

c. second molar is lost after eruption of a permanent first molar

d. second molar is lost before eruption of a permanent first molar

83. Dental Fluorosis is most likely to occur on a:

a. two-year-old child ingesting 1 ppm of fluoride

b. 17-year-old patient ingesting 1 ppm of fluoride

c. on a 3-year-old child ingesting 0.50 ppm of fluoride

d. none of the above.

84. Sudden and firm commands used to get the child’s attention:

a. HOME

b. Tell, show and do

c. voice control

d. Hypnodontics

85. Restore with bonding agent and composite

a. Extrusion

b. dentin fracture

c. intrusion

d. enamel fracture

Downloaded by John Erickson Bangayan (bangayan1903064@ceu.edu.ph)


lOMoARcPSD|19937673

CLUSTER VII: ORTHODONTICS AND PEDIATRIC DENTISTRY

e. enamel-dentin fracture.

86. In treating a pediatric patient, one must establish good communication with the

a. child only, since he is the one to be treated

b. parents only, because they are the ones who will pay the bill

c. the caretaker or the yaya who takes care of the child

d. both parent and child

87. This is an increase in size by expansion from within

a. sutural apposition

b. interstitial growth

c. maturation

d. development

88. . To avoid vomiting and complications during treatment with sedation, no milk or solid food should be taken___ before
the scheduled procedure.

a. 4 hrs.

b. 6 hrs.

c. 8 hrs.

d. after midnight

89. . Anodontia, diagnosed in a 5-year-old child, primarily affects the growth of the:

a. Midface

b. mandible

c. maxilla

d. alveolar bone

e. maxilla & mandible

90. Transillumination of soft tissues is useful in detecting which of the following problems in a child?

a. Koplick’s spot

b. Sialolithiasis

c. Aortic stenosis

d. Sickle cell disease

Downloaded by John Erickson Bangayan (bangayan1903064@ceu.edu.ph)


lOMoARcPSD|19937673

CLUSTER VII: ORTHODONTICS AND PEDIATRIC DENTISTRY

e. abnormal frenum attachment

91. Slow progress in molar uprighting in an adult patient is usally due to:

a. overextended bands

b. an overcontoured spring

c. lack of anchorage control

d. the occlusion not being relieved

e. the stabilizing wire not being passive

92. . Fluoride reduces the incidence of carious lesions on the

a. pit and fissure

b. occlusal surfaces

c. smooth surfaces

d. lingual surfaces of teeth

93. Indicated restoration on a primary anterior tooth with small mesial and distal caries and a cervical caries but without
pulp involvement:

a. stainless steel crown,

b. composite resin

c. strip-off crown,

94. Induce formation of apical closure of young permanent molar using CAOH

a. Apexogenesis

b. apexification

c. apeximation

d. apex formation

95. . The least desirable method used in child management is:

a. tell-show do technique,

b. voice control,

c. gift before treatment,

d. hand-over-mouth technique.

Downloaded by John Erickson Bangayan (bangayan1903064@ceu.edu.ph)


lOMoARcPSD|19937673

CLUSTER VII: ORTHODONTICS AND PEDIATRIC DENTISTRY

96. A light bluish, dome-shaped lesion on the inside lip of a 2-year old child is most likely a:

a. Mucocele

b. Melanoma

c. Hematoma

d. hemangioma

e. sucking callous

97. Pulpotomy with formocresol of primary tooth:

a. result in lower success rate compared to calcium hydroxide,

b. result in higher success rate compared to calcium hydroxide,

c. should never be used,

d. induces formation of a dentin bridge at site of amputation,

e. none of the above.

98. Using a topical fluoride rinse before acid etch direct bonding of orthodontic brackets is contraindicated because
fluoride:

a. decreases the solubility of enamel

b. increases the pH of the etching agent

c. causes copious amounts of saliva

d. directly reacts chemically with the bonding agent.

99. The tooth in the mandibular arch most likely to be malposed in cases of arch space discrepancy is the:

a. first molar

b. first premalor

c. second molar

d. second premolar

100. Gingival margin of a tooth that will receive a stainless steel crown should have a ___finish line:

a. Shoulder

b. feather edge

c. BOTH

Angeli
PRE-BOARD THEORETICAL EXAMINATION

ORTHODONTICS AND PEDIATRIC DENTISTRY

Downloaded by John Erickson Bangayan (bangayan1903064@ceu.edu.ph)


lOMoARcPSD|19937673

CLUSTER VII: ORTHODONTICS AND PEDIATRIC DENTISTRY

1. The age at which tooth displacement occurs most frequently is influenced by:

A. The degree of calcification of the cementum


B. The degree of calcification of the dentin
C. The stage of development of the root
D. The stage of development of the crown
E. None of the above.

2. Increase in lower intercanine width is due to:

A. labial positioning of permanent anterior teeth as they erupt


B. divergence of alveolar process
C. dental tipping of canine into the primate spaces during eruption of permanent incisors
D. expanding “V” principle
E. None of the above.

3. The crown of lower 6 erupt, tipped somewhat lingually and do not upright until the eruption of lower 7. As the lower 6
uprights, it:

A. increases the arch perimeter C. increases the arch width


B. decreases the arch perimeter D. decreases the arch width

4. Lower arch perimeter from mixed to permanent dentition:

A. increases slightly
B. decreases
C. remains the same
D. increases slightly but has an equal chance to increase or decrease.

5. These are present at birth, having appeared as normal part of prenatal maturation of the

neuromusculature:

A. conditioned reflexes C. voluntary activities


B. unconditioned reflexes D. both B & C E. both A & C

6. Transition from infantile to mature swallow takes place over several months depending on timing of maturation but most
children achieve mature swallow by ___ yrs.

A. 1 ½ B. 3 C. 5 D. 7 E. none of these

7. The most accepted and widely used classification system is:

A. Simon’s B. Angle’s C. Dewey-Anderson D. Ackermann & Profitt

8. Dewey-Anderson modifications for Class II malocclusion:

A. types 1-3 B. types 1-5 C. types 0-5 D. none of


these

9. A system of classification in which five characteristics and interrelationships are described by:

A. Simon’s B. Angle’s C. Dewey-Anderson D. Ackermann & Profitt

10. Normal relationship in the primary dentition is:

Downloaded by John Erickson Bangayan (bangayan1903064@ceu.edu.ph)


lOMoARcPSD|19937673

CLUSTER VII: ORTHODONTICS AND PEDIATRIC DENTISTRY

A. procumbent C. slightly retrusive


B. same as in adults D. almost vertical

11. Overbite from 3 to 5 years old,

A. remains relatively the same B. decreases C. increases

12. Precise bite/jaw relationship is not yet seen or established at birth because of:

A. immaturity of neuromuscular control C. absence of teeth


B. ill-defined eminentia articularis D. both A & B E. all of these

13. Spacing of incisors at age 7-9 (ugly duckling) stage :

A. should be treated at once


B. is an abnormal stage of dentitional development
C. should be left undisturbed; let nature take its course
D. refer the case to a specialist; this is a difficult case

14. Which one is an incorrect description of Nolla’s stages of calcification?

A. Stage 4, 2/3 of the crown completed


B. Stage 6, crown almost completed
C. Stage 7, 1/3 of the root completed
D. Stage 9, root almost completed, open apex
15. Which of the following sequence of eruption is not favorable for maintaining arch

length?

A. 6-1-2-4-5-3-7 upper C. 6-1-2-3-4-5-7 lower


B. 1-6-2-4-5-3-7 upper D. 1-6-2-3-4-5-7 lower
E. 6-1-2-7-4-5-3 upper

16. Mesiodistal measurement of CDE- mesiodistal widths of erupting 345 yields:

A. lack of space
B. excess space
C. no additional space in the posterior segment

17. Space differential in an anterior segment (BA/AB) - 21/12 gives :

A. excess space B. no space differential C. negative space

18. This phenomenon occurs in patients with spaced primary dentition and a straight terminal relationship of primary molars.
The permanent mandibular first molars emerge at age 6, move the primary molars mesially, close the space distal to the
primary canines and correct the straight terminal plane relationship to a mesial step relationship.

A. early mesial shift B. homeostatic occlusion C. late mesial shift.

19. Type of malocclusion that is prone to fracture of the anterior teeth:

A. Class II Division 2 C. Class I Type 3


B. Class II Division 1 D. Class I Type 1 E. none of these

20. Maximum number of catridges that can be used if a child weighs 54 pounds.

A. 1 B. 2 C. 3 D. 4 E. 5

Downloaded by John Erickson Bangayan (bangayan1903064@ceu.edu.ph)


lOMoARcPSD|19937673

CLUSTER VII: ORTHODONTICS AND PEDIATRIC DENTISTRY

21. The following types of fluoride can be applied every 6 months , except

A. acidulated phosphate fluoride C. sodium fluoride


B. stannous fluoride D. two of the above

22. An adjunctive diagnostic tool for treatment planning orthodontics that gives information on possible growth pattern and can
give an accurate bone age picture of the patient:

A. hand & wrist C. panoramic


B. cephalometrics D. tomogram E. periapical radiograph

23. Which of the following is the order of tooth extraction in a serial extraction procedure?

I. Primary canines II. Selected premolars III. Primary first molars

A. I, II, III B. I, III, II C. II, I, III D. II, III, I E. III, I, II

24. The distal step in the primary dentition will result in:

A. end to end molar relationship C. Class I molar relationship


B. Class III molar relationship D. Class II molar relationship

25. This is a process of reshaping and resizing as a consequence of progressive continuous relocation:

A. drift C. remodeling
B. displacement D. translation E. none of these.

26. The tooth movement most readily accomplished with a removable appliance is:

A. tipping C. root movement


B. torquing D. bodily movement E. none of these

27. A 12 year old boy exhibits a deep anterior vertical overlap, maxillary centrals are inclined lingually, and maxillary lateral
incisors protrude. Maxillary molars are in an anterior relationship with mandibular molars. The Angle classification is:

A. Class I C. Class II, Division 1, Subdivision


B. Class II, Division 1 D. Class II, Division 2 E. Class III

28. Recurring tooth rotations occur most frequently after orthodontic correction because of:

A. density of the cortical bone


B. persistence of tongue and finger habits

C. persistence of free gingival and transseptal fibers


D. persistence of oblique fibers of the periodontal ligaments

29. The best alternative diagnostic aid in the absence of cephalometric head plate:

A. study cast C. facial photograph


B. radiograph D. both A & C E. none of these

30. An elongating force will move the tooth:

Downloaded by John Erickson Bangayan (bangayan1903064@ceu.edu.ph)


lOMoARcPSD|19937673

CLUSTER VII: ORTHODONTICS AND PEDIATRIC DENTISTRY

A. towards the socket C. on its axis E. none of these


B. away from the occlusal plane D. towards the occlusal plane

31. Appliance indicated for bruxism:

A. oral screen B. buccal shield C. night guard D. lip bumper

32. Open bite is term applied when:

A. torsiversion is present
B. there is an abnormal buccolingual relationship of the teeth

C. there is an excessive vertical overlap of the incisors


D. there is localized absence of occlusion, while the remaining teeth are in occlusion
E. none of the above.

33. A 5-year old boy who lives in an area where fluoride content of the drinking water averages 0.75ppm should be
supplemented with how much fluoride per day?

A. 0 mg. B. 0.25 mg C. 0.50 mg. D.


1.00mg.

34. Generally speaking, intruded primary teeth should be:

A. observed C. extracted
B. repositioned by hand D. treated pulpally E. none of these

35. An intruded primary tooth which is found in a lingual or encroaching relationship to the developing permanent
tooth as confirmed with a lateral radiograph of the anterior segment:

A. observed C. extracted
B. repositioned by hand D. treated pulpally E. none of these.

36. The development of dentition is delayed, there is complete primary dentition at age 15, delayed eruption of permanent
teeth, and presence of supernumerary teeth. This is most likely to be:

A. 21-Trisomy syndrome C. hypothyroidism


B. cleidocranial dysostosis D. hypopituitarism
E. achondroplastic dwarfism

37. Which of the following conditions is usually present in a Class II, Division 2 malocclussion?

A. Open bite
B. Steep mandibular plane
C. Mesiocclusion of permanent first molars
D. Lingual inclination of maxillary central incisors

38. Excessive orthodontic force used to move a tooth may:

1. cause hyalinization
2. cause undermining resorption
3. crush the periodontal ligament
A. 1 & 2 B. 2 & 3 C. 1 & 3 D. 1, 2 & 3 E. 3 only

Downloaded by John Erickson Bangayan (bangayan1903064@ceu.edu.ph)


lOMoARcPSD|19937673

CLUSTER VII: ORTHODONTICS AND PEDIATRIC DENTISTRY

39. A bimaxillary protrusion occurs when:

A. large horizontal overlap with biteral crossbite exits


B. the maxillary dentition is forward with respect to basal bone
C. both dental arches are forward with respect to basal bone
D. none of the choices.

40. A narrow maxillary arch with respect to midsagittal plane is said to be in:

A. Protraction C. contraction
B. retraction D. distraction E. both C & D

41. A distal shoe space maintainer is indicated when a primary:

A. incisor is avulsed
B. first molar is prematurely lost
C. second molar is lost after eruption of a permanent first molar
D. second molar is lost before eruption of a permanent first molar

42. Cranial base grows primarily by:

A. intramembranous
B. endochondral
C. combination of intramembranous and endochondral growth at the synchondrosis

43. At age 0 – 5, which has grown the most at 85%?

A. Neurocranium B. maxilla C. mandible

44. The increase in length of the maxilla is due to:

A. apposition on the posterior border of the maxillary tuberosity


B. progressive movement of the entire zygomatic process in posterior direction
C. growth in the median suture
D. Both A & B
E. none of these.

45. Nasomaxillary morphology and growth can be altered. The sutural system adapts to:

A. posterior forces C. transverse forces


B. anterior traction D. all of the choices.

46. The principal force that produce the downward and forward displacement of the

mandible:

A. condylar cartilage C. ramal growth


B. corpus D. soft tissue development E. none of these

47. Addition of alveolar bone, as the teeth erupt and slight additions on the body of the mandible and coronoid notch increase
___ of the mandible

A. length B. height C. width

48. When is the best time to influence, retard and redirect growth?

Downloaded by John Erickson Bangayan (bangayan1903064@ceu.edu.ph)


lOMoARcPSD|19937673

CLUSTER VII: ORTHODONTICS AND PEDIATRIC DENTISTRY

A. after puberty C. just before and during puberty


B. before puberty D. time element is not important

49. Apposition on the posterior border of ramus with remodeling resorption on the anterior border of mandible increases its:

A. antero-posterior dimension B. height C. width

50. In the mouth of a neonate, the lower gum pad is ____ the upper gum pad:

A. ahead B. behind C. in line with

51. Terminal plane is found?

A. distal of D B. mesial of D C. distal of E D. mesial of E

52. Primate space is found:

A. mesial of the lower cuspid and distal of upper cuspid

B. distal of lower cuspid and distal of upper cuspid


C. mesial of lower cuspid and mesial of upper cuspid
D. mesial of upper cuspid and distal of lower cuspid

53. Primary molars with multisurface caries ( 3 or more) are best restored using:

A. GIC C. stainless steel crown


B. Posterior composite D. amalgam restoration E. none

54. Enamel thickness of primary teeth is ___ than the permanent teeth:

A. thinner B. of the same thickness C. thicker

55. Primary teeth have pulpal inflammation when the bacterial infested dentin is __ mm from the pulp:

A. 0.6 B. 0.8 C. 1.6 D. 1.8 E. none

56. Formocresol pulpotomy at the site of amputation produces:

A. dentin bridge C. calcified barrier


B. zone of fixation D. calcific globule E. none of these

57. Ellis classification with crown fracture and exposure of the dental pulp:

A. Class I B. Class II C. Class III D. Class IV

58. In inferior nerve block for a child patient, the injection must be made ___ than for an adult patient.

A. slightly higher B. in level C. slightly lower and more posteriorly

Downloaded by John Erickson Bangayan (bangayan1903064@ceu.edu.ph)


lOMoARcPSD|19937673

CLUSTER VII: ORTHODONTICS AND PEDIATRIC DENTISTRY

59. The maximum number of catridges of 2% lidocaine with 1:100,000 epinephrine for a 40 pound child patient is

A. 3 B. 5 C. 7 D. 9 E. 10

60. In inferior alveolar nerve block, the barrel of the syringe should be placed on the other side of the arch:

A. between C & D C. between E & 6


B. between D & E D. on top of D E. none

61. Controlled alteration of voice volume, tone or pace in order to influence and direct the patient’s behavior:

A. TSD B. positive reinforcement C. distraction D. voice control

62. Parent should be allowed inside the treatment room when treating the following:

A. infant to 41 months C. 7 –8 years old


B. handicapped patients D. both A & B E. all of these.

63. In nitrous oxide oxygen sedation, once the patient takes on a distant gaze with sagging eyelids, local anesthesia is given and the
concentration of nitrous oxide oxygen is reduced to:

A. 40-60 B. 30-70 C. 20-80 D. 10-90 E. none of these.

64. A tiny spicule of nonviable bone overlying the crown of an erupting permanent molar.

A. eruption sequestrum C. natal teeth


B. eruption hematoma D. neonatal teeth D. dental lamina cyst

65. White lesion found at the crest of dental ridges:

A. eruption sequestrum C. Bohn’s nodule


B. dental lamina cyst E. Epstein pearl E. none of these

66. Reason why matrix band is difficult to adapt on a primary molar.

A. large interproximal contacts C. exaggerated cervical bulge


B. direction of enamel rods D. none of these

67. Bluish purple elevated area of tissue, which develops a few weeks before eruption:

A. eruption sequentrum C. eruption hematoma


B. ectopic eruption D. none of these

68. Formocresol has mutagenic and carcinogenic potential so the best replacement as medicament for pulpotomy is:

A. electrosurgery C. enriched collagen solution


B. gluteraldehyde D. still formocresol as of late

69. Treatment for young permanent teeth with exposed vital but infected coronal pulp with immature root development.

A. apexification B. apexogenesis C. DPC D. IPC

Downloaded by John Erickson Bangayan (bangayan1903064@ceu.edu.ph)


lOMoARcPSD|19937673

CLUSTER VII: ORTHODONTICS AND PEDIATRIC DENTISTRY

70. Obturation of primary tooth canals is done using:

A. gutta percah B. silver point C. ZOE D. none of the choices

71. Injury to the teeth of children is often accompanied by open wounds of the oral tissues, abrasion of the facial tissues or even puncture
wounds. The dentist must recognize the possibility of the development of:

A. replacement resorption C. hepatitis


B. tetanus D. pulpal death E. none of these.

72. This is partial or complete obliteration of the pulp chamber and canal, which is a frequently observed reaction to trauma :

A. pulpal hyperemia C. external resorption


B. internal resorption D. calcific metamorphosis

73. The most commonly avulsed tooth in both primary and permanent dentitions:

A. mandibular central incisor C. mandibular lateral incisor


B. maxillary central incisor D. maxillary lateral incisor

74. Splinting of avulsed tooth should allow some degree of mobility. Rigid stabilization seems to stimulate:

A. internal resorption C. replacement resorption


B. external resorption D. calcific metamorphosis

75. Stabilization of replanted avulsed teeth without complications:

A. 2-3 days C. 7-14 weeks


B. 7-14 days D. 2-3 weeks

76. When operative or surgical procedure are undertaken for the mandibular primary molars:

A. inferior alveolar nerve must be blocked


B. nasopalatine nerve block is indicated
C. infiltration technique is sufficient

D. none of the choices

77. Bilateral inferior alveolar nerve block, especially in younger children is:

A. routinely done C. discouraged unless absolutely necessary


B. made with caution D. none of these

78. Children who receive an inferior alveolar injection may bite the lip, tongue and inner surface of the cheek. The resulting ulceration is
known as:

A. ulcerative, necrotizing zone C. ulcer


B. traumatic ulcer D. none of these.

79. Method of early examination where dentist and parent are seated face to face with their knees touching. The upper legs form the
examination table.

A. knee position C. knee to arm position


B. bent position D. knee to knee position

80. Delayed development in the milestones of social behavior, solitary play and withdrawal from other people, absence of eye contact,
rote and compulsive behavior which if interrupted may provide temper tantrums are characteristics of:

Downloaded by John Erickson Bangayan (bangayan1903064@ceu.edu.ph)


lOMoARcPSD|19937673

CLUSTER VII: ORTHODONTICS AND PEDIATRIC DENTISTRY

A. Schizophrenia C. ADHD
B. Autism D. ADD E. mental retardation

81. Its purpose is to gain the attention of a highly oppositional child.

A. Retraining C. hypnodontics

B. Aversive conditioning D. behavior shaping E. none of these

82. Extensive fracture of the crown involving considerable dentin but not the dental pulp. Ellis and Davey classification is:

A. I B. II C. III D. IV E. none of these.

83. The use of calcium hydroxide as a pulp capping agent and as a primary pulpotomy medication is contraindicated because
of the development of:

A. calcific metamorphosis C. internal root resorption


B. abscess D. necrotic pulp E. none of these.

84. Communicative management technique:

i. distraction III. Positive reinforcement


ii. voice control IV. Physical restraint
A. I & II B. I & III C. I & IV D. I, II & III E. II, III & IV

85. Euphemism or word substitute for rubber dam:

A. rack B. button C. raincoat D. cloth E. none of these.

86. A callused or unusually clean digit suggests:

A. cheek biting B. finger sucking C. finger biting D. toe biting

87. Clubbing of the fingers or a bluish color in the nail beds suggests:

A. Rheumatic heart disease C. leukemia


B. Congenital heart disease D. Anemia E. none of these

88. Interested in the dental procedures, laughing and enjoying:

A. Definitely negative C. Positive


B. Negative D. Definitely positive

89. Sudden firm commands used to get the child’s attention:

A. HOME C. modulated voice


B. Voice control D. flexibility

90. The blood filled cyst is most frequently seen during eruption of primary second molar:

Downloaded by John Erickson Bangayan (bangayan1903064@ceu.edu.ph)


lOMoARcPSD|19937673

CLUSTER VII: ORTHODONTICS AND PEDIATRIC DENTISTRY

A. Eruption hematoma C. Eruption sequestrum


B Ectopic eruption D. all of the above

91. Injury to the teeth of children is often accompanied by open wounds, abrasion of facial tissues and puncture wounds. The
dentist must check the child’s immunization status regarding:

A. MMR B. tetanus C. Hib D. polio

92. Treatment of choice when the patient is seen within an hour or two after the injury of tooth #21, vital exposure is small and
sufficient crown remains:

A. DPC B. pulpotomy C. apexogenesis D. apexification E.pulpectomy

93. Treatment for non-vital tooth with open apex:

A. IPC C. Pulpotomy
B. DPC D. Pulpectomy E. Apexification

94. The amount of pulp tissue to be removed in formocresol pulpotomy procedure

A. half of the coronal pulp


B. coronal pulp tissue down to the cervical constriction of each root canal
C. infected portion of coronal pulp tissue

D. total removal of pulp tissue including radicular pulp

95. Primary second molars usually erupt during ages:

C. 8-14 months C. 30-36 months


D. 14-20 months D. 36-48 months E. 20-30 months

96. A 4-year old child has a traumatized central incisor with a Class III (Ellis) fracture. The injury occurred about one month ago, and
examination indicates that the pulp is necrotic. There are no other pathologic findings. Treatment of choice is:

A. watchful observation
B. extraction and use of a space maintainer
C. pulpectomy and root canal filling using gutta-percha points and cement
D. endodontic treatment and root canal filling with a resorbable paste.

97. Definitely negative behavior.

A. Frankl 1 B. Frankl 2 C. Frankl 3 D. Frankl 4

98. The growth movement of the mandible is complimented by the growth of the maxilla, which is:

A. down and forward C. downward and backward


B. forward only, D. upward and backward

99. This is a contraindication of HOM

Downloaded by John Erickson Bangayan (bangayan1903064@ceu.edu.ph)


lOMoARcPSD|19937673

CLUSTER VII: ORTHODONTICS AND PEDIATRIC DENTISTRY

C. spoiled child
D. defiant child
C. very young patient
D. uncooperative behavior but capable of understanding

100. In infant oral care, clean mouth with gauze after feedings and at bed time done during:

A. 0-6 mons B. 6-12 mos C. 12-24 mos. D. 24-36 mos

“There is no revenge so complete as forgiveness.”

Ortho-pedo

When force is applied, bone reflexes that produces deformation of crystalline materials and allows flow of electric current necessary for
tooth movement

Peizoelectricity

Cephalometrics is useful in assessing

Tooth to tooth, bone to bone, tooth to bone

Most essential factor related to correction of an anterior crossbite

Space available mesiodistally

Frankfort-horizontal is a reference plane constructed by joining

Nasion and Sella

Palatal expansion appliance is

Not A space maintainer

Space closure is least likely to occur after loss of which of the ff teeth

Primary maxillary central incisors

Crowding that occurs with mandibular incisors after age 18 is most often result of

Physiologic maturational change

Undesirable side effect most commonly associated with use of a finger spring to tip the crown of a tooth

Tendency for the root apex to move in the direction opposite from the crown

After age 6, greatest increase in size of the mandible occurs

Distal to the first molars

Distal shoe space maintainer is indicated when a primary

Second molar is lost prior to eruption of a permanent first molar

V principle of growth is best illustrated by

Downloaded by John Erickson Bangayan (bangayan1903064@ceu.edu.ph)


lOMoARcPSD|19937673

CLUSTER VII: ORTHODONTICS AND PEDIATRIC DENTISTRY

Mandibular ramus

Anterior crossbite in the primary dentition usually indicates a developing

Class iii maloclussion

Mixed dentition analysis determines:

Space available versus space required

Major disadvantage of treatment using cervical headgear

Extrusion of maxillary molars

Asymmetrical anterior open bite with normal posterior occlusion is characteristic of

Thumb sucking

If the norm for the cephalometric angle SNA is 82 degree, a patient's reading of 90 degree for SNA most likely indicates

Thumb sucking

Maloclussion should be treated between the ages of

None

Rx of a 5 year old patient show permanent maxillary first molars inclined mesially with resulting resorption of the distal portions of the
roots of primary second molars. Condition is

Ectopic eruption

Fractured maxillary anterior teeth generally occur most often in children with

Class Ii division 1 malocclusion

On the first dental visit, basic fear of a child under age 2 is related to

Anxiety over being separated from parent

A child with down's syndrome is

Affectionate, fearful of quick movements, capable of learning dental procedure

A 3 1/2 year old child has an acute fever, diarrhea, oral vesicular lesions and gingival tenderness.

Acute herpetic stomatitis

Transillumination of soft tissues is useful in detecting which

Sialolithiasis

Least likely to result from persistent long term thumb sucking

Deep overbite

Least desirable method used in child management

Gift before treatment

A child who is reluctant to accept dental treatment, exhibit negative attitude but not so pronounced is

Frankl II

Amount of pulp tissue to be removed in formocresol pulpotomy procedure

Downloaded by John Erickson Bangayan (bangayan1903064@ceu.edu.ph)


lOMoARcPSD|19937673

CLUSTER VII: ORTHODONTICS AND PEDIATRIC DENTISTRY

Coronal pulp tissue down to the cervical constriction of each root canal

A sequelae likely in a child with a history of generalized growth failure in first six months of life

Dentinogenesis imperfecta

Average age at which calcification of crowns of permanent central incisors is completed

4-5 years of age

Amalgam is most often the restorative material of choice for primary teeth. Most important modification in its use for children is in

Cavity preparation

Most frequent cause of fracture of a root tip during extraction of a primary molar is

Root resorption between apex and bifurcation

Most common cause of sinus tracts on gingival tissue of children is

Chronic periapical abscess

Light, bluish, dome shaped lesion on the inside lip of a 2 year old child is most likely a

Mucocele

Primary second molars usually erupts during

20-30 months

A child in long term remission of acute leukemia has dental problems characterized by unusual susceptibility to

Oral infection

Contraindication of HOM

For very young patient

Induce formation of apical closure of young permanent molar using CAOH

Apexification

Growth of the mandible is complimented by the growth of the maxilla, which is

Upward and backward

Direction of displacement of the mandible in an individual with developing class ii maloclussion

Down and backward

Child refuses to accept treatment or open the mount minimally

Frankl 2

Definetely negative behavior

Frankl 1

Child has good rapport with the dentist

Frankl 3

Treatment option contraindicated in patients who are not able to breathe nasally

Nitrous oxide and oxygen inhalation

Downloaded by John Erickson Bangayan (bangayan1903064@ceu.edu.ph)


lOMoARcPSD|19937673

CLUSTER VII: ORTHODONTICS AND PEDIATRIC DENTISTRY

Treatment option for patients who have sustained extensive orofacial and dental trauma

General anesthesia

With a flush terminal plane, permanent first molars will

Erupt immediately into an end to end relationship

Arch shape and symmetry are best evaluated from the

Frontal photograph

Bodily force that moves the central incisor mesially produces

Stretching of the periodontal fiber on the distal side

Infant oral care, clean mouth with gauze after feedings and at bed time done during

0-6 mons

Space differential between combine width of CDE and 345 is

Positive

Bite plane therapy for deep bite, active Hawley's Plate are examples of

Limited corrective orthodontics

Normal sequential events from fertilization to death

Development

___ will give rise to tissues that will become the mandible

1st branchial arch

Emphasized how cartilage of nasal septum during growth paced the growth of maxilla

Scott's hypothesis

Skeletal tissues grow in response to soft tissue growth

Moss' hypothesis

Cranial vault grows primarily by cartilage growth at

Fontanelles

Cranial base grows primarily by ___ growth and synchondrosis

Endochondral

Over 90% of brain growth is achieved by age

Neurocranium follows what Scammon's curve

Neural

Growth of brain case is primarily by proliferation and ossification of

Sutural connective tissue

Nasimaxillary complex is hafted to the cranium by the ff sutures except

Downloaded by John Erickson Bangayan (bangayan1903064@ceu.edu.ph)


lOMoARcPSD|19937673

CLUSTER VII: ORTHODONTICS AND PEDIATRIC DENTISTRY

Frontotemporal

As the maxilla decends. There is ____ on the orbit floor, ___ on the nasal floor and ___ on the inferior palatal surface

+-+

Increase in length of maxilla is due to

Apposition on the post border of mx tuberosity and progressive movement of entire zygomatic process in posterior direction

Mechanism for maxillary growth

Sutural, ossification, surface apposition:resorption, translation

Nasomaxillary complex mechanism of growth

Intramembranous

Sutures of nasomaxillary complex are oblique and more or less parallel with each other, thus growth in these areas would serve to displace
the maxilla

Downward and forward

The maxilla is displaced downward and forward by growth in ___ parts of the bone

Posterior and superior

What characteristic growth pattern is shown in the vertical section, through the coronoid process

+ lingual
- half of the buccal surface

What characteristic growth pattern is shown in the vertical section through the basal Part of the ramus

- lingual
+ on buccal surface

When teeth are lost, alveolar bone

Resorbs

Mandibular growth follows what growth curve

General

At age 5-10 mandible is ___ completed

65%

Predominant direction of mandibular growth is

Superior and posterior

Displacement of the mandible occurs in what direction

Downward and forward

Increase in mandibular length is accomplished by

+posterior border
- anterior border of ramus

Growth of the posterior border of the mandible with additive growth at the ends of the V increases the terminal points. This increases the
___ of the mandible

Downloaded by John Erickson Bangayan (bangayan1903064@ceu.edu.ph)


lOMoARcPSD|19937673

CLUSTER VII: ORTHODONTICS AND PEDIATRIC DENTISTRY

Width

Condylar growth with significant alveolar growth increase the __ of the mandible

Height

Some mandibles grow more forward than downward. If there is predominance of forward growth ___ is the result

Class III

When is the best time to influence, retard or redirect growth

Just before and during puberty

Order form most rapid to least amount of growth for the cranium

Depth, width, height

Continued apposition of alveolar bone on the free borders of the alveolar process as the teeth erupts increases the ___ of maxilla

Height

Order form greatest to least incremental change for the face

Height, depth, width

At age 0-5, which has grown the most 85%

Neurocranium

Because if the differential growth of the maxilla and the mandible at age 0-5 the normal profile at birth is

Conves

Maxillary inter canine width is completed in girls at age

12

Second peak for girls

7-9

For boys, maximum condylar changes concurrently with sutural and skeletal height peaks 3 years later that for girls

False

Treatment timing must be based on the individuals own pattern of growth

True

Primate space if found:

Mesial of the upper cuspid and distal of lower cuspid

Normal terminal plane relationship

Flush/straight

Most common sequence of eruption in upper arch

6124537

Tooth begins eruptive movement

Stage 6

Downloaded by John Erickson Bangayan (bangayan1903064@ceu.edu.ph)


lOMoARcPSD|19937673

CLUSTER VII: ORTHODONTICS AND PEDIATRIC DENTISTRY

Periapical lesions, pulpitis and pulpotomy of a primary molar will __ the eruption of successor premolar

Hasten

If a primary tooth is extracted prior to the onset of permanent eruptive movement, the permanent tooth is likely to

Delayed

Which one is an incorrect description of nolla's stages classification

Stage 6 crown almost completed

Space differential in the anterior segment (BA/AS) - 21/12 gives

Negative space

Proper crown angulation of a permanent incisor is one where the gingival portion of the long axis of the crown should be ___ to the incisal
portion

Distal

Rotated teeth occupy less space within the arch

False

Plane of occlusion for permanent teeth

Flat

PEDO

____ is most common childhood disease. it is ___ times more common than asthma.
tactile evaluation with an _____
visual examination with good ____; tooth must be ___ and ___

caries, 5, explorer, light, dry, clean

____ is the radiograph of choice to diagnose inter proximal decay when contacts are closed.

proximal caries begins just below contact area and spreads ___ then ____.

t/f: surface susceptibility is the result of contact & it differs between 1st and 2nd primary molars

bitewing, laterally, gingivally, true

surface susceptibility of 1st primary molar:

____(distal/mesial) surface experiences much more decay b/c of firm contact w/ the 2nd primary molar whereas there is a space b/w it
and the canine

distal

surface susceptibility of 2nd primary molar:

____(mesial/distal) surface has caries incidence ____x more than the other surface b/c there is no contact until the 1st permanent molar
comes in

following establishment of distal contact after eruption of 1st permanent molar, little difference in susceptibility remains b/w the two
surfaces

mesial, 10

in an 8 year old patient, the ____(mesial/distal/both) surfaces has a higher chance of surface susceptibility in the 2nd primary molar

both (has contacts on both sides of the molar now)

Downloaded by John Erickson Bangayan (bangayan1903064@ceu.edu.ph)


lOMoARcPSD|19937673

CLUSTER VII: ORTHODONTICS AND PEDIATRIC DENTISTRY

t/f: caries susceptibility of the distal surface of the 1st molar essentially the same as that of the mesial surface of the second molar

true

_____ caries is more prevalent than proximal caries until age 7

among primary teeth, the ____ molars, both maxillary and mandibular, are more susceptible to occlusal caries because of the more
fissured and pitted occlusal surface

occlusal, second

t/f: smooth surface caries can occur on the buccal, lingual, mesial, and distal

smooth surface caries begins as a ___ ___, early decalcification (which is reversible at this stage) and progresses to frank cavitation on the
enamel surfaces

true, white spot

before the age of 7, a child is more likely to get _____ caries.

after the age of 7, a child is more likely to get ____ caries

occlusal, interproximal

when charting caries, circle the tooth # or letters that are present in _____

chart new carious lesions in _____

chart existing amalgam in _____

chart existing composite as a ____ ___

chart existing sealant was a blue "___"

chart existing SSC with ____ ___

blue, red, blue, blue outline, S, blue stripes

a ____ ____ should take into consideration:


-dental age
-patient's behavior
-caries risk assessment (CAT)
-parental compliance and likelihood of timely recall
-the patient's ability to cooperate for treatment
-finances

treatment plan

restorative txt should be based upon the results of an appropriate clinical exam and ideally be apart of a comprehensive txt plan. the ____
____ should take into consideration:

dental age
patient's behavior
caries risk assessment: CAT
parental compliance and likelihood of timely recall
the patient's ability to cooperate for treatment
finances

treatment plan

steps to develop txt plan:

____ _____ and ____:

Downloaded by John Erickson Bangayan (bangayan1903064@ceu.edu.ph)


lOMoARcPSD|19937673

CLUSTER VII: ORTHODONTICS AND PEDIATRIC DENTISTRY

-complete eval of pt's medical, dental, and social hx


-radiographic eval
-clinical eval: extra-oral, intra-oral, charting caries
-risk assessment: high, moderate, low
-diagnosis

____ ____:
-must develop one for each tooth including alternatives
-discuss plan w/ faculty member assigned to you
-present plan to parent in lay terms
-get informed consent from parent inside the clinic, not in waiting room in order to be in compliance with HIPAA regulations
-have parent sign txt plan in computer and print them a copy

data gathering and diagnosis, treatment plan

t/f: it is acceptable to present the treatment plan to the pt's parent in the waiting room

false

treatment options:

1. ______ dentistry:
-provide anticipatory guidance
-establish dental home for pt
-oral hygiene instructions
-dietary counseling
-fluoride: varnish, foam, silver diamine, mouth rinses, toothpaste, -water
-sealants

2. _______ dentistry:
-class I composite or amalgam
-class II composite or amalgam
-SSC
-Strip crowns
-esthetic crown

3. ___ therapy:
-IPC
-pulpotomy
-pulpectomy

4. ____

5. ____ ____

preventative, restorative, pulp, extractions, space maintenance

when considering dental age, a 3 year old patient who has a carious lesion on #L should probably be restored with a ____

crown

t/f: when filling out a pediatric treatment planning sheet, you should write something for each tooth

t/f: there is a section at the bottom of the form to include extra treatments such as nitrous, sedation, etc.

t/f: the student dentist, faculty member, and parent must all sign the form, indicating informed consent

true (fluoride, active surveillance/recall, prep/restoration, diagnosis, alternative), true, true

advantages of rubber dam to the patient:


1. ____: prevents swallowing and aspiration of materials
2. ____: separates tooth and procedure and mouth
3. protects ____ ____, especially wiggly tongues

Downloaded by John Erickson Bangayan (bangayan1903064@ceu.edu.ph)


lOMoARcPSD|19937673

CLUSTER VII: ORTHODONTICS AND PEDIATRIC DENTISTRY

4. helps keep ____ ____


5. allows safe use of caustic materials such as _____
6. prevents ____ breathing during nasal administration of nitrous oxide
7. prevents ___ ___ in mouth

safety, security, soft tissues, mouth open, formocresol, mouth, bad taste

advantages of rubber dam to operator:


1. ___, ____, ____ operating field
2. reduction of _____ ____ by elimination of talking, spitting, swallowing
3. ____ ____ superior to other isolation methods
4. aids ___ and ____ control
5. ____ improved and dentist can work under less stress

dry, accessible, visible, operating time, moisture control, tongue, lip, behavior

rarely rubber dams cannot be placed. list 3 situations.

presence of fixed orthodontic appliance, newly erupted tooth can't retain a clamp, child is unable to breathe through nose

can rubber dams be used in a pt with latex allergy?

yes (but make sure it is latex free)

armamentarium for rubber dam application:

1. ____: 5x5 inch; medium/heavy; dark color better

2. ____: metal or plastic

3. ____: secures rubber dam to mouth

4. _____: must be attached to clamp before placing it in mouth; if it falls into mouth it can easily be inhaled into the lung

5. clamp _____: to seat the clamp onto the tooth

6. rubber dam ____: punch minimal number of holes required

rubber dam, frame, clamp, floss, forceps, punch

____ clamps have jaws Angled toward the gingiva, so these will grip better on a partially erupted tooth (ex. 14A, 8A); have long very
traumatic points to help in placement of clamp

most commonly used clamps for ____ molars are 7, 8, and 14

most commonly used clamps for _____ molars are 3, 5, and 7

t/f: need anesthesia to apply rubber dam clamp

A, permanent, primary, true

for class I restorations, how many teeth are isolated/clamped?

for SSCs, you can make a ____ or "____ ___" while allows for quick placement of the dam but does sacrifice a little moisture control,
especially in the ____ (maxilla/mandible)

1, slit, Mickey Mouse, mandible

you should punch the rubber dam _____(on/off) the frame

on (to make positioning the holes and punching them easier)

placing the rubber dam:

Downloaded by John Erickson Bangayan (bangayan1903064@ceu.edu.ph)


lOMoARcPSD|19937673

CLUSTER VII: ORTHODONTICS AND PEDIATRIC DENTISTRY

the punched dam should be lightly stretched on the frame. this should be done ______(before/after) placing the clamp to make the
process as smooth as possible. place the clamp using the clamp forces, and check its stability with firm ___ ___ on both sides. once the
clamp is in site, use the fingers to stretch the ____ over the clamp bow and wings. a little lubrication may help. Then if appropriate, stretch
the dam over the remaining teeth to be isolated. secure it _____(posteriorly/anteriorly) with a wedge, piece of dam, or floss. Adjust the
tension of the dam if needed.

if placing composites, ____ the edges of the rubber dam around the gingival margins with a blunt instrument.

remove the whole lot together when you are done, using the forceps

before, finger pressure, hole, anteriorly, invert

Do's or Dont's for a rubber dam??

1. use euphemisms to explain the idea to the child


2. describe the sensations
3. check that nostrils are not covered
4. leave the child with the clamp on the tooth without the dam over it
5. place the clamp in the mouth without a long piece of floss attached

do, do, do, don't (they will bit it, dislodge it, and injure themselves), don't

5 euphemisms to describe rubber dam armamentarium

ring, umbrella, raincoat, string, picture frame

list 2 ways to describe the sensations/noises the child will feel or hear during rubber dam placement

ring will hug your tooth, umbrella/raincoat will make a noise like popcorn

the purpose of placing the rubber dam is to obtain ____ while doing restorative and pulp therapy procedures

the procedure should start with ____ ___ and placement of the appropriate ____

the rubber dam has been placed properly when there is no ____ contamination

the rubber dam must be centered horizontally on the face so that the upper lip is covered but the _____ are not

isolation, local anesthesia, clamp, saliva, nostrils

to punch the rubber dam, you must place it on the frame first. that is because it is _____ ____ ____ ___ _____.

easier to punch the whole

when restoring 1 tooth surface, only punch ____ hole b/c you only need to isolate the minimum number of teeth.

when restoring an _____ lesion and when placing an ____, isolate 1 tooth anterior and 1 tooth posterior.

t/f: when restoring several teeth, you should isolate the whole quadrant

1, interproximal, SSC, true

clamps:

____ is used for first primary molar


____ is used for second primary molar
____ and ____ is used for first permanent molar
_____ is used for partially erupted permanent molar

2A, 3, 14, 7, 14A

a ___-___ inch piece of dental floss should be attached to the hub of the rubber dam clamp before placement in the mouth in case the
clamp is dislodged form the tooth in a pharyngeal direction.

Downloaded by John Erickson Bangayan (bangayan1903064@ceu.edu.ph)


lOMoARcPSD|19937673

CLUSTER VII: ORTHODONTICS AND PEDIATRIC DENTISTRY

the clamp should be seated on the tooth with forceps from _____ (buccal to lingual/lingual to buccal)

t/f: after seating the clamp, remove forceps and use finger pressure to gently press clamp in gingival direction to ensure it is adequately
seated and stable

12-18, lingual to buccal, true

t/f: placing the rubber dam and frame together after placing the clamp is easier than placing all at the same time

true (with dam on frame, stretch most posterior col of RD over the hub and wings of clamp)

the most common reason sealant failure if lack of care for proper _____ of etched enamel from contamination with saliva.

t/f: isolation is easy to achieve even when molars are not well erupted in children

*child's behavior and cooperation also impact isolation efficiency

isolation, false

rapid formation of a tenacious surface coating occurs over etched enamel surfaces exposed to _____. this occurs within a few
____(seconds/minutes) and cannot be removed with water-spray unless the exposure is less than ____ second(s).

the use of ____ bonding agent may help improve sealant retention in areas that are difficult to isolate, but is not a substitute for good
isolation technique.

saliva, seconds, one, hydrophilic

____ and ____ are two methods of isolation traditionally used when placing sealants in pediatric dentistry. Isolite can also be used when
appropriate.

at BCD, we generally use ____ and _____ isolation, unless there is another tooth in the quadrant which requires restorative treatment, in
which case we place the sealant using the rubber dam.

rubber dam, cotton roll, cotton roll, dry angle

sealant placement:

1._____ the tooth: with pumice slurry in prophy cup or toothbrush prophy (*do NOT use prophy paste b/c the glycerin interferes w/ etch)

2. _____ the tooth: then check with explorer

3. _____ the tooth: place dry angle, cotton rolls, and saliva ejectors; always place dry angle over parotid duct

4. ____: w/ phosphoric acid for 15 seconds. cover 2-3mm of cuspal inclines and be sure to cover all grooves

5. ____ the etched surface for 30 seconds with air-water spray. if necessary, change out cotton rolls; do not contaminate w/ saliva

6. _____ for 15 seconds; check that etched enamel has frosted appearance; if not, repeat the etch.

7. apply ____ ____ & air-dry it.

8. ____ bonding agent for 20 seconds

9. apply the _____ to the etched surface. allow its to flow into the pits and fissures. do not obliterate the occlusal anatomy with sealant.
use micro brush or explorer to carry sealant into supplemental fissures. flow it into buccal or lingual groove.

10. cure for ____ seconds minimum. cure buccal/lingual grooves separately for 20 seconds

11. if you have good isolation, delay cure by 10 seconds which allows sealant to penetrate deeper into etched enamel

12. explore sealed surface -- check for voids, retention

13. wipe unpolymerized air-inhibited layer with cotton roll as you remove them from mouth. this layer tastes bad

Downloaded by John Erickson Bangayan (bangayan1903064@ceu.edu.ph)


lOMoARcPSD|19937673

CLUSTER VII: ORTHODONTICS AND PEDIATRIC DENTISTRY

14. evaluate ____

clean, rinse, isolate, etch, rinse, dry, bonding agent, cure, sealant, 20, occlusion

preventative recommendations for pregnant women:


brush teeth twice daily w/ fluoridated toothpaste, especially before bedtime, and floss daily

-increase in sex hormones during pregnancy which can cause an increase in _____ growth
-increase in progesterone can cause decrease in ____ formation; therefor easier for gingiva to become inflamed
-pregnancy ____: painful, affecting mom's ability to eat and sleep, which is not ideal for growing fetus
-pregnancy _____: cyst filled w/ blood vessels; may consider excision and biopsy if painful; otherwise might not want to address

bacterial, collagen, gingivitis, granuloma

preventative recommendations for pregnant women:


consume prenatal vitamins, including folic acid

-increase in sex hormones can cause increase in ___ metabolism


-_____ is important for developing fetus, reduces risk of birth defects such as cleft lip and palate and spina bifida

folate, folic acid

preventative recommendations for pregnant women:


limit foods containing fermentable carbohydrates including juice, soda, sports drinks between meals, sugary snacks including cookies,
crackers, chips to mealtimes only

-frequent between-meal consumption of these foods increases ___ risk


-mom needs to be mindful of cravings and snacking; make good snacking choices like ___ and ____
-advise a diet of foods high in protein, calcium, phosphorous, and vitamin A, C, and D

caries, cheese, unsweetened yogurt

preventative recommendations for pregnant women:


pregnant pts should be educated on effects of xylitol on oral health and encouraged to chew xylitol-contains gum or other xylitol-contains
products, four to five times s day, after consuming food

-increase in sex hormones can cause ____, thereby increasing caries risk
-chewing gum can stimulate ____ flow, rehydrate and neutralize pH
-bacteria cannot break ____ down as a sugar source

xerostomia, saliva, xylitol

preventative recommendations for pregnant women:


morning sickness

-frequent nausea and vomiting can cause ____ of tooth surfaces


-many pregnant women develop acid reflux (no committing): if taking meds like Tums, need to be aware of ___ content
-avoid tooth brushing directly after vomiting as the effect of erosion can be exacerbated by brushing an already demineralized tooth
surface; wait at least ____ minutes before brushing
-can rinse with plain water or water w/ one teaspoon of baking soda
-can use a fluoride-containing mouth rinse (alcohol free) or apply MI paste immediately before bedtime to help remineralize teeth

erosion, sugar, 30

preventative recommendations for pregnant women:


counsel on alcohol/smoking/opioid usage -- harmful effects to fetus

t/f: any amount of alcohol at any stage of pregnancy can affect fetus

true

Downloaded by John Erickson Bangayan (bangayan1903064@ceu.edu.ph)


lOMoARcPSD|19937673

CLUSTER VII: ORTHODONTICS AND PEDIATRIC DENTISTRY

list 6 preventive recommendations to address physiological changes during pregnancy

*emphasize to mom that good oral health for mother can man good oral health for their babies

brush twice daily with fluoride especially before bedtime and floss, prenatal vitamins including folic acid, limit fermentable carbohydrates
and sugary snacks to mealtimes only, xylitol chewing gum, morning sickness, counsel on alcohol/smoking/opioid use

research studies show association b/w periodontal disease and ____ birth and low birth ____. should encourage pregnant women to
maintain their routine recall appts.

research studies show association b/w untreated caries in pregnant mothers and early childhood caries in children (_____ transmission)

premature, weight, vertical

t/f: it is okay to delay treatment just because a woman is pregnant

point to provide pregnant mother:


-routine restorative dental care is recommended to reduce level of caries-causing bacteria in pregnant woman's mouth
-untreated caries can become painful: don't wan't mothers self-medicating
-untreated caries can lead to active infection which can be life threatening, stressing the mother and growng fetus leading to pre-term
labor
-greater risk of vertical transmission and early childhood caries

false

t/f: diagnostic radiographs are considered safe during pregnancy

a pregnant woman's ___ and ___ should be protected from radiation

true, abdomen, neck

positioning pregnant patient:

-never leave patient in supine position, especially in ___ trimester

-pregnant women are at increased risk for ____ ____ as a result of reduced gastro-esophageal sphincter tone and delayed gastric

-about 15-20% of pregnant women in 3rd trimester are at risk for "postural hypotensive syndrome" from lying flat on their back, which
causes the uterus to press on the inferior vena cava and impede venous return to heart which can lead to light headedness. place the pt in
a _____ ____ position; can prop their left side up using a pilot or folded blanket under the hip to move the uterus off the vena cava

third, gastric aspiration, semi-reclined

t/f: nitrous oxide use during pregnancy is controversial

t/f: amalgam, composite resins, glass ionomer, gold and porcelain restorations are some of possible dental restorative materials that can be
used in pregnant women

true, true

____ and ____ are two local anesthetics that are safe to use in pregnant pts, but ____ should be avoided b/c it is too long lasting

____ if safe for pain management; ____ and ____ should be avoided

the _____ and _____ family of systemic antibiotics should be avoided

lidocaine, mepivacaine, marraine, acetaminophen, ibuprofen, NSAIDS, tetracycline, fluoroquinolones ("floxacin")

2 indications for P&F sealants:

1. ____ erupted primary or permanent teeth w/ ___, uncoalesced fissures


2. ____ caries on occlusal fissures

newly, deep, incipient

Downloaded by John Erickson Bangayan (bangayan1903064@ceu.edu.ph)


lOMoARcPSD|19937673

CLUSTER VII: ORTHODONTICS AND PEDIATRIC DENTISTRY

2 contraindications of P&F sealants:

1. not indicated for use on a tooth with existing ____ caries


2. not indicated when ____ covers part of the tooth

interproximal, operculum

technique of application of P&F sealants:

1. ___ the teeth: use plain pumice & water; not prophy paste; explorer to clean out grooves (prophy paste has glycerin, mouthwash has
essential oils-- so DON'T use to clean)

2. ____: most critical step for retention; rubber damn is gold standard; cotton rolls & dry-angles are more practical

3. ____: 15 seconds with 35% phosphoric acid; dry tooth surface, gentry rub this onto the tooth surface and include cusp inclines, pits, and
fissures

4. _____: must remove acid and reaction precipitates from etch; air-water spray for minimum of 30 sec per tooth using high volume water
and suction

5. maintain ____: replace cotton rolls; re-etch if contaminated w/ saliva; moisture will interfere with bonding

6. ____ the etched enamel: critical step (will fail if not done); 15 seconds for each tooth; enamel should look frosted

7. ____ agent: use underneath sealants to enhance retention; hydrophilic properties; air dry 5-7 seconds to reduce pooling and to
evaporate ethanol or acetone, cure for 20 seconds

8. ____ application: place minimal amount; use explorer or micro brush to apply to secondary tissues, delay cure for 10 seconds for better
penetration, cure for 20 sec minimum

9. ____ the sealant: use explorer to look for voids and retention; occlusion

10. wipe the ____ surface w/ cotton roll as you remove them from mouth: this layer tastes bad

clean, isolate, etch, wash, isolation, dry, bonding, sealant, check, unpolymerized

for both mandibular and maxillary teeth, apply the sealant at the ____(mesial/distal) part of the occlusal surface and allow it to flow in the
opposite direction.

don't forget the ____ groove of mandibular molars

don't forget the ____ groove of maxillary molars

mesial, buccal, lingual

at recall exams, check sealant for wear, marginal breakdown, and leakage. caries can develop rapidly in the protected niche under a leaky
sealant. a partly retained sealant _____ (increases/decreases) the risk of caries.

the need for repair is greatest in the first ____ months after initial placement

increases, 6

application of sealant to the developmental pits and fissure areas of teeth is one of the preventive measures that inhibit the initiation of
dental _____ by forming a ______ barrier.

caries, nonpermeable

The class II composite restoration is used to restore small carious defects that involve the proximal surfaces of posterior teeth. this
restoration offers the advantages of improved ____ and ____ of tooth structure.

esthetics, conservation

Downloaded by John Erickson Bangayan (bangayan1903064@ceu.edu.ph)


lOMoARcPSD|19937673

CLUSTER VII: ORTHODONTICS AND PEDIATRIC DENTISTRY

5 steps of class II composite restoration for primary molars:

1. ____ and place rubber dam


2. _____ tooth surface
3. ____ placement
4. _____ placement
5. _____ the composite

anesthetize, prep, matrix, composite, finish

class II composite preparations:

-prep should be ____ (more/less) shallow and narrow than class II amalgam
-isthmus width is approximately ____ of the intercuspal width with occlusal walls flowing smoothly into the proximal box with proximal box
margins in cleansable areas
-a ____ should be inserted into the embrasure b/w the tooth to be prepped and the adjacent tooth to protect the interproximal rubber
dam material from the action of the high speed bur. the wedge also better defines the gingival extent of the contact area.
-establish proximal box and break contact. the axial wall is placed just into dentin so that the horizontal depth of the axial wall is ___mm.
the axial wall is slightly ____(convex/concave) B-L and parallel to outer enamel surface
-the box is ____ and the buccal and lingual walls of the prep are parallel to buccal and lingual surfaces of the tooth. this conserves tooth
structure and provides retention for the proximal segment.
-all internal line angles are _____(rounded/squared) to decrease the stress on the amalgam and increase bulk of amalgam on pulpoaxial
region
-proximal margins are extended so that the tip of an _____ can just reach all margins
-the gingival floor is just below contact area to preserve as much enamel as possible
-remove _____ and all debris with slow speed round bur

more, 1/3, wedge, 1, convex, truncated, rounded, explorer, caries

class II composite placement:

1. ____: apply 35% phosphoric acid w/ micro brush or syringe for 15 seconds

2. ____ and ____: rinse for 20 seconds w/ air-water spray

3. ____ agent: cure for 20 seconds - surface must look shiny

4. _____ placement: must be placed in increments of 2mm for cure to be effective and to reduce shrinkage

5. remove matrix band

6. adjust occlusion with articulating paper

etch, wash and dry, bonding, composite

4 indications for SSC

1. _____ decay:
-related to capacity of tooth to retain diff restorations
-extensive occlusal decay that undermines 1 or more surfaces with more than 1/3 root remaining
-decay on 3 or more surfaces of primary tooth with more than 1/3 root remaining
-proximal decay that crosses line angle of primary tooth with 1/3 root remaining
-proximal decay on 1st primary molar prior to eruption of first permanent molar

2. following _____ therapy


-primary tooth that has had pulpotomy or pulpectomy
-young permanent tooth that has had pulp therapy but is not mature enough to receive a cast restoration

3. ____ defects
-hypoplasia, AI, DI

4. crown _____
-especially primary teeth and posterior permanent teeth

Downloaded by John Erickson Bangayan (bangayan1903064@ceu.edu.ph)


lOMoARcPSD|19937673

CLUSTER VII: ORTHODONTICS AND PEDIATRIC DENTISTRY

extensive, pulp, developmental, fracture

An SSC is indicated in certain situations of extensive decay:

1. extensive occlusal decay which undermines ____ or more surfaces with 1/3 of the root remaining.

2. decay on ___ or more surfaces of a primary tooth with 1/3 of the root remaining

3. proximal decay which crosses the ___ ___ of a primary tooth with 1/3 of root remaining

4. proximal decay on a first primary molar prior to the eruption of the ____ ____ ____. (class II amalgams in these teeth do not remain
intact for more than 2-3 years and these teeth are not expected to exfoliate within that time frame)

1, 3, line angle, first permanent molar

tooth prep for SSC:

1. local anesthesia and rubber dam

2. occlusal reduction: remove enough tooth structure (usually ___-___mm) with a 330 or 331 for clearance; should be uniform in depth
and follow occlusal anatomy

3. secondary occlusal reduction: 45 degree angle that is ___-___mm wide; more pronounced on stamp cusp; narrows occlusal table

4. proximal reduction: remove enough tooth structure with ___ A diamond slightly convergent to break contacts so the SSC can pass w/ the
teeth as it is seated; explorer should pass through

5. buccal and lingual reductions: only reduce areas that interfere with seating of the crown, not entire surface

6. ____ line angles: blend proximal onto B and L

7. remove _____ and perform any pulp therapy as necessary


-remove all w/ slow speed bur; only leave some caries if doing IPC

8. apply ___ protection: CaOH or ZOE on deep areas of dentin

8. check occlusal _____: remove rubber dam, check for 1mm

1-1.5, 1-2, 1/8, round, caries, pulp, clearance

purpose of tooth prep for SSC is to remove ____ and only that tooth structure which may interfere with the ____ of the SSC

caries, seating

t/f: SSC prep has no defined margins, no finish lines, and no inherent retentive features

true

An SSC is retained by ____ or "bending" the margins inward so that the crown will be held on mechanically by undercuts inherent to the
tooth's anatomy.

crimping

basic selection criteria for SSC:

1. restore tooth's original ____ dimension


2. restore ____ contact
3. SSC should fit ____(loosely/tightly) over tooth without binding against tooth structure
4. margin of crown should be ____(supragingival/subgingival)

MD, occlusal, loosely, subgingival

Downloaded by John Erickson Bangayan (bangayan1903064@ceu.edu.ph)


lOMoARcPSD|19937673

CLUSTER VII: ORTHODONTICS AND PEDIATRIC DENTISTRY

preparing SSC for seating:

1. establish MD width: use the ___ ____ for adjusting crown MD width

2. establish height and length


-trim margin w/ crown and collar scissors such that marginal ridges of crown are even w/ marginal ridges of adjacent teeth and gingival
margin of crown is ____mm below gingival all around the tooth

3. contour/crimp margins to adapt more closely to cervical construction of tooth: use ____ plier
-crown should only be removed by _____
-crown should "_____" onto the tooth

4. establish fit of SSC: place crown and check margins


-man: seat L to B
-max: seat B to L

5. finish margins and polish

howe plier, 1, crimping, discoid-cleoid, snap

purpose of cementing SSC is to fill in gaps which will exist between crown and tooth surfaces, especially ____, and seals the _____ of the
crown since they will not be perfectly adapted in all areas

occlusal, margins

the cement we use for SSC crown cementation is ____ which is a reinforced ZOE cement.

t/f: no specific powder-liquid ratio, just mix to "mayonnaise like" consistency

1/f: fill crown entirely

t/f: cement takes 3-5 min to set, then can clean excess with explorer or scaler and knotted floss

t/f: ZOE creates burning sensation on contact w/ skin and mucous membrane

temrex, true, true, true, true

when prepping an SSC, any excessive buccal or lingual ___ ____ should be reduced resulting in a feather edge finish line. this allows the
crown to be seated.

excessive reduction removes ____ required for retention of the crown

cervical bulge, undercuts

when selecting an SSC crown, the crown should restore the _____ width of the original tooth to preserve the integrity of the dental arch.

exception: leave open contact if it was there beforehand

M-D

If an SSC does not seat until it is level with the adjacent teeth, the SSC might be too (long/short).
the occlusal reduction of the tooth might have been insufficient or a ledge may be keeping the crown from seating.

if the crown is too long, it might cause gingival ____.

if the crown is too long, reduce the margins of the crown with curved crown and bridge ____ in the areas impinging of the soft tissue until
the crown can be seated into proper occlusion.
The margins should be trimmed to 1mm into the gingival sulcus.

long, irritation, scissors

be able to evaluate errors in SSC prep

Downloaded by John Erickson Bangayan (bangayan1903064@ceu.edu.ph)


lOMoARcPSD|19937673

CLUSTER VII: ORTHODONTICS AND PEDIATRIC DENTISTRY

...

when inserting an SSC into the mouth:

_____: seat lingual to buccal

____: seat buccal to lingual

mandibular, maxillary

______ of an SSC provides a tight fit of the crown to the tooth and the proper anatomical form of the crown to protect the gingiva and
restores contact areas where appropriate.

contouring

when an SSC is properly fitted, it should ____ into place when being seated, should fit _____, and appropriate contact areas and occlusion
should be restored.

snap, tightly

steps of strip crown (Unitek):

1. select ____ of composite resin, then place rubber dam

2. select primary incisor crown from with a MD width ____ to that of tooth to be restored

3. remove decay w/ large round bur & do pulp therapy if needed

4. reduce incisal edge ___mm using fine tapered diamond

5. reduce interproximal spaces 0.5-1mm to allow crown form to slip easily over tooth. IP walls should have a ____ edge.

6. reduce facial surface 0.5-1mm and lingual surface 0.5mm. create feather edge margin; round all line angles

7. place small undercut on ____ (facial/lingual) surface in gingival third of tooth with 1/4 round bur or inverted cone. serves as mechanical
lock.

8. trim crown former to CEJ. it should fit ____mm below the gingival crest and be of comparable height to adjacent teeth. Collar has to be
removed so that the form will fit over the prep.

9. punch a small hole on the ____(facial/lingual) surface near incisal edge w/ an explorer to act as a vent for the escape of trapped air as
the crown is placed

10. place calcium hydroxide or glass ionomer over deepest areas of prep & etch remaining enamel 15-60 seconds. rinse and dry, apply
dentin-bonding agent.

11. fill crown former _____ full w/ resin, make a well in the center, and seat onto tooth. use a _____ motion and don't apply pressure
directly on the ____ of the crown former. remove gingival excess with explorer. make changes to positioning prior to curing.

12. cure from F and L

13. remove celluloid former w/ round bur on lingual aspect so that any damage done to the restoration is on the back side of the tooth.

14. remove rubber dam and evaluate occlusion. Facial surface should not be disturbed b/c you will be unable to recreate the glassy finish..
fine diamond and abrasive disks on lingual surface.

shade, equal, 1.5, feathered, facial, 1, lingual, 2/3, rocking motion, incisal edge

the undercut created on the facial surface of a strip crown prep serves as a _____ ____.

mechanical lock

t/f: when placing a strip crown, changes to the position and alignment of the crown can be changed after curing the composite.

Downloaded by John Erickson Bangayan (bangayan1903064@ceu.edu.ph)


lOMoARcPSD|19937673

CLUSTER VII: ORTHODONTICS AND PEDIATRIC DENTISTRY

false (all changes should be made before the composite is cured)

indications for anterior restorations on primary teeth:


-anterior decay, after trauma
-challenge: check restorability & root resorption

restorative options:
1. _____ fillings
pro: conservative
cons: case selection important, technique sensitive, recurrent caries is an issue

2. ____ crown
pros: esthetic, more conservative than other esthetic options
cons: highly technique sensitive, risk of fracture

3. ____
pros: easier to fit than other crowns, cost effective
cons: esthetics

4. pre-_____ SSC
pros: improved esthetics over SSC
cons: esthetics, more aggressive prep than SSC

5. _____
pros: great esthetic option
cons: difficult to fit, technique sensitive, costly

composite, strip, SSC, veneered, zirconia

______ is the removal of potentially inflamed coronal portion of the pulp, leaving healthy radicular pulp tissue intact, followed by txt of
pulp stumps with a medicament

*remove b/c the coronal pulp tissue adjacent to the carious exposure may show evidence of inflammation which could proceed to
degenerative change. removal of this tissue leaves essentially normal pulp tissue in the roots.

pulpotomy

steps of pulpotomy:

1. local anesthesia and rubber dam

2. ____ opening: occlusal approach w/ high speed bur; remove superficial caries first then all peripheral decay before removing caries
directly above pulp chamber to minimize contamination

3. access to _____: with high speed bur, remove dentin covering the pulp chamber; remove entire roof; there should be hemorrhage (good
sign!!! b/c remaining pulp tissue is vital); pulp must bleed (indication of vitality) in order to continue

4. removal of pulp tissue from chamber: use sharp ___ excavator to excise tissue at orifices to root canals

5. evaluate pulp tissue: indicates health status of pulp; for pulpotomy, the primary tooth must be vital.

6. _____ control: use water dampened cotton pellets with light to moderate pressure over pulp stumps; tissue tags or remaining pieces of
pulp tissue in chamber are most common reason for uncontrollable hemorrhage. continued hemorrhage w/ no tissue tags means there
could be inflammation that extends further than the coronal portion.

7. placement of _____: formocresol (1:5 dilution); place drop on clean, dry cotton pellet and squeeze out excess with 2x2 glaze. place pellet
over pulp stumps for ____ minutes. do NOT leave pellet in tooth. pulp stumps brown/black after removal of pellet.

8. placement of ____ _____: zinc-oxide and eugenol (ZOE); material will resorb if any gets in the roots; mix to doughy consistency and
compact w/ wet cotton pellet; base will seal the tooth against leakage.

access, pulp chamber, spoon, hemorrhage, medicament, 5, pulp paste

Downloaded by John Erickson Bangayan (bangayan1903064@ceu.edu.ph)


lOMoARcPSD|19937673

CLUSTER VII: ORTHODONTICS AND PEDIATRIC DENTISTRY

when trying to maintain hemorrhage control during a pulpotomy, ____ _____ or remaining pieces of pulp tissue in the chamber are the
most common reason for uncontrollable hemorrhage.

continued hemorrhage in the absence of tissue tags is a likely indicator that the ______ extends further than the coronal portion, either
partially or completely into the radicular tissue; if this is the case, other txt options are indicated.

the ability to control hemorrhage is one of the criteria used to determine the _____ of a tissue in the root canals and should be
determined _____(before/after) placing medicament.

tissue tags, inflammation, health, before (HEMORRHAGE CONTROL MUST BE OBTAINED BEFORE MEDICAMENT PLACEMENT)

the tooth is (vital/non-vital) when performing pulptotomy

vital

restoration for pulptotomy:

following a pulpotomy, the crown of the tooth is left compromised, weakened, dehydrated, and easily prone to fracture. a ______
restoration must be placed for posterior teeth

expose a follow up radiograph in 6 months

arch length has been preserved b/c we got to keep the natural tooth

SSC

how do you explain a pulpotomy to a parent?

the tooth has a very ___ ____ which looks on the x ray like it will go into the _____ of the tooth. we will need to do a procedure where we
remove the ____ half of the nerve and put some _____ on the nerve that is left. It is something like a ____ ____ on a permanent tooth, but
much more simple and less expensive.

deep cavity, nerve, top, medicine, root canal

no hemorrhage indicates a necrotic pulp and vital pulp therapy is not indicated. the options in this situation are _____ or _____

pulpectomy, extraction

If pulpotomy is performed on an anterior tooth and it is to be restored with composite resin or a strip crown, you need to cover the ZOE
with ___ ____ b/c oxygen released from the ZOE will inhibit the proper cure of composite if it is in direct contact.

glass ionomer

_____ is a procedure in which infected dentin is removed and the affected dentin is left in the tooth.

medicaments and bases are placed on the affected dentin intended to seal the tooth and kill remaining microorganisms

indirect pulp cap (IPC)

justification for IPC:

the carious lesion of dentin consists of layers of varying degrees of demineralization caused by the acids produced by the bacteria. there
are two major areas of composition in carious dentin which are important in IPCs:

1. the more superficial layers are softer, contain the majority of bacteria and are called _____ dentin

2. the deeper layers are less demineralized, have fewer organisms and are called _____ dentin
-some is left to avoid pulp exposure
-remaining microorganisms die if cut off from moisture so it is the most important to get a good seal against leakage
-microorganism's death is enhanced by certain agents which can be placed over this layer (CaOH, ZOE).
-can remineralize if sealed under a base of glass ionomer or IRM

infected, affected

Downloaded by John Erickson Bangayan (bangayan1903064@ceu.edu.ph)


lOMoARcPSD|19937673

CLUSTER VII: ORTHODONTICS AND PEDIATRIC DENTISTRY

steps for IPC:

1. local anesthesia and rubber dam

2. removal of _____ caries


-use round bur on high or slow speed
-remove all peripheral caries (need clean margins on enamel)
-leave demineralized dentin on floor

3. placement of medicament:
-______: bacteriostatic and highly alkaline; protects pulp in case of microexposure during caries removal; must be used in conjunction with
base to provide seal
-_____: cytotoxic (kills bacteria); used as base to seal against micro leakage and prevent regrowth of bacteria
-______: use when ZOE is contraindicated as with composites or VLC materials; used as base to seal against micro leakage

4. restore the tooth

superficial, calcium hydroxide, ZOE, glass ionomer

t/f: calcium hydroxide provides a seal in IPC

t/f: ZOE provides a seal in IPC

t/f: glass ionomer provides a seal in IPC

false (not a base), true, true

how to explain IPC to parent:

the tooth has a very ___ ____ almost into the nerve. If we remove all of the decay, we may expose the nerve and have to do a ____ ____.
we are going to remove most of the ____ decay and place some ____ in the tooth to give the nerve time to heal. we will place a ____ filling
in the tooth while we wait to see if it will work. if the tooth has no pain and the x-rays look good in 3 months, the tooth will be responding
positively. the worst outcome is that we have to do the ____ ____ anyway. the best is that the nerve will ____ and not need the root canal.

deep cavity, root canal, worst, medicine, temporary, root canal, live

the differentiate the infected dentin from affected dentin when prepping an IPC:

_____ dentin is softened and will appear wet. it will gum the flutes of the bur.

_____ dentin appears as a powder when cut with the bur and more pressure is required to make the bur cut; may still be discolored.

infected, affected

isthmus of a class II amalgam is ____ the intercuspal width.

rounded internal line angles decreases ____ on the amalgam.

rounded pulpoaxial line angle increases ____ of amalgam in this area.

the ____ shape of the box conserves tooth structure and provides retention for the proximal segment.

the axial wall is placed just into dentin so that the horizontal depth of the axial wall is ____mm

1/3, stress, bulk, truncated, 1

2 matrix materials for a class II amalgam are the _____ band or the ____ band

the band should extend approximately ___mm above the marginal ridge and just below the gingival seat

tofflemire, T, 1

Downloaded by John Erickson Bangayan (bangayan1903064@ceu.edu.ph)


lOMoARcPSD|19937673

CLUSTER VII: ORTHODONTICS AND PEDIATRIC DENTISTRY

which is worse: undercarving or overcarving amalgam?

undercarving leaves flash of amalgam which will fracture and possibly result in a crevice defect -- potential for food entrapment and
recurrent caries

overcarving creates a slight step-down at the margin which has potential for future deterioration.

undercarving

primary teeth are small and preparations are only ___-____ mm deep.

1.25-1.5

use of _____ radiographs:

-caries section: usually IP


-periapical pathology
-root resorption
-abnormality in number: supernumerary or missing
-abnormality in morphology: dens in dente, dense invaginstus, fusion, gemination
-abnormalities in size: micro, macro
-assessment of tooth development
-traumatic injuries diagnosis
-pathology: cyst, odontoma

anterior

use of _____ radiograph:

-occlusal caries: proximity to pulp chamber b/c least superimposition


-proximal caries: radiograph of choice for interproximal caries when contacts are closed
-pulpal pathology
-relationship of permanent tooth bud to primary tooth roots
-resorption pattern of primary tooth roots
-eruption disturbances: ectopic eruption of permanent 1st molar
-periodontal disturbances
-check integrity of restorations

bitewing

use of _____ radiographs:

-pulpal involvement of primary/permanent tooth


-periapical pathology
-root resorption
-crown and root development of permanent tooth buds
-relationship of permanent tooth buds to primary roots
-presence or absence of permanent tooth buds
-traumatic injuries
-pathology
-follow up dental txt

periapical

use of _____ radiographs:

-dental age
-examine amount of root resorption on primary teeth
-examine presence of anomalies in number and position
-look for bilateral symmetry in presence and position of teeth
-look for supernumerary teeth
-look for missing teeth: max laterals and 2nd premolars
-abnormalities of eruption: ectopic eruption of canines and first molars

Downloaded by John Erickson Bangayan (bangayan1903064@ceu.edu.ph)


lOMoARcPSD|19937673

CLUSTER VII: ORTHODONTICS AND PEDIATRIC DENTISTRY

-pathology of teeth and supporting structures


-gross caries and gross periapical pathology
-fractures

panoramic

when evaluating ____ ____ with panoramic radiographs:

-which teeth are erupted? primary and permanent? compare with tooth eruption charts
-examine amount of root development on unerupted permanent teeth
-examine position of permanent tooth bud beneath primary tooth: how much bone is left between crown of permanent tooth and
furcation of primary tooth.

on average it takes 6 months for a tooth to go through 1mm of bone

dental age

look at eruption charts under tab 8

...

ORTHO-PEDO

1. A supra erupted mand. canine with respect to Frankfurt Horizontal plane is said to be in?
A. Aracon
  
B. Abstracon 
C. Protracon 
D. Retracon
E. Contracon 
1. A supra erupted mand. canine with respect to Frankfurt Horizontal plane is said to be in?
A. Aracon
  
B. Abstracon 
C. Protracon 
D. Retracon 
E. Contracon 

Paul
Ortho-Pedo

1. Mandible comes from branchial arch


a. 3rd
b. 1st
c. 2nd
d. 4th
2. The sutures of the nasomaxillary complex are oblique and more or less parallel with each other thus, growth in these areas
would serve to displace the maxilla
a. Upward and backward
b. Downward
c. Downward and forward
d. Forward and upward
3. Effect of abnormal eruptive path
a. Space loss
b. Delayed eruption
c. Increase in arch length
d. Elongation of adjacent teeth
e. Deflection of eruption
4. Proximal caries, if not restore, will ___ the arch length
a. Increase
b. Decrease
c. Not affect
d. No answer
5. Neutroclusion with maxillary anteriors are lingual in relation to the lower anteriors and there is mesial drifting of molars
a. Class 1 type 3 and 4
b. Class 1 type 3 and 5

Downloaded by John Erickson Bangayan (bangayan1903064@ceu.edu.ph)


lOMoARcPSD|19937673

CLUSTER VII: ORTHODONTICS AND PEDIATRIC DENTISTRY

c. Class 1 type 5 and 4


d. No answer
6. The space that allows an increase in the mandibular intercanine width is
a. Interdental space
b. Primate space
c. Anterior intermaxillary space
d. Leeway space
7. The ff are vertical planes, except
a. Y-axis
b. Palatal
c. Facial plane
d. Axis of 1
8. A bimaxillary protrusion occurs when
a. Both dental arches are forward with respect to basal bone
b. Large horizontal overlap with bilateral crossbite exits
c. The maxillary dentition is forward with respect to basal bone
d. None of the choices
9. Neutrocclusion with maxillary anteriors are lingual in relation to the lower anteriors and there is mesial drifting of molars
a. Class 1 type 3 and 5
b. Class 1 type 5 and 4
c. Class 1 type 3 and 4
d. No answer
10. Two or more teeth moving in opposite directions and pitted against each other by the appliance. Usually, the resistance to each
other is equal and opposite. The anchorage is
a. Extraoral
b. Simple
c. Stationary
d. Reciprocal
11. The V principle of growth is found in the ff structures of the skull except
a. Mandible
b. Alveolar process
c. Orbits of the eyes
d. Palate
12. Interstitial growth is observed at which of the ff sites?
I. Spheno-occipital synchondrosis
II. Maxillary tuberosity
III. Mandibular condyle
IV. Zygomaticomaxillary suture
V. Apex of an erupting premolar
a. I,II
b. I,III
c. I,IV
d. II,V
e. II,IV
13. Complete unilateral lip-jaw palate cleft
a. Class I
b. Class IV
c. Class II
d. Class III
14. The use of nitrous oxide is classified as a
a. Physical restraint
b. Psychological approach
c. Pharmacologic approach
d. No answer
15. Transillumination of soft tissues is useful in detecting which of the ff problems in a child?
a. Aortic stenosis
b. Abnormal frenum attachment
c. Sickle cell disease
d. Sialolithiasis
e. Koplik’s spot
16. Pulpotomy with formocresol of primary tooth
a. Should never be used

Downloaded by John Erickson Bangayan (bangayan1903064@ceu.edu.ph)


lOMoARcPSD|19937673

CLUSTER VII: ORTHODONTICS AND PEDIATRIC DENTISTRY

b. Result in higher success rate compared to CaOH


c. Result in lower success rate compared to CaOH
d. Induces formation of a dentin bridge at site of amputation
17. Radiographs of a 5 year old patient show permanent maxillary 1st molars inclined mesially with resulting resorption of the distal
portions of the roots of primary 2nd M. the condition described is
a. Infrafollicular resorption
b. Ankylosis
c. Premature eruption
d. Internal resorption
e. Ectopic eruption
18. Device that assess the arterial hemoglobin oxygen saturation and pulse rate
a. Automated vital signs monitor
b. Pretracheal stethoscope
c. Pulse oximeter
d. None
19. The tooth in the mandibular arch most likely to be malposed in cases of arch space discrepancy is the
a. 2nd PM
b. 1st M
c. 2nd M
d. 1st PM
20. In inferior nerve block for a child px, the injection must be made __ than for an adult px
a. Slightly higher
b. Slightly lower and more posteriorly
c. In level
d. No answer
21. A child in the “ugly duckling” stage is characterized by
a. Absence of max incisors
b. A flat nasal bone
c. Diastema bet 2 central incisors
d. Presence of only one maxillary central incisor
22. The least desirable method used in child management is
a. Tell-show do technique
b. Voice control
c. Hand over mouth technique
d. Gift before tx
23. In acute ingestion of fluoride, the ff can be given to the px to counteract its effect, except
a. Milk of magnesia
b. 1 tsp of ipecac syrup
c. Milk
d. Alum
24. A 4 year old child has traumatized central incisor with a class III (Ellis) fracture. The injury occurred about 1 month ago, and
examination indicates that the pulp is necrotic. There are no other pathologic findings. Tx of choice is
a. Endodontic tx and root canal filling with a resorbable paste
b. Pulpectomy and root canal filling using gutta-percha points and cement
c. Extraction and use of a space maintainer
d. Watchful observation
25. Which of the ff approaches is best for a child suffering from cerebral palsy
a. Use of sedation
b. Psychological approaches
c. Towel method
d. HOM exercise
26. Slow progress in molar uprighting in an adult px is usually due to
a. An over contoured spring
b. Lack of anchorage control
c. Overextended bands
d. The occlusion not being relieved
27. The prognosis of a cervical 3rd root fracture
a. Is favorable
b. Depends on whether tooth is discolored
c. Is not favorable
d. No answer
28. Frankl behavior rating characterized by refusal of tx and extreme negativism

Downloaded by John Erickson Bangayan (bangayan1903064@ceu.edu.ph)


lOMoARcPSD|19937673

CLUSTER VII: ORTHODONTICS AND PEDIATRIC DENTISTRY

a. Rating 1
b. Rating 4
c. Rating 3
d. Rating 2

ORTHODONTICS AND PEDIATRICS

1. The ffng are vertical planes, except:

a. V-axis b. Palatal c. Axis of 1 d. Facial plane

2. The V principle of growth is found in the ffng structures of the skull except

a. Alveolar process b. Palate c. Orbits of the eye d. Mandible

3. Increase in breadth means increase in

a. Length b. Height c. Vertical Dimension d. Depth e. Width

4. In deep bite the postural vertical dimension is/has:

a. In harmony with occlusal vertical dimension

b. No answer text provided

c. No relations with occlusal vertical dimension

d. Not in harmony with occlusal vertical dimension

5. Bone ossification can be detected with the use of:

a. Hand and wrist xray b. BMR c. Electromyographic exam

e. Biostatic

6. The maxilla is displaced downward and forward by growth in __ parts of the bone

a. Posterior and superior

b. Posterior and anterior

c. Anterior and superior

d. Anterior and inferior

7. It is considered to be the best space maintainer

a. Cron and crib

b. Functional space maintainer

c. A well-restored deciduous teeth

d. Fixed space maintainer

8. Generalized osteoclastic activity along the walls of the alveolar socket is the bone response to

a. Elongating forces b. Rotating forces c. Depressing Force d. Extrusion

9. Neutroclusion with maxillary anteriors are lingual relation to the lower anteriors and there is mesial drifting of molars

a. No answer text provided

b. Class 1 type 3 and 4

c. Class 1 type 3 and 5

d. Class 1 type 5 and 4

10. Gonion, menton, and pogonion are cephalometric landmarks located on the

Downloaded by John Erickson Bangayan (bangayan1903064@ceu.edu.ph)


lOMoARcPSD|19937673

CLUSTER VII: ORTHODONTICS AND PEDIATRIC DENTISTRY

a. Midline b. Mandible c. Skeletal profile d. Bony chin

11. A linguoverted max. incisor with respect to the orbital plane is said to be in?

a. Attraction b. Abstraction c. Contraction d. Protraction e. Retraction

12. The cranial vault increases rapidly in size the first few years of post natally and completes approximately 90% of its growth by 6
years of age. This growth is typical of which of the ffng types of tissues?

a. Genital b. General (somatic) c. Lymphoid d. Dental e. Neural

13. At age 4-5 what normal sign of primary dentition augers well for the erupting permanent incisors in terms of space availability:

a. Class 1 cuspid relationship

b. Flush terminal plane

c. Growth spaces, interdental spaces

d. Upright vertical incisor relationship

14. It is the growth of the mandible which is responsible for the major increase in length

a. Ramus of the mandible

b. Alveolar process

c. Lingual tuberosity

d. Condyle

15. When one side of the arch is intact and there are several primary teeth missing on the other side, use:

a. A nance lingual arch

b. A soldered fixed lingual arch

c. A transpalatal arch

d. A partial denture space maintainer

e. A distal shoe

16. Bodily force that moves the central incisor mesially produces:

a. Compression of the periodontal fiber in the distal side

b. Osteoclastic activity on the distal side

c. Osteoblastic activity on the mesial side

d. Stretching of the periodontal fiber on the distal side

17. Nasomaxillary complex is hafted to the cranium by the ffng sutures except:

a. Frontomaxillary b. Zygomaticomaxillary c. Frontotemporal d. Pterygopalatine e.


Zygomaticotemporal

18. As arch perimeter increase, arch length?

a. Decreases b. No answer text provided c. Increases d. No change

19. Growth of maxilla follows the __ growth curve

a. Genital b. Lymphoid c. Neural d. General / bodily

20. A linguoverted max. premolar with respect to midsagittal plane is said to be in?

a. Abstraction b. Attraction c. Contraction d. Refraction e. Protraction

21. Frankfort-horizontal is a reference plane constructed by joining which of the ffng landmarks?

a. Nasion and sella

b. Porion and sella

Downloaded by John Erickson Bangayan (bangayan1903064@ceu.edu.ph)


lOMoARcPSD|19937673

CLUSTER VII: ORTHODONTICS AND PEDIATRIC DENTISTRY

c. Porion and nasion

d. Porion and orbitale

22. This will decrease arch perimeter during transitional period:

a. Late mesial shift of 1st permanent molar

b. Labial position of permanent incisors

c. Distal tipping of lower cuspid

d. No answer text provided

23. Which of the ffng developmental space will cause decrease in arch perimeter when pre-empted?

a. Interdental space b. Leeway space c. Primate space d. Inter-occlusal space

24. How will extraction of primary maxillary central incisor in a 5 year old child with incisal spacing affect the size of the intercanine
space?

a. No change will occur in the size of the intercanine space

b. The intercanine space will increase in size

c. No answer text provided

25. Inclined planes should be left in the mouth for more than 2 months to prevent creation of;

a. Anterior cross bite b. Anterior open bite c. Posterior cross bite d. Posterior open bite

26. Serial extraction is done if the space deficiency to align the is

a. 6mm b. 5mm c. 3mm d. 8mm

27. A dolichofacial face would likely to have __ arch form

a. Long and tapering b. Broad and square c. Round d. Average

28. The best appliance for 7 years old child with class 1 type 3 (inlocked maxillary central incisor) is

a. Cross bite elastics

b. Myofunctional appliance

c. Band and crib

d. Mandibular acrylic inclined plane

29. From birth to 5 years the dominance of growth is on the

a. Nasomaxillary structures

b. Mandible

c. Cranial structures

d. Muscular components of the body

30. Interstitial growth is observed at which of the ffng sites?

1. Spheno-occipital synchondrosis

2. Maxillary tuberosity

3. Mandibular condyle

4. Zygomaticomaxillary suture

5. Apex of an erupting premolar

a. 1&3 b. 1&4 c. 2&5 d. 2&4 e. 1&2

31. Which of the ffng is the most essential factor related to correction of an anterior crossbite?

Downloaded by John Erickson Bangayan (bangayan1903064@ceu.edu.ph)


lOMoARcPSD|19937673

CLUSTER VII: ORTHODONTICS AND PEDIATRIC DENTISTRY

a. Depth of the crossbite b. Age of the patient c. Shape of the tooth involved d. Space
available mediodistally

32. As arch perimeter increase, arch length?

a. Increases b. No answer text provided c. no change d. Decreases

33. The “V” principle of growth is best illustrated by the:

a. Mandibular ramus b. Nasal septum c. Speno-occipital Synchondrosis d. Mandibular symphysis

34. The order from greatest to least change of the dimension of the cranium:

a. Depth, width, height

b. Height, depth, width

c. Width, height, depth

d. Depth, height, width

35. The relationship of the upper and lower gum pads is such that

a. The upper is within the lower

b. The lower is anterior to the upper

c. It presents a convex profile

d. The upper is posterior to the lower

36. The most common malocclusion in the mixed dentition period is:

a. Class II Div. 1 b. Class I type 2 c. Antterior open bite d. Posterior open bite e.
Crowding

37. At age 4-5 what normal sign of primary dentition augers well for the erupting permanent incisors in terms of space availability:

a. Flush terminal plane

b. Growth spaces, interdental spaces

c. Upright vertical incisor relationship

d. Class I cuspid relationship

38. The ffng are factors that will compensate incisor liablity except:

a. Favorable tooth size ratio

b. Intercanine width growth

c. Labial positioning of permanent incisors

d. Upright position of primary incisors

39. Neutroclusion with max. anterior are lingual in relation to the lower anteriors and there is mesial drifting of molars.

a. Class 1 type 3 and 4

b. Class 1 type 5 and 4

c. No answer text provided

d. Class 1 type 3 and 5

40. Facial structures follow the

a. Neural growth

b. All of these

c. Bodily growth curve

d. None of the above

Downloaded by John Erickson Bangayan (bangayan1903064@ceu.edu.ph)


lOMoARcPSD|19937673

CLUSTER VII: ORTHODONTICS AND PEDIATRIC DENTISTRY

41. The profile of the patient with protruded mandible and retruded maxilla is:

a. No answer text provided

b. Concave

c. Convex

d. Straight

42. The sutures of nasomaxillary complex are oblique and more less parallel with each other thus growth in these area would serve
to displace the maxilla:

a. Forward and upward

b. Downward

c. Downward and forward

d. Upward and backward

43. Mesial surface of E can be disliked in case the 3 cannot erupt because of insufficient space. This is:

a. Contingency of extraction

b. Occlusal equilibration

c. Observation

d. Space regaining

44. Cranial vault grows primarily by cartilage growth at:

a. Disappears at birth b. Fontanelles c. Sutures d. Synchondrosis

45. Complete unilateral lip-jaw palate cleft

a. Class I b. Class III c. Class IV d. Class II

46. The basic form of arch is determined:

a. By environmental factors b. by muscles c. Two of the choices d. All of these e. In intrauterine life

47. A supra erupted max canine with respect to frankhurt horizontal plane is said to be in?

a. Contraction b. Retraction c. Abstraction d. Protraction d.


Attraction

48. The order from greatest to least change of the dimensions of the cranium:

a. Depth, height, width

b. Depth, width, height

c. Width, height, depth

d. Height, depth, width

49. Orthodontic correction of which of the following is most easily retained?

a. Rotation b. Diastema c. Posterior crossbite d. Anterior Crossbite e. Anterior crossbite e. Expansion

50. A supra erupted mand canine with respect to Frankfort Horizontal Plane is said to be in?

a. Abstraction b. Protraction c. Attraction d. Contraction e. Retraction

51. The least desirable method used in child management is:

a. Gift before treatment

b. Hand-over-mouth technique

c. Voice control

d. Tell-show do technique

Downloaded by John Erickson Bangayan (bangayan1903064@ceu.edu.ph)


lOMoARcPSD|19937673

CLUSTER VII: ORTHODONTICS AND PEDIATRIC DENTISTRY

52. To avoid ingestion of lethal dose of fluoride, it is safe to dispense up to __ of fluoride

a. 700mg b. 120mg c. 500mg d. 300mg

53. In infant oral care, clean mouth with gauze after feedings and at the bed time done during:

a. 6-12mos b. 0-6mos c. 24-36 mos d. 12-24mos

54. In Acute ingestion of fluoride, the ffng can be given to the patient to counteract its effect, except;

a. Milk of magnesia b. Milk c. 2tsp of ipecac syrup d. Alum

55. Child’s arms and legs can be immobilized with:

a. Plastic bowl b. Head positioner c. Posey Straps d.Forearm-body support

56. Periapical lesions, pulpitis and pulpotomy of a primary molar will __ the eruption of successor premolar.

a. Have no effect on b. No answer text provided c. Delay d. Hasten

57. Primary second molars usually erupt during ages:

a. 30-36months b. 20-30months c. 8-14months d. 36-48months

e. 14-20 months

58. In festooning and trimming a stainless steel crown, greater length is necessary in the region of the mesiofacial bulge in primary:

a. Maxillary canine

b. First molar

c. Maxillary lateral incisor

d. Second molar

59. If a primary tooth has been extracted before succedaneous tooth has begun eruptive movement, its eruption will be ___

a. Delayed b. Not affected c. No answer text provided d. hastened

60. Method of early examination where the dentist and parent are seated face to face

a. Cradling position b. Leg position c. Knee position d. Knee to knee position e. All

61. The most common sequence of eruption for the upper arch:

a. 6125437 b. 6124537 c. 6123457 d. 6142537

62. The purpose of pulp treatment and root canal filling is to:

a. Remove pulpal tissue b. Prolong the usefulness of a tooth to function in mastication

b. C. No answer text provided d. Enlarge the root canal

63. Least reduction in tooth preparation for posterior SSC:

a. Lingual b. Buccal c. Occlusal d. Proximal surfaces

64. In pedodontic triangle the apex of the triangle is:

a. Dentist b. Parent c. Assistant d. Child

65. Mandibular teeth are best anesthetized with;

a. Infraorbital injection b. Mental injection c. An inferior alveolar and long buccal injection d. No answer
text provided

66. This is growth movement which mean movement of whole bone as a unit

a. Deposition b. Drift c. Relocation d. Displacement

67. This is an increase in size by expansion from within

a. Sutural apposition b. Maturation c. Development d. Interstitial growth

Downloaded by John Erickson Bangayan (bangayan1903064@ceu.edu.ph)


lOMoARcPSD|19937673

CLUSTER VII: ORTHODONTICS AND PEDIATRIC DENTISTRY

68. Which of the ffng approaches is best for a child suffering from cerebral palsy

a. HOM exercise b. Psychological approaches c. Use of sedation d. Towel method

69. Grinding of the sharp edges is sufficient

a. Dentin fracture b. Lateral Luxation c. Extrusion d. Enamel Fracture

70. Type of fear which is produced by direct physical stimulation of sense organ

a. Associative fear b. Subjective fear c. Objective Fear d. Acquired Fear

71. Anodontia, diagnosed in a 5-year-old child, primarily affects the growth of the:

a. Maxilla b. Maxilla & mandible c. Alveolar Bone d. Mandible e. Midface

72. The maximum number of cartridges of 2% lidocaine with 1:100,000 epinephrine for 40 pound child patient is:

a. 7 b. 3 c. 5 d. 9 e. 10

73. Treatment for crown fracture involving the pulp of a primary incisor;

a. Formocresol pulpotomy b. DPC c. Extraction d. Pulpectomy

74. Device that assess the arterial hemoglobin oxygen saturation and pulse rate

a. Pulse oximeter b. None c. Pretracheal stethoscope d. Automated vital sign monitor

75. Spoiled children are with parents who are:

a. Overindulging b. Dominating c. Rejecting d. No answer text provided

76. Transillumination of soft tissues is useful in detecting which of the ffng problem in a child?

a. Abnormal frenum attachment b. Aortic stenosis c. Koplick’s spot

c. Sialothiasis d. Sickle cell disease

77. The most common cause of sinus tracts in gingival tisuues of children is;

a. Periapical cyst b. Chronic Periapical Abscess c. Acute Periapical abscess d. Pericementitis

78. A child in long term remission of acute leukemia has dental problems characterized by unusual susceptibility to:

a. Development of jaw deformities b. Oral infection c. Dental caries d. Periodontal bone loss

79. Maxilla is formed from the maxillary processes and ___

a. Tuberculum impar b. Medial nasal process c. Lateral Processess d. Globular


process

80. The most important predisposing factor to injury of the anterior teeth is:

a. Rotated anterior teeth b. Crossbite of anterior teeth c. No answer text provided d. Protruding anterior
teeth

81. Pulpotomy with calcium hydroxide in the primary teeth:

a. Result in acute inflammation of pulp

b. No answer text provided

c. Result in necrosis of pulp

d. May form calcific barrier on the pulp stump

82. Device that assess the arterial hemoglobin oxygen saturation and pulse rate:

a. Pulse oximeter b. Automated vital sign monitor c. none d. Pretracheal stethoscope

83. In inferior nerve block for a child patient, the injection must be made _ than for an adult patient

a. Slightly lower and more posteriorly

b. In level

Downloaded by John Erickson Bangayan (bangayan1903064@ceu.edu.ph)


lOMoARcPSD|19937673

CLUSTER VII: ORTHODONTICS AND PEDIATRIC DENTISTRY

c. No answer text provided

d. Slightly higher

84. Guiding of behavior through eye contact, posture and facial expression

a. TSD b. Non-verbal communication c. Positive reinforcement d. Distraction e. HOM exercise

85. In managing abnormal oral habits, therapy must be:

a. Of physical control b. A means of punishment c. Of conditioning responses

e. Mechanical

86. In incorrigible 4 year- old child who keeps on kicking and throwing objects in the clinic can be managed by:

a. General Anesthesia b. All of these c. Tell-show-do-technique d. Physical


resistraint and conscious sedation

87. A 4 year old child has traumatized central incisor with a Class III (Ellis) fracture. The injury occurred about one month ago, and
examination indicates that pulp is necrotic. There are no other pathologic findings, treatment of choice:

a. Watchful observation

b. Pulpectomy and root canal filing using gutta percha points and cements

c. Extraction and use of a space maintainer

d. Endodontic treatment and root canal filling with a resorbable paste

88. Restore with bonding agent and composite

a. Dentin fracture b. Enamel fracture c. Enamel-dentin fracture d. Extrusion e. Intrusion

89. Indicated restoration on a primary anterior tooth with small mesial and distal caries and cervical caries but without pulp
involvement:

a. Strip-off crown b. Composite Resin c. Stainless steel crown d. no answer text provided

90. The treatment option for patients who have sustained extensive orofacial and dental trauma

a. General anesthesia b. No treatment c. Nitrous Oxide& Oxygen inhalation d.


Conscious sedation

91. The purpose of pulp treatment and root canal filling is to:

a. Remove pulpal tissue

b. Prolong the usefulness of a tooth to function in mastication

c. Enlarge the root canal

d. No answer text provided

92. The ultimate objective of pulp capping is:

a. No answer text provided

b. To preserve the pulp and odontoblast so that secondary dentin will be deposited

c. To prevent bacterial contamination

d. To prevent pain

93. The least desirable method used in a child management is:

a. Voice control

b. Gift before treatment

c. Tell-show do technique

d. Hand-over-mouth technique

Downloaded by John Erickson Bangayan (bangayan1903064@ceu.edu.ph)


lOMoARcPSD|19937673

CLUSTER VII: ORTHODONTICS AND PEDIATRIC DENTISTRY

94. Medium-size punch hole is used for:

a. Maxillary permanent incisors

b. Lower permanent incisors

c. Permanent molars

d. Primary molars

95. Point of injection for mandibular blocking in pediatric is _ the occlusal plane

a. Below or at the level

b. No answer text provided

c. Higher than

d. Above

96. In pedodontic triangle the apex of the triangle is:

a. Assistant b. Child c. Parent d. Dentist

97. Quadrant used in the gluteal region when intramuscular sedation is used:

a. Lower outer quadrant

b. Lower inner quadrant

c. Upper inner quadrant

d. Upper outer quadrant

98. Transillumination of soft tissue is useful in detecting which of the ffng problems in a child?

a. Sickle cell diseases

b. Abnormal frenum attachment

c. Aortic stenosis

d. Sialothiasis

e. Koplick’s spot

99. Frankl behavior rating characterized by refusal of treatment and extreme negativism

a. Rating 3 b. Rating 4 c. Rating 2 d. Rating 1

100. The tx option contraindicated in patients who are not able to breathe nasally.

a. Nitrous oxide & oxygen inhalation

b. No treatment

c. Conscious sedation

d. General anesthesia

Downloaded by John Erickson Bangayan (bangayan1903064@ceu.edu.ph)


lOMoARcPSD|19937673

S7 Blank - Q and A's for Ortho and Pedo

Dentistry (Centro Escolar University)

Studocu is not sponsored or endorsed by any college or university


Downloaded by John Erickson Bangayan (bangayan1903064@ceu.edu.ph)
lOMoARcPSD|19937673

● ORTHODONTICS AND PEDIATRIC DENTISTRY

Which of the developmental space will cause a decrease in


arch perimeter when pre-empted?
primate space B. interocclusal space C.
interdental space D. leeway space

This is an increase in size by expansion from within


sutural apposition C. interstitial growth
maturation D. development

This is growth movement which mean movement of whole bone as


a unit
A. drift B. displacement C. deposition D.
relocation E. remodeling

Mandible comes from ? branchial arch first


B. second C. third D. fourth

Maxilla is formed from the maxillary processes and ?


medial nasal process C. globular process
B. lateral processes D. tuberculum impar

In young child, paradoxical excitement occurs most frequently


following premedication with:
a narcotic B. a barbiturate C. nitrous oxide
D. an amphetamine

The maximum amount of anesthesia that can be given to a


36pound child is:
4 carpules B. 3 carpules C. 2 carpules
D. 1 carpule

Dental Fluorosis is most likely to occur on a:


two-year-old child ingesting 1 ppm of fluoride
17-year-old patient ingesting 1 ppm of fluoride
on a 3-year-old child ingesting 0.50 ppm of
fluoride D. none of the above.

Pulpotomy with formocresol of primary tooth:


result in lower success rate compared to calcium
hydroxide, result in higher success rate compared to
calcium hydroxide,
should never be used,

Downloaded by John Erickson Bangayan (bangayan1903064@ceu.edu.ph)


lOMoARcPSD|19937673

induces formation of a dentin bridge at site of


amputation, none of the above.

A 4-year-old child visited a dentist for the first time and


received prophylaxis
quite well. Before he left, the dentist was glad
that he gave the child a toy.
This kind of gesture is called:
classical conditioning C. positive reinforcement,
bribery, B. public relations
E. none of the above.

If a primary tooth has been extracted before succedaneous


tooth has begun eruptive movement, its eruption will be __
hastened B. delayed C. not affected
Which of the following conditions is usually present in a
Class II, Division 2 malocclussion? Open bite
Steep mandibular plane
Mesiocclusion of permanent first molars
Lingual inclination of maxillary central incisors

Excessive orthodontic force used to move a tooth may:


cause hyalinization cause
undermining resorption
crush the periodontal
ligament
A. 1 & 2 B. 2 & 3 C. 1 & 3 D. 1, 2 & 3 E. 3 only

Severely crowded mandibular incisors usually result from:


mesial migration of teeth premature
loss of primary teeth presence of
supernumerary teeth D. tooth
size-arch length discrepancy
How will extraction of a primary maxillary central incisor in
a 5-year old child with incisal spacingaffect the size of
the intercanine space?
The intercanine space will increase in size.
The intercanine space will decrease in size.
No change will occur in the size of the intercanine
space.
The major criterion to differentiate between a true Class
III and a pseudo-Class III malocclusion is: degree of
anterior crossbite presence of a bilateral crossbite
C. existence of a forward shift of the mandible during
closure

Downloaded by John Erickson Bangayan (bangayan1903064@ceu.edu.ph)


lOMoARcPSD|19937673

D. occlusal relationship between maxillary and


mandibular first molars

Orthodontic correction of which of the following is most


easily retained?
Diastema C. Expansion
B. Rotation D. Posterior crossbite E. Anterior
crossbite

A bimaxillary protrusion occurs when:


large horizontal overlap with biteral crossbite exits
the maxillary dentition is forward with respect to
basal bone
both dental arches are forward with respect to basal bone
D. none of the choices.

When a simple tipping force is applied to the crown of a


singlerooted tooth, the center of rotation is usually
located:
at the apex D. at the cervical line
5mm beyond the apex E. one-third the root length
from the apex
two-third the root length from the
apex.
Which of the following is the most common orofacial
malformation that produces malocclusion?
Cleft palate C. Ectodermal dysplasia
Pierre Robin syndrome D. Osteogenesis imperfecta E.
Cleidocranial dysostosis

The cranial vault increases rapidly in size the first few


years postnatally and completes approximately 90% of its
growth by 6 years of age. This growth is typical of which of
the following types of tissues?
Neural B. dental C. genital D. lymphoid E.
general (somatic)

The following are the conditions found on the area of


pressure in the presence of heavy forces: area of
hyalinization occlusion of blood vessels stretched
periodontal fibers undermining resoprtion
A. 1, 2& 3 B. 1, 2 & 4 C. 2 , 3 & 4 D. 3 only
E. 1 & 4 only

Downloaded by John Erickson Bangayan (bangayan1903064@ceu.edu.ph)


lOMoARcPSD|19937673

Generalized osteoclastic activity along the walls of the


alveolar socket is the bone response to: depressing force
C. extrusion
B. elongating force D. rotating force E.
both B & D

A 4-year old child has a traumatized central incisor with a


Class III (Ellis) fracture. The injury occurred about one
month ago, and examination indicates that the pulp is
necrotic. There are no other pathologic findings. Treatment
of choice is:
watchful observation
extraction and use of a space maintainer
pulpectomy and root canal filling using gutta-percha
points and cement
endodontic treatment and root canal filling with a
resorbable paste.

Supervision of a child’s development of occlusion is most


critical at ages:
3-6 years B. 7-10 years C. 11-14 years E. 14-
17 years

A child with congenital heart disease requires special


treatment planning for dental care because of potential
problems with:
bleeding
local infection
systemic infection
enamel hypoplasia
A. 1 , 2 & 3 B. 1 & 3 C. 1 only D. 3 only E.
1,2,3 & 4

In festooning and trimming a stainless steel crown, greater


length is necessary in the region of the mesiofacial bulge
in a primary:
first molar B. second molar C. maxillary canine
D. maxillary lateral incisor

Using a topical fluoride rinse before acid etch direct


bonding of orthodontic brackets is contraindicated because
fluoride:
decreases the solubility of enamel increases the
pH of the etching agent causes copious amounts

Downloaded by John Erickson Bangayan (bangayan1903064@ceu.edu.ph)


lOMoARcPSD|19937673

of saliva directly reacts chemically with the


bonding agent.

If a 7-year old patient loses a primary mandibular canine


about the same time the adjacent lateral incisor is erupting
or shortly thereafter, the dentist should be alert to the
possibility of:
a tongue habit a
developing
crossbite
an early eruption of the permanent canine
lingual collapse of mandibular anterior teeth

Which of the ff. approaches is best for a child suffering


from celebral palsy
HOM exercise B. towel method C. use of sedation
D. psychological approaches

Type of fear which is produced by direct physical stimulation


of sense organ associative fear B. objective fear
C. subjective fear D. acquired fear

Agent used to reduce pain without affecting consciousness


hypnotic drug B. analgesic C. barbiturate
D. general anesthesia

The drug that modifies fear and anxiety


A. flanax B. ethamizid C. celestamine
D. seconal

A general characteristic of a 12 year old child mother


as the center of his world C. exhibits
tantrums
separation anxiety D. rejects parental
authority

This is a psychological approach to manage a child


nitrous oxide inhalation C. giving diazepam
mutisensory techniques D. HOM

The major source of anchorage in a maxillary Hawley appliance


is
the :
molar clasp C. acrylic portion

Downloaded by John Erickson Bangayan (bangayan1903064@ceu.edu.ph)


lOMoARcPSD|19937673

B. labial bow of D. interproximal clasp E. bracket


the band

A narrow maxillary arch with respect to midsagittal plane is


said to be in:
Protraction C. contraction
B. retraction D. distraction E. both C & D

Complete unilateral lip-jaw palate cleft.


A. Class I B. Class II D. C. Class III
Class IV

Anodontia, diagnosed in a 5-year-old child, primarily affects


the growth of the:
Midface C. mandible
B. maxilla D. alveolar bone E. maxilla
& mandible

Which of the following are likely to occur during orthodontic


therapy?
gingival irritation
cementum resorption
increased mobility of
teeth
demineralization of enamel adjacent to appliances in
patients with poor oral hygiene
1, 3 & 4 C. 2 & 4 only E. 3 & 4 only
B. 1 & 4 only D. 1, 2, 3 & 4

A distal shoe space maintainer is indicated when a primary:


incisor is avulsed first
molar is prematurely lost
second molar is lost after eruption of a permanent first
molar
second molar is lost before eruption of a permanent first
molar

The normal downward and forward direction of facial growth


results from upward and backward growth of maxillary
sutures and the mandibular condyle
vertical eruption and mesial drift of the dentitions
interstitial growth in the maxilla and the mandible
epithelial induction at growth centers
A. 1 & 2 B. 1 & 3 C. 1 & 4 D. 2 & 3 E. 2 &

Downloaded by John Erickson Bangayan (bangayan1903064@ceu.edu.ph)


lOMoARcPSD|19937673

An 8-year-old girl has 2mm diastema between permanent


maxillary central incisors. Permanent maxillary lateral
incisors are in position. The diastema is probably the
result of:
thumb-sucking
an abnormal frenum attachment
a normal developmental
process
A. 1 & 2 B. 1 & 3 C. 2 & 3 D. 2 only E. 3 only
When comparing cementum to bone in their responses to
orthodontic forces, cementum resorbs: more readily C.
not at all
B. less readily D. under lighter loads E. by
indirect (undermining) resoprtion

The tooth in the mandibular arch most likely to be malposed


in cases of arch space discrepancy is the:
first molar B.. first premalor C. second molar D.
second premolar

Slow progress in molar uprighting in an adult patient is


usally due to:
overextended bands D. an overcontoured
spring
lack of anchorage control E. the occlusion not
being relieved
the stabilizing wire not being passive

A 9-year-old patient has a slightly convex profile and a


suspected tooth mass-arch length (circumference)
discrepancy. Before instituting space management procedures,
the dentist should:
complete a space analysis C. complete a
metric analysis cephalo-
seek orthodontic consultation D. all of the above
E. Either A or C above

A primary maxillary anterior tooth in a 4-year-old child was


traumatically intruded into the tissues so that only half
the tooth is visible. The most appropriate treatment is to:

Downloaded by John Erickson Bangayan (bangayan1903064@ceu.edu.ph)


lOMoARcPSD|19937673

extract the tooth.


perform a pulpotomy
administer no treatment
place orthodontic bands on adjacent teeth and draw the
tooth down with elastics

Which of the following orthodontic movements of teeth are


most difficult to accomplish?
Tipping
Rotation
Intrusion
Extrusion
Translation
A. 1 & 2 B. 1 & 3 C. 1 & 5 D. 3 & 4 E. 3 & 5

Interstitial growth is observed at which of the following


sites?
Spheno-occipital syndrondrosis
Maxillary tuberosity
Mandibular condyle
Zygomaticomaxillary suture
Apex of an erupting premolar
A. 1 & 2 B. 1 & 3 C. 1 & 4 D. 2 & 4 E. 2 & 5

When force is applied, bone reflexes that produces


deformation of crystalline materials and allows flow of
electric current necessary for tooth movement:
deposition C. hyalinization
B. piezoelectricity D. undermining resoprtion
Cephalometrics is useful in assessing which of the following
relationships?
1. tooth-to-tooth
2. bone-to-bone
3. tooth-to-bone
A. 1 & 2 B. 1 & 3 C. 2 & 3 D. 1,2 & 3 E. 3 only
Which of the following is the most essential factor related
to correction of an anterior crossbite?
age of the patient C. depth of the
corssbite shape of the tooth involved D. space
available mesiodistally

Downloaded by John Erickson Bangayan (bangayan1903064@ceu.edu.ph)


lOMoARcPSD|19937673

Frankfort-horizontal is a reference plane constructed by


joining which of the following landmarks?
Porion and sella C. Porion and nasion
Porion and orbitale D. Nasion and Sella

Which of the following is not a space maintainer?


Lingual arch D. Nance holding arch
Class III restoration E. Stainless steel crown
Palatal expansion appliance

Space closure is least likely to occur after loss of which of


the following teeth?
Primary mandibular canines D. Primary mandibular
second molars
Primary maxillary first molars E. Primary maxillary
central incisors
Permanent maxillary central incisors

Crowding that occurs with mandibular incisors after age 18 is


most often the result of:
orthodontic relapse C. eriodontal disease
impacted thid molars D. a physiologic
matura-
tional change

The undesirable side-effect most commonly associated with use


of a finger spring to tip the crown of a tooth is:
Pain D. gingival irritation
tendency for the tooth to intrude E. severe mobility
of the tooth
tendency for the root apex to move in the direction
opposite from the crown.

After the age 6, the greatest increase in size of the


mandible occurs:
at the symphysis C. along the lower border
between the canines D. distal to the first molars
A distal shoe space maintainer is indicated when a primary:
canine is in crossbite first
molar is prematurely lost
second molar is lost after eruption of a permanent first
molar
second molar is lost prior to eruption of a permanent
first molar

Downloaded by John Erickson Bangayan (bangayan1903064@ceu.edu.ph)


lOMoARcPSD|19937673

The <V= principle of growth is best illustrated by the:


nasal spetum C. mandibular symphysis
B. mandibular ramus D. spheno-occipital synchondrosis

Anterior crossbite in the primary dentition usually indicates


a developing:
Class I malocclusion C. Class II malocclusion
Class III malocclusion D. Class IV maloccusion E.
none of the above

A mixed dentition analysis determines:


intercanine width D. skeletal growth pattern
discrepancies in jaw size E. size of permanent incisors
space available versus space required

Gonion, menton and pogonion are cephalometric landmarks


located on the
midline B. bony chin C. mandible D. skeletal
profile

A major disadvantage of treatment using cervical headgear is:


impaction of maxillary canines D. extrusion of
illary incisors max-
extrusion of maxillary molars E. potential
deform-
ity of the neck
psychologic trauma due to
appearance
Asymmetrical anterior open bite with normal posterior
occlusion is characteristic of:
thumb-sucking C. abnormal swallowing habits
mouth breathing D. both A & B E. both B &
C

If the norm for the cephalometric angle SNA is 82°, a


patient’s reading of 90° for SNA most likely indicates:
maxillary protrusion C. the patient’s ethnic
background
protrusive maxillary incisors D. dysplasia of the
anterior cranial base.

Downloaded by John Erickson Bangayan (bangayan1903064@ceu.edu.ph)


lOMoARcPSD|19937673

Ideally, a malooclusion should be treated between the ages


of:
5 and 8 years D. 8 and 10 years
10 and 12 years E. 12 and 14 years
None of the above. The age at which a malocclusion
is treated should depend on the problem involved.
Radiographs of a 5-year old patient show permanent maxillary
first molars inclined mesially with resulting resoprtion of
the distal portions of the roots of primary second molars.
The condition described is:
ankylosis C. ectopic eruption premature
eruption D. internal resorption
E. intrafollicular resoprtion
Fractured maxillary anterior teeth generally occur most often
in children with which of the following dental conditions?
Class I malocclusion D. Class II, Division
1 malocclusion
Class II, Division 2 malocclusion E. Class III
malocclusion
Marked incisor crowding

On the first dental visit, the basic fear of a child under


the age of 2 is related to: fear of an injection
anxiety over being separated from a parent
anxiety regarding the instruments used to perform dental
treatment
not understanding the reason for dental treatment.

A child with Down’s syndrome is characterized as being:


affectionate C. fearful of quick movements
capable of learning dental procedures D. both A & B
E. all of these

A 3 ½ year old child has an acute fever, diarrhea, oral


vesicular lesions and gingival tenderness. The most likely
diagnosis is:
thrush D. drug allergy
aphthous ulcerations E. . acute herpetic stoma-
titis
C. necrotizing ulcerative gingivitis

Transillumination of soft tissues is useful in detecting


which of the following problems in a child?

Downloaded by John Erickson Bangayan (bangayan1903064@ceu.edu.ph)


lOMoARcPSD|19937673

Koplick’s spot C. Sialolithiasis


Aortic stenosis D. Sickle cell disease E. abnormal
frenum attachment

Which of the following is least likely to result from


persistent long-term thumb-sucking?
a deep overbite D. protrusion of maxillary incisor
constriction of maxillary arch E. rotation of maxillary
lateral incisor
lingual inclination of mandibular
incisors
The least desirable method used in child management is:
tell-show do technique, C. voice control,
gift before treatment, D. hand-over-mouth
technique.

A child who is reluctant to accept dental treatment


,exhibit negative attitude but not so pronounced is
classified under Frankl ?
I B. II C. III D. IV

The amount of pulp tissue to be removed in formocresol


pulpotomy procedure half of the coronal pulp
coronal pulp tissue down to the cervical constriction of
each root canal
C. infected portion of coronal pulp tissue
D. total removal of pulp tissue including radicular pulp

Which of the following dental sequelae is likely in a child


with a history of generalized growth failure (failure to
thrive) in the first six months of life?
Retrusive maxilla C. Enamel hypoplasia
Retrusive mandible D. Small permanent teeth E.
Dentinogenesis imperfecta

The average age at which calcification of crowns of permanent


central incisors is completed is:
birth B. 2-3 years of age C. 4-5 years of age D. 6-7
years of age

Amalgam is most often the restorative material of choice for


primary teeth. The most important modification in its use
for children is in:
cavity preparation D. use of a rubber dam

Downloaded by John Erickson Bangayan (bangayan1903064@ceu.edu.ph)


lOMoARcPSD|19937673

use of a zinc-free alloy loy E. condensation of the al-


placement of matrix bands

The most frequent cause of fracture of a root tip during


extraction of a primary molar is: ankylosis of the tooth
improper use of cowhorn forceps presence of a
supernumerary premolar
root resorption between the apex and the bifurcation
asymmetric root resorption in which only one root is
completely resorbed.

The most common cause of sinus tracts in gingival tissues of


children is:
pericementitis C. periapical cyst
acute periapical abscess D. chronic periapical
ab-
scess

A light bluish, dome-shaped lesion on the inside lip of a


2-year old child is most likely a:
Mucocele B. melanoma C. hematoma D. hemangioma
E. sucking callous

Primary second molars usually erupt during ages:


8-14 months C. 14-20 months
20-30 months D. 30-36 months E. 36-48 months

A child in long-term remission of acute leukemia has


dental problems characterized by unusual susceptibility
to: dental caries C. oral infection
periodontal bone loss D. development of jaw
deformities E. all of the above.

This is a contraindication of HOM spoiled child


C. very young patient defiant child D.
uncooperative behavior but capable of understanding

Induce formation of apical closure of young permanent molar


using CAOH
apexogenesis C. apexification
B. apeximation D. apex formation

Downloaded by John Erickson Bangayan (bangayan1903064@ceu.edu.ph)


lOMoARcPSD|19937673

The growth movement of the mandible is complimented by the


growth of the
maxilla, which is:
down and forward C. downward and backward
B. forward only, D. upward and backward

Direction of displacement of the mandible in an individual


with developing Class II malocclusion:
down and forward, C. down and backward,
B. forward only, D. forward and upward,
E. upward and backward.

Child refuses to accept treatment or open the mount


minimally:
A. Frankl 1 B. Frankl 2 C. Frankl 3 D. Frankl 4

Definitely negative behavior.


A. Frankl 1 B. Frankl 2 C. Frankl 3 D. Frankl 4

Child has good rapport with the dentist.


A. Frankl 1 B. Frankl 2 C. Frankl 3
D. Frankl 4.

The treatment option contraindicated in patients who are not


able to breathe nasally.
no treatment C. nitrous oxide & oxygen
inhalation
B. conscious sedation D. genera anesthesia

The treatment option for patients who have sustained extensive


orofacial & or dental trauma no treatment C.
nitrous oxide & oxygen inhalation
B. conscious sedation D. general anesthesia

With a flush terminal plane, permanent first molars will:


initially be Class II
initially be Class III
immediately assume a normal relationship
erupt immediately into an end-to-end
relationship

Arch shape & symmetry are best evaluated from the:


study cast C. frontal photograph
B. panoramic x-ray D. lateral cephalogram E. intraoral
photograph

Downloaded by John Erickson Bangayan (bangayan1903064@ceu.edu.ph)


lOMoARcPSD|19937673

Bodily force that moves the central incisor mesially produces:


stretching of the periodontal fiber on the distal
side compression of the periodontal fiber in the
distal side osteoblastic activity on the mesial
side osteoclastic activity on the distal side

In infant oral care, clean mouth with gauze after feedings and at
bed time done during:
A. 0-6 mons B. 6-12 mons C. 12-24 mons. D. 24-
36 mons

Space differential between combine width of CDE and 345 is__


A. positive B . negative C. zero D. 2.2 mm E.
2.4 mm

● ORTHODONTICS AND PEDIATRIC DENTISTRY

1. Bite plane therapy for deep bite, active Hawley’s plate are
examples of:
A. Preventive Orthodontics C. Limited Corrective Orthodontics
B. Interceptive Orthodontics D. Extensive Corrective
Orthodontics E. none of the choices

2. The normal sequential events from fertilization to death:


A. growth B. differentiation C. translocation D.
development E. none of the choices

3. ______will give rise to tissues that will become the


mandible
A. Reichert’s cartilage C. Meckel’s cartilage B. 3rd
branchial arch D. 1st branchial arch E.
none of the choices
4. Emphasized how cartilage of nasal septum during growth paced
the growth of maxilla
A. Genetic Theory C. Moss’ Hypothesis
B. Sicher’s Hypothesis D. Scott’s Hypothesis E. Pe-
trovic’s Hypothesis

5. Skeletal tissues grow in response to soft tissue growth



A. Genetic Theory C. Moss Hypothesis
B. Sicher’s Hypothesis D. Scott’s Hypothesis
E. Petrovic’s
Hypothesis

Downloaded by John Erickson Bangayan (bangayan1903064@ceu.edu.ph)


lOMoARcPSD|19937673

6. Cranial vault grows primarily by cartilage growth at:


A. synchondrosis B. sutures C. fontanelles D. disappears
at birth
7. Cranial base grows primarily by__ growth at the
synchondrosis: A. intramembranous B. endochondral
C. combination of intramembranous and endochondral
Over 90% of brain growth is achieved by age: A. 5 B. 6
C. 7 D. 8 E. 15

Neurocranium follows what Scammon’s curve?


A. lymphoid B. neural C. general D. bodily E.
genital type

10. Growth of brain case is primarily by proliferation and


ossification of:
A. synchondrosis B. sutural connective tissue C.
fontanelles D. none of the choices
11. Nasomaxillary complex is hafted to the cranium by the
following sutures except: A.
frontomaxillary C. zygomaticotemporal E.
zygomaticomaxillary
B. pterygopalatine D. frontotemporal

12. As the maxilla decends, there is __ on the orbital floor, __


on the nasal floor and __ on the inferior palatal surface:
A. ++- B. -+- C. +-+ D. +++ E. --+

13. The increase in length of the maxilla is due to:


A. apposition on the posterior border of the maxillary
tuberosity B. progressive movement of the entire
zygomatic process in posterior direction D. A&B
C. growth in the median suture
E. none of the choices

14. Mechanism for maxillary growth:


A. sutural connective tissue proliferation C. surface
apposition / resorption
B. ossification D. translation
E. all of the choices

15. The mechanism for bone growth of the nasomaxillary complex


is: A. intramembranous B. endochondral C.
combination

Downloaded by John Erickson Bangayan (bangayan1903064@ceu.edu.ph)


lOMoARcPSD|19937673

16. The sutures of the nasomaxillary complex are oblique and


more or less parallel with each other thus, growth in these
areas would serve to displace the maxilla:
A. downward B. forward and upward C. downward and
forward D. upward and backward

17. Continued apposition of alveolar bone on the free borders of


the alveolar process as the teeth erupt increases the __ of the
maxilla.
A. height B. width C. length

18. Increase in the width of the maxilla involves palatal


growth following the expanding V principle. The buccal segments
move: A. downward and forward B. posterior and superior
C. upward and posterior D. upward

19. The maxilla Is displaced downward and forward by growth in


__ parts of the bone:
A. anterior and inferior B. posterior and superior C.
anterior and superior D. posterior and anterior

20. What characteristic growth pattern is shown in the vertical


section through the coronoid process?
A. (+) on the lingual, (-) on the buccal C. (-)
on the lingual, (+) on the buccal surface
B. (+) on the lingual, (-) on half of the buccal surface D. (-)
on the lingual, (-) on the buccal surface

21. What characteristic growth pattern is shown in the vertical


section through the basal part of the ramus? A. (+) on the
lingual, (-) on the buccal C. (-) on the lingual,
(+) on the buccal surface
B. (+) on the lingual, (-) on half of the buccal surface
D. (-) on the lingual, (-) on the buccal
surface
22. When teeth are lost, alveolar bone: A. persists B.
resorbs C. no effect

23. Mandibular growth follows what growth curve? A.


lymphoid
B. neural C. general D. genital

24. At age 5-10, the mandible is __ completed. A. 45% B.


40% C. 65% D. 96%

Downloaded by John Erickson Bangayan (bangayan1903064@ceu.edu.ph)


lOMoARcPSD|19937673

25. The predominant direction of mandibular growth is:


A. superior and inferior C. superior and
posterior B. downward and forward D. downward
E. superior
26. Displacement of the mandible occurs in what direction?
A. superior and inferior B. superior and posterior C.
downward and forward D. downward E. superior

27. Increase in mandibular length is accomplished by:


A. (+) on the inferior border of the ramus
B. (-) on the inferior border of the ramus
C. (-) on the posterior border, (+) on the anterior border of
the ramus
D. (+) on the posterior border, (-) on the anterior border of
the ramus

28. Growth of the posterior border of the mandible with additive


growth at the ends of the <V= increases the terminal points.
This increases the __ of the mandible.
A. width B. height C. depth

29. Condylar growth with significant alveolar growth increase


the __ of the mandible.
A. width B. height C. depth

30. Some mandibles grow more forward than downward. If there is


predominance of forward growth __ is the result.
A. Class I B. Class II C. Class III D. open
bite

31. When is the best time to influence , retard or redirect


growth?
A. after puberty C. before puberty
B. just before and during puberty D. time
element is not important

32. The order form most rapid to least amount of growth for the
cranium:
A. height, depth, width B. depth, width, height C.
depth, height, width D. width, height, depth

33. The order form greatest to least incremental change for the
face:

Downloaded by John Erickson Bangayan (bangayan1903064@ceu.edu.ph)


lOMoARcPSD|19937673

A. depth, height, width B. width, depth height C.


height, depth, width D. height, width, depth

34. At age 0-5, which has grown the most at 85%? A.


neurocranium B. maxilla C. mandible

35. Because of the differential growth of the maxilla and the


mandible at age 0-5, the normal profile at birth is: A.
straight
B. convex C. concave

36. Maxillary intercanine width is completed in girls at age:


A. 7-8 B. 9-10 C. 11 D. 12 E. 18

37. Second peak for girls: A. 3 B. 6-7 C. 7-9 D.


11-12 E. 14-15

38. For boys, maximum condylar changes concurrently with sutural


and skeletal height peaks 3 years later that for girls.
A.
true B. false

39. Treatment timing must be based on the individual’s own


pattern of growth. A. true B. false

40. Primate space if found:


A. mesial of the lower cuspid and distal of upper cuspid
B. distal of the lower cuspid and distal of the upper cuspid
C. mesial of the lower cuspid and mesial of the upper cuspid
D. mesial of the upper cuspid and distal of the lower cuspid
41. Normal terminal plane relationship[: A. mesial step
B. distal step C. flush/straight

42. The most common sequence of eruption for the upper arch:
A. 6124537 B. 6123457 C. 6142537 D.
6125437

43. The stage at which a tooth begins its eruptive movement:


A. stage 0 B. stage 2 C. stage 3 D. stage 6
E. stage 10

44.Periapical lesions, pulpitis and pulpotomy of a primary molar


will___ the eruption of successor premolar. A. hasten B.
delay C. have no effect on

Downloaded by John Erickson Bangayan (bangayan1903064@ceu.edu.ph)


lOMoARcPSD|19937673

45. If a primary tooth is extracted prior to the onset of


permanent eruptive movement (prior to root formation), the
permanent tooth is likely to be:
A. delayed B. accelerated C. no effect on
eruption

46. Which one is an incorrect description of Nolla’s stages of


calcification?
A. Stage 4, 2/3 of crown completed C. Stage 7, 1/3 of
root completed
B. Stage 6, crown almost completed D. Stage 9, root
almost complete, open apex

47. Space differential in the anterior segment ( BA/AB ) -


21/12 gives:
A. excess space B. no space differential C.
negative space

48. The proper crown angulation of a permanent incisor is one where the gingival portion of the
long axis of the crown should be __ to the incisal portion.
A. mesial B. distal C. incisal D. cervical

49. Rotated teeth occupy less space within the arch: A. true B. false

50. Plane of occlusion for permanent teeth: A. flat B. exaggerated

51. Posterior crown inclination from canine to molar:


A. lingual crown inclination B. buccal crown inclination
C. upright

52. Size of teeth correlates well with stature. Size of teeth is sex-linked.
A. 1st statement is true, 2nd is false C. both are true
B. 1st statement is false, 2nd is true D. both are false

53. The most prevalent malocclusion in the primary dentition:


A. Class II Div. 1 B. Class I type 5 C. Class III D. anterior openbite E. Class II types 1

54. A dolichofacial face would likely to have ___ arch form.


A. round B. average C. broad and square D. long and tapering

55. A face that is broad and short is:


A. dolichofacial B. brachyfacial C. mesofacial D. normofacial

Downloaded by John Erickson Bangayan (bangayan1903064@ceu.edu.ph)


lOMoARcPSD|19937673

56. The most common malocclusion in the mixed dentition period is:
A. Class II Div. 1 B. anterior open bite C. posterior open bite D. crowding E. Class I type 2

57. A type of conditioning where in a specific response is


immediately rewarded
Classical conditioning C. operator conditioning
Operant conditioning D. conditioning E. classic conditioning

58. The patients history of childhood diseases are under


A. familial history B. social history C. medical history D. dental history

59. The history on the behavior of the child toward dentistry:


A. familial history B. medical history C. Dental history
D. history of present illness

60. Method of early examination where the dentist and parent are
seated face to face
A. Knee position B. leg position C. cradling position
D. Knee to knee position E. all

61. In pedodontic triangle the apex of the triangle is:


A. Child B. Dentist C. Parent D. Assistant

62. Parent who are mistrusting are:


Overprotective Parent B. Neglectful Parent C. Manipulative
Parent D. Hostile parent

63. Parent who are demanding and can extend to directing the
course of the treatment is a :
A. Overprotective Parent B. Neglectful Parent C.
Manipulative Parent D. Hostile parent

64. Panoramic Radiographic Film:


A. 5 < x 7= B. 5= x 12= C. 2=x 3= D. 8= x 10=

65. Frankl behavior rating characterized by refusal of treatment


and extreme negativism:
A. rating 1 B. rating 2 C. rating 3 D. rating 4
66. A period that child undergoes the phase of separation
anxiety:
A. Toddler B. school age C. Pre school D.
Adolescent

Downloaded by John Erickson Bangayan (bangayan1903064@ceu.edu.ph)


lOMoARcPSD|19937673

67. In Nitous oxygen sedation, once a petient takes a distant


gaze the concentration of the nitous oxide- oxygen is reduces
to:
A. 30-70 B. 20-80 C. 40-60 D. 10-90 E. none

68. Device that assess the arterial hemoglobin oxygen saturation


and pulse rate:
A. automated vital sign monitor B. pulse oximeter C.
pretracheal stethoscope D. none

69. Quadrant used in the gluteal region when intramuscular


sedation is used:
A, upper inner quadrant B. upper outer quadrant C. Lower
inner quadrant D. Lower outer quadrant

70. The most unstable form of fluoride used for topical


application: A. Acidulated Phosphate Fluoride B. Stannous
Fluoride C.
Sodium Fluoride . D. None

71. A sudden firm command use dto get the child’s attention:
A. Voice control B. Multisensory communication C. .Aversive
conditioning D. HOME

72. Procedure which slowly develops behavior by reinforcing


successive approximation of the desired behavior until desired
behavior develop:
A. Pre appointment behavior modification C. behavior shaping
Behavior management D. Retraining

73. Parents should be allowed inside a treatment room when:


A. infant to 41 months B. handicapped patient C. 7-8 Years
old
D. A & B E. None

74. A technique of diverting the attention of the patient from


what may be perceived as an umpleasant procedure:
A. tell show do C. Voice control
B. Positive reinforcement D. Distraction E Non
verbal
communication

Downloaded by John Erickson Bangayan (bangayan1903064@ceu.edu.ph)


lOMoARcPSD|19937673

75. Plastics applied to the occlusal surface of posterior teeth


by simple acid etch method to prevent dental caries & early
carious lesion from developing further
A. Composite C. Fluoride
B. Preventive Restorative Resin D. Glass Ionomer
E. None

76. Medicament used in type A PRR:


A. Glass ionomer B. Bonding agent C. Composite D. Pit and
fissure sealant E. all of these

77. Distance of punched holes on the rubber dam for primary


teeth:
A. 1.0mm B. 1.5 mm C. 2.0mm D. 2.5 mm E. 3.0mm

78. Gingival cavosurface beveling is not indicated in Class II


cavity preparation for deciduous teeth, because the direction of
the enamel rods are toward:
A. Cervical B. Occlusal C. gingival D. Apical E. all
of these

79. Etching time for deciduous dentition is ----- compared to


permanent dentition:
A. shorter B. the same C. longer D. none

80. The least amount of tooth surface reduction in stainless


steel crown preparation is:
A. Occlusal B. lingual C. Proximal D. Buccal
E. none

81. Ellis classification of tooth fracture when the coronal pulp


is exposed:
A. Class I B. Class II C. Class III D. Class IV
E. Class V

82. Ellis classification of tooth fracture when a tooth is lost


as a result of trauma
A. Class I B. Class II C. Class III D. Class IV
E. Class V

83. Ellis classification of a non vital traumatized tooth:


A. Class I B. Class II C. Class III D. Class IV
E. Class V

Downloaded by John Erickson Bangayan (bangayan1903064@ceu.edu.ph)


lOMoARcPSD|19937673

84. Treatment of choice for an immature non vital permanent


tooth to stimulate root closure:
A. Indirect pulp capping B. Pulpectomy C. Pulpotomy D.
Apexogenesis E. Apexification

85. Materials that is used to induced apexification:


A. calcium hydroxide B. Zinc oxide eugenol C. Formocresol D.
IRM E. Gutta percha

86. Superficial wound due to scrapping of the mucosa:


A. laceration B. abrasion C. Contusion D.
Attrition E. Erosion

87. Hypoplastic enamel is a result in the disturbance at:


A. initiation stage B. Appostion stage C. .
Morphodifferentation
D. Histodifferentiation E. None
88. Macrodontia incisor crown with two root canals:
A. Flexion B. Fusion C. Gemination D. Supernumerary
root E. none

89. Presence of enamel at the furcation of the root:


A. enameloma B. Enamelin C. ameloblast D. Epstein Pearl
E. all of the above

90. Radiograph picture of a caries is generally--- than the


actual caries:
A. smaller B. the same C. larger D. deeper E. B&D

91. Primary molars with multisurface caries ( 3 or more) are


best restored using:
A.GIC B. Posterior composite C. stainless steel crown
D. amalgam restoration E. none

92. Primary teeth have pulpal inflammation when the bacterial


infested dentin is __ mm from the pulp:
A. 0.6 B. 0.8 C. 1.6 D. 1.8 E. none

93. Requires retentive groove in tooth preparation:

Downloaded by John Erickson Bangayan (bangayan1903064@ceu.edu.ph)


lOMoARcPSD|19937673

A. anterior SSC B. strip crown C. posterior SSC D.


none of these

94. T-cut made on celluloid strip crown is placed on the ___


surface?
A. labial B. lingual C. mesial D. distal

95. The following are favorable storage medium for avulsed


tooth, except:
A. Hank’s buffered saline C. pasteurized bovine milk
B.Isotonic saline D. human saliva E.
water
96.Treatment for non-vital tooth with open apex:

A.IPC B. Pulpotomy C. DPC D. Pulpectomy E.


Apexification

97. This method is not indicated on children who are emotionally


immature
A. TSD B Voice control E.
Positive reinforcement
B. Distraction D. Non Verbal communication

98. Diverts the attention of the patient from what may be


perceived as an unpleasant procedure
A. TSD C Voice control E. Positive
reinforcement
B. Distraction D. Non Verbal communication

99. Conveying reinforcement and guiding behavior through


contact, posture and facial expression
A. TSD C Voice control E.
Positive reinforcement
B. Distraction D. Non Verbal communication

100. Method used to strengthen the occurrence of desired


behavior
through verbal praise, giving tokens and toys
A. TSD C Voice control
B. Positive reinforcement D. Distraction E. Non
Verbal communication

Downloaded by John Erickson Bangayan (bangayan1903064@ceu.edu.ph)


lOMoARcPSD|19937673

● ORTHODONTICS AND PEDIATRIC DENTISTRY

Which of the following conditions is usually present in a Class


II, Division 2 malocclussion?
Open bite
Steep mandibular plane
Mesiocclusion of permanent first molars
Lingual inclination of maxillary central incisors

Excessive orthodontic force used to move a tooth may:


cause hyalinization cause
undermining resorption
crush the periodontal
ligament
A. 1 & 2 B. 2 & 3 C. 1 & 3 D. 1, 2 & 3 E. 3 only

Severely crowded mandibular incisors usually result from:


mesial migration of teeth premature
loss of primary teeth presence of
supernumerary teeth D. tooth
size-arch length discrepancy
How will extraction of a primary maxillary central incisor in
a 5-year old child with incisal spacingaffect the size of
the intercanine space?
The intercanine space will increase in size.
The intercanine space will decrease in size.
No change will occur in the size of the intercanine
space.
The major criterion to differentiate between a true Class
III and a pseudo-Class III malocclusion is: degree of
anterior crossbite presence of a bilateral crossbite
C. existence of a forward shift of the mandible during
closure
occlusal relationship between maxillary and mandibular
first molars

Orthodontic correction of which of the following is most


easily retained?
Diastema C. Expansion
B. Rotation D. Posterior crossbite E. Anterior
crossbite

A bimaxillary protrusion occurs when:

Downloaded by John Erickson Bangayan (bangayan1903064@ceu.edu.ph)


lOMoARcPSD|19937673

large horizontal overlap with biteral crossbite exits


the maxillary dentition is forward with respect to
basal bone
both dental arches are forward with respect to basal bone
D. none of the choices.

The following are the conditions found on the area of


pressure in the presence of heavy forces: area of
hyalinization occlusion of blood vessels stretched
periodontal fibers undermining resoprtion
A. 1, 2& 3 B. 1, 2 & 4 C. 2 , 3 & 4 D. 3 only
E. 1 & 4 only

Generalized osteoclastic activity along the walls of the


alveolar socket is the bone response to: depressing force
C. extrusion
B. elongating force D. rotating force E. both B & D

The major source of anchorage in a maxillary Hawley appliance


is the :
molar clasp C. acrylic portion
B. labial bow of D. interproximal clasp E. bracket
the band

A narrow maxillary arch with respect to midsagittal plane is


said to be in:
Protraction C. contraction
B. retraction D. distraction E. both C & D

A distal shoe space maintainer is indicated when a primary:


incisor is avulsed first molar is prematurely lost
second molar is lost after eruption of a permanent first
molar
second molar is lost before eruption of a permanent first
molar

Complete unilateral lip-jaw palate cleft.


A. Class I B. Class II C. Class III
D. Class IV

Anodontia, diagnosed in a 5-year-old child, primarily affects


the growth of the:
Midface C. mandible
B. maxilla D. alveolar bone E. maxilla

Downloaded by John Erickson Bangayan (bangayan1903064@ceu.edu.ph)


lOMoARcPSD|19937673

& mandible

Which of the following are likely to occur during orthodontic


therapy?
gingival irritation
cementum resorption
increased mobility of
teeth
demineralization of enamel adjacent to appliances in
patients with poor oral hygiene
1, 3 & 4 C. 2 & 4 only E. 3 & 4 only
B. 1 & 4 only D. 1, 2, 3 & 4

The normal downward and forward direction of facial growth


results from upward and backward growth of maxillary
sutures and the mandibular condyle
vertical eruption and mesial drift of the dentitions
interstitial growth in the maxilla and the mandible
epithelial induction at growth centers
A. 1 & 2 B. 1 & 3 C. 1 & 4 D. 2 & 3 E. 2 &
4

An 8-year-old girl has 2mm diastema between permanent


maxillary central incisors. Permanent maxillary lateral
incisors are in position. The diastema is probably the
result of:
thumb-sucking
an abnormal frenum attachment
a normal developmental
process
A. 1 & 2 B. 1 & 3 C. 2 & 3 D. 2 only E. 3 only
When comparing cementum to bone in their responses to
orthodontic forces, cementum resorbs: more readily C.
not at all
B. less readily D. under lighter loads E. by
indirect (undermining) resoprtion

The tooth in the mandibular arch most likely to be malposed


in cases of arch space discrepancy is the:
first molar C. first premalor
B. second molar D. second premolar

Slow progress in molar uprighting in an adult patient is


usally due to:

Downloaded by John Erickson Bangayan (bangayan1903064@ceu.edu.ph)


lOMoARcPSD|19937673

overextended bands an overcontoured


spring lack of anchorage control
the occlusion not being relieved
the stabilizing wire not being
passive

The <V= principle of growth is best illustrated by the:


nasal spetum C. mandibular symphysis
B. mandibular ramus D. spheno-occipital synchondrosis

Anterior crossbite in the primary dentition usually indicates


a developing:
Class I malocclusion
Class II malocclusion
Class III malocclusion
Class IV maloccusion
none of the above

A mixed dentition analysis determines:


intercanine width skeletal growth
pattern discrepancies in jaw size
size of permanent incisors space
available versus space required
With a flush terminal plane, permanent first molars will:
initially be Class II
initially be Class III
immediately assume a normal relationship erupt
immediately into an end-to-end relationship

Arch shape & symmetry are best evaluated from the:


study cast C. frontal photograph
B. panoramic x-ray D. lateral cephalogram E.
intraoral photograph

Bodily force that moves the central incisor mesially produces:


stretching of the periodontal fiber on the distal
side compression of the periodontal fiber in the
distal side osteoblastic activity on the mesial side
osteoclastic activity on the distal side

A 9-year-old patient has a slightly convex profile and a


suspected tooth mass-arch length (circumference)
discrepancy. Before instituting space management procedures,
the dentist should:

Downloaded by John Erickson Bangayan (bangayan1903064@ceu.edu.ph)


lOMoARcPSD|19937673

complete a space analysis


complete a cephalometric
analysis seek orthodontic
consultation all of the above
Either A or C above

A primary maxillary anterior tooth in a 4-year-old child was


traumatically intruded into the tissues so that only half
the tooth is visible. The most appropriate treatment is to:
extract the tooth.
perform a pulpotomy
administer no treatment
place orthodontic bands on adjacent teeth and draw the
tooth down with elastics

Which of the following orthodontic movements of teeth are


most difficult to accomplish?
Tipping
Rotation
Intrusion
Extrusion
Translation
A. 1 & 2 B. 1 & 3 C. 1 & 5 D. 3 & 4 E. 3 & 5
Interstitial growth is observed at which of the following
sites?
Spheno-occipital syndrondrosis
Maxillary tuberosity
Mandibular condyle
Zygomaticomaxillary suture
Apex of an erupting premolar
A. 1 & 2 B. 1 & 3 C. 1 & 4 D. 2 & 4 E. 2 & 5

When force is applied, bone reflexes that produces


deformation of crystalline materials and allows flow of
electric current necessary for tooth movement:
deposition C. hyalinization
B. piezoelectricity D. undermining resoprtion
Cephalometrics is useful in assessing which of the
following relationships? tooth-to-tooth bone-to-bone
tooth-to-bone
A. 1 & 2 B. 1 & 3 C. 2 & 3 D. 1,2 & 3 E. 3 only
Which of the following is the most essential factor related
to correction of an anterior crossbite?

Downloaded by John Erickson Bangayan (bangayan1903064@ceu.edu.ph)


lOMoARcPSD|19937673

age of the patient depth of


the corssbite shape of the
tooth involved space
available mesiodistally
Frankfort-horizontal is a
reference plane
constructed by joining
which of the following
landmarks?
Porion and sella
Porion and nasion
Porion and orbitale
Nasion and Sella

Which of the following is not a space maintainer?


Lingual arch
Nance holding arch
Class III restoration
Stainless steel crown
Palatal expansion appliance

Space closure is least likely to occur after loss of which of


the following teeth?
Primary mandibular canines
Primary mandibular second molars
Primary maxillary first molars
Primary maxillary central incisors
Permanent maxillary central incisors

Crowding that occurs with mandibular incisors after age 18


is most often the result of: orthodontic relapse
periodontal disease impacted thid molars a physiologic
maturational change

The undesirable side-effect most commonly associated with use


of a finger spring to tip the crown of a tooth is:
pain
gingival irritation
tendency for the tooth to intrude
severe mobility of the tooth
tendency for the root apex to move in the direction
opposite from the crown.

After the age 6, the greatest increase in size of the


mandible occurs:

Downloaded by John Erickson Bangayan (bangayan1903064@ceu.edu.ph)


lOMoARcPSD|19937673

at the symphysis C. along the lower border


between the canines D. distal to the first molars
A distal shoe space maintainer is indicated when a primary:
canine is in crossbite first
molar is prematurely lost
second molar is lost after eruption of a permanent first
molar
second molar is lost prior to eruption of a permanent
first molar

Gonion, menton and pogonion are cephalometric landmarks


located on the
midline C. bony chin
B. mandible D. skeletal
profile
A major disadvantage of treatment using cervical headgear is:
impaction of maxillary canines
extrusion of maxillary incisors
extrusion of maxillary molars
potential deformity of the neck
psychologic trauma due to
appearance
Asymmetrical anterior open bite with normal posterior
occlusion is characteristic of:
thumb-sucking mouth
breathing abnormal
swallowing habits both
A & B both B & C

If the norm for the cephalometric angle SNA is 82°, a


patient’s reading of 90° for SNA most likely indicates:
maxillary protrusion the patient’s
ethnic background protrusive
maxillary incisors dysplasia of the
anterior cranial base.
Ideally, a malooclusion should be treated between the ages
of:
5 and 8 years
8 and 10 years
10 and 12 years
12 and 14 years
None of the above. The age at which a malocclusion
is treated should depend on the problem involved.

Downloaded by John Erickson Bangayan (bangayan1903064@ceu.edu.ph)


lOMoARcPSD|19937673

Radiographs of a 5-year old patient show permanent maxillary


first molars inclined mesially with resulting resoprtion of
the distal portions of the roots of primary second molars.
The condition described is:
ankylosis ectopic
eruption premature
eruption internal
resorption
intrafollicular
resoprtion

Fractured maxillary anterior teeth generally occur most often


in children with which of the following dental conditions?
Class I malocclusion
Class II, Division 1 malocclusion
Class II, Division 2 malocclusion
Class III malocclusion
Marked incisor crowding

When a simple tipping force is applied to the crown of a


singlerooted tooth, the center of rotation is usually
located:
at the apex at the
cervical line 5mm
beyond the apex
one-third the root length from the apex
two-third the root length from the
apex.
Which of the following is the most common orofacial
malformation that produces malocclusion?
Cleft palate
Ectodermal dysplasia
Pierre Robin syndrome
Osteogenesis imperfecta
Cleidocranial dysostosis

The cranial vault increases rapidly in size the first few


years postnatally and completes approximately 90% of its
growth by 6 years of age. This growth is typical of which of
the following types of tissues?
neural dental
genital
lymphoid
general
(somatic)

Downloaded by John Erickson Bangayan (bangayan1903064@ceu.edu.ph)


lOMoARcPSD|19937673

On the first dental visit, the basic fear of a child under


the age of 2 is related to: fear of an injection
anxiety over being separated from a parent
anxiety regarding the instruments used to perform dental
treatment
not understanding the reason for dental treatment.

A child with Down’s syndrome is characterized as being:


affectionate
fearful of quick movements capable
of learning dental procedures
both A & B E.
all of these

A 3 ½ year old child has an acute fever, diarrhea, oral


vesicular lesions and gingival tenderness. The most likely
diagnosis is:
thrush drug allergy aphthous
ulcerations acute herpetic
stomatitis necrotizing
ulcerative gingivitis

Transillumination of soft tissues is useful in detecting


which of the following problems in a child?
Koplick’s spot
Sialolithiasis
Aortic stenosis Sickle
cell disease abnormal
frenum attachment

Which of the following is least likely to result from


persistent long-term thumb-sucking?
a deep overbite
protrusion of maxillary incisor
constriction of maxillary arch rotation
of maxillary lateral incisor lingual
inclination of mandibular incisors
The least desirable method used in child management is:
tell-show do technique,
voice control, gift
before treatment,
hand-over-mouth
technique.

Downloaded by John Erickson Bangayan (bangayan1903064@ceu.edu.ph)


lOMoARcPSD|19937673

A child who is reluctant to accept dental treatment


,exhibit negative attitude but not so pronounced is
classified under Frankl ?
I B. II C. III D. IV
The amount of pulp tissue to be removed in formocresol
pulpotomy procedure half of the coronal pulp
coronal pulp tissue down to the cervical constriction of
each root
canal
C. infected portion of coronal pulp tissue
D. total removal of pulp tissue including radicular pulp

Which of the following dental sequelae is likely in a child


with a history of generalized growth failure (failure to
thrive) in the first six months of life?
Retrusive maxilla
Enamel hypoplasia
Retrusive mandible
Small permanent teeth
Dentinogenesis imperfecta

In festooning and trimming a stainless steel crown, greater


length is necessary in the region of the mesiofacial bulge
in a primary: first molar second molar maxillary canine
maxillary lateral incisor

Using a topical fluoride rinse before acid etch direct bonding


of orthodontic brackets is contraindicated because fluoride:
decreases the solubility of enamel increases the pH of the
etching agent causes copious amounts of saliva directly reacts
chemically with the bonding agent.

If a 7-year old patient loses a primary mandibular canine


about the same time the adjacent lateral incisor is erupting
or shortly thereafter, the dentist should be alert to the
possibility of:
a tongue habit a
developing
crossbite
an early eruption of the permanent canine
lingual collapse of mandibular anterior
teeth
The average age at which calcification of crowns of permanent
central incisors is completed is:

Downloaded by John Erickson Bangayan (bangayan1903064@ceu.edu.ph)


lOMoARcPSD|19937673

birth
2-3 years of age
4-5 years of age
6-7 years of age

Amalgam is most often the restorative material of choice for


primary teeth. The most important modification in its use
for children is in:
cavity preparation use
of a rubber dam use of
a zinc-free alloy
condensation of the
alloy placement of
matrix bands

The most frequent cause of fracture of a root tip during


extraction of a primary molar is: ankylosis of the tooth
improper use of cowhorn forceps presence of a
supernumerary premolar
root resorption between the apex and the bifurcation
asymmetric root resorption in which only one root is
completely resorbed.

The most common cause of sinus tracts in gingival tissues of


children is:
pericementitis
periapical cyst acute
periapical abscess
chronic periapical
abscess

A light bluish, dome-shaped lesion on the inside lip of a


2-year old child is most likely a:
mucocele
melanoma
hematoma
hemangioma
sucking
callous

Primary second molars usually erupt during ages:


8-14 months
14-20 months
20-30 months
30-36 months

Downloaded by John Erickson Bangayan (bangayan1903064@ceu.edu.ph)


lOMoARcPSD|19937673

36-48 months

A child in long-term remission of acute leukemia has dental


problems characterized by unusual susceptibility to:
dental caries oral infection
periodontal bone loss
development of jaw
deformities all of the
above.

Supervision of a child’s development of occlusion is most


critical at ages:
3-6 years
7-10 years
11-14 years
14-17 years

A 4-year old child has a traumatized central incisor with a


Class III (Ellis) fracture. The injury occurred about one
month ago, and examination indicates that the pulp is
necrotic. There are no other pathologic findings. Treatment
of choice is:
watchful observation
extraction and use of a space maintainer
pulpectomy and root canal filling using gutta-percha
points and cement
endodontic treatment and root canal filling with a
resorbable paste.

A child with congenital heart disease requires special


treatment planning for dental care because of potential
problems with:
bleeding local
infection
systemic
infection
enamel
hypoplasia
A. 1 , 2 & 3 B. 1 & 3 C. 1 only D. 3 only E.
1,2,3 & 4

Which of the ff. approaches is best for a child suffering


from celebral palsy

Downloaded by John Erickson Bangayan (bangayan1903064@ceu.edu.ph)


lOMoARcPSD|19937673

HOM exercise
towel method use of
sedation
psychological
approaches

Type of fear which is produced by direct physical stimulation


of sense organ associative fear objective fear
subjective fear acquired fear

Agent used to reduce pain without affecting


consciousness hypnotic drug analgesic
barbiturate general anesthesia

The drug that modifies fear and anxiety


A. flanax B. ethamizid C. celestamine
D. seconal

A general characteristic of a 12 year old


child mother as the center of his world
exhibits tantrums separation anxiety
rejects parental authority

This is a psychological approach to manage a


child nitrous oxide inhalation giving
diazepam mutisensory techniques
HOM

This is a contraindication of
HOM spoiled child very
young patient defiant
child
uncooperative behavior but capable of understanding

Induce formation of apical closure of young permanent molar


using CAOH apexogenesis C. apexification
B. apeximation D. apex formation

The growth movement of the mandible is complimented by the


growth of the
maxilla, which is:
down and forward C. downward and backward
B. forward only, D. upward and backward

Downloaded by John Erickson Bangayan (bangayan1903064@ceu.edu.ph)


lOMoARcPSD|19937673

Direction of displacement of the mandible in an individual


with developing Class II malocclusion:
down and forward, C. down and backward,
B. forward only, forward and upward,
upward and backward.

Child refuses to accept treatment or open the mount


minimally:
A. Frankl 1 B. Frankl 2 C. Frankl 3 D. Frankl 4

Definitely negative behavior.


A. Frankl 1 B. Frankl 2 C. Frankl 3 D. Frankl 4

Child has good rapport with the dentist.


A. Frankl 1 B. Frankl 2 C. Frankl 3
D. Frankl 4.

The treatment option contraindicated in patients who are not


able to breathe nasally.
no treatment C. nitrous oxide & oxygen inhalation
B. conscious sedation D. genera anesthesia
The treatment option for patients who have sustained
extensive orofacial & or dental trauma no treatment
C. nitrous oxide & oxygen inhalation
B. conscious sedation D. general anesthesia

In infant oral care, clean mouth with gauze after feedings and
at bed time done during:
A. 0-6 mons B. 6-12 mons C. 12-24 mons. D. 2436
mons

Space differential between combine width of CDE and 345 is__


A. positive B . negative C. zero D.
2.2 mm E. 2.4 mm

Which of the developmental space will cause a decrease in


arch perimeter when pre-empted?
primate space C. interocclusal space
B, interdental space D. leeway
space
This is an increase in size by expansion from within
sutural apposition C. interstitial growth
maturation D. development

Downloaded by John Erickson Bangayan (bangayan1903064@ceu.edu.ph)


lOMoARcPSD|19937673

This is growth movement which mean movement of whole bone as


a unit
A. drift B. displacement C. deposition D.
relocation E. remodeling

Mandible comes from ? branchial arch first


B. second C. third D. fourth

Maxilla is formed from the maxillary processes and ?


medial nasal process C. globular process
B. lateral processes D. tuberculum
impar

In young child, paradoxical excitement occurs most frequently


following premedication with:
a narcotic a
barbiturate
nitrous
oxide an
amphetamine

The maximum amount of anesthesia that can be given to a 36-pound


child is:
4 carpules
3 carpules
2 carpules
1 carpule

Dental Fluorosis is most likely to occur on a:


two-year-old child ingesting 1 ppm of fluoride
17-year-old patient ingesting 1 ppm of fluoride
on a 3-year-old child ingesting 0.50 ppm of
fluoride D. none of the above.

Pulpotomy with formocresol of primary tooth:


result in lower success rate compared to calcium hydroxide,
result in higher success rate compared to calcium hydroxide,
should never be used,
induces formation of a dentin bridge at site of amputation,
none of the above.

A 4-year-old child visited a dentist for the first time and


received prophylaxis
quite well. Before he left, the dentist was glad that
he gave the child a toy.

Downloaded by John Erickson Bangayan (bangayan1903064@ceu.edu.ph)


lOMoARcPSD|19937673

This kind of gesture is


called: classical
conditioning positive
reinforcement,
bribery, public
relations none of the
above.

If a primary tooth has been extracted before succedaneous tooth


has begun eruptive movement, its eruption will be __ hastened
B. delayed C. not affected

● ORTHODONTICS AND PEDIATRIC DENTRISTRY Shade AE if the


answer is not in the choices given.
Fractured maxillary anterior teeth generally occur most often
in children with which of the following dental conditions?
Class I malocclusion D. Class II, Division 1 malocclusion
Class II, Division 2 malocclusion E. Class III malocclusion
Marked incisor crowding

When a simple tipping force is applied to the crown of a


singlerooted tooth, the center of rotation is usually
located:
at the apex D. at the cervical line
5mm beyond the apex E. one-third the root length from the apex
two-third the root length from the apex.

Which of the following is the most common orofacial


malformation that produces malocclusion?
Cleft palate C. Ectodermal dysplasia
Pierre Robin syndrome D. Osteogenesis imperfect E. Cleidocranial dysos-
tosis

The cranial vault increases rapidly in size the first few


years postnatally and completes approximately 90% of its
growth by 6 years of age. This growth is typical of which of
the following types of tissues?
Neural B. dental C. genital D. lymphoid E. general (somatic)

On the first dental visit, the basic fear of a child under


the age of 2 is related to: fear of an injection

Downloaded by John Erickson Bangayan (bangayan1903064@ceu.edu.ph)


lOMoARcPSD|19937673

anxiety over being separated from a parent anxiety regarding the


instruments used to perform dental treatment not understanding
the reason for dental treatment.

The maximum amount of anesthesia that can be given to a


36-pound child is:
4 carpules B. 3 carpules C. 2 carpules D. 1 carpule

Dental Fluorosis is most likely to occur on a:


two-year-old child ingesting 1 ppm of fluoride 17-year-old
patient ingesting 1 ppm of fluoride
on a 3-year-old child ingesting 0.50 ppm of fluoride
D. none of the above.

Pulpotomy with formocresol of primary tooth:


result in lower success rate compared to calcium hydroxide,
result in higher success rate compared to calcium hydroxide,
should never be used, induces formation of a dentin bridge at
site of amputation, none of the above.

A 4-year-old child visited a dentist for the first time and


received prophylaxis
quite well. Before he left, the dentist was glad
that he gave the child a toy.
This kind of gesture is called:
A. classical conditioning C. positive
reinforcement,
B. bribery, D. public relations E.
none of the above.

If a primary tooth has been extracted before succedaneous


tooth has begun eruptive movement, its eruption will be __
hastened B. delayed C. not affected

The most common cause of sinus tracts in gingival tissues of


children is:
Pericementitis C. periapical cyst
acute periapical abscess D. chronic periapical abscess

Downloaded by John Erickson Bangayan (bangayan1903064@ceu.edu.ph)


lOMoARcPSD|19937673

The most frequent cause of fracture of a root tip during


extraction of a primary molar is: ankylosis of the tooth improper use
of cowhorn forceps presence of a supernumerary premolar root resorption between
the apex and the bifurcation asymmetric root resorption in which only one root is
completely resorbed.

A light bluish, dome-shaped lesion on the inside lip of a


2-year old child is most likely a:
Mucocele B. melanoma C. hematoma D. hemangioma E. sucking callous

Primary second molars usually erupt during ages:


8-14 months C. 14-20 months
20-30 months D. 30-36 months E. 36-48 months

A child in long-term remission of acute leukemia has dental


problems characterized by unusual susceptibility to:
dental caries C. oral infection
periodontal bone loss D. development of jaw deformities E. all of the
above.

Agent used to reduce pain without affecting consciousness


hypnotic drug B. analgesic C. barbiturate D. gen-
eral anesthesia

The drug that modifies fear and anxiety


A. flanax B. ethamizid C. celestamine
D. seconal

A general characteristic of a 12 year old child


mother as the center of his world C. exhibits tantrums
separation anxiety D. rejects parental authority

This is a psychological approach to manage a child


nitrous oxide inhalation C. giving diazepam
mutisensory techniques D. HOM

This is a contraindication of HOM


spoiled child C. very young patient
defiant child D. uncooperative behavior but capable of
understanding

Downloaded by John Erickson Bangayan (bangayan1903064@ceu.edu.ph)


lOMoARcPSD|19937673

Child has good rapport with the dentist.


A. Frankl 1 B. Frankl 2 C. Frankl 3
D. Frankl 4.

The treatment option contraindicated in patients who are not


able to breathe nasally.
no treatment C. nitrous oxide & oxygen inhalation
B. conscious sedation D. genera anesthesia
The treatment option for patients who have sustained
extensive orofacial & or dental trauma no treatment C. nitrous
oxide & oxygen inhalation B. conscious sedation D. general anesthesia

In infant oral care, clean mouth with gauze after feedings and
at bed time done during:
A. 0-6 mons B. 6-12 mons C. 12-24 mons.
D. 24-36 mons

Which of the developmental space will cause a decrease in


arch perimeter when pre-empted?
primate space C. interocclusal space
B, interdental space D. leeway
space
This is an increase in size by expansion from
within sutural apposition C. interstitial growth
maturation D. development

This is growth movement which mean movement of whole bone as


a unit
A. drift B. displacement C. deposition D.
relocation E. remodeling

The normal downward and forward direction of facial growth


results from upward and backward growth of maxillary
sutures and the mandibular condyle
vertical eruption and mesial drift of the dentitions
interstitial growth in the maxilla and the mandible
epithelial induction at growth centers
A. 1 & 2 B. 1 & 3 C. 1 & 4 D. 2 & 3 E. 2 &
4

Downloaded by John Erickson Bangayan (bangayan1903064@ceu.edu.ph)


lOMoARcPSD|19937673

An 8-year-old girl has 2mm diastema between permanent


maxillary central incisors. Permanent maxillary lateral
incisors are in position. The diastema is probably the
result of:
thumb-sucking
an abnormal frenum attachment
a normal developmental
process
A. 1 & 2 B. 1 & 3 C. 2 & 3 D. 2 only E. 3 only
When comparing cementum to bone in their responses to
orthodontic forces, cementum resorbs:
more readily C. not at all
B. less readily D. under lighter loads E. by
indirect (undermining) resoprtion

The tooth in the mandibular arch most likely to be malposed


in cases of arch space discrepancy is the:
first molar B. second molar C. first premalor D. second
premolar

Generalized osteoclastic activity along the walls of the


alveolar socket is the bone response to: depressing force C.
extrusion
B. elongating force D. rotating force E. both B & D

The major source of anchorage in a maxillary Hawley appliance


is the :
molar clasp C. acrylic portion
B. labial bow D. interproximal clasp E.
bracket of the band

A narrow maxillary arch with respect to midsagittal plane is


said to be in:
Protraction C. contraction
B. retraction D. distraction E. both C & D

A distal shoe space maintainer is indicated when a primary:


incisor is avulsed
first molar is prematurely lost second molar is lost after eruption
of a permanent first molar second molar is lost before eruption
of a permanent first molar

Downloaded by John Erickson Bangayan (bangayan1903064@ceu.edu.ph)


lOMoARcPSD|19937673

Complete unilateral lip-jaw palate cleft.


A. Class I B. Class II D. C. Class III
Class IV

Space differential between combine width of CDE and 345 is__


A. positive B . negative C. zero D.
2.2 mm E. 2.4 mm

The least desirable method used in child management is:


tell-show do technique, C. voice control, gift before
treatment, D. hand-over-mouth technique.

In festooning and trimming a stainless steel crown, greater


length is necessary in the region of the mesiofacial bulge
in a primary:
first molar B. second molar C. maxillary canine D. maxillary lateral in-
cisor

Which of the following conditions is usually present in a


Class
II, Division 2 malocclussion?
Open bite C. Mesiocclusion of permanent first
molars
Steep mandibular plane D. Lingual inclination of
maxillary central incisors

Severely crowded mandibular incisors usually result from:


mesial migration of teeth C. premature loss of primary teeth
presence of supernumerary teeth D. tooth size-arch length discrepancy

Excessive orthodontic force used to move a tooth may:


cause hyalinization cause
undermining resorption
crush the periodontal
ligament
A. 1 & 2 B. 2 & 3 C. 1 & 3 D. 1, 2 & 3 E. 3 only

How will extraction of a primary maxillary central incisor in


a 5-year old child with incisal spacingaffect the size of
the intercanine space?
The intercanine space will increase in size.

Downloaded by John Erickson Bangayan (bangayan1903064@ceu.edu.ph)


lOMoARcPSD|19937673

The intercanine space will decrease in size.


No change will occur in the size of the intercanine space.

Orthodontic correction of which of the following is most


easily retained?
Diastema C. Expansion
B. Rotation D. Posterior crossbite E. Anterior
crossbite

A bimaxillary protrusion occurs when:


large horizontal overlap with biteral crossbite exits the
maxillary dentition is forward with respect to basal bone
both dental arches are forward with respect to basal bone
D. none of the choices.

The following are the conditions found on the area of


pressure in the presence of heavy forces: area of
hyalinization occlusion of blood vessels stretched
periodontal fibers undermining resoprtion
A. 1, 2& 3 B. 1, 2 & 4 C. 2 , 3 & 4 D. 3 only
E. 1 & 4 only

Anodontia, diagnosed in a 5-year-old child, primarily affects


the growth of the:
Midface C. mandible
B. maxilla D. alveolar bone E. maxilla
& mandible

Which of the following are likely to occur during orthodontic


therapy?
gingival irritation
cementum resorption
increased mobility of
teeth
demineralization of enamel adjacent to appliances in
patients with poor oral hygiene
1, 3 & 4 B. 1 & 4 only C. 2 & 4 only D. 1, 2, 3 & 4 E. 3 & 4 only

Slow progress in molar uprighting in an adult patient is


usally due to:
overextended bands D. an overcontoured spring

Downloaded by John Erickson Bangayan (bangayan1903064@ceu.edu.ph)


lOMoARcPSD|19937673

lack of anchorage control E. the occlusion not being relieved


the stabilizing wire not being passive

The <V= principle of growth is best illustrated by the:


nasal spetum C. mandibular symphysis
B. mandibular ramus D. spheno-occipital synchondrosis

Anterior crossbite in the primary dentition usually indicates


a developing:
Class I malocclusion C. Class II malocclusion
Class III malocclusion D. Class IV malocclusion E. none of the above

A mixed dentition analysis determines:


intercanine width D. skeletal growth pattern
discrepancies in jaw size E. size of permanent incisors
space available versus space required

A primary maxillary anterior tooth in a 4-year-old child was


traumatically intruded into the tissues so that only half
the tooth is visible. The most appropriate treatment is to:
extract the tooth.
perform a pulpotomy
administer no treatment
place orthodontic bands on adjacent teeth and draw the tooth down with elastics

A child who is reluctant to accept dental treatment


,exhibit negative attitude but not so pronounced is
classified under Frankl ?
I B. II C. III D. IV

The amount of pulp tissue to be removed in formocresol


pulpotomy procedure
half of the coronal pulp
coronal pulp tissue down to the cervical constriction of each root canal
C. infected portion of coronal pulp tissue
D. total removal of pulp tissue including radicular pulp

Downloaded by John Erickson Bangayan (bangayan1903064@ceu.edu.ph)


lOMoARcPSD|19937673

Which of the following dental sequelae is likely in a child


with a history of generalized growth failure (failure to
thrive) in the first six months of life?
Retrusive maxilla C. Enamel hypoplasia
Retrusive mandible D. Small permanent teeth E. Dentinogenesis im-
perfecta

Which of the following orthodontic movements of teeth are


most difficult to accomplish?
Tipping
Rotation
Intrusion
Extrusion
Translation
A. 1 & 2 B. 1 & 3 C. 1 & 5 D. 3 & 4 E. 3 & 5
Interstitial growth is observed at which of the following
sites?
Spheno-occipital syndrondrosis
Maxillary tuberosity
Mandibular condyle
Zygomaticomaxillary suture
Apex of an erupting premolar
A. 1 & 2 B. 1 & 3 C. 1 & 4 D. 2 & 4 E. 2 & 5

When force is applied, bone reflexes that produces


deformation of crystalline materials and allows flow of
electric current necessary for tooth movement: deposition C.
hyalinization
B. piezoelectricity D. undermining resoprtion
Cephalometrics is useful in assessing which of the
following relationships? tooth-to-tooth bone-to-bone
tooth-to-bone
A. 1 & 2 B. 1 & 3 C. 2 & 3 D. 1,2 & 3 E. 3 only
Which of the following is the most essential factor related
to correction of an anterior crossbite? age of the patient depth of
the corssbite shape of the tooth involved space available mesiodistally

Crowding that occurs with mandibular incisors after age 18 is


most often the result of:
orthodontic relapse C. periodontal disease
impacted thid molars D. a physiologic maturational change

Downloaded by John Erickson Bangayan (bangayan1903064@ceu.edu.ph)


lOMoARcPSD|19937673

The undesirable side-effect most commonly associated with use


of a finger spring to tip the crown of a tooth is: Pain D.
gingival irritation
tendency for the tooth to intrude E. severe mobility of the tooth
tendency for the root apex to move in the direction opposite from the crown.

After the age 6, the greatest increase in size of the


mandible occurs:
at the symphysis C. along the lower border
between the canines D. distal to the first
molars

A distal shoe space maintainer is indicated when a primary:


canine is in crossbite first
molar is prematurely lost
second molar is lost after eruption of a permanent first molar
second molar is lost prior to eruption of a permanent first molar

If the norm for the cephalometric angle SNA is 82°, a


patient’s reading of 90° for SNA most likely indicates:
maxillary protrusion C. the patient’s ethnic background protrusive
maxillary incisors D. dysplasia of the anterior cranial base.

Ideally, a malooclusion should be treated between the ages


of:
5 and 8 years D. 8 and 10 years
10 and 12 years E. 12 and 14 years
None of the above. The age at which a malocclusion is treated should depend on
the problem involved.

Frankfort-horizontal is a reference plane constructed by


joining which of the following landmarks?
Porion and sella C. Porion and nasion
Porion and orbitale D. Nasion and Sella

Which of the following is not a space maintainer?


Lingual arch C. Nance holding arch

Downloaded by John Erickson Bangayan (bangayan1903064@ceu.edu.ph)


lOMoARcPSD|19937673

Class III restoration D. Stainless steel crown E. Palatal


expansion appliance

Space closure is least likely to occur after loss of which of


the following teeth?
Primary mandibular canines D. Primary mandibular second mo-
lars
Primary maxillary first molars E. Primary maxillary central incisors Permanent
maxillary central incisors

Radiographs of a 5-year old patient show permanent maxillary


first molars inclined mesially with resulting resoprtion of
the distal portions of the roots of primary second molars.
The condition described is:
Ankylosis C. ectopic eruption
premature eruption D. internal resorption E. intrafollicular
resoprtion

A child with Down’s syndrome is characterized as being:


Affectionate C. fearful of quick movements
capable of learning dental procedures D. both A & B E. all of these

A 3 ½ year old child has an acute fever, diarrhea, oral


vesicular lesions and gingival tenderness. The most likely
diagnosis is:
Thrush C. drug allergy
aphthous ulcerations D. acute herpetic stomatitis E. necrotizing
ulcerative gingivitis

Transillumination of soft tissues is useful in detecting


which of the following problems in a child?
Koplick’s spot C. Sialolithiasis
Aortic stenosis D. Sickle cell disease E. abnormal frenum
attachment

Which of the following is least likely to result from


persistent long-term thumb-sucking?

Downloaded by John Erickson Bangayan (bangayan1903064@ceu.edu.ph)


lOMoARcPSD|19937673

a deep overbite D. protrusion of maxillary incisor constriction of


maxillary arch E. rotation of maxillary lateral incisor lingual
inclination of mandibular incisors

A 9-year-old patient has a slightly convex profile and a


suspected tooth mass-arch length (circumference)
discrepancy. Before instituting space management procedures,
the dentist should:
complete a space analysis C. complete a cephalometric analysis
seek orthodontic consultation D. all of the above E. Either A or C

Using a topical fluoride rinse before acid etch direct


bonding of orthodontic brackets is contraindicated because
fluoride: decreases the solubility of enamel C. increases the pH of the etching
agent causes copious amounts of saliva D. directly reacts chemically with the
bonding agent.

If a 7-year old patient loses a primary mandibular canine


about the same time the adjacent lateral incisor is erupting
or shortly thereafter, the dentist should be alert to the
possibility of:
a tongue habit C. a developing crossbite
an early eruption of the permanent canine D. Lingual collapse of mandibular
anterior teeth

The average age at which calcification of crowns of permanent


central incisors is completed is:
Birth B. 2-3 years of age C. 4-5 years of age D. 6-7 years of age

Amalgam is most often the restorative material of choice for


primary teeth. The most important modification in its use
for children is in:
cavity preparation C. use of a rubber dam
use of a zinc-free alloy D. condensation of the alloy E. placement of matrix bands

Supervision of a child’s development of occlusion is most


critical at ages:
3-6 years B. 7-10 years C. 11-14 years D. 14-17 years

A 4-year old child has a traumatized central incisor with a


Class III (Ellis) fracture. The injury occurred about one

Downloaded by John Erickson Bangayan (bangayan1903064@ceu.edu.ph)


lOMoARcPSD|19937673

month ago, and examination indicates that the pulp is


necrotic. There are no other pathologic findings. Treatment
of choice is:
watchful observation
extraction and use of a space maintainer pulpectomy and root canal
filling using gutta-percha points and cement endodontic treatment
and root canal filling with a resorbable paste.

A child with congenital heart disease requires special


treatment planning for dental care because of potential
problems with:
bleeding local
infection
systemic
infection
enamel
hypoplasia
A. 1 , 2 & 3 B. 1 & 3 C. 1 only D. 3 only E.
1,2,3 & 4

With a flush terminal plane, permanent first molars will:


initially be Class II C. immediately assume a normal relationship
initially be Class III D. erupt immediately into an end-to-end
relationship

Arch shape & symmetry are best evaluated from the:


study cast C. frontal photograph
B. panoramic x-ray D. lateral cephalogram E.
intraoral photograph

Bodily force that moves the central incisor mesially produces:


stretching of the periodontal fiber on the distal side
compression of the periodontal fiber in the distal
side osteoblastic activity on the mesial side
osteoclastic activity on the distal side

Which of the ff. approaches is best for a child suffering


from celebral palsy
HOM exercise C. towel method
use of sedation D. psychological approaches

Downloaded by John Erickson Bangayan (bangayan1903064@ceu.edu.ph)


lOMoARcPSD|19937673

Type of fear which is produced by direct physical stimulation


of sense organ
associative fear B. objective fear C. subjective fear D. acquired fear

Induce formation of apical closure of young permanent molar


using CAOH
apexogenesis B. apexification C. apeximation D. apex formation

Gonion, menton and pogonion are cephalometric landmarks


located on the midline B. mandible C. bony chin D. skeletal profile

A major disadvantage of treatment using cervical headgear is:


impaction of maxillary canines D. extrusion of maxillary incisors extrusion of
maxillary molars E. potential deformity of the neck psychologic trauma
due to appearance

Asymmetrical anterior open bite with normal posterior


occlusion is characteristic of:
thumb-sucking C. mouth breathing
abnormal swallowing habits B. both A & B E. both B & C

The growth movement of the mandible is complimented by the


growth of the
maxilla, which is:
down and forward C. downward and backward
B. forward only, D. upward and backward

Direction of displacement of the mandible in an individual


with developing Class II malocclusion:
down and forward, C. down and backward,
B. forward only, D. forward and upward,
E. upward and backward.

Child refuses to accept treatment or open the mount


minimally:
A. Frankl 1 B. Frankl 2 C. Frankl 3
D. Frankl 4

Definitely negative behavior.


A. Frankl 1 B. Frankl 2 C. Frankl 3 D. Frankl 4

Downloaded by John Erickson Bangayan (bangayan1903064@ceu.edu.ph)


lOMoARcPSD|19937673

Mandible comes from ? branchial arch


first B. second C. third D. fourth

Maxilla is formed from the maxillary processes and ?


medial nasal process C. globular process
B. lateral processes D. tuberculum
impar

Protrusiveness or retrusiveness of the chin point can be known


by analyzing the:
A. SNB B. FMLA C. IMPA D. FMA E. FH/NP

In young child, paradoxical excitement occurs most


frequently following premedication with: a narcotic C. a
barbiturate nitrous oxide D. an amphetamine

● ORTHODONTICS AND PEDIATRIC DENTISTRY

1. The principal growth site of the cranial base believed to be responsible for its
anteroposterior growth is?
spheno-occipital C. spheno-ethmoidal intersphenoidal
D. intraoccipital

2. Which of the following is not a source of extra space for the resolution of permanent
incisor crowding in the lower arch?
increase in intercanine width
labial positioning of the permanent incisors distal
movement of the canines into the primate space
deposition of bone at the posterior border of the ramus

3. Which of the following factors will not increase maxillary width?


growth of the palate following <V= principle
deposition at the fronto-maxillary suture deposition
at the lateral walls
deposition at the median palatine sutre

4. The major mechanism for growth of the cranial case is the:


expansion of the cartilage cells interstitial
growth of bone apposition of new bone at
the synchondroses apposition of new
bone at the sutures.

Downloaded by John Erickson Bangayan (bangayan1903064@ceu.edu.ph)


lOMoARcPSD|19937673

5. The vertical lengthening of the maxillary complex is brought about by a composite of


factors, except:
deposition of bone on the oral side of the palate with compensating resopriton on
the entire oral side
deposition on the posterior facing cortical surface of the maxillary tuberosity
deposition on the various sutures where it contacts the other bones above it

6. The main growth site of the mandible which is responsible for its increase ion
height?
Condyle C. symphysis
gonial angle D. posterior border of the ramus E. inferior border of the body

7. The sequence of completion of facial growth by planes of space is?


depth, width, height C. height, depth, width
width, depth, height D. width, height, depth E. depth, height, width

8. The theory which states that the growth of the craniofacial bones is caused by the
soft tissues adjacent to them.
Functional matrix C. Sutural Dominance theory Cartilaginous
growth theory D. Limborgh’s theory

9. The <V= principle of growth is best illustrated by the?


nasal septum
posterior border of the mandibular ramus mandibular
symphysis
intersphenoidal synchondrosis

10. Space for eruption of permanent mandibular molars is created by:


apposition at the anterior border of the ramus
apposition at the alveolar process
resorption at the anterior border of the ramus resorption
at the posterior border of the ramus

11. Displacement of the mandible due to its growth at the condyle and posterior border
of the ramus is?
A. primary B. secondary C. tertiary D. cortical drift

12. Chin cup is used to intercept a?


Class 1 type 3 C. Class II Class III
D. Developing Class III

Downloaded by John Erickson Bangayan (bangayan1903064@ceu.edu.ph)


lOMoARcPSD|19937673

13. A direct growth movement that is produced by deposition on one side of the orbital
plate with resorption on the opposite side is?
Displacement C. translation
cortical drift D. remodeling

14.. The following are distinctive structural features related to cartilage of the cranial
base, except:
pressure- tolerant C. grows interstitially and appositionally
matrix is non-vascular D. grows appositionally

15. What is the chief factor in the formation of the alveolar process?
normal process of growth
eruption of teeth
lengthening of the condyle
overall growth of the maxilla and mandible

16. Which of the following is not a characteristic of a mature swallow?


active contractions of the muscles of the lips
tongue tip is placed against alveolar process behind the upper
incisors teeth are together during swallowing both A & B
both A & C

17. These are six soft spots present between the bones of the skull roof.
Sutures B. synchondroses C. fontanelles D. cartilages

18. Class I permanent molar relationship can be achieved through the following, except
Late mesial shift after the loss of second primary molar
Greater forward growth of the mandible than the
maxilla Combination of both None of the above.

19. An inherent disposition of most teeth to drift mesially even before they are in
occlusion:
mesial drifting tendency
anterior component of force D. both A & B
physiologic movement of teeth E. both B & C

20. Spaces between the primary anteriors:


interdental space
primate space D. both A & B
Nance leeway space E. both B & C

21. Primate space is found between the:


maxillary canine and first molar
mandibular primary canine and lateral

Downloaded by John Erickson Bangayan (bangayan1903064@ceu.edu.ph)


lOMoARcPSD|19937673

incisor maxillary canine and lateral


incisors none of the above

22. At what stage in Nolla’s classification does a permanent tooth start to erupt?
Stage 7 C. Stage 6
Stage 5 D. Stage 8 E. Stage 4

23. From the flush terminal plane relationship of molars in the primary dentition, the
permanent first molar relationship in the permanent dentition can become the
following in the transition period, except:
Class I C. Class III
Class II D. end to end / cusp to cusp E. none of the choices

24. Which of the following are normal signs of primary dentition?


ovoid arch form deep
overjet and overbite
straight terminal plane
primate spaces
A. I, II, III & IV B. II, III & IV C. I, III & IV D. I, II & IV

25. It is the difference in size between the primary teeth and their permanent successor
in the posterior segment:
posterior liability C. Posterior size discrepancy
Nance Leeway space D. Late mesial shift

26. Mouth breather who seldom approximate their lips do muscle exercises. This can
be varied
by. A. playing wind instrument B. blowing exercise C. A & B D. none

27. Child is 10years old. Tooth no. 55 has been exfoliated but tooth no. 65 is still very
intact.
What must be done?
A. none, wait for 65 to exfoliate C. do percussion test before extracting 65
B. Extract 65 at once D. take x-ray to evaluate

28. When is the proper time to remove a space maintainer?


crown successor visible clinically and it is in stage 8 radiographically
B. bone barrier has resorbed
C. crown of successor is fully
erupted
D. age 12.

29. Two or more tooth moving opposite directions and pitted against each other is
equal and opposite. The anchorage is.

Downloaded by John Erickson Bangayan (bangayan1903064@ceu.edu.ph)


lOMoARcPSD|19937673

A. simple B. stationary C. reciprocal D. none.

30. The following are caused by thumbsucking habit Except.


A. anterior open bite B. protrusion of upper incisor C. expansion of upper arch

31. Mesial surface of E can disked in the case of 3 cannot erupt because of insufficient
space,
This is.
A. contingency of extraction B. occlusal equilibrium C. space regaining D.
none.

32. In a condition characterized by partial or complete absence of clavicle.


A. torticollis B. cerebral palsy C. cleidocranial dysostosis

33. A condition characterized by foreshortening of sternocleidomastoid muscle. A.


torticollis B. cerebral palsy C. cleidocranial dysostosis

34. Type of force recommended for removable orthodontic appliance.


A. continuous B. functional C. dissipating D. intermittent

35. Type of force which can be seen in fixed appliance is.


A. continuous B. functional C. dissipating D. intermittent

36. The ages of bone development for girls is.


A. 7,9,15,18 B. 8,10,12,17 C. 6,7,12,13 D. 7,8,11,12

37. It can detect thyroid disturbances which has an oral manifestation.


A. hand & wrist x-ray B. electromyographic exam C. BMR D. biostatic

38. Osteoclastic activity occurs on the.


A. tension side B. pressure side C. stress D. compression

39. Bone deposition occurs on the. A. tension side B. pressure side C. stress D.
compression

40. The tooth is push towards the socket and pulled away from occlusal plane.
A. elongating force B. rotation C. depressing D. heavy force

41. Neutroclusion with labioversion of max centrals & buccoversion of mand. 1st
premolar.
A. Class 1 type 2 & 3 B. Class 1 type 2 & 4 C. Class 1 type 2 & 5

Downloaded by John Erickson Bangayan (bangayan1903064@ceu.edu.ph)


lOMoARcPSD|19937673

42. A supraverted mand. Premolar with respect to Frankfurt horizontal plane is said it
be in.
A. contraction B. abstraction C. Attraction D. distraction

43.. The resorption occurs from behind an immediate site of pressure, there is
accumula-
tion of fluid and blood vessels are
occluded.
direct resorption C. undetermining resorption
B. frontal osteoclastic attack D. all of the Above.

44. It refers to perfect arrangement of teeth when jaw’s are closed & condyies are at
rest in the glenoid fossa.
A. normal occlusion B. ideal occlusion C. malocclusion

45. The incidence of malocclusion is very high in.


homogeneous population B. Heterogeneous population C. Eskimos D. none of the
above

46. The predominant type of malocclusion seen in mixed dentition is. A. Crowding
B. anterior open bite C. Class II Div. 1

47. A brachycephalic individual is associated with___ arch form:


A. square, B. ovoid, C. long and narrow, D. none of the above.

48. Increase in maxillary arch perimeter can be due to the following, except:
labial tipping or eruption of permanent incisors,
diverging increase in the height of alveolar bone,
interproximal wear out of tooth surfaces.

49. Direction of displacement of the mandible in an individual with developing Class


II malocclusion:
down and forward C. down and backward forward only, D. forward
and upward, E. upward and backward.

50. The growth movement of the mandible is complimented by the growth of the
maxilla, which is:
down and forward, C. down and backward,
forward only D. upward and backward.

A type of conditioning where in a specific response is immediately rewarded


Classical conditioning C. operator conditioning
Operant conditioning D. conditioning E. classic conditioning

Downloaded by John Erickson Bangayan (bangayan1903064@ceu.edu.ph)


lOMoARcPSD|19937673

A type of preventive dentistry where the main concern is to slow down the process of a
disease:
A. Primary prevention C. tertiary prevention
B. Secondary prevention D. All of the Above

Characteristic behavior that is based on belief:


A. Phenomelogical C. Behavioral
B. Neurobiological D. Psychoanalytical E. none

The patients history of childhood diseases are under


A. familial history C. social
history
B. medical history D. dental history

The history on the behavior of the child toward dentistry:


A. familial history C. medical history
B. Dental history D. history of present illness

Method of early examination where the dentist and parent are seated face to face
A. Knee position C. leg position
B. cradling position D. Knee to knee position E. all

In pedodontic triangle the apex of the triangle is:


A. Child B. Dentist C. Parent D. Assistant

Parent who are mistrusting are:


A. Overprotective Parent C. Neglectful Parent
B. Manipulative Parent D. Hostile parent

Parent who are demanding and can extend to directing the course of the treatment
is a
:
A. Overprotective Parent C. Neglectful Parent
B. Manipulative Parent D. Hostile parent

A parent who postponed the dental treatment of his/her child is


A. Overprotective Parent C. Neglectful Parent
B. Manipulative Parent D. Hostile parent

Panoramic Radiographic Film:


A. 5 < x 7= B. 2=x 3= C. 5= x 12= D.. 8= x 10=

Downloaded by John Erickson Bangayan (bangayan1903064@ceu.edu.ph)


lOMoARcPSD|19937673

Radiograph examination for a three years old patient with no apparent abnormalities
and open contact:
A. 2 posterior bitewing C. 4 film series
B. 8 film series D. 12 film Series E. None

Radiographic examination needed for a 6-7 years old with no apparent


abnormalities:
A. 2 posterior bitewing C. 4 film series
B. 8 film series D. 12 film Series E. None

Frankl behavior rating characterized by refusal of treatment and extreme negativism:


A. rating 1 B. rating 2 C. rating 3 D. rating 4

A period that child undergoes the phase of separation anxiety:


A. Toddler B. Pre school C. school age D. Adolescent

Stage where one exhibit absolute confidence that they are right about everything,
and their judgement are infallible:
A. Toddler B. Pre school C. school age D. Adolescent

In Nitous oxygen sedation, once a petient takes a distant gaze the concentration of
the nitous oxide- oxygen is reduces to:
A. 30-70 B. 40-60 C. 20-80 D. 10-90 E. none

Device that assess the arterial hemoglobin oxygen saturation and pulse rate:
A. automated vital sign monitor C. pulse oximeter
B. pretracheal stethoscope D. none

Quadrant used in the gluteal region when intramuscular sedation is used:


A. upper inner quadrant C. upper outer quadrant
B. Lower inner quadrant D. Lower outer quadrant

The most unstable form of fluoride used for topical application:


Acidulated Phosphate Fluoride C. Stannous Fluoride
Sodium Fluoride D. None

Retention of PFS is through:


A. Light curing C. Drilling dentin
B. Acid etching D. Application of fluoride

A sudden firm command use to get the child’s attention:


A. Voice control C. Multisensory communication
B. Aversive conditioning D. HOME

Downloaded by John Erickson Bangayan (bangayan1903064@ceu.edu.ph)


lOMoARcPSD|19937673

Procedure which slowly develops behavior by reinforcing successive approximation


of the desired behavior until desired behavior develop:
A. Pre appointment behavior modification C. behavior shaping
B. Behavior management D. Retraining

Parents should be allowed inside a treatment room when:


A. infant to 41 months C. handicapped patient
B. 7-8 Years old D. A & C E. None

A technique of diverting the attention of the patient from what may be perceived as
an umpleasant procedure:
A. tell show do C. Voice control
B. Positive reinforcement D. Distraction E Non verbal commu-
nication

Plastics applied to the occlusal surface of posterior teeth by simple acid etch method
to prevent dental caries & early carious lesion from developing further
A. Composite C. Fluoride
B. Preventive Restorative Resin D. Glass Ionomer E. None

Medicament used in type A PRR:


A. Glass ionomer C. Bonding agent
B. Composite D. Pit and fissure sealant E. all of these

Medicament used for Type C PRR:


A. Glass ionomer C. Bonding agent
B. Composite D. Pit and fissure sealant E. all of these

Advantage of Rubber dam utilization:


A. Control saliva C. Aids management
B. Provides Protection D. all of these E. None

Distance of punched holes on the rubber dam for primary teeth:


A. 1.0mm C. 2.0mm
B. 1.5 mm D. 2.5 mm E. 3.0mm
Gingival cavosurface beveling is not indicated in Class II cavity preparation for
deciduous teeth, because the direction of the enamel rods are toward:
A. Cervical B. Occlusal C. gingival D. Apical E. all of these

Etching time for deciduous dentition is ----- compared to permanent dentition:


A. shorter B. longer C. the same D. none

The least amount of tooth surface reduction in stainless steel crown preparation is:

Downloaded by John Erickson Bangayan (bangayan1903064@ceu.edu.ph)


lOMoARcPSD|19937673

A. Occlusal B. lingual C. Proximal D. Buccal E. none

Ellis classification of tooth fracture when the coronal pulp is exposed:


A. Class I B. Class II C. Class III D. Class IV E. Class V

Ellis classification of tooth fracture when a tooth is lost as a result of trauma


A. Class I B. Class II C. Class III D. Class IV E. Class V

Ellis classification of a non vital traumatized tooth:


A. Class I B. Class II C. Class III D. Class IV E. Class V

Treatment of choice for an immature non vital permanent tooth to stimulate root
closure:
A. Indirect pulp capping C. Pulpotomy
B. Pulpectomy D. Apexogenesis E. Apexification

Use of formocresol:
A. to stop bleeding C. Stimulate secondary dentin
B. render area of fixation D. Dissolve the pulp tissue E. all of these

Materials that is used to induced apexification:


A. calcium hydroxide C. Formocresol
B. Zinc oxide eugenol D. IRM E. Gutta percha

Treatment of choice for a pulp exposed primary molar except:


A. Direct pulp capping C. pulpotomy
B. Pulpectomy D. all of these E. none of the above
Superficial wound due to scrapping of the mucosa:
A. laceration B. contusion C. abrasion D. Attrition E.
Erosion

Hypoplastic enamel is a result in the disturbance at:


A. initiation stage C. Appostion stage
B. Morphodifferentation D. Histodifferentiation E. None

Macrodontia incisor crown with two root canals:


A. Flexion B. Gemination C. Fusion D. Supernumerary root E.
none

Presence of enamel at the furcation of the root:


A. enameloma C. Enamelin
B. ameloblast D. Epstein Pearl E. all of the above

Radiograph picture of a caries is generally--- than the actual caries:

Downloaded by John Erickson Bangayan (bangayan1903064@ceu.edu.ph)


lOMoARcPSD|19937673

A. smaller B. larger C. the same D. deeper E. B&D

97. Rain coat:


A. cotton B. rubber dam C. tooth paint D. tooth camera E. pointer

98. Anesthesia:
A. rubber dam B. tooth paint C. Sleeping juice D. tooth camera E. pointer

99. Explorer:
A. Pointer B. toothbrush C. Tooth paint D. Injection E. none

100. Pit and Fissure sealant:


A. Pointer B. toothbrush C. Tooth paint D. Injection E. none

101. Excavator:
A. Pointer B. toothbrush C. Tooth paint D. Injection E. none

● ORTHODONTICS AND PEDIATRIC DENTISTRY


Shade AE if the answer is not found in the given
choices.

A supra erupted mand. canine with respect to Frankfurt Horizontal


plane is said to be in?
A. attraction B. Abstraction C. Protraction D.
Retraction E. Contraction.

A supra erupted max. canine with respect to Frankfurt Horizontal


Plane is said to be in?
A. attraction B. Abstraction C. Protraction D. Retraction
E. Contraction.

A linguoverted max. premolar with respect to Midsaggital plane is


said to be in?
A. attraction B. Abstraction C. Protraction D. Re
traction E. Contraction. -

A linguoverted max. incisor with respect to Orbital Plane is said


to be in?
A. attraction B. Abstraction C. Protraction D.
Retraction E. Contraction.

Neutroclusion with maxillary anteriors are lingual in relation to


the lower anteriors and there is mesial drifting of molars.

Downloaded by John Erickson Bangayan (bangayan1903064@ceu.edu.ph)


lOMoARcPSD|19937673

A. Class 1 type 3 and 5 B. Class 1 type 3 and 4 C.


Class 1 type 5 and 4.

The following are vertical planes, except:


Axis of 1 B. v-axis C. facial plane D. palatal

Bone ossification can be detected with the use of:


A, hand and wrist x-ray B. BMR C. electromyographic exam
D. biostatic

The least desirable method used in child management is:


A. tell-show do technique, C. gift before treatment,
B. hand-over-mouth technique. D. voice control

Spoiled children are with parents who are:


A, dominating, B. rejecting, C
overindulging,
A mechanical aid for keeping the mouth open and operates on a
reverse scissors action: A.finger guard C. padded
and wrapped tongue blade
B. Molt mouth prop D. rubber bite block
E. Mc Kesson bite block

Child’s arms and legs can be immobilized with:


A. forearm-body support B. plastic bowl C. Posey
straps D. head positioner

To avoid vomiting and complications during treatment with


sedation, no milk or solid food should be taken___ before the
scheduled procedure. A. 4 hrs. B. 6 hrs. C. 8 hrs. D.
after midnight

For sedation via intramuscular route, the ___ of the gluteal


region is the safest. A. upper, inner
B. upper outer C. lower inner D. lower outer
Stabilization period for teeth with fractured roots:
A. 2-3 months B. 2-3 days C. 2-3 weeks
D. 7-14 days

The correct angulation of the inclined plane in relation to the


tooth in crossbite is: A. . 15º B. 25º C. 35º D. 45º E. 55º

The following are sequelae of untreated crossbite, except:


A. faceting on the labial surface of the tooth in
crossbite
B. faceting of the labial surface of the tooth opposing
the inlocked tooth

Downloaded by John Erickson Bangayan (bangayan1903064@ceu.edu.ph)


lOMoARcPSD|19937673

C. abrasion
D. periodontal involvement.

The appliance to be used in correcting the anterior crossbite is


determined by the:
A. amount of overbite B. age of the patient C.
cooperation of the patient D. all of these

Inclined planes should not be left in the mouth for more than 2
months to prevent creation of:
A. anterior open bite B. posterior open bite C. anterior
cross bite D. posterior cross bite.

The best appliance for 7 years old child with Class 1 Type 3
(inlocked maxillary central incisor) is A. cross bite elastics
C. myofunctional appliance
B. band and crib D. mandibular acrylic inclined plane.

Appliance for correcting Class 1 deep bite in a growing child.


A. tongue depressor B. band and crib C. inclined
plane
D. maxillary bite planes.

The overbite reduction achieved by the bite plane is due largely


to:
A. intrusion of posterior teeth C. overeruption of pos-
terior teeth
B. mesial drifting of posterior teeth D. buccal and lingual
movement of posterior teeth.

In adult patient with excessive overbite the lower facial height


in relation to the upper and lower middle thirds is:
A. normal B. markedly short C. markedly long.

In deep bite the postural vertical dimension is/has:


A. in harmony with occlusal vertical dimension
B. not in harmony with occlusal vertical dimension
C. no relations with occlusal vertical dimension
Pulpotomy with formocresol of primary tooth:
A. result in lower success rate compared to calcium hydroxide,
B. result in higher success rate compared to calcium hydroxide,
C. should never be used,
D. induces formation of a dentin bridge at site of amputation,
The prognosis of a cervical third root fracture:

Downloaded by John Erickson Bangayan (bangayan1903064@ceu.edu.ph)


lOMoARcPSD|19937673

A is not favorable, B. is favorable, C. depends on whether


tooth is discolored

The most important predisposing factor to injury of the anterior


teeth is:
A. rotated anterior teeth, B. protruding anterior teeth
C. crossbite of anterior teeth,
The ultimate objective of pulp capping is:
A. to preserve the pulp and odontoblast so that secondary
dentin will be deposited
B. to prevent bacterial contamination, C. to
prevent pain,

The purpose of pulp treatment and root canal filling is to:


A. remove pulpal tissue,
B. prolong the usefulness of a tooth to function in
mastication, C. enlarge the root canal,

Stage of Nolla’s clacification wherein tooth begins eruptive


movement:
2 B. 5 C. 6 D. 7 E. 10.

Which of the facial radiographs is best used in assessing whether


the patient is gummy or not? A. frontal view with lips repose
B. lateral view C. smiling photograph.
Lip profile is influenced more by:
A. lip size B. lip tonicity C. lip shape D.
deposition of upper incisors

The profile of the patient with protruded mandible and retruded


maxilla is:
A. straight B. convex C. concave.

Which of the following cannot be assessed from cephalometric


radiographs?
A. mandibular retrusion C. direction of mandibular growth
B. incisor D. adequacy of dental arch pe-
inclination rimeter.

Treatment for crown fracture involving the pulp of a primary


incisor:
A. DPC B. formocresol pulpotomy C. pulpectomy D. extraction

The dental procedure that produces the greatest negative response


in children:

Downloaded by John Erickson Bangayan (bangayan1903064@ceu.edu.ph)


lOMoARcPSD|19937673

A. extraction B. injection C. cavity preparation


D. all of these

In inferior nerve block for a child patient, the injection must be


made__ than for an adult patient. A. slightly higher B. in
level C. slightly lower and more posteriorly.

The maximum number of cartridges of 2% lidocaine with 1:100,000


epinephrine for a 40 pound child patient is:
A. 3 B. 5 C. 7 D. 9 E. 10.

In inferior alveolar nerve blocked, the barrel of the syringe


should be directed on the plane:
A. between C & D B. between D & E C. between E &
6 D. on top of D.

Indicated restoration on a primary anterior tooth with small


mesial and distal caries and a cervical caries but without pulp
involvement: A. stainless steel crown, B. composite
resin C. strip-off crown,

In pulpectomy of primary teeth, filling of root canal should be:


A. 0.5 - 1 mm above the radiographic apex,
B. 2 - 3 mm above the radiographic apex, C. 4 - 5 mm above
the radiographic apex,

The type of fear that is based on the feelings and attitude that
have been suggested to the child by others without having had
the experience:
A. objective fear B. subjective fear C. BOTH

On the first dental visit, the basic fear of the children below 2
years old is concerned with.
A. injections C.. anxiety & being
separated from parents
B. instruments used in dental treatment D. not
understanding the reason for treatment.

Giving gifts to children.


A, should promised to the child for every appointment to ensure
cooperation.
B, should be considered as a token for
friendship C, both A&B D, Both B&C.

The arrangement of 2 forces of equal magnitude and opposite but


noncollinear lines of action:

Downloaded by John Erickson Bangayan (bangayan1903064@ceu.edu.ph)


lOMoARcPSD|19937673

A. center of rotation B. couple C. moment


D. center of resistance

When one side of the arch is intact and there are several primary
teeth missing on the other side, use:
A. a Nance lingual arch C. a distal shoe
B. a transpalatal arch D. a partial denture space maintainer
E.a soldered fixed lingual arch

Choice of nursing nipple to prevent oral habit formation :


A. Evenflo B. Mimiflo C. Nuk Sauger nipple D.
nonphysiologic nipple

Mesial surface of E can be disked in case the 3 cannot erupt


because of insufficient space. This is:
A. contingency of extraction B. occlusal equilibration C.
space regaining D. observation

In managing abnormal oral habits, therapy must be:


A. mechanical C. of physical control
B. of conditioning responses D. a means of punishment
Sudden and firm commands used to get the child’s attention:
A. HOME B. Tell, show and do C. voice control
D. Hypnodontics

Uses the principles of learning theory:


A. behavior modification B. behavior management C.
behavior shaping

Poor personal experiences in the dental office:


A. overprotective parents C. hostile parents
B. neglectful parents D. manipulative parents.

Reluctant to accept treatment:


A. definitely negative B. negative C.positive
D. definitely positive.

They have minimal apprehensions and are reasonably relaxed:


A. enthusiastic B. cooperative C. timid D. defiant.

In treating a pediatric patient, one must establish good


communication with the
A. child only, since he is the one to be treated
B. parents only, because they are the ones who will pay the bill
C. the caretaker or the yaya who takes care of the child
D. both parent and child

Downloaded by John Erickson Bangayan (bangayan1903064@ceu.edu.ph)


lOMoARcPSD|19937673

The use of nitrous oxide is classified as a


A. physical restraint B. pharmacologic approach C.
psychological approach
Guiding of behavior through eye contact, posture and facial
expression.
A. distraction C. positive reinforcement
B. non-verbal communication D. HOM exercise
E. TSD

Partial or complete immobilization of the patient to protect him


from injury while providing dental care.
A. conscious sedation C. positive reinforcement
B. HOM exercise D. physical restraint E. nitrous
oxide and oxygen inhalation

The following are factors that will compensate incisor liability,


except:
A. intercanine width growth C. labial posi-
tioning of permanent incisors
C. upright position of primary D. favorable tooth
incisors size ratio

Crowding of permanent posterior teeth may be due to:


A. mesial drifting of permanent first molar C. crowding of
terior teeth an-
B. labioversion of anterior D. spacing of
teeth rior teeth ante-

This will decrease lower arch perimeter during transitional


period:
A. late mesial shift of first permanent molar
B. distal tipping of lower cuspid C. labial position of
permanent incisors

As arch perimeter increase, arch length?


A. increases B. decreases C. no change.

This is measured from the distal of second primary molar to distal


of second primary molar on the other side following the contour
of the arch.
A. arch width B. intercanine width C. arch perimeter
D. arch length

Which of the developmental space will cause a decrease in arch


perimeter when pre-empted?
A. primate space B. interdental space C. leeway space D.
inter-occlusal space

Downloaded by John Erickson Bangayan (bangayan1903064@ceu.edu.ph)


lOMoARcPSD|19937673

Upper arch width increases significantly more than that of lower


arch due to?
A. diverging alveolar growth C. vertical alveolar growth B.
labial positioning of permanent incisors D. distal tipping of
cuspid

Point of injection for mandibular blocking in pediatric patient is


___ the occlusal plane.
A. above, B. below or at the level, C. higher than,
An incorrigible 4-year-old child who keeps on kicking and throwing
objects in the clinic can be managed by:
A. tell-show-do technique, C. physical
restraint
and conscious sedation
B. general anesthesia, D. all of these
If a child complains of pain on a direct pulp capped tooth, the
first treatment option should be:
A. indirect pulp capping, B. pulpotomy, C.
pulpectomy, D. extraction.

The following irrigating solution can be used as irrigant in


pulpectomy procedure except:
A. normal saline solution, B. sodium hypochlorite
and water solution,
B. distilled water, D. anesthetic solution,
The first dental appointment of a child patient should be:
A. before 1 year old, B. 2 1/2 to 3 years old, C. 6
years old

Least reduction in tooth preparation for posterior SSC:


A. buccal B. lingual C. occlusal ces. D. proximal
surfa-
Occlusal reduction for SSc :

A. 0.25-0.5mm B. 1-1.5mm C. 2-3mm D. 5mm.

Grinding of the sharp edges is sufficient.


A. lateral luxation B. dentin fracture C. enamel fracture
D. extrusion.

Restore with bonding agent and composite


A. extrusion C. dentin fracture
B. intrusion D. enamel fracture E. enamel-dentin
fracture.

In acute ingestion of fluoride, the following can be given to the


patient to counteract its effect, except:

Downloaded by John Erickson Bangayan (bangayan1903064@ceu.edu.ph)


lOMoARcPSD|19937673

A. 2 tsp of ipecac syrup B. milk C. milk of magnesia


D. alum

Gingival margin of a tooth that will receive a stainless steel


crown should have a ___finish line:
A. shoulder B. feather edge C.
BOTH
Chewable fluoride tablets:
A. systemic B. local C. BOTH

.Mandibular teeth are best anesthetized with:


A. infraorbital injection C. mental
injection
B. an inferior alveolar and long buccal injection C. both B
&
C

Pulpotomy with calcium hydroxide in the primary teeth:


A. may form calcific barrier on the pulp stump
B. result in acute inflammation of pulp
C. result in necrosis of pulp

Lack of anatomic detail of dental restoration produces:


A. increased arch length B. elongation of adjacent
teeth
B. elongation of opposing teeth D. A and B

Child is 11 years old. Tooth # 55 has been exfoliated but tooth #


65 is still very intact. What must be done?
A. none, wait for 65 to exfoliate C. do percussion test
before extracting 65
B. extract 65 at once D. retain 65 and place a band and
loop for 55

Two or more teeth moving in opposite directions and pitted against


each other by the appliance. Usually, the resistance to each
other is equal and opposite. The anchorage is:
A. simple B. stationary C. reciprocal D.
extraoral
The following are sources of extraoral anchorage except:
A. cervical B. occipital C.cranial D. facial E.
mucosa

The order from greatest to least change of the dimensions of the


cranium:
A. height, depth, width C. depth, width, height
B. depth, height , width D. width, height, depth

Downloaded by John Erickson Bangayan (bangayan1903064@ceu.edu.ph)


lOMoARcPSD|19937673

At age 4-5, what normal sign of primary dentition augers well for
the erupting permanent incisors in terms of space availability:
A. Class I cuspid relationship C. growth
spaces, interdental spaces
B. upright vertical incisor relationship D. flush
terminal plane

The basic form of the arch is determined:


A. by environmental factors B. by muscles B.
in intrauterine life D. two of the choices E. all of these

The ffg are true regarding the deciduous maxillary central


incisor, except:
A. MD diameter of crown greater than its cervicoincisal length
B. developmental lines are usually not evident
C. well-developed marginal ridges on the lingual
D. root shape is conical

To avoid ingestion of lethal dose of fluoride, it is safe to


dispense up to __ of fluoride.
A. 120mg B. 300 mg C. 500 mg. D. 700 mg.

Medium-size punch hole is used for:


A. primary molars C. permanent molars
B. maxillary permanent incisors D. lower permanent
incisors.
Effect of abnormal eruptive path:
A. space loss C. delayed eruption
E. deflection of eruption
B. increase in arch length D. elongation of adjacent
teeth

Proximal caries, if not restored, will: ... the arch length


A. increase B. decrease C. not affect

Lack of anatomic detail of dental restoration produces:


A. increased arch length C. elongation of
adjacent
teeth
B. elongation of opposing teeth D. A and B
Trident factor of oral habit which is now considered as the most
impt.
contributory factor to the dev’t of malocclusion:
A. duration B. frequency C. intensity

Downloaded by John Erickson Bangayan (bangayan1903064@ceu.edu.ph)


lOMoARcPSD|19937673

The ff. are true regarding thumb sucking EXCEPT:


A. it is a learned habit, not a symptom of a deeper emotional
disturbance
B. for the first 3 years, damage to occlusion is confined to
the anterior segment and is usually temporary
C. normal from 1-1 1/2 then disappears at age 2 with proper
attention to nursing
D. habit beyond age 4 can be controlled by badgering attempts
of parents to break the habit

Growth of maxilla follows the__ growth curve.


A. general / bodily B. lymphoid C. neural D. genital
At age 5-10, the mandible is__ completed.
A. 45% B. 40% C. 65% D.
96%
Lack of anatomic detail of dental restoration produces:
A. increased arch length C. elongation of
adjacent
teeth
B. elongation of opposing teeth D. A and B
At age 4-5, what normal sign of primary dentition augers well for
the erupting permanent incisors in terms of space availability:
A. Class 1 cuspid relationship C. growth spaces,
interdental spaces
B. upright vertical incisor relationship D. flush terminal
plane.
The basic form of the arch is determined:
A. by environmental factors B. by muscles C. in
intrauterine life D. two of the choices
This aims to recognize and eliminate potential irregularities and
malpositions in the developing craniofacial complex:
A. Preventive Orthodontics C. Interceptive Ortho-
dontics
B. Limited Corrective Orthodontics D. Extensive Corrective
Orthodontics

The action taken to preserve the integrity of what appears to be


normal occlusion at a specific time:
A. Preventive Orthodontics C. Interceptive Orthodontics
B. Limited Corrective Orthodontics D. Extensive
Corrective Orthodontics

● ORTHODONTICS AND PEDIATRIC DENTISTRY

Which of the developmental space will cause a decrease in arch


perimeter when pre-empted?

Downloaded by John Erickson Bangayan (bangayan1903064@ceu.edu.ph)


lOMoARcPSD|19937673

primate space B. interocclusal space C. interdental


space D. leeway space

This is an increase in size by expansion from within


sutural apposition C. interstitial growth
maturation D. development

This is growth movement which mean movement of whole bone as a


unit
A. drift B. displacement C. deposition D.
relocation E. remodeling

Mandible comes from ? branchial arch first


B. second C. third D. fourth

Maxilla is formed from the maxillary processes and ?


medial nasal process C. globular process
B. lateral processes D. tuberculum impar

In young child, paradoxical excitement occurs most frequently


following premedication with:
a narcotic B. a barbiturate C. nitrous oxide D. an
amphetamine

The maximum amount of anesthesia that can be given to a 36-pound


child is:
4 carpules B. 3 carpules C. 2 carpules D.
1 carpule

Dental Fluorosis is most likely to occur on a:


two-year-old child ingesting 1 ppm of fluoride
17-year-old patient ingesting 1 ppm of fluoride
on a 3-year-old child ingesting 0.50 ppm of
fluoride D. none of the above.

Pulpotomy with formocresol of primary tooth:


result in lower success rate compared to calcium
hydroxide, result in higher success rate compared to
calcium hydroxide, should never be used, induces
formation of a dentin bridge at site of amputation, none
of the above.

A 4-year-old child visited a dentist for the first time and


received prophylaxis
quite well. Before he left, the dentist was glad that
he gave the child a toy.

Downloaded by John Erickson Bangayan (bangayan1903064@ceu.edu.ph)


lOMoARcPSD|19937673

This kind of gesture is called:


classical conditioning C. positive reinforcement,
bribery, B. public relations E. none
of the above.

If a primary tooth has been extracted before succedaneous tooth


has begun eruptive movement, its eruption will be __
hastened B. delayed C. not affected

Which of the following conditions is usually present in a Class


II, Division 2 malocclussion? Open bite
Steep mandibular plane
Mesiocclusion of permanent first molars
Lingual inclination of maxillary central incisors

Excessive orthodontic force used to move a tooth may:


cause hyalinization cause
undermining resorption
crush the periodontal ligament
A. 1 & 2 B. 2 & 3 C. 1 & 3 D. 1, 2 & 3 E. 3 only

Severely crowded mandibular incisors usually result from:


mesial migration of teeth premature
loss of primary teeth presence of
supernumerary teeth D. tooth
size-arch length discrepancy
How will extraction of a primary maxillary central incisor in a
5-year old child with incisal spacingaffect the size of the
intercanine space?
The intercanine space will increase in size.
The intercanine space will decrease in size.
No change will occur in the size of the intercanine
space.
The major criterion to differentiate between a true Class III
and a pseudo-Class III malocclusion is: degree of anterior
crossbite presence of a bilateral crossbite
C. existence of a forward shift of the mandible during
closure D. occlusal relationship between maxillary and
mandibular first molars

Orthodontic correction of which of the following is most easily


retained?
Diastema C. Expansion
B. Rotation D. Posterior crossbite E.
Anterior crossbite

Downloaded by John Erickson Bangayan (bangayan1903064@ceu.edu.ph)


lOMoARcPSD|19937673

A bimaxillary protrusion occurs when:


large horizontal overlap with biteral crossbite exits the
maxillary dentition is forward with respect to basal bone
both dental arches are forward with respect to basal bone
D. none of the choices.

When a simple tipping force is applied to the crown of a


single-rooted tooth, the center of rotation is usually located:
at the apex D. at the cervical line
5mm beyond the apex E. one-third the root length from the
apex
two-third the root length from the
apex.
Which of the following is the most common orofacial malformation
that produces malocclusion?
Cleft palate C. Ectodermal dysplasia
Pierre Robin syndrome D. Osteogenesis imperfecta E.
Cleidocranial dysostosis

The cranial vault increases rapidly in size the first few years
postnatally and completes approximately 90% of its growth by 6
years of age. This growth is typical of which of the following
types of tissues?
Neural B. dental C. genital D. lymphoid E.
general (somatic)
The following are the conditions found on the area of pressure in
the presence of heavy forces:
area of hyalinization
occlusion of blood vessels
stretched periodontal fibers
undermining resoprtion
A. 1, 2& 3 B. 1, 2 & 4 C. 2 , 3 & 4 D. 3 only E.
1 & 4 only

Generalized osteoclastic activity along the walls of the alveolar


socket is the bone response to:
depressing force C. extrusion
B. elongating force D. rotating force E. both B &
D

A 4-year old child has a traumatized central incisor with a Class


III (Ellis) fracture. The injury occurred about one month ago,
and examination indicates that the pulp is necrotic. There are
no other pathologic findings. Treatment of choice is: watchful
observation
extraction and use of a space maintainer

Downloaded by John Erickson Bangayan (bangayan1903064@ceu.edu.ph)


lOMoARcPSD|19937673

pulpectomy and root canal filling using gutta-percha points


and cement
endodontic treatment and root canal filling with a resorbable
paste.

Supervision of a child’s development of occlusion is most critical


at ages:
3-6 years B. 7-10 years C. 11-14 years E. 14-
17 years

A child with congenital heart disease requires special treatment


planning for dental care because of potential problems with:
bleeding local
infection
systemic
infection enamel
hypoplasia
A. 1 , 2 & 3 B. 1 & 3 C. 1 only D. 3 only E.
1,2,3 & 4

In festooning and trimming a stainless steel crown, greater


length is necessary in the region of the mesiofacial bulge in a
primary: first molar B. second molar C. maxillary canine
D. maxillary lateral incisor

Using a topical fluoride rinse before acid etch direct bonding of


orthodontic brackets is contraindicated because fluoride:
decreases the solubility of enamel increases
the pH of the etching agent causes copious
amounts of saliva directly reacts chemically
with the bonding agent.

If a 7-year old patient loses a primary mandibular canine about


the same time the adjacent lateral incisor is erupting or
shortly thereafter, the dentist should be alert to the
possibility of:
a tongue habit a developing crossbite an
early eruption of the permanent canine
lingual collapse of mandibular anterior
teeth

Which of the ff. approaches is best for a child suffering from


celebral palsy
HOM exercise B. towel method C. use of sedation D.
psychological approaches

Downloaded by John Erickson Bangayan (bangayan1903064@ceu.edu.ph)


lOMoARcPSD|19937673

Type of fear which is produced by direct physical stimulation of


sense organ associative fear B. objective fear C.
subjective fear D.
acquired fear

Agent used to reduce pain without affecting consciousness


hypnotic drug B. analgesic C. barbiturate D. general
anesthesia

The drug that modifies fear and anxiety


A. flanax B. ethamizid C. celestamine D.
seconal

A general characteristic of a 12 year old child mother as the


center of his world C. exhibits tantrums separation
anxiety D. rejects parental authority

This is a psychological approach to manage a child


nitrous oxide inhalation C. giving
diazepam mutisensory techniques D. HOM

The major source of anchorage in a maxillary Hawley appliance is


the :
molar clasp C. acrylic portion
B. labial bow D. interproximal clasp E. bracket of
the band

A narrow maxillary arch with respect to midsagittal plane is said


to be in:
Protraction C. contraction
B. retraction D. distraction E. both C & D

Complete unilateral lip-jaw palate cleft.


A. Class I B. Class II C. Class III D.
Class IV

Anodontia, diagnosed in a 5-year-old child, primarily affects the


growth of the:
Midface C. mandible
B. maxilla D. alveolar bone E. maxilla & mandible

Which of the following are likely to occur during orthodontic


therapy? gingival irritation cementum resorption increased
mobility of teeth
demineralization of enamel adjacent to appliances in
patients with poor oral hygiene
1, 3 & 4 C. 2 & 4 only E. 3 & 4 only

Downloaded by John Erickson Bangayan (bangayan1903064@ceu.edu.ph)


lOMoARcPSD|19937673

B. 1 & 4 only D. 1, 2, 3 & 4

A distal shoe space maintainer is indicated when a primary:


incisor is avulsed first
molar is prematurely lost
second molar is lost after eruption of a permanent first
molar second molar is lost before eruption of a permanent
first molar

The normal downward and forward direction of facial growth


results from upward and backward growth of maxillary sutures
and the mandibular condyle
vertical eruption and mesial drift of the dentitions
interstitial growth in the maxilla and the mandible
epithelial induction at growth centers
A. 1 & 2 B. 1 & 3 C. 1 & 4 D. 2 & 3 E. 2 & 4
An 8-year-old girl has 2mm diastema between permanent maxillary
central incisors. Permanent maxillary lateral incisors are in
position. The diastema is probably the result of:
thumb-sucking an abnormal
frenum attachment a normal
developmental process
A. 1 & 2 B. 1 & 3 C. 2 & 3 D. 2 only E. 3 only
When comparing cementum to bone in their responses to orthodontic
forces, cementum resorbs:
more readily C. not at all
B. less readily D. under lighter loads
E. by
indirect (undermining) resoprtion

The tooth in the mandibular arch most likely to be malposed in


cases of arch space discrepancy is the:
first molar B.. first premalor C. second molar D. second
premolar

Slow progress in molar uprighting in an adult patient is usally


due to:
overextended bands D. an overcontoured spring
lack of anchorage control E. the occlusion not
being relieved
the stabilizing wire not being passive

A 9-year-old patient has a slightly convex profile and a


suspected tooth mass-arch length (circumference) discrepancy.
Before instituting space management procedures, the dentist
should:

Downloaded by John Erickson Bangayan (bangayan1903064@ceu.edu.ph)


lOMoARcPSD|19937673

complete a space analysis C. complete a


cephalometric analysis
seek orthodontic consultation D. all of the above E.
Either A or C above

A primary maxillary anterior tooth in a 4-year-old child was


traumatically intruded into the tissues so that only half the
tooth is visible. The most appropriate treatment is to:
extract the tooth.
perform a pulpotomy
administer no treatment
place orthodontic bands on adjacent teeth and draw the tooth
down with elastics

Which of the following orthodontic movements of teeth are most


difficult to accomplish?
Tipping
Rotation
Intrusion
Extrusion
Translation
A. 1 & 2 B. 1 & 3 C. 1 & 5 D. 3 & 4 E. 3 & 5

Interstitial growth is observed at which of the following sites?


Spheno-occipital syndrondrosis
Maxillary tuberosity
Mandibular condyle
Zygomaticomaxillary suture
Apex of an erupting premolar
A. 1 & 2 B. 1 & 3 C. 1 & 4 D. 2 & 4 E. 2 & 5

When force is applied, bone reflexes that produces deformation of


crystalline materials and allows flow of electric current
necessary for tooth movement:
deposition C. hyalinization
B. piezoelectricity D. undermining resoprtion
Cephalometrics is useful in assessing which of the following
relationships?
1. tooth-to-tooth
2. bone-to-bone
3. tooth-to-bone
A. 1 & 2 B. 1 & 3 C. 2 & 3 D. 1,2 & 3 E. 3 only

Downloaded by John Erickson Bangayan (bangayan1903064@ceu.edu.ph)


lOMoARcPSD|19937673

Which of the following is the most essential factor related to


correction of an anterior crossbite?
age of the patient C. depth of the corssbite
shape of the tooth involved D. space available
mesiodistally

Frankfort-horizontal is a reference plane constructed by joining


which of the following landmarks?
Porion and sella C. Porion and nasion
Porion and orbitale D. Nasion and
Sella
Which of the following is not a space maintainer?
Lingual arch D. Nance holding arch
Class III restoration E. Stainless steel crown
Palatal expansion appliance

Space closure is least likely to occur after loss of which of the


following teeth?
Primary mandibular canines D. Primary mandibular
second molars
Primary maxillary first molars E. Primary maxillary
central incisors
Permanent maxillary central incisors

Crowding that occurs with mandibular incisors after age 18 is most


often the result of:
orthodontic relapse C. eriodontal disease
impacted thid molars D. a physiologic maturational
change

The undesirable side-effect most commonly associated with use of a


finger spring to tip the crown of a tooth is:
Pain D. gingival irritation
tendency for the tooth to intrude E. severe mobility of the
tooth
tendency for the root apex to move in the direction opposite
from the crown.

After the age 6, the greatest increase in size of the mandible


occurs:
at the symphysis C. along the lower border
between the canines D. distal to the first
molars
A distal shoe space maintainer is indicated when a primary:
canine is in crossbite first
molar is prematurely lost

Downloaded by John Erickson Bangayan (bangayan1903064@ceu.edu.ph)


lOMoARcPSD|19937673

second molar is lost after eruption of a permanent first


molar second molar is lost prior to eruption of a permanent
first molar

The <V= principle of growth is best illustrated by the:


nasal spetum C. mandibular symphysis
B. mandibular ramus D. spheno-occipital
synchondrosis

Anterior crossbite in the primary dentition usually indicates a


developing:
Class I malocclusion C. Class II malocclusion
Class III malocclusion D. Class IV maloccusion E.
none of the above

A mixed dentition analysis determines:


intercanine width D. skeletal growth pattern
discrepancies in jaw size E. size of permanent
incisors space available versus space required

Gonion, menton and pogonion are cephalometric landmarks located on


the midline B. bony chin C. mandible D. skeletal
profile

A major disadvantage of treatment using cervical headgear is:


impaction of maxillary canines D. extrusion of maxil-
lary incisors
extrusion of maxillary molars E. potential deformity of the
neck
psychologic trauma due to
appearance
Asymmetrical anterior open bite with normal posterior occlusion
is characteristic of:
thumb-sucking C. abnormal swallowing habits
mouth breathing D. both A & B E. both B &
C

If the norm for the cephalometric angle SNA is 82°, a patient’s


reading of 90° for SNA most likely indicates:
maxillary protrusion C. the patient’s ethnic
ground back-
protrusive maxillary D. dysplasia of the anterior
incisors cranial base.

Ideally, a malooclusion should be treated between the ages of:


5 and 8 years D. 8 and 10 years

Downloaded by John Erickson Bangayan (bangayan1903064@ceu.edu.ph)


lOMoARcPSD|19937673

10 and 12 years E. 12 and 14 years


None of the above. The age at which a malocclusion is
treated should depend on the problem involved.
Radiographs of a 5-year old patient show permanent maxillary
first molars inclined mesially with resulting resoprtion of the
distal portions of the roots of primary second molars. The
condition described is:
ankylosis C. ectopic eruption
premature eruption D. internal resorption E.
intrafollicular resoprtion

Fractured maxillary anterior teeth generally occur most often in


children with which of the following dental conditions?
Class I malocclusion D. Class II, Division 1
malocclusion
Class II, Division 2 malocclusion sion E. Class III malocclu-
Marked incisor crowding

On the first dental visit, the basic fear of a child under the age
of 2 is related to:
fear of an injection
anxiety over being separated from a parent
anxiety regarding the instruments used to perform dental
treatment
not understanding the reason for dental treatment.

A child with Down’s syndrome is characterized as being:


affectionate C. fearful of quick
movements capable of learning dental procedures D. both A & B
E.
all of these

A 3 ½ year old child has an acute fever, diarrhea, oral vesicular


lesions and gingival tenderness. The most likely diagnosis is:
thrush D. drug allergy
aphthous ulcerations E. . acute herpetic stoma-
titis
C. necrotizing ulcerative gingivitis

Transillumination of soft tissues is useful in detecting which of


the following problems in a child?
Koplick’s spot C. Sialolithiasis
Aortic stenosis D. Sickle cell disease E.
abnormal frenum attachment

Downloaded by John Erickson Bangayan (bangayan1903064@ceu.edu.ph)


lOMoARcPSD|19937673

Which of the following is least likely to result from persistent


long-term thumb-sucking?
a deep overbite D. protrusion of maxillary
incisor constriction of maxillary arch E. rotation of
maxillary lateral incisor
lingual inclination of mandibular
incisors
The least desirable method used in child management is:
tell-show do technique, C. voice control,
gift before treatment, D. hand-over-mouth
technique.

A child who is reluctant to accept dental treatment ,exhibit


negative attitude but not so pronounced is classified under
Frankl ?
I B. II C. III D. IV

The amount of pulp tissue to be removed in formocresol pulpotomy


procedure half of the coronal pulp
coronal pulp tissue down to the cervical constriction of
each root canal
C. infected portion of coronal pulp tissue
D. total removal of pulp tissue including radicular pulp
Which of the following dental sequelae is likely in a child with
a history of generalized growth failure (failure to thrive) in
the first six months of life?
Retrusive maxilla C. Enamel hypoplasia
Retrusive mandible D. Small permanent teeth E.
Dentinogenesis imperfecta

The average age at which calcification of crowns of permanent


central incisors is completed is:
birth B. 2-3 years of age C. 4-5 years of age D. 6-7 years
of age

Amalgam is most often the restorative material of choice for


primary teeth. The most important modification in its use for
children is in:
cavity preparation D. use of a rubber dam
use of a zinc-free alloy E. condensation of the alloy
placement of matrix bands

The most frequent cause of fracture of a root tip during


extraction of a primary molar is:

Downloaded by John Erickson Bangayan (bangayan1903064@ceu.edu.ph)


lOMoARcPSD|19937673

ankylosis of the tooth improper


use of cowhorn forceps presence
of a supernumerary premolar
root resorption between the apex and the bifurcation
asymmetric root resorption in which only one root is
completely resorbed.

The most common cause of sinus tracts in gingival tissues of


children is:
pericementitis C. periapical cyst
acute periapical abscess D. chronic periapical
abscess
A light bluish, dome-shaped lesion on the inside lip of a 2-year
old child is most likely a:
Mucocele B. melanoma C. hematoma D. hemangioma E.
sucking callous

Primary second molars usually erupt during ages:


8-14 months C. 14-20 months
20-30 months D. 30-36 months E. 36-48
months
A child in long-term remission of acute leukemia has dental
problems characterized by unusual susceptibility to: dental
caries C. oral infection
periodontal bone loss D. development of jaw deformities E.
all of the above.

This is a contraindication of HOM


spoiled child C. very young patient
defiant child D. uncooperative behavior but capable of
understanding

Induce formation of apical closure of young permanent molar using


CAOH apexogenesis C.
apexification
B. apeximation D. apex formation

The growth movement of the mandible is complimented by the growth


of
the
maxilla, which is:
down and forward C. downward and backward
B. forward only, D. upward and backward

Direction of displacement of the mandible in an individual with


developing Class II malocclusion:

Downloaded by John Erickson Bangayan (bangayan1903064@ceu.edu.ph)


lOMoARcPSD|19937673

down and forward, C. down and backward,


B. forward only, D. forward and upward, E. upward and
backward.

Child refuses to accept treatment or open the mount minimally:


A. Frankl 1 B. Frankl 2 C. Frankl 3 D. Frankl 4

Definitely negative behavior.


A. Frankl 1 B. Frankl 2 C. Frankl 3 D. Frankl 4

Child has good rapport with the dentist.


A. Frankl 1 B. Frankl 2 C. Frankl 3
D. Frankl 4.

The treatment option contraindicated in patients who are not able


to breathe nasally.
no treatment C. nitrous oxide & oxygen
inhalation
B. conscious sedation D. genera anesthesia

The treatment option for patients who have sustained extensive


orofacial & or dental trauma no treatment C.
nitrous oxide & oxygen inhalation
B. conscious sedation D. general anesthesia

With a flush terminal plane, permanent first molars will:


initially be Class II
initially be Class III
immediately assume a normal relationship
erupt immediately into an end-to-end
relationship

Arch shape & symmetry are best evaluated from the:


study cast C. frontal photograph
B. panoramic x-ray D. lateral cephalogram E. intraoral
photograph

Bodily force that moves the central incisor mesially produces:


stretching of the periodontal fiber on the distal
side compression of the periodontal fiber in the
distal side osteoblastic activity on the mesial
side osteoclastic activity on the distal side

In infant oral care, clean mouth with gauze after feedings and at
bed time done during:
A. 0-6 mons B. 6-12 mons C. 12-24 mons. D. 24-

Downloaded by John Erickson Bangayan (bangayan1903064@ceu.edu.ph)


lOMoARcPSD|19937673

36 mons

Space differential between combine width of CDE and 345 is__


A. positive B . negative C. zero D. 2.2 mm E.
2.4 mm

● ORTHODONTICS AND PEDIATRIC DENTISTRY

1. Gonion, menton and pogonion are cepahlometric landmarks located on the :


A. midline B. mandible C. bony chin D. skeletal profile

2. Complete unilateral lip-jaw palate cleft.


A. Class I B. Class II C. Class III D. Class IV

3. The best alternative diagnostic aid in the absence of cephalometric head plate:
A. study cast C. facial photograph
B. radiograph D. both A & C E. none of these

4. A narrow maxillary arch with respect to midsagittal plane is said to be in:


A. protraction B. retraction C. contraction D. distraction

5. The downward and forward direction of facial growth results from:


A. upward and backward growth of the maxillary sutures and the mandibular condyle
B. vertical eruption and mesial drift of the dentitions
C. interstitial growth in the maxilla and the mandible
D. epithelial induction at the growth centers

6. The condyle of the mandible grows by:


A. membrane bone growth C. appositional bone growth
B. instertitial bone growth D. proliferation of cartilage E. none of these

7. Spinning bead retainer is the appliance of choice for:


A. night grinding C. thumb sucking
B. tongue thrusting D. mouth breathing E. none of these

8. Asymmetrical anterior open bite with normal posterior occlusion is characteristic of:
A. thumb-sucking C. abnormal swallowing habits
B. mouth breathing D. all of these E. none of these

9. An adjunctive diagnostic tool for treatment planning orthodontics that gives information on
possible growth pattern and can give an accurate bone age picture of the patient:
A. hand & wrist C. panoramic
B. cephalometrics D. tomogram E. periapical radiograph

Downloaded by John Erickson Bangayan (bangayan1903064@ceu.edu.ph)


lOMoARcPSD|19937673

10. Frankfort-horizontal is a reference plane constructed by joining which of the following


landmarks?
A. porion and sella C. porion and orbitale
B. porion and nasion D. nasion and sella E. none of these

11. The distal step in the primary dentition will result in:
A. end to end molar relationship C. Class I molar relationship
B. Class III molar relationship D. Class II molar relationship

12. The space created upon exfoliation of primary molars which is conducive to proper
relationship of the permanent 1st molars is:
A. negative freeway space C. negative leeway space
B. positive freeway space D. positive leeway space

13. This is a process of reshaping and resizing as a consequence of progressive continuous


relocation:
A. drift C. remodeling
B. displacement D. translation E. none of these.

14. A 5-year old boy who lives in an area where fluoride content of the drinking water averages
0.75ppm should be supplemented with how much fluoride per day?
A. 0 mg. B. 0.25 mg C. 0.50 mg. D. 1.00mg.

15. The following is the difference in size between the primary and their permanent successor in
the posterior segment.
A. posterior liability C. posterior size discrepancy
B. nance’s leeway space D. late mesial shift E. none of these.

16. Precursor of the mandible:


A. tuberculum impar C. condyle
B. meckel’s cartilage D. copula E. none of these

17. Deep mentolabial sulcus:


A. hypertonic upper lip C. hypertonic lower lip
B. hypotonic upper lip D. hypotonic lower lip

18. Active mentalis muscle:


A. hyperactive upper lip C. hyperactive lower lip
B. hypoactive lower lip D. hypoactive upper lip

19. A desirable zone of flame in soldering metals is:


A. reducing B. oxidizing C. combustion D. air blast zone

20. Which of the following factors will not increase maxillary width?
A. growth of the palate following <V= principle
B. deposition at the fronto-maxillary suture
C. deposition at the lateral walls

Downloaded by John Erickson Bangayan (bangayan1903064@ceu.edu.ph)


lOMoARcPSD|19937673

D. deposition at the median palatine sutre

21. The treatment option contraindicated in patients who are not able to breathe nasally.
A. no treatment C. nitrous oxide & oxygen inhalation
B. conscious sedation D. genera anesthesia

22. The major mechanism for growth of the cranial case is the:
A. expansion of the cartilage cells
B. interstitial growth of bone
C. apposition of new bone at the synchondroses D. apposition of new bone at the sutures.

23. . The main growth site of the mandible which is responsible for its increase ion height?
A. Condyle C. symphysis
B. gonial angle D. posterior border of the ramus E. inferior border of the body

24. The sequence of completion of facial growth by planes of space is?


A. depth, width, height C. height, depth, width
B. width, depth, height D. width, height, depth E. depth, height, width

25. The theory which states that the growth of the craniofacial bones is caused by the soft
tissues adjacent to them.
A. Functional matrix C. Sutural Dominance theory
B. Cartilaginous growth theory D. Limborgh’s theory

26. The <V= principle of growth is best illustrated by the?


A. nasal septum
B. posterior border of the mandibular ramus
C. mandibular symphysis
D. intersphenoidal synchondrosis

27. Chin cup is used to intercept a?


A. Class 1 type 3 C. Class II
B. Class III D. Developing Class III

28. A direct growth movement that is produced by deposition on one side of the orbital plate
with resorption on the opposite side is?
A. Displacement C. translation
B. cortical drift D. remodeling

29. The following are distinctive structural features related to cartilage of the cranial base,
except:
A. pressure- tolerant C. grows interstitially and appositionally
B. matrix is non-vascular D. grows appositionally

30. What is the chief factor in the formation of the alveolar process?
A. normal process of growth
B. eruption of teeth

Downloaded by John Erickson Bangayan (bangayan1903064@ceu.edu.ph)


lOMoARcPSD|19937673

C. lengthening of the condyle


D. overall growth of the maxilla and mandible

31. Which of the following is not a characteristic of a mature swallow?


A. active contractions of the muscles of the lips
B. tongue tip is placed against alveolar process behind the upper incisors
C. teeth are together during swallowing
D. both A & B E. both A & C

32. These are six soft spots present between the bones of the skull roof.
A. Sutures B. synchondroses C. fontanelles D. cartilages

33. Spaces between the primary anteriors:


A. interdental space
B. primate space D. both A & B
C. Nance leeway space E. both B & C

34. Primate space is found between the:


A. maxillary canine and first molar
B. mandibular primary canine and lateral incisor
C. maxillary canine and lateral incisors
D. none of the above

35. At what stage in Nolla’s classification does a permanent tooth start to erupt?
A. Stage 7 C. Stage 6
B. Stage 5 D. Stage 8 E. Stage 4

36. From the flush terminal plane relationship of molars in the primary dentition, the permanent
first molar relationship in the permanent dentition can become the following in the transition
period, except:
A. Class I C. Class III
B. Class II D. end to end / cusp to cusp E. none of the choices

37. Two or more tooth moving opposite directions and pitted against each other is equal and
opposite. The anchorage is.
A. simple B. stationary C. reciprocal D. none.

38. The following are caused by thumbsucking habit Except.


A. anterior open bite B. protrusion of upper incisor C. expansion of upper arch

39. In a condition characterized by partial or complete absence of clavicle.


A. torticollis B. cerebral palsy C. cleidocranial dysostosis

40. A condition characterized by foreshortening of sternocleidomastoid muscle. A.


torticollis B. cerebral palsy C. cleidocranial dysostosis

41. Type of force recommended for removable orthodontic appliance.

Downloaded by John Erickson Bangayan (bangayan1903064@ceu.edu.ph)


lOMoARcPSD|19937673

A. continuous B. functional C. dissipating D. intermittent

42. Type of force which can be seen in fixed appliance is.


A. continuous B. functional C. dissipating D. intermittent

43. Osteoclastic activity occurs on the.


A. tension side B. pressure side C. stress D. compression

44. Bone deposition occurs on the.


A. tension side B. pressure side C. stress D. compression

45. The tooth is push towards the socket and pulled away from occlusal plane.
A. elongating force B. rotation C. depressing D. heavy force

46. Neutroclusion with labioversion of max centrals & buccoversion of mand. 1st
premolar. A. Class 1 type 2 & 3 B. Class 1 type 2 & 4 C. Class 1 type 2 & 5

47. A supraverted mand. Premolar with respect to Frankfurt horizontal plane is said it be
in. A. contraction B. abstraction C. Attraction D. distraction

48. The resorption occurs from behind an immediate site of pressure, there is
accumulation of fluid and blood vessels are occluded.
A. direct resorption C. undetermining resorption
B. frontal osteoclastic attack D. all of the Above.

49. It refers to perfect arrangement of teeth when jaw’s are closed & condyies are at rest in the
glenoid fossa.
A. normal occlusion B. ideal occlusion C. malocclusion

50. The incidence of malocclusion is very high in.


A. homogeneous population B. Heterogeneous population
C. Eskimos D. none of the above

51. The predominant type of malocclusion seen in mixed dentition is. A. Crowding
B. anterior open bite C. Class II Div. 1

52. A brachycephalic individual is associated with___ arch form:


A. square B. ovoid C. long and narrow D. none of the above.

53. Method of early examination where the dentist and parent are seated face to face
A. Knee position C. leg position
B. cradling position D. Knee to knee position E. all

54. In pedodontic triangle the apex of the triangle is:


A. Child B. Dentist C. Parent D. Assistant

55. Parent who are mistrusting are:


A. Overprotective Parent C. Neglectful Parent

Downloaded by John Erickson Bangayan (bangayan1903064@ceu.edu.ph)


lOMoARcPSD|19937673

B. Manipulative Parent D. Hostile parent

56. Parent who are demanding and can extend to directing the course of the treatment is a :
A. Overprotective Parent C. Neglectful Parent
B. Manipulative Parent D. Hostile parent

57. A parent who postponed the dental treatment of his/her child is


A. Overprotective Parent C. Neglectful Parent
B. Manipulative Parent D. Hostile parent

58. Panoramic Radiographic Film:


A. 5 < x 7= B. 2=x 3= C. 5= x 12= D.. 8= x 10=

59. Radiograph examination for a three years old patient with no apparent abnormalities and
open contact:
A. 2 posterior bitewing C. 4 film series
B. 8 film series D. 12 film Series E. None

60. Radiographic examination needed for a 6-7 years old with no apparent abnormalities:
A. 2 posterior bitewing C. 4 film series
B. 8 film series D. 12 film Series E. None

61. Frankl behavior rating characterized by refusal of treatment and extreme negativism:
A. rating 1 B. rating 2 C. rating 3 D. rating 4

62. A period that child undergoes the phase of separation anxiety:


A. Toddler B. Pre school C. school age D. Adolescent

63. Stage where one exhibit absolute confidence that they are right about everything, and their
judgement are infallible:
A. Toddler B. Pre school C. school age D. Adolescent

64. The most unstable form of fluoride used for topical application:
Acidulated Phosphate Fluoride C. Stannous Fluoride
Sodium Fluoride D. None

65. Retention of PFS is through:


A. Light curing C. Drilling dentin
B. Acid etching D. Application of fluoride

66. A sudden firm command use to get the child’s attention:


A. Voice control C. Multisensory communication
B. Aversive conditioning D. HOME

67. Procedure which slowly develops behavior by reinforcing successive approximation of the
desired behavior until desired behavior develop:
A. Pre appointment behavior modification C. behavior shaping

Downloaded by John Erickson Bangayan (bangayan1903064@ceu.edu.ph)


lOMoARcPSD|19937673

B. Behavior management D. Retraining

68. Parents should be allowed inside a treatment room when:


A. infant to 41 months C. handicapped patient
B. 7-8 Years old D. A & C E. None

69. Medicament used in type A PRR:


A. Glass ionomer C. Bonding agent
B. Composite D. Pit and fissure sealant E. all of these

70. Medicament used for Type C PRR:


A. Glass ionomer C. Bonding agent
B. Composite D. Pit and fissure sealant E. all of these

71. Advantage of Rubber dam utilization:


A. Control saliva C. Aids management
B. Provides Protection D. all of these E. None

72. Distance of punched holes on the rubber dam for primary teeth:
A. 1.0mm C. 2.0mm
B. 1.5 mm D. 2.5 mm E. 3.0mm

73. Gingival cavosurface beveling is not indicated in Class II cavity preparation for deciduous
teeth, because the direction of the enamel rods are toward:
A. Cervical B. Occlusal C. gingival D. Apical E. all of these

74. Ellis classification of a non vital traumatized tooth:


A. Class I B. Class II C. Class III D. Class IV E. Class V

75. Treatment of choice for an immature non vital permanent tooth to stimulate root closure:
A. Indirect pulp capping C. Pulpotomy
B. Pulpectomy D. Apexogenesis E. Apexification

76. Use of formocresol:


A. to stop bleeding C. Stimulate secondary dentin
B. render area of fixation D. Dissolve the pulp tissue E. all of these

77. Materials that is used to induced apexification:


A. calcium hydroxide C. Formocresol
B. Zinc oxide eugenol D. IRM E. Gutta percha

78. Treatment of choice for a pulp exposed primary molar except:


A. Direct pulp capping C. pulpotomy
B. Pulpectomy D. all of these E. none of the above

79. The average SNB angle for Filipino children with Class 1 occlusion is: A. 82.0º B. . 82.5º
C. 84.0º D. 84.5º E. 103º

Downloaded by John Erickson Bangayan (bangayan1903064@ceu.edu.ph)


lOMoARcPSD|19937673

80. The ANB angle of a patient with severe Class 2 skeletal malocclusion would most likely be:
A.
–4 B. 0 C. 2 D. 8.

81. If the mandibular and Frankfurt horizontal plane converge and meet at a point close behind
the face, the direction of growth potential is :
A. balanced vertically and horizontally B. more vertical
C. more horizontal D. none.

82, Protrusiveness or retrusiveness of the chin point can be known by analyzing the:
A. SNB B. FMLA C. IMPA D. FMA E. FH/NP.

83. Physiologic tooth movements include the following, except:


A. eruption B. functional movement C. gaining of space with an appliance
D. mesial drifting in the absence of adjacent teeth E. none.

84. Force level decline abruptly to zero.


A. coil spring B. fix braces C. removable appliance D. arch wire.

85. Unilateral distocclusion with linguoversion of the maxillary central incisors.


A. Class 2 Div 1 B. Class 2 Div. 2 subd. C. Class 2 Div. 1 subd.

86. MB cusp of maxillar 6 occludes with the MB groove of mand 6 with crowding of the anterior
teeth,
:
A. Class II type 1 B. Class I type 1 C. Class I type 3

87. MB cusp of max. 6 occludes with the MB groove of mand 6 with buccoversion of 2nd
premolar: A. Class II type 3 B. Class I type 3 C. Class I type 5.

88. Requires retentive groove in tooth preparation:


A. anterior SSC B. SOC C. polycarbonate crown

89. Retention is achieved through acid etching the tooth preparation:


A. anterior SSC B. SOC C. polycarbonate crown

90. Deciduous incisor is subjected to pulpectomy, final restoration is:


A. anterior SSC B. SOC C. polycarbonate crown.

91. A supra erupted mand. canine with respect to Frankfurt Horizontal plane is said to be in?
A. attraction B. Abstraction C. Protraction
D. Retraction E. Contraction.

92. A supra erupted max. canine with respect to Frankfurt Horizontal Plane is said to be in?
A. attraction B. Abstraction C. Protraction
D. Retraction E. Contraction.

Downloaded by John Erickson Bangayan (bangayan1903064@ceu.edu.ph)


lOMoARcPSD|19937673

93. When one side of the arch is intact and there are several primary teeth missing on the other
side, use:
A. a Nance lingual arch C. a distal shoe
B. a transpalatal arch E D. a partial denture space maintainer
.a soldered fixed lingual arch

94. Child has good rapport with the dentist.


A. Frankl 1 B. Frankl 2 C. Frankl 3 D. Frankl 4.

95. The order from greatest to least change of the dimensions of the cranium:
A. height, depth, width C. depth, width, height
B. depth, height , width D. width, height, depth

96. At age 4-5, what normal sign of primary dentition augers well for the erupting permanent
incisors in terms of space availability:
A. Class I cuspid relationship C. growth spaces, interdental spaces
B. upright vertical incisor relationship D. flush terminal plane

97. The basic form of the arch is determined:


A. by environmental factors B. by muscles
B. in intrauterine life D. two of the choices E. all of these

98. The drug that modifies fear and anxiety


A. flanax B.ethamizid C. celestamine D. seconal

99. This is a psychological approach to manage a child


A. nitrous oxide inhalation C. giving diazepam
B. mutisensory techniques D. HOM

100. This is a contraindication of HOM


A. spoiled child C. very young patient
B. defiant child D. uncooperative behavior but capable of understanding

ORTHODONTICS AND PEDIATRIC DENTISTRY

1. Which of the following is the most common orofacial malformation that produces
malocclusion?
A. Cleft palate C. Ectodermal dysplasia
B. Pierre Robin syndrome D. Osteogenesis imperfecta
E.Cleidocranial dysostosis

Downloaded by John Erickson Bangayan (bangayan1903064@ceu.edu.ph)


lOMoARcPSD|19937673

2. The cranial vault increases rapidly in size the first few years postnatally and completes
approximately 90% of its growth by 6 years of age. This growth is typical of which of the
following types of tissues?
A.Neural B. dental C. genital D. lymphoid E. general (somatic)

3. The following are the conditions found on the area of pressure in the presence of heavy
forces: area of hyalinization occlusion of blood vessels stretched periodontal fibers
undermining resoprtion
A. 1, 2& 3 B. 1, 2 & 4 C. 2 , 3 & 4 D. 3 only E. 1 & 4 only

4. Generalized osteoclastic activity along the walls of the alveolar socket is the bone response
to:
A. depressing force C. extrusion
B. elongating force D. rotating force E. both B & D

5. A 4-year old child has a traumatized central incisor with a Class III (Ellis) fracture. The injury
occurred about one month ago, and examination indicates that the pulp is necrotic. There are
no other pathologic findings. Treatment of choice is:
A. watchful observation
B. extraction and use of a space maintainer
C. pulpectomy and root canal filling using gutta-percha points and cement D. endodontic
treatment and root canal filling with a resorbable paste.

6. Supervision of a child’s development of occlusion is most critical at ages:


A. 3-6 years B. 7-10 years C. 11-14 years E. 14-17 years

7. A child with congenital heart disease requires special treatment planning for dental care
because of potential problems with:
1 .bleeding
2. local infection
3. systemic infection
4. enamel hypoplasia
A. 1 , 2 & 3 B. 1 & 3 C. 1 only D. 3 only E. 1,2,3 & 4

8. In festooning and trimming a stainless steel crown, greater length is necessary in the region
of the mesiofacial bulge in a primary:
A. first molar B. second molar C. maxillary canine D. maxillary lateral incisor

9. Using a topical fluoride rinse before acid etch direct bonding of orthodontic brackets is
contraindicated because fluoride:
A. decreases the solubility of enamel
B. increases the pH of the etching agent
C .causes copious amounts of saliva D. directly
reacts chemically with the bonding agent.

10. If a 7-year old patient loses a primary mandibular canine about the same time the adjacent
lateral incisor is erupting or shortly thereafter, the dentist should be alert to the possibility of:

Downloaded by John Erickson Bangayan (bangayan1903064@ceu.edu.ph)


lOMoARcPSD|19937673

A. a tongue habit
B. a developing crossbite
C. an early eruption of the permanent canine
D. lingual collapse of mandibular anterior teeth

11. Which of the ff. approaches is best for a child suffering from celebral palsy
A. HOM exercise B. towel method C. use of sedation D. psychological approaches

12. Type of fear which is produced by direct physical stimulation of sense organ
A. associative fear B. objective fear C. subjective fear D. acquired fear

13. Agent used to reduce pain without affecting consciousness


A. hypnotic drug B. analgesic C. barbiturate D. general anesthesia

14. The drug that modifies fear and anxiety


A. flanax B. ethamizid C. celestamine D. seconal

15. A general characteristic of a 12 year old child


A. mother as the center of his world C. exhibits tantrums
B. separation anxiety D. rejects parental authority

16. This is a psychological approach to manage a child


A. nitrous oxide inhalation C. giving diazepam
B. mutisensory techniques D. HOM

17. The major source of anchorage in a maxillary Hawley appliance is the :


A. molar clasp C. acrylic portion
B. labial bow D. interproximal clasp E. bracket of the band

18. A narrow maxillary arch with respect to midsagittal plane is said to be in:
A. Protraction C. contraction
B. retraction D. distraction E. both C & D

19. Complete unilateral lip-jaw palate cleft.


A. Class I B. Class II C. Class III D. Class IV

20. Anodontia, diagnosed in a 5-year-old child, primarily affects the growth of the:
A. Midface C. mandible
B. maxilla D. alveolar bone E. maxilla & mandible

21. Which of the following are likely to occur during orthodontic therapy?
1. Gingival irritation
2. cementum resorption
3. increased mobility of teeth
4. demineralization of enamel adjacent to appliances in patients with poor oral hygiene
A. 1, 3 & 4 C. 2 & 4 only E. 3 & 4 only
B. 1 & 4 only D. 1, 2, 3 & 4

Downloaded by John Erickson Bangayan (bangayan1903064@ceu.edu.ph)


lOMoARcPSD|19937673

22. A distal shoe space maintainer is indicated when a primary:


A. incisor is avulsed
B. first molar is prematurely lost
C. second molar is lost after eruption of a permanent first molar
D. second molar is lost before eruption of a permanent first molar

23. The normal downward and forward direction of facial growth results from
1. upward and backward growth of maxillary sutures and the mandibular condyle
2. vertical eruption and mesial drift of the dentitions
3. interstitial growth in the maxilla and the mandible
4. epithelial induction at growth centers
A. 1 & 2 B. 1 & 3 C. 1 & 4 D. 2 & 3 E. 2 & 4

24. An 8-year-old girl has 2mm diastema between permanent maxillary central incisors.
Permanent maxillary lateral incisors are in position. The diastema is probably the result of:
1. thumb-sucking
2. an abnormal frenum attachment
3. a normal developmental process
A. 1 & 2 B. 1 & 3 C. 2 & 3 D. 2 only E. 3 only

25. When comparing cementum to bone in their responses to orthodontic forces, cementum
resorbs:
A. more readily C. not at all
B. less readily D. under lighter loads E. by indirect (undermining) resoprtion

26. The tooth in the mandibular arch most likely to be malposed in cases of arch space
discrepancy is the:
A. first molar C.. first premalor
B. second molar D. second premolar

27. Slow progress in molar uprighting in an adult patient is usally due to:
A. overextended bands D. an overcontoured spring
B. lack of anchorage control E. the occlusion not being relieved
C. the stabilizing wire not being passive

28. A 9-year-old patient has a slightly convex profile and a suspected tooth mass-arch length
(circumference) discrepancy. Before instituting space management procedures, the dentist
should:
A. complete a space analysis C. complete a cephalometric analysis
B. seek orthodontic consultation D. all of the above E. Either A or C above

29. A primary maxillary anterior tooth in a 4-year-old child was traumatically intruded into the
tissues so that only half the tooth is visible. The most appropriate treatment is to:
A. extract the tooth.
B. perform a pulpotomy
C. administer no treatment

Downloaded by John Erickson Bangayan (bangayan1903064@ceu.edu.ph)


lOMoARcPSD|19937673

D. place orthodontic bands on adjacent teeth and draw the tooth down with elastics

30. Which of the following orthodontic movements of teeth are most difficult to accomplish?
1. Tipping
2. Rotation
3. Intrusion
4. Extrusion
5. Translation
A. 1 & 2 B. 1 & 3 C. 1 & 5 D. 3 & 4 E. 3 & 5

31. Interstitial growth is observed at which of the following sites?


1. Spheno-occipital syndrondrosis
2. Maxillary tuberosity
3. Mandibular condyle
4. Zygomaticomaxillary suture
5. Apex of an erupting premolar
A. 1 & 2 B. 1 & 3 C. 1 & 4 D. 2 & 4 E. 2 & 5

32. Which of the developmental space will cause a decrease in arch perimeter when
pre-empted?
A. primate space B. interocclusal space
C. interdental space D. leeway space

33. This is an increase in size by expansion from within


A.sutural apposition C. interstitial growth
B. maturation D. development

34.This is growth movement which mean movement of whole bone as a unit


A. drift B. displacement
C. deposition D. relocation E. remodeling

35. Mandible comes from ? branchial arch


A. first B. second C. third D.
fourth
36. Maxilla is formed from the maxillary processes and ?
A. medial nasal process C. globular process
B. lateral processes D. tuberculum impar

37. In young child, paradoxical excitement occurs most frequently following


premedication with: A. a narcotic B. a barbiturate C. nitrous oxide D. an
amphetamine

38. The maximum amount of anesthesia that can be given to a 36-pound


child is: A. 4 carpules B. 3 carpules C. 2 carpules D. 1 carpule

39. Dental Fluorosis is most likely to occur on a: A. two-year-old child


ingesting 1 ppm of fluoride

Downloaded by John Erickson Bangayan (bangayan1903064@ceu.edu.ph)


lOMoARcPSD|19937673

B.17-year-old patient ingesting 1 ppm of fluoride


C. on a 3-year-old child ingesting 0.50 ppm of
fluoride
D. none of the above.

40. Pulpotomy with formocresol of primary tooth:


A. result in lower success rate compared to calcium hydroxide,
B. result in higher success rate compared to calcium hydroxide,
C. should never be used,
D. induces formation of a dentin bridge at site of amputation,
E. none of the above.

41. If a primary tooth has been extracted before succedaneous tooth has begun eruptive
movement, its eruption will be __
A. hastened B. delayed C. not affected

42. Which of the following conditions is usually present in a Class II, Division 2 malocclussion?
Open bite
Steep mandibular plane
Mesiocclusion of permanent first molars
D. Lingual inclination of maxillary central incisors

43. Excessive orthodontic force used to move a tooth


may: cause hyalinization cause
undermining resorption crush the
periodontal ligament
A. 1 & 2 B. 2 & 3 C. 1 & 3 D. 1, 2 & 3 E. 3 only

44. Severely crowded mandibular incisors usually result from:


A. mesial migration of teeth
B. premature loss of primary teeth
C. presence of supernumerary teeth
D. tooth size-arch length discrepancy

45. How will extraction of a primary maxillary central incisor in a 5-year old child with incisal
spacing affect the size of the intercanine space?
A. The intercanine space will increase in size.
B. The intercanine space will decrease in size. C. No change will occur in the
size of the intercanine space.

The major criterion to differentiate between a true Class III and a pseudo-Class III malocclusion
is: A. degree of anterior crossbite
B. presence of a bilateral crossbite
C. existence of a forward shift of the mandible during closure
D. occlusal relationship between maxillary and mandibular first molars

47. Orthodontic correction of which of the following is most easily retained?

Downloaded by John Erickson Bangayan (bangayan1903064@ceu.edu.ph)


lOMoARcPSD|19937673

A. Diastema C. Expansion
B. Rotation D. Posterior crossbite E. Anterior crossbite

48. A bimaxillary protrusion occurs when:


A. large horizontal overlap with biteral crossbite exits
B. the maxillary dentition is forward with respect to basal bone
C. both dental arches are forward with respect to basal bone
D. none of the choices.

49. When a simple tipping force is applied to the crown of a single-rooted tooth, the center of
rotation is usually located:
A. at the apex D. at the cervical line
B. 5mm beyond the apex E. one-third the root length from the apex
C two-third the root length from the apex.

50. Bite plane therapy for deep bite, active Hawley’s plate are examples of:
A. Preventive Orthodontics C. Limited Corrective Orthodontics
B. Interceptive Orthodontics D. Extensive Corrective Orthodontics

51. The normal sequential events from fertilization to death:


A. growth C. differentiation E. translocation B.
development D. none of the choices

52.. ______will give rise to tissues that will become the mandible
A. Reichert’s cartilage C. Meckel’s cartilage
B. 3rd branchial arch D. 1st branchial arch E. none of the choices

53. Emphasized how cartilage of nasal septum during growth paced the growth of maxilla
A. Genetic Theory C. Moss’ Hypothesis
B. Sicher’s Hypothesis D. Scott’s Hypothesis E. Petrovic’s Hypothesis

54. Skeletal tissues grow in response to soft tissue growth


A. Genetic Theory C. Moss’ Hypothesis
B. Sicher’s Hypothesis D. Scott’s Hypothesis E. Petrovic’s
Hypothesis

55. Cranial vault grows primarily by cartilage growth at:


A. synchondrosis B. sutures C. fontanelles D. disappears at birth

56. Cranial base grows primarily by__ growth at the synchondrosis:


A. intramembranous C. endochondral
B. combination of intramembranous and endochondral

57. Over 90% of brain growth is achieved by age: A. 5 B. 6 C. 7 D. 8 E. 15


58. Neurocranium follows what Scammon’s curve?

A. lymphoid B. neural C. general D. bodily E. genital type

Downloaded by John Erickson Bangayan (bangayan1903064@ceu.edu.ph)


lOMoARcPSD|19937673

59. Growth of brain case is primarily by proliferation and ossification of:


A. synchondrosis B. sutural connective tissue C.
fontanelles D. none of the choices

60. Nasomaxillary complex is hafted to the cranium by the following sutures except:
A. frontomaxillary C. zygomaticotemporal E. zygomaticomaxillary
B. pterygopalatine D. frontotemporal

61. As the maxilla decends, there is __ on the orbital floor, __ on the nasal floor and __ on the
inferior palatal surface:
A. ++- B. -+- C. +-+ D. +++ E. --+

62. The increase in length of the maxilla is due to:


A. apposition on the posterior border of the maxillary tuberosity
B. progressive movement of the entire zygomatic process in posterior direction D. A&B
C. growth in the median suture E. none of the choices

63. Mechanism for maxillary growth:


A. sutural connective tissue proliferation C. surface apposition / resorption
B. ossification D. translation E. all of the
choices

64. The mechanism for bone growth of the nasomaxillary complex is: A.
intramembranous B. endochondral C. combination

65. The sutures of the nasomaxillary complex are oblique and more or less parallel with
each other thus, growth in these areas would serve to displace the maxilla: A.
downward C. forward and upward B. downward and forward
D. upward and backward

66. Continued apposition of alveolar bone on the free borders of the alveolar process as the
teeth erupt increases the __ of the maxilla.
A. height B. width C. length

67. Increase in the width of the maxilla involves palatal growth following the expanding V
principle. The buccal segments move:
A. downward and forward B. posterior and superior
C. upward and posterior D. upward

68. The maxilla Is displaced downward and forward by growth in __ parts of the bone:
A. anterior and inferior C. posterior and superior
B. anterior and superior D. posterior and anterior

69. What characteristic growth pattern is shown in the vertical section through the coronoid
process?
A. (+) on the lingual, (-) on the buccal

Downloaded by John Erickson Bangayan (bangayan1903064@ceu.edu.ph)


lOMoARcPSD|19937673

B.. (-) on the lingual, (+) on the buccal surface


C.(+)
on the lingual, (-) on half of the buccal surface
D. (-) on the lingual, (-) on the buccal surface

70. Normal terminal plane relationship:


A. mesial step B. distal step C. flush/straight

71. The most common sequence of eruption for the upper arch:
A. 6124537 B. 6123457 C. 6142537 D. 6125437

72. The stage at which a tooth begins its eruptive movement:


A. stage 0 B. stage 2 C. stage 3 D. stage 6 E. stage 10

73.Periapical lesions, pulpitis and pulpotomy of a primary molar will___ the eruption of
successor premolar.
A. hasten B. delay C. have no effect on

74. If a primary tooth is extracted prior to the onset of permanent eruptive movement (prior to
root formation), the permanent tooth is likely to be:
A. delayed B. accelerated C. no effect on eruption

75. Which one is an incorrect description of Nolla’s stages of calcification?


A. Stage 4, 2/3 of crown completed C. Stage 7, 1/3 of root completed
B. Stage 6, crown almost completed D. Stage 9, root almost complete, open apex

76. Space differential in the anterior segment ( BA/AB ) - 21/12 gives:


A. excess space B. no space differential C. negative space

77. The proper crown angulation of a permanent incisor is one where the gingival portion of the
long axis of the crown should be __ to the incisal portion.
A. mesial B. distal C. incisal D. cervical

78. Rotated teeth occupy less space within the arch: A. true B. false

79. Plane of occlusion for permanent teeth: A. flat B. exaggerated

80. Posterior crown inclination from canine to molar:


A. lingual crown inclination B. buccal crown inclination C. upright

81. Size of teeth correlates well with stature. Size of teeth is sex-linked.
A. 1st statement is true, 2nd is false C. both are true
B. 1st statement is false, 2nd is true D. both are false

82. The most prevalent malocclusion in the primary dentition:


A. Class II Div. 1 C. Class I type 5 E. Class III
B. anterior openbite D. Class II types 1

Downloaded by John Erickson Bangayan (bangayan1903064@ceu.edu.ph)


lOMoARcPSD|19937673

83. A dolichofacial face would likely to have ___ arch form.


A. round C. average
B. broad and square D. long and tapering

84. A face that is broad and short is:


A. dolichofacial B. brachyfacial C. mesofacial D. normofacial

85. The most common malocclusion in the mixed dentition period is:
A. Class II Div. 1 B. anterior open bite
C. posterior open bite D. crowding E. Class I type 2

86. A type of conditioning where in a specific response is immediately rewarded


Classical conditioning C. operator conditioning
Operant conditioning D. conditioning E. classic conditioning

87. The patients history of childhood diseases are under


A. familial history B. social history C. medical history D. dental history

88. The history on the behavior of the child toward dentistry:


A. familial history B. medical history
C. Dental history D. history of present illness

89. Method of early examination where the dentist and parent are seated face to face
A. Knee position B. leg position
C. cradling position D. Knee to knee position E. all

90. In pedodontic triangle the apex of the triangle is:


A. Child B. Dentist C. Parent D. Assistant

91. Parent who are mistrusting are:


Overprotective Parent C. Neglectful Parent
Manipulative Parent D. Hostile parent

92. Parent who are demanding and can extend to directing the course of the treatment is a :
A. Overprotective Parent B. Neglectful Parent C.
Manipulative Parent D. Hostile parent

93. Etching time for deciduous dentition is ----- compared to permanent dentition:
A. shorter B. the same C. longer D. none

94. Panoramic Radiographic Film:


A. 5 < x 7= B. 5= x 12= C. 2=x 3= D. 8= x 10=

95. Frankl behavior rating characterized by refusal of treatment and extreme negativism: A.
rating 1 B. rating 2 C. rating 3 D. rating 4

Downloaded by John Erickson Bangayan (bangayan1903064@ceu.edu.ph)


lOMoARcPSD|19937673

96. A period that child undergoes the phase of separation anxiety:


A. Toddler B. school age C. Pre school D. Adolescent

97. In Nitous oxygen sedation, once a petient takes a distant gaze the concentration of the
nitous oxide- oxygen is reduces to:
A. 30-70 B. 20-80 C. 40-60 D. 10-90 E. none

98. Device that assess the arterial hemoglobin oxygen saturation and pulse rate:
A. automated vital sign monitor C. pulse oximeter
B. pretracheal stethoscope D. none

99. Quadrant used in the gluteal region when intramuscular sedation is used: A.upper
inner quadrant B. upper outer quadrant C. Lower
inner quadrant D. Lower outer quadrant

100. The most unstable form of fluoride used for topical application:
A. Acidulated Phosphate Fluoride B. Stannous Fluoride
C. Sodium Fluoride . D. None

Downloaded by John Erickson Bangayan (bangayan1903064@ceu.edu.ph)


1. By what mechanism does the mandibular condyle grow?

- Endochondral bone formation

2. By what mechanism does the mandible grow downward and forward as viewed on successive
superimposed head films?

- Condylar growth

3. What are the active growth sites of the mandible?

- Posterior Border of the Ramus

- Condyle, Coronoid process

- Alveolar Process

* All of the above

4. According to current concepts of mandibular growth, what is the mechanism of action of the
following mandibular expansion appliance?

- Arch expansion by dental tipping

5. What are the active growth sites of the maxilla?

- Maxillary tuberosity areas

- Frontonasal process

- Midpalatal and transpalatal suture

*All of the above

6. How does the maxilla grow in width?


- Suture growth

7. In assessing overall facial growth, what are important considerations in treatment planning?
- Amount of growth to be expected

- Direction of growth expected

- General physical development

* All of the above


8. What are the typical profile changes that occur during the transition from childhood to adulthood?
- Flattening of the profile typically occurs with maturation

9. What is the response of alveolar bone to orthodontic forces?


- Bone is deposited at tension sites and resorbed at pressure sites

10. Change in the pressure side of the alveolar crest when orthodontic forces exceed physiologic limits
are called
- Undermining resorption

11. What are some of the potential negative tissue responses to heavy forces?
- Root resorption
- Excessive tooth mobility
- Discomfort
*All of the above

12. An example of extraoral anchorage is


- Diastema closure by elastic traction, tipping the crowns together

13. An example of extraoral anchorage is


- Chin cup
- Cervical headgear
- Occipital headgear
*All of the above

14. Which of the following is not an example of intermaxillary anchorage?


- Retraction of maxillary incisors by tipping them lingually, using maxillary molars as anchorage

15. Given an orthodontic force of 30 grams applied in a lingual direction to the crown of a maxillary
central incisor, the areas of the bone resorption or osteoclastic activity are
- A and E

16. The areas of bone despostion or osteoblastic activity in the above illustration are
- B and D

17. The type of anchorage being used in the illustration below is


- Simple Anchorage
- Reciprocal anchorage
- Intraoral anchorage
- Intermaxillary anchorage
*All of the above

18. Serious and difficult open bite cases are often characterized by which, of the following
cephalometric findings?
- Palatal plane tipped upward in the anterior in conjunction with a steep mandibular plane

19. Which of the following cannot be assessed with cephalometric radiographs?


- Adequecy of dental arch perimeter
20. A 7 year old patient demonstrates unilateral buccal crossbite in centric occlusion, deviation of the
mandibular midline, bilateral construction of the maxillary arch, and a lateral deviation from centric
relation to centric occlusion. What is the most likely treatment for this case?
- Bilateral palatal expansion

21. A mixed dentition patient with anterior crossbite of al incisors in centric occlusion and Class I molar
relationship can be positioned in centric relation so that the incisors are in an edge-to-edge relationship.
If the crossbite is correctable through maxillary incisor advancement and centric relation and centric
occlusion are then coincident, what will be the molar classification?
- Class II

22. A patient presents with a history of cleft palate and anterior crossbite. What is the most likely
skeletal cephalometric finding?
- Maxillary Retrusion

23. How does the evaluation of posterior arch length influence the management of intermediate
segment deficiency?
- Evaluation of the posterior segment may help determine bicuspid removal

24. What is the incidence of anterior open bite among children and adolescents?
- 5%

25. In an early mixed dentition case with insufficient space in the anterior segment for erupting
permanent lateral incisors, what treatment is indicated?
- Disc the deciduous cuspids

26. What is the rationale for selective early removal of deciduous teeth in cases involving deficient arch
length?
- To promote permanent tooth alignment by creating space
- To hasten eruption of underlying permanent teeth
- To reduce subsequent orthodontic treatment time
- To begin the several steps leading to selected bicuspid extractions
* All of the above
1. By what mechanism does the mandibular condyle grow?

- Endochondral bone formation

2. By what mechanism does the mandible grow downward and forward as viewed on successive
superimposed head films?

- Condylar growth

3. What are the active growth sites of the mandible?

- Posterior Border of the Ramus

- Condyle, Coronoid process

- Alveolar Process

* All of the above

4. According to current concepts of mandibular growth, what is the mechanism of action of the
following mandibular expansion appliance?

- Arch expansion by dental tipping

5. What are the active growth sites of the maxilla?

- Maxillary tuberosity areas

- Frontonasal process

- Midpalatal and transpalatal suture

*All of the above

6. How does the maxilla grow in width?


- Suture growth

7. In assessing overall facial growth, what are important considerations in treatment planning?
- Amount of growth to be expected

- Direction of growth expected

- General physical development

* All of the above


8. What are the typical profile changes that occur during the transition from childhood to adulthood?
- Flattening of the profile typically occurs with maturation

9. What is the response of alveolar bone to orthodontic forces?


- Bone is deposited at tension sites and resorbed at pressure sites

10. Change in the pressure side of the alveolar crest when orthodontic forces exceed physiologic limits
are called
- Undermining resorption

11. What are some of the potential negative tissue responses to heavy forces?
- Root resorption
- Excessive tooth mobility
- Discomfort
*All of the above

12. An example of extraoral anchorage is


- Diastema closure by elastic traction, tipping the crowns together

13. An example of extraoral anchorage is


- Chin cup
- Cervical headgear
- Occipital headgear
*All of the above

14. Which of the following is not an example of intermaxillary anchorage?


- Retraction of maxillary incisors by tipping them lingually, using maxillary molars as anchorage

15. Given an orthodontic force of 30 grams applied in a lingual direction to the crown of a maxillary
central incisor, the areas of the bone resorption or osteoclastic activity are
- A and E

16. The areas of bone despostion or osteoblastic activity in the above illustration are
- B and D

17. The type of anchorage being used in the illustration below is


- Simple Anchorage
- Reciprocal anchorage
- Intraoral anchorage
- Intermaxillary anchorage
*All of the above

18. Serious and difficult open bite cases are often characterized by which, of the following
cephalometric findings?
- Palatal plane tipped upward in the anterior in conjunction with a steep mandibular plane

19. Which of the following cannot be assessed with cephalometric radiographs?


- Adequecy of dental arch perimeter
20. A 7 year old patient demonstrates unilateral buccal crossbite in centric occlusion, deviation of the
mandibular midline, bilateral construction of the maxillary arch, and a lateral deviation from centric
relation to centric occlusion. What is the most likely treatment for this case?
- Bilateral palatal expansion

21. A mixed dentition patient with anterior crossbite of al incisors in centric occlusion and Class I molar
relationship can be positioned in centric relation so that the incisors are in an edge-to-edge relationship.
If the crossbite is correctable through maxillary incisor advancement and centric relation and centric
occlusion are then coincident, what will be the molar classification?
- Class II

22. A patient presents with a history of cleft palate and anterior crossbite. What is the most likely
skeletal cephalometric finding?
- Maxillary Retrusion

23. How does the evaluation of posterior arch length influence the management of intermediate
segment deficiency?
- Evaluation of the posterior segment may help determine bicuspid removal

24. What is the incidence of anterior open bite among children and adolescents?
- 5%

25. In an early mixed dentition case with insufficient space in the anterior segment for erupting
permanent lateral incisors, what treatment is indicated?
- Disc the deciduous cuspids

26. What is the rationale for selective early removal of deciduous teeth in cases involving deficient arch
length?
- To promote permanent tooth alignment by creating space
- To hasten eruption of underlying permanent teeth
- To reduce subsequent orthodontic treatment time
- To begin the several steps leading to selected bicuspid extractions
* All of the above
White patch which can be rubbed off and is found on the labial
or buccal mucosa is:
pachyderma oris C. lichen planus
leukoplakia D. ptyalism E. none of these.

Which of the following odontogenic cysts does not require clini-


cal or radiographic presence of a tooth for diagnosis?
Apical C. Eruption
B. Residual D. Dentigerous E. Lateral
periodontal

Brown to black pigmentation caused by adrenal insufficiency is


commonly associated with:
angular cheilitis C. candidiasis
perleche D. addison’s disease E. syphilitic lesion

Severe form of cellulites involving all spaces of the floor of


the mandible is:
Ranula B. dermoid cysts C. Ludwig’s angina D. sialolithiasis

Decotisyl is a:
antiarrhythmic C. analgesic
anticoagulant D. adrenocorticosteroid E. antifungal

ORTHODONTICS AND PEDIATRIC DENTISTRY

Which of the developmental space will cause a decrease in arch


perimeter when pre-empted?
primate space B. interocclusal space C. interden-
tal space D. leeway space

This is an increase in size by expansion from within


sutural apposition C. interstitial growth
maturation D. development

This is growth movement which mean movement of whole bone as a


unit
A. drift B. displacement C. deposition D. relo-
cation E. remodeling

Mandible comes from ? branchial arch


first B. second C. third D. fourth

Maxilla is formed from the maxillary processes and ?


medial nasal process C. globular process
B. lateral processes D. tuberculum impar

In young child, paradoxical excitement occurs most frequently


following premedication with:
a narcotic B. a barbiturate C. nitrous oxide
D. an amphetamine

The maximum amount of anesthesia that can be given to a 36-


pound child is:
4 carpules B. 3 carpules C. 2 carpules
D. 1 carpule

Dental Fluorosis is most likely to occur on a:


two-year-old child ingesting 1 ppm of fluoride
17-year-old patient ingesting 1 ppm of fluoride
on a 3-year-old child ingesting 0.50 ppm of fluoride
D. none of the above.

Pulpotomy with formocresol of primary tooth:


result in lower success rate compared to calcium hydroxide,
result in higher success rate compared to calcium hydrox-
ide,
should never be used,
induces formation of a dentin bridge at site of amputation,
none of the above.

A 4-year-old child visited a dentist for the first time and


received prophylaxis
quite well. Before he left, the dentist was glad
that he gave the child a toy.
This kind of gesture is called:
classical conditioning C. positive reinforcement,
bribery, B. public relations
E. none of the above.

If a primary tooth has been extracted before succedaneous tooth


has begun eruptive movement, its eruption will be __
hastened B. delayed C. not affected
Which of the following conditions is usually present in a Class
II, Division 2 malocclussion?
Open bite
Steep mandibular plane
Mesiocclusion of permanent first molars
Lingual inclination of maxillary central incisors

Excessive orthodontic force used to move a tooth may:


cause hyalinization
cause undermining resorption
crush the periodontal ligament
A. 1 & 2 B. 2 & 3 C. 1 & 3 D. 1, 2 & 3 E. 3 only

Severely crowded mandibular incisors usually result from:


mesial migration of teeth
premature loss of primary teeth
presence of supernumerary teeth
D. tooth size-arch length discrepancy

How will extraction of a primary maxillary central incisor in a


5-year old child with incisal spacingaffect the size of the
intercanine space?
The intercanine space will increase in size.
The intercanine space will decrease in size.
No change will occur in the size of the intercanine space.

The major criterion to differentiate between a true Class III


and a pseudo-Class III malocclusion is:
degree of anterior crossbite
presence of a bilateral crossbite
C. existence of a forward shift of the mandible during clo-
sure
D. occlusal relationship between maxillary and mandibular
first molars

Orthodontic correction of which of the following is most easily


retained?
Diastema C. Expansion
B. Rotation D. Posterior crossbite E. Anterior
crossbite

A bimaxillary protrusion occurs when:


large horizontal overlap with biteral crossbite exits
the maxillary dentition is forward with respect to basal
bone
both dental arches are forward with respect to basal bone
D. none of the choices.

When a simple tipping force is applied to the crown of a single-


rooted tooth, the center of rotation is usually located:
at the apex D. at the cervical line
5mm beyond the apex E. one-third the root length
from the apex
two-third the root length from the apex.

Which of the following is the most common orofacial malfor-


mation that produces malocclusion?
Cleft palate C. Ectodermal dysplasia
Pierre Robin syndrome D. Osteogenesis imperfecta E.
Cleidocranial dysostosis

The cranial vault increases rapidly in size the first few years
postnatally and completes approximately 90% of its growth by
6 years of age. This growth is typical of which of the fol-
lowing types of tissues?
Neural B. dental C. genital D. lymphoid E.
general (somatic)

The following are the conditions found on the area of pressure


in the presence of heavy forces:
area of hyalinization
occlusion of blood vessels
stretched periodontal fibers
undermining resoprtion
A. 1, 2& 3 B. 1, 2 & 4 C. 2 , 3 & 4 D. 3 only
E. 1 & 4 only

Generalized osteoclastic activity along the walls of the alveo-


lar socket is the bone response to:
depressing force C. extrusion
B. elongating force D. rotating force E.
both B & D

A 4-year old child has a traumatized central incisor with a


Class III (Ellis) fracture. The injury occurred about one
month ago, and examination indicates that the pulp is ne-
crotic. There are no other pathologic findings. Treatment of
choice is:
watchful observation
extraction and use of a space maintainer
pulpectomy and root canal filling using gutta-percha points
and cement
endodontic treatment and root canal filling with a resorba-
ble paste.

Supervision of a child’s development of occlusion is most criti-


cal at ages:
3-6 years B. 7-10 years C. 11-14 years E. 14-
17 years

A child with congenital heart disease requires special treatment


planning for dental care because of potential problems with:
bleeding
local infection
systemic infection
enamel hypoplasia
A. 1 , 2 & 3 B. 1 & 3 C. 1 only D. 3 only E.
1,2,3 & 4

In festooning and trimming a stainless steel crown, greater


length is necessary in the region of the mesiofacial bulge in
a primary:
first molar B. second molar C. maxillary canine
D. maxillary lateral incisor

Using a topical fluoride rinse before acid etch direct bonding


of orthodontic brackets is contraindicated because fluoride:
decreases the solubility of enamel
increases the pH of the etching agent
causes copious amounts of saliva
directly reacts chemically with the bonding agent.

If a 7-year old patient loses a primary mandibular canine about


the same time the adjacent lateral incisor is erupting or
shortly thereafter, the dentist should be alert to the possi-
bility of:
a tongue habit
a developing crossbite
an early eruption of the permanent canine
lingual collapse of mandibular anterior teeth

Which of the ff. approaches is best for a child suffering from


celebral palsy
HOM exercise B. towel method C. use of sedation
D. psychological approaches

Type of fear which is produced by direct physical stimulation of


sense organ
associative fear B. objective fear C. subjective
fear D. acquired fear

Agent used to reduce pain without affecting consciousness


hypnotic drug B. analgesic C. barbiturate
D. general anesthesia

The drug that modifies fear and anxiety


A. flanax B. ethamizid C. celestamine
D. seconal

A general characteristic of a 12 year old child


mother as the center of his world C. exhibits
tantrums
separation anxiety D. rejects parental
authority

This is a psychological approach to manage a child


nitrous oxide inhalation C. giving diazepam
mutisensory techniques D. HOM

The major source of anchorage in a maxillary Hawley appliance is


the :
molar clasp C. acrylic portion
B. labial bow D. interproximal clasp E. bracket
of the band

A narrow maxillary arch with respect to midsagittal plane is


said to be in:
Protraction C. contraction
B. retraction D. distraction E. both C & D

Complete unilateral lip-jaw palate cleft.


A. Class I B. Class II C. Class III
D. Class IV

Anodontia, diagnosed in a 5-year-old child, primarily affects


the growth of the:
Midface C. mandible
B. maxilla D. alveolar bone E. maxilla
& mandible

Which of the following are likely to occur during orthodontic


therapy?
gingival irritation
cementum resorption
increased mobility of teeth
demineralization of enamel adjacent to appliances in pa-
tients with poor oral hygiene
1, 3 & 4 C. 2 & 4 only E. 3 & 4 only
B. 1 & 4 only D. 1, 2, 3 & 4

A distal shoe space maintainer is indicated when a primary:


incisor is avulsed
first molar is prematurely lost
second molar is lost after eruption of a permanent first
molar
second molar is lost before eruption of a permanent first
molar

The normal downward and forward direction of facial growth re-


sults from
upward and backward growth of maxillary sutures and the
mandibular condyle
vertical eruption and mesial drift of the dentitions
interstitial growth in the maxilla and the mandible
epithelial induction at growth centers
A. 1 & 2 B. 1 & 3 C. 1 & 4 D. 2 & 3 E. 2 &
4
An 8-year-old girl has 2mm diastema between permanent maxillary
central incisors. Permanent maxillary lateral incisors are in
position. The diastema is probably the result of:
thumb-sucking
an abnormal frenum attachment
a normal developmental process
A. 1 & 2 B. 1 & 3 C. 2 & 3 D. 2 only E. 3 only

When comparing cementum to bone in their responses to orthodon-


tic forces, cementum resorbs:
more readily C. not at all
B. less readily D. under lighter loads E. by
indirect (undermining) resoprtion

The tooth in the mandibular arch most likely to be malposed in


cases of arch space discrepancy is the:
first molar B.. first premalor C. second molar D.
second premolar

Slow progress in molar uprighting in an adult patient is usally


due to:
overextended bands D. an overcontoured
spring
lack of anchorage control E. the occlusion not
being relieved
the stabilizing wire not being passive

A 9-year-old patient has a slightly convex profile and a sus-


pected tooth mass-arch length (circumference) discrepancy.
Before instituting space management procedures, the dentist
should:
complete a space analysis C. complete a cephalo-
metric analysis
seek orthodontic consultation D. all of the above
E. Either A or C above

A primary maxillary anterior tooth in a 4-year-old child was


traumatically intruded into the tissues so that only half the
tooth is visible. The most appropriate treatment is to:
extract the tooth.
perform a pulpotomy
administer no treatment
place orthodontic bands on adjacent teeth and draw the
tooth down with elastics

Which of the following orthodontic movements of teeth are most


difficult to accomplish?
Tipping
Rotation
Intrusion
Extrusion
Translation
A. 1 & 2 B. 1 & 3 C. 1 & 5 D. 3 & 4 E. 3 & 5
Interstitial growth is observed at which of the following
sites?
Spheno-occipital syndrondrosis
Maxillary tuberosity
Mandibular condyle
Zygomaticomaxillary suture
Apex of an erupting premolar
A. 1 & 2 B. 1 & 3 C. 1 & 4 D. 2 & 4 E. 2 & 5

When force is applied, bone reflexes that produces deformation


of crystalline materials and allows flow of electric current
necessary for tooth movement:
deposition C. hyalinization
B. piezoelectricity D. undermining resoprtion

Cephalometrics is useful in assessing which of the following


relationships?
1. tooth-to-tooth
2. bone-to-bone
3. tooth-to-bone
A. 1 & 2 B. 1 & 3 C. 2 & 3 D. 1,2 & 3 E. 3 only

Which of the following is the most essential factor related to


correction of an anterior crossbite?
age of the patient C. depth of the corssbite
shape of the tooth involved D. space available mesi-
odistally

Frankfort-horizontal is a reference plane constructed by joining


which of the following landmarks?
Porion and sella C. Porion and nasion
Porion and orbitale D. Nasion and Sella

Which of the following is not a space maintainer?


Lingual arch D. Nance holding arch
Class III restoration E. Stainless steel crown
Palatal expansion appliance

Space closure is least likely to occur after loss of which of


the following teeth?
Primary mandibular canines D. Primary mandibular sec-
ond molars
Primary maxillary first molars E. Primary maxillary
central incisors
Permanent maxillary central incisors

Crowding that occurs with mandibular incisors after age 18 is


most often the result of:
orthodontic relapse C. eriodontal disease
impacted thid molars D. a physiologic matura-
tional change

The undesirable side-effect most commonly associated with use of


a finger spring to tip the crown of a tooth is:
Pain D. gingival irritation
tendency for the tooth to intrude E. severe mobility of
the tooth
tendency for the root apex to move in the direction oppo-
site from the crown.

After the age 6, the greatest increase in size of the mandible


occurs:
at the symphysis C. along the lower border
between the canines D. distal to the first molars

A distal shoe space maintainer is indicated when a primary:


canine is in crossbite
first molar is prematurely lost
second molar is lost after eruption of a permanent first
molar
second molar is lost prior to eruption of a permanent first
molar

The “V” principle of growth is best illustrated by the:


nasal spetum C. mandibular symphysis
B. mandibular ramus D. spheno-occipital synchondro-
sis

Anterior crossbite in the primary dentition usually indicates a


developing:
Class I malocclusion C. Class II malocclusion
Class III malocclusion D. Class IV maloccusion E.
none of the above

A mixed dentition analysis determines:


intercanine width D. skeletal growth pattern
discrepancies in jaw size E. size of permanent inci-
sors
space available versus space required

Gonion, menton and pogonion are cephalometric landmarks located


on the
midline B. bony chin C. mandible D. skeletal
profile

A major disadvantage of treatment using cervical headgear is:


impaction of maxillary canines D. extrusion of max-
illary incisors
extrusion of maxillary molars E. potential deform-
ity of the neck
psychologic trauma due to appearance

Asymmetrical anterior open bite with normal posterior occlu-


sion is characteristic of:
thumb-sucking C. abnormal swallowing habits
mouth breathing D. both A & B E. both B &
C

If the norm for the cephalometric angle SNA is 82°, a patient’s


reading of 90° for SNA most likely indicates:
maxillary protrusion C. the patient’s ethnic
background
protrusive maxillary incisors D. dysplasia of the
anterior cranial base.

Ideally, a malooclusion should be treated between the ages of:


5 and 8 years D. 8 and 10 years
10 and 12 years E. 12 and 14 years
None of the above. The age at which a malocclusion is
treated should depend on the problem involved.

Radiographs of a 5-year old patient show permanent maxillary


first molars inclined mesially with resulting resoprtion of
the distal portions of the roots of primary second molars.
The condition described is:
ankylosis C. ectopic eruption
premature eruption D. internal resorption
E. intrafollicular resoprtion

Fractured maxillary anterior teeth generally occur most often


in children with which of the following dental conditions?
Class I malocclusion D. Class II, Division
1 malocclusion
Class II, Division 2 malocclusion E. Class III
malocclusion
Marked incisor crowding

On the first dental visit, the basic fear of a child under the
age of 2 is related to:
fear of an injection
anxiety over being separated from a parent
anxiety regarding the instruments used to perform dental
treatment
not understanding the reason for dental treatment.

A child with Down’s syndrome is characterized as being:


affectionate C. fearful of quick move-
ments
capable of learning dental procedures D. both A & B
E. all of these

A 3 ½ year old child has an acute fever, diarrhea, oral vesicu-


lar lesions and gingival tenderness. The most likely diagno-
sis is:
thrush D. drug allergy
aphthous ulcerations E. . acute herpetic stoma-
titis
C. necrotizing ulcerative gingivitis

Transillumination of soft tissues is useful in detecting which


of the following problems in a child?
Koplick’s spot C. Sialolithiasis
Aortic stenosis D. Sickle cell disease E. ab-
normal frenum attachment

Which of the following is least likely to result from persis-


tent long-term thumb-sucking?
a deep overbite D. protrusion of maxillary in-
cisor
constriction of maxillary arch E. rotation of maxil-
lary lateral incisor
lingual inclination of mandibular incisors

The least desirable method used in child management is:


tell-show do technique, C. voice control,
gift before treatment, D. hand-over-mouth technique.

A child who is reluctant to accept dental treatment ,exhibit


negative attitude but not so pronounced is classified under
Frankl ?
I B. II C. III D. IV

The amount of pulp tissue to be removed in formocresol pulpotomy


procedure
half of the coronal pulp
coronal pulp tissue down to the cervical constriction of
each root canal
C. infected portion of coronal pulp tissue
D. total removal of pulp tissue including radicular
pulp

Which of the following dental sequelae is likely in a child


with a history of generalized growth failure (failure to
thrive) in the first six months of life?
Retrusive maxilla C. Enamel hypoplasia
Retrusive mandible D. Small permanent teeth E.
Dentinogenesis imperfecta
The average age at which calcification of crowns of permanent
central incisors is completed is:
birth B. 2-3 years of age C. 4-5 years of age D. 6-7
years of age

Amalgam is most often the restorative material of choice for


primary teeth. The most important modification in its use for
children is in:
cavity preparation D. use of a rubber dam
use of a zinc-free alloy E. condensation of the al-
loy
placement of matrix bands

The most frequent cause of fracture of a root tip during ex-


traction of a primary molar is:
ankylosis of the tooth
improper use of cowhorn forceps
presence of a supernumerary premolar
root resorption between the apex and the bifurcation
asymmetric root resorption in which only one root is com-
pletely resorbed.

The most common cause of sinus tracts in gingival tissues of


children is:
pericementitis C. periapical cyst
acute periapical abscess D. chronic periapical ab-
scess

A light bluish, dome-shaped lesion on the inside lip of a 2-year


old child is most likely a:
Mucocele B. melanoma C. hematoma D. hemangioma
E. sucking callous

Primary second molars usually erupt during ages:


8-14 months C. 14-20 months
20-30 months D. 30-36 months E. 36-48 months

A child in long-term remission of acute leukemia has dental


problems characterized by unusual susceptibility to:
dental caries C. oral infection
periodontal bone loss D. development of jaw deformi-
ties E. all of the above.

This is a contraindication of HOM


spoiled child C. very young patient
defiant child D. uncooperative behavior but capa-
ble of understanding

Induce formation of apical closure of young permanent molar us-


ing CAOH
apexogenesis C. apexification
B. apeximation D. apex formation
The growth movement of the mandible is complimented by the
growth of the
maxilla, which is:
down and forward C. downward and backward
B. forward only, D. upward and backward

Direction of displacement of the mandible in an individual with


developing Class II malocclusion:
down and forward, C. down and backward,
B. forward only, D. forward and upward,
E. upward and backward.

Child refuses to accept treatment or open the mount minimally:


A. Frankl 1 B. Frankl 2 C. Frankl 3 D. Frankl 4

Definitely negative behavior.


A. Frankl 1 B. Frankl 2 C. Frankl 3 D. Frankl 4

Child has good rapport with the dentist.


A. Frankl 1 B. Frankl 2 C. Frankl 3
D. Frankl 4.

The treatment option contraindicated in patients who are not


able to breathe nasally.
no treatment C. nitrous oxide & oxygen inha-
lation
B. conscious sedation D. genera anesthesia

The treatment option for patients who have sustained extensive orofa-
cial & or dental trauma
no treatment C. nitrous oxide & oxygen inhalation
B. conscious sedation D. general anesthesia

With a flush terminal plane, permanent first molars will:


initially be Class II
initially be Class III
immediately assume a normal relationship
erupt immediately into an end-to-end relationship

Arch shape & symmetry are best evaluated from the:


study cast C. frontal photograph
B. panoramic x-ray D. lateral cephalogram E. intraoral pho-
tograph

Bodily force that moves the central incisor mesially produces:


stretching of the periodontal fiber on the distal side
compression of the periodontal fiber in the distal side
osteoblastic activity on the mesial side
osteoclastic activity on the distal side

In infant oral care, clean mouth with gauze after feedings and at bed
time done during:
A. 0-6 mons B. 6-12 mons C. 12-24 mons. D. 24-
36 mons
Space differential between combine width of CDE and 345 is__
A. positive B . negative C. zero D. 2.2 mm
E. 2.4 mm

ORTHODONTICS AND PEDIATRIC DENTISTRY

1. Bite plane therapy for deep bite, active Hawley’s plate are ex-
amples of:
A. Preventive Orthodontics C. Limited Corrective Orthodon-
tics
B. Interceptive Orthodontics D. Extensive Corrective Ortho-
dontics E. none of the choices

2. The normal sequential events from fertilization to death:


A. growth B. differentiation C. translocation D. de-
velopment E. none of the choices

3. ______will give rise to tissues that will become the mandible


A. Reichert’s cartilage C. Meckel’s cartilage
B. 3rd branchial arch D. 1st branchial arch E.
none of the choices

4. Emphasized how cartilage of nasal septum during growth paced


the growth of maxilla
A. Genetic Theory C. Moss’ Hypothesis
B. Sicher’s Hypothesis D. Scott’s Hypothesis E. Pe-
trovic’s Hypothesis

5. Skeletal tissues grow in response to soft tissue growth


A. Genetic Theory C. Moss’ Hypothesis
B. Sicher’s Hypothesis D. Scott’s Hypothesis
E. Petrovic’s Hypothesis

6. Cranial vault grows primarily by cartilage growth at:


A. synchondrosis B. sutures C. fontanelles D. disappears at
birth

7. Cranial base grows primarily by__ growth at the synchondrosis:


A. intramembranous B. endochondral C. combination
of intramembranous and endochondral

8. Over 90% of brain growth is achieved by age: A. 5 B. 6


C. 7 D. 8 E. 15

9. Neurocranium follows what Scammon’s curve?


A. lymphoid B. neural C. general D. bodily E.
genital type

10. Growth of brain case is primarily by proliferation and ossifi-


cation of:
A. synchondrosis B. sutural connective tissue C. fonta-
nelles D. none of the choices
11. Nasomaxillary complex is hafted to the cranium by the follow-
ing sutures except: A. fronto-
maxillary C. zygomaticotemporal E. zygo-
maticomaxillary
B. pterygopalatine D. frontotemporal

12. As the maxilla decends, there is __ on the orbital floor, __


on the nasal floor and __ on the inferior palatal surface:
A. ++- B. -+- C. +-+ D. +++ E. --+

13. The increase in length of the maxilla is due to:


A. apposition on the posterior border of the maxillary tuberosity
B. progressive movement of the entire zygomatic process in poste-
rior direction D. A&B
C. growth in the median suture
E. none of the choices

14. Mechanism for maxillary growth:


A. sutural connective tissue proliferation C. surface appo-
sition / resorption
B. ossification D. translation
E. all of the choices

15. The mechanism for bone growth of the nasomaxillary complex is:
A. intramembranous B. endochondral C. combination

16. The sutures of the nasomaxillary complex are oblique and more
or less parallel with each other thus, growth in these areas would
serve to displace the maxilla:
A. downward B. forward and upward C. downward and
forward D. upward and backward

17. Continued apposition of alveolar bone on the free borders of


the alveolar process as the teeth erupt increases the __ of the
maxilla.
A. height B. width C. length

18. Increase in the width of the maxilla involves palatal growth


following the expanding V principle. The buccal segments move:
A. downward and forward B. posterior and superior C. up-
ward and posterior D. upward

19. The maxilla Is displaced downward and forward by growth in __


parts of the bone:
A. anterior and inferior B. posterior and superior C. an-
terior and superior D. posterior and anterior

20. What characteristic growth pattern is shown in the vertical


section through the coronoid process?
A. (+) on the lingual, (-) on the buccal C. (-)
on the lingual, (+) on the buccal surface
B. (+) on the lingual, (-) on half of the buccal surface D. (-)
on the lingual, (-) on the buccal surface
21. What characteristic growth pattern is shown in the vertical
section through the basal part of the ramus? A. (+) on the
lingual, (-) on the buccal C. (-) on the lingual, (+)
on the buccal surface
B. (+) on the lingual, (-) on half of the buccal surface
D. (-) on the lingual, (-) on the buccal surface

22. When teeth are lost, alveolar bone: A. persists B. re-


sorbs C. no effect

23. Mandibular growth follows what growth curve? A. lymphoid


B. neural C. general D. genital

24. At age 5-10, the mandible is __ completed. A. 45% B.


40% C. 65% D. 96%

25. The predominant direction of mandibular growth is:


A. superior and inferior C. superior and posterior
B. downward and forward D. downward E. su-
perior

26. Displacement of the mandible occurs in what direction? A.


superior and inferior B. superior and posterior C. down-
ward and forward D. downward E. superior

27. Increase in mandibular length is accomplished by:


A. (+) on the inferior border of the ramus
B. (-) on the inferior border of the ramus
C. (-) on the posterior border, (+) on the anterior border of the
ramus
D. (+) on the posterior border, (-) on the anterior border of the
ramus

28. Growth of the posterior border of the mandible with additive


growth at the ends of the “V” increases the terminal points. This
increases the __ of the mandible.
A. width B. height C. depth

29. Condylar growth with significant alveolar growth increase the


__ of the mandible.
A. width B. height C. depth

30. Some mandibles grow more forward than downward. If there is


predominance of forward growth __ is the result.
A. Class I B. Class II C. Class III D.
open bite

31. When is the best time to influence , retard or redirect


growth?
A. after puberty C. before puberty
B. just before and during puberty D. time ele-
ment is not important
32. The order form most rapid to least amount of growth for the
cranium:
A. height, depth, width B. depth, width, height C.
depth, height, width D. width, height, depth

33. The order form greatest to least incremental change for the
face:
A. depth, height, width B. width, depth height C.
height, depth, width D. height, width, depth

34. At age 0-5, which has grown the most at 85%? A. neurocra-
nium B. maxilla C. mandible

35. Because of the differential growth of the maxilla and the man-
dible at age 0-5, the normal profile at birth is: A. straight
B. convex C. concave

36. Maxillary intercanine width is completed in girls at age:


A. 7-8 B. 9-10 C. 11 D. 12 E. 18

37. Second peak for girls: A. 3 B. 6-7 C. 7-9 D.


11-12 E. 14-15

38. For boys, maximum condylar changes concurrently with sutural


and skeletal height peaks 3 years later that for girls. A.
true B. false

39. Treatment timing must be based on the individual’s own pattern


of growth. A. true B. false

40. Primate space if found:


A. mesial of the lower cuspid and distal of upper cuspid
B. distal of the lower cuspid and distal of the upper cuspid
C. mesial of the lower cuspid and mesial of the upper cuspid
D. mesial of the upper cuspid and distal of the lower cuspid

41. Normal terminal plane relationship[: A. mesial step


B. distal step C. flush/straight

42. The most common sequence of eruption for the upper arch:
A. 6124537 B. 6123457 C. 6142537 D.
6125437

43. The stage at which a tooth begins its eruptive movement:


A. stage 0 B. stage 2 C. stage 3 D. stage 6
E. stage 10

44.Periapical lesions, pulpitis and pulpotomy of a primary molar


will___ the eruption of successor premolar. A. hasten B.
delay C. have no effect on
45. If a primary tooth is extracted prior to the onset of perma-
nent eruptive movement (prior to root formation), the permanent
tooth is likely to be:
A. delayed B. accelerated C. no effect on
eruption

46. Which one is an incorrect description of Nolla’s stages of


calcification?
A. Stage 4, 2/3 of crown completed C. Stage 7, 1/3 of
root completed
B. Stage 6, crown almost completed D. Stage 9, root al-
most complete, open apex

47. Space differential in the anterior segment ( BA/AB ) - 21/12


gives:
A. excess space B. no space differential C.
negative space

48. The proper crown angulation of a permanent incisor is one where the gingival portion of the
long axis of the crown should be __ to the incisal portion.
A. mesial B. distal C. incisal D. cervical

49. Rotated teeth occupy less space within the arch: A. true B. false

50. Plane of occlusion for permanent teeth: A. flat B. exaggerated

51. Posterior crown inclination from canine to molar:


A. lingual crown inclination B. buccal crown inclination
C. upright

52. Size of teeth correlates well with stature. Size of teeth is sex-linked.
A. 1st statement is true, 2nd is false C. both are true
B. 1st statement is false, 2nd is true D. both are false

53. The most prevalent malocclusion in the primary dentition:


A. Class II Div. 1 B. Class I type 5 C. Class III D. anterior openbite E. Class II types 1

54. A dolichofacial face would likely to have ___ arch form.


A. round B. average C. broad and square D. long and tapering

55. A face that is broad and short is:


A. dolichofacial B. brachyfacial C. mesofacial D. normofacial

56. The most common malocclusion in the mixed dentition period is:
A. Class II Div. 1 B. anterior open bite C. posterior open bite D. crowding E. Class I type
2

57. A type of conditioning where in a specific response is immedi-


ately rewarded
Classical conditioning C. operator conditioning
Operant conditioning D. conditioning E. classic con-
ditioning
58. The patients history of childhood diseases are under
A. familial history B. social history C. medical history D. dental history

59. The history on the behavior of the child toward dentistry:


A. familial history B. medical history C. Dental history
D. history of present illness

60. Method of early examination where the dentist and parent are
seated face to face
A. Knee position B. leg position C. cradling position
D. Knee to knee position E. all

61. In pedodontic triangle the apex of the triangle is:


A. Child B. Dentist C. Parent D. Assistant

62. Parent who are mistrusting are:


Overprotective Parent B. Neglectful Parent C. Manipulative
Parent D. Hostile parent

63. Parent who are demanding and can extend to directing the course
of the treatment is a :
A. Overprotective Parent B. Neglectful Parent C. Manipu-
lative Parent D. Hostile parent

64. Panoramic Radiographic Film:


A. 5 “ x 7” B. 5” x 12” C. 2”x 3” D. 8” x 10”

65. Frankl behavior rating characterized by refusal of treatment and


extreme negativism:
A. rating 1 B. rating 2 C. rating 3 D. rating 4

66. A period that child undergoes the phase of separation anxiety:


A. Toddler B. school age C. Pre school D. Ado-
lescent

67. In Nitous oxygen sedation, once a petient takes a distant gaze


the concentration of the nitous oxide- oxygen is reduces to:
A. 30-70 B. 20-80 C. 40-60 D. 10-90 E. none

68. Device that assess the arterial hemoglobin oxygen saturation and
pulse rate:
A. automated vital sign monitor B. pulse oximeter C.
pretracheal stethoscope D. none

69. Quadrant used in the gluteal region when intramuscular sedation


is used:
A, upper inner quadrant B. upper outer quadrant C. Lower in-
ner quadrant D. Lower outer quadrant

70. The most unstable form of fluoride used for topical application:
A. Acidulated Phosphate Fluoride B. Stannous Fluoride C.
Sodium Fluoride . D. None
71. A sudden firm command use dto get the child’s attention:
A. Voice control B. Multisensory communication C. .Aversive
conditioning D. HOME

72. Procedure which slowly develops behavior by reinforcing succes-


sive approximation of the desired behavior until desired behavior
develop:
A. Pre appointment behavior modification C. behavior shap-
ing
Behavior management D. Retraining

73. Parents should be allowed inside a treatment room when:


A. infant to 41 months B. handicapped patient C. 7-8 Years old
D. A & B E. None

74. A technique of diverting the attention of the patient from what


may be perceived as an umpleasant procedure:
A. tell show do C. Voice control
B. Positive reinforcement D. Distraction E Non verbal
communication

75. Plastics applied to the occlusal surface of posterior teeth by


simple acid etch method to prevent dental caries & early carious
lesion from developing further
A. Composite C. Fluoride
B. Preventive Restorative Resin D. Glass Ionomer
E. None

76. Medicament used in type A PRR:


A. Glass ionomer B. Bonding agent C. Composite D. Pit and
fissure sealant E. all of these

77. Distance of punched holes on the rubber dam for primary teeth:
A. 1.0mm B. 1.5 mm C. 2.0mm D. 2.5 mm E. 3.0mm

78. Gingival cavosurface beveling is not indicated in Class II cavity


preparation for deciduous teeth, because the direction of the enamel
rods are toward:
A. Cervical B. Occlusal C. gingival D. Apical E. all
of these

79. Etching time for deciduous dentition is ----- compared to per-


manent dentition:
A. shorter B. the same C. longer D. none

80. The least amount of tooth surface reduction in stainless steel


crown preparation is:
A. Occlusal B. lingual C. Proximal D. Buccal
E. none

81. Ellis classification of tooth fracture when the coronal pulp is


exposed:
A. Class I B. Class II C. Class III D. Class IV
E. Class V

82. Ellis classification of tooth fracture when a tooth is lost as


a result of trauma
A. Class I B. Class II C. Class III D. Class IV
E. Class V

83. Ellis classification of a non vital traumatized tooth:


A. Class I B. Class II C. Class III D. Class IV
E. Class V

84. Treatment of choice for an immature non vital permanent tooth


to stimulate root closure:
A. Indirect pulp capping B. Pulpectomy C. Pulpotomy D.
Apexogenesis E. Apexification

85. Materials that is used to induced apexification:


A. calcium hydroxide B. Zinc oxide eugenol C. Formocresol D.
IRM E. Gutta percha

86. Superficial wound due to scrapping of the mucosa:


A. laceration B. abrasion C. Contusion D. At-
trition E. Erosion

87. Hypoplastic enamel is a result in the disturbance at:


A. initiation stage B. Appostion stage C. . Morphodifferentation
D. Histodifferentiation E. None
88. Macrodontia incisor crown with two root canals:
A. Flexion B. Fusion C. Gemination D. Supernumerary
root E. none

89. Presence of enamel at the furcation of the root:


A. enameloma B. Enamelin C. ameloblast D. Epstein Pearl
E. all of the above

90. Radiograph picture of a caries is generally--- than the actual


caries:
A. smaller B. the same C. larger D. deeper E. B&D

91. Primary molars with multisurface caries ( 3 or more) are best


restored using:
A.GIC B. Posterior composite C. stainless steel crown
D. amalgam restoration E. none

92. Primary teeth have pulpal inflammation when the bacterial in-
fested dentin is __ mm from the pulp:
A. 0.6 B. 0.8 C. 1.6 D. 1.8 E.
none
93. Requires retentive groove in tooth preparation:
A. anterior SSC B. strip crown C. posterior SSC D.
none of these

94. T-cut made on celluloid strip crown is placed on the ___ sur-
face?
A. labial B. lingual C. mesial D. distal

95. The following are favorable storage medium for avulsed tooth,
except:
A. Hank’s buffered saline C. pasteurized bovine milk
B.Isotonic saline D. human saliva E.
water

96.Treatment for non-vital tooth with open apex:


A.IPC B. Pulpotomy C. DPC D. Pulpectomy E.
Apexification

97. This method is not indicated on children who are emotionally


immature
A. TSD B Voice control E. Positive
reinforcement
B. Distraction D. Non Verbal communication

98. Diverts the attention of the patient from what may be perceived
as an unpleasant procedure
A. TSD C Voice control E. Positive
reinforcement
B. Distraction D. Non Verbal communication

99. Conveying reinforcement and guiding behavior through contact,


posture and facial expression
A. TSD C Voice control E. Positive
reinforcement
B. Distraction D. Non Verbal communication

100. Method used to strengthen the occurrence of desired behavior


through verbal praise, giving tokens and toys
A. TSD C Voice control
B. Positive reinforcement D. Distraction E. Non
Verbal communication
ORTHODONTICS AND PEDIATRIC DENTISTRY

Which of the following conditions is usually present in a Class


II, Division 2 malocclussion?
Open bite
Steep mandibular plane
Mesiocclusion of permanent first molars
Lingual inclination of maxillary central incisors

Excessive orthodontic force used to move a tooth may:


cause hyalinization
cause undermining resorption
crush the periodontal ligament
A. 1 & 2 B. 2 & 3 C. 1 & 3 D. 1, 2 & 3 E. 3 only

Severely crowded mandibular incisors usually result from:


mesial migration of teeth
premature loss of primary teeth
presence of supernumerary teeth
D. tooth size-arch length discrepancy

How will extraction of a primary maxillary central incisor in a


5-year old child with incisal spacingaffect the size of the
intercanine space?
The intercanine space will increase in size.
The intercanine space will decrease in size.
No change will occur in the size of the intercanine space.

The major criterion to differentiate between a true Class III


and a pseudo-Class III malocclusion is:
degree of anterior crossbite
presence of a bilateral crossbite
C. existence of a forward shift of the mandible during clo-
sure
occlusal relationship between maxillary and mandibular
first molars

Orthodontic correction of which of the following is most easily


retained?
Diastema C. Expansion
B. Rotation D. Posterior crossbite E. Anterior
crossbite

A bimaxillary protrusion occurs when:


large horizontal overlap with biteral crossbite exits
the maxillary dentition is forward with respect to basal
bone
both dental arches are forward with respect to basal bone
D. none of the choices.

The following are the conditions found on the area of pressure


in the presence of heavy forces:
area of hyalinization
occlusion of blood vessels
stretched periodontal fibers
undermining resoprtion
A. 1, 2& 3 B. 1, 2 & 4 C. 2 , 3 & 4 D. 3 only
E. 1 & 4 only

Generalized osteoclastic activity along the walls of the alveo-


lar socket is the bone response to:
depressing force C. extrusion
B. elongating force D. rotating force E.
both B & D

The major source of anchorage in a maxillary Hawley appliance


is the :
molar clasp C. acrylic portion
B. labial bow D. interproximal clasp E. bracket
of the band

A narrow maxillary arch with respect to midsagittal plane is


said to be in:
Protraction C. contraction
B. retraction D. distraction E. both C & D

A distal shoe space maintainer is indicated when a primary:


incisor is avulsed
first molar is prematurely lost
second molar is lost after eruption of a permanent first
molar
second molar is lost before eruption of a permanent first
molar

Complete unilateral lip-jaw palate cleft.


A. Class I B. Class II C. Class III
D. Class IV

Anodontia, diagnosed in a 5-year-old child, primarily affects


the growth of the:
Midface C. mandible
B. maxilla D. alveolar bone E. maxilla
& mandible

Which of the following are likely to occur during orthodontic


therapy?
gingival irritation
cementum resorption
increased mobility of teeth
demineralization of enamel adjacent to appliances in pa-
tients with poor oral hygiene
1, 3 & 4 C. 2 & 4 only E. 3 & 4 only
B. 1 & 4 only D. 1, 2, 3 & 4

The normal downward and forward direction of facial growth re-


sults from
upward and backward growth of maxillary sutures and the
mandibular condyle
vertical eruption and mesial drift of the dentitions
interstitial growth in the maxilla and the mandible
epithelial induction at growth centers
A. 1 & 2 B. 1 & 3 C. 1 & 4 D. 2 & 3 E. 2 &
4

An 8-year-old girl has 2mm diastema between permanent maxillary


central incisors. Permanent maxillary lateral incisors are in
position. The diastema is probably the result of:
thumb-sucking
an abnormal frenum attachment
a normal developmental process
A. 1 & 2 B. 1 & 3 C. 2 & 3 D. 2 only E. 3 only

When comparing cementum to bone in their responses to orthodon-


tic forces, cementum resorbs:
more readily C. not at all
B. less readily D. under lighter loads
E. by indirect (undermining) resoprtion

The tooth in the mandibular arch most likely to be malposed in


cases of arch space discrepancy is the:
first molar C. first premalor
B. second molar D. second premolar

Slow progress in molar uprighting in an adult patient is usally


due to:
overextended bands
an overcontoured spring
lack of anchorage control
the occlusion not being relieved
the stabilizing wire not being passive

The “V” principle of growth is best illustrated by the:


nasal spetum C. mandibular symphysis
B. mandibular ramus D. spheno-occipital synchondro-
sis

Anterior crossbite in the primary dentition usually indicates a


developing:
Class I malocclusion
Class II malocclusion
Class III malocclusion
Class IV maloccusion
none of the above

A mixed dentition analysis determines:


intercanine width
skeletal growth pattern
discrepancies in jaw size
size of permanent incisors
space available versus space required

With a flush terminal plane, permanent first molars will:


initially be Class II
initially be Class III
immediately assume a normal relationship
erupt immediately into an end-to-end relationship

Arch shape & symmetry are best evaluated from the:


study cast C. frontal photograph
B. panoramic x-ray D. lateral cephalogram E. in-
traoral photograph

Bodily force that moves the central incisor mesially produces:


stretching of the periodontal fiber on the distal side
compression of the periodontal fiber in the distal side
osteoblastic activity on the mesial side
osteoclastic activity on the distal side

A 9-year-old patient has a slightly convex profile and a sus-


pected tooth mass-arch length (circumference) discrepancy. Be-
fore instituting space management procedures, the dentist
should:
complete a space analysis
complete a cephalometric analysis
seek orthodontic consultation
all of the above
Either A or C above

A primary maxillary anterior tooth in a 4-year-old child was


traumatically intruded into the tissues so that only half the
tooth is visible. The most appropriate treatment is to:
extract the tooth.
perform a pulpotomy
administer no treatment
place orthodontic bands on adjacent teeth and draw the
tooth down with elastics

Which of the following orthodontic movements of teeth are most


difficult to accomplish?
Tipping
Rotation
Intrusion
Extrusion
Translation
A. 1 & 2 B. 1 & 3 C. 1 & 5 D. 3 & 4 E. 3 & 5

Interstitial growth is observed at which of the following


sites?
Spheno-occipital syndrondrosis
Maxillary tuberosity
Mandibular condyle
Zygomaticomaxillary suture
Apex of an erupting premolar
A. 1 & 2 B. 1 & 3 C. 1 & 4 D. 2 & 4 E. 2 & 5

When force is applied, bone reflexes that produces deformation


of crystalline materials and allows flow of electric current
necessary for tooth movement:
deposition C. hyalinization
B. piezoelectricity D. undermining resoprtion

Cephalometrics is useful in assessing which of the following


relationships?
tooth-to-tooth
bone-to-bone
tooth-to-bone
A. 1 & 2 B. 1 & 3 C. 2 & 3 D. 1,2 & 3 E. 3 only

Which of the following is the most essential factor related to


correction of an anterior crossbite?
age of the patient
depth of the corssbite
shape of the tooth involved
space available mesiodistally
Frankfort-horizontal is a reference plane constructed by joining
which of the following landmarks?
Porion and sella
Porion and nasion
Porion and orbitale
Nasion and Sella

Which of the following is not a space maintainer?


Lingual arch
Nance holding arch
Class III restoration
Stainless steel crown
Palatal expansion appliance

Space closure is least likely to occur after loss of which of


the following teeth?
Primary mandibular canines
Primary mandibular second molars
Primary maxillary first molars
Primary maxillary central incisors
Permanent maxillary central incisors

Crowding that occurs with mandibular incisors after age 18 is


most often the result of:
orthodontic relapse
periodontal disease
impacted thid molars
a physiologic maturational change

The undesirable side-effect most commonly associated with use of


a finger spring to tip the crown of a tooth is:
pain
gingival irritation
tendency for the tooth to intrude
severe mobility of the tooth
tendency for the root apex to move in the direction oppo-
site from the crown.

After the age 6, the greatest increase in size of the mandible


occurs:
at the symphysis C. along the lower border
between the canines D. distal to the first molars

A distal shoe space maintainer is indicated when a primary:


canine is in crossbite
first molar is prematurely lost
second molar is lost after eruption of a permanent first
molar
second molar is lost prior to eruption of a permanent first
molar

Gonion, menton and pogonion are cephalometric landmarks located


on the
midline C. bony chin
B. mandible D. skeletal profile

A major disadvantage of treatment using cervical headgear is:


impaction of maxillary canines
extrusion of maxillary incisors
extrusion of maxillary molars
potential deformity of the neck
psychologic trauma due to appearance

Asymmetrical anterior open bite with normal posterior occlu-


sion is characteristic of:
thumb-sucking
mouth breathing
abnormal swallowing habits
both A & B
both B & C

If the norm for the cephalometric angle SNA is 82°, a patient’s


reading of 90° for SNA most likely indicates:
maxillary protrusion
the patient’s ethnic background
protrusive maxillary incisors
dysplasia of the anterior cranial base.

Ideally, a malooclusion should be treated between the ages of:


5 and 8 years
8 and 10 years
10 and 12 years
12 and 14 years
None of the above. The age at which a malocclusion is
treated should depend on the problem involved.

Radiographs of a 5-year old patient show permanent maxillary


first molars inclined mesially with resulting resoprtion of
the distal portions of the roots of primary second molars. The
condition described is:
ankylosis
ectopic eruption
premature eruption
internal resorption
intrafollicular resoprtion

Fractured maxillary anterior teeth generally occur most often


in children with which of the following dental conditions?
Class I malocclusion
Class II, Division 1 malocclusion
Class II, Division 2 malocclusion
Class III malocclusion
Marked incisor crowding

When a simple tipping force is applied to the crown of a single-


rooted tooth, the center of rotation is usually located:
at the apex
at the cervical line
5mm beyond the apex
one-third the root length from the apex
two-third the root length from the apex.

Which of the following is the most common orofacial malfor-


mation that produces malocclusion?
Cleft palate
Ectodermal dysplasia
Pierre Robin syndrome
Osteogenesis imperfecta
Cleidocranial dysostosis

The cranial vault increases rapidly in size the first few years
postnatally and completes approximately 90% of its growth by 6
years of age. This growth is typical of which of the following
types of tissues?
neural
dental
genital
lymphoid
general (somatic)

On the first dental visit, the basic fear of a child under the
age of 2 is related to:
fear of an injection
anxiety over being separated from a parent
anxiety regarding the instruments used to perform dental
treatment
not understanding the reason for dental treatment.

A child with Down’s syndrome is characterized as being:


affectionate
fearful of quick movements
capable of learning dental procedures
both A & B
E. all of these

A 3 ½ year old child has an acute fever, diarrhea, oral vesicu-


lar lesions and gingival tenderness. The most likely diagnosis
is:
thrush
drug allergy
aphthous ulcerations
acute herpetic stomatitis
necrotizing ulcerative gingivitis

Transillumination of soft tissues is useful in detecting which


of the following problems in a child?
Koplick’s spot
Sialolithiasis
Aortic stenosis
Sickle cell disease
abnormal frenum attachment

Which of the following is least likely to result from persis-


tent long-term thumb-sucking?
a deep overbite
protrusion of maxillary incisor
constriction of maxillary arch
rotation of maxillary lateral incisor
lingual inclination of mandibular incisors

The least desirable method used in child management is:


tell-show do technique,
voice control,
gift before treatment,
hand-over-mouth technique.

A child who is reluctant to accept dental treatment ,exhibit


negative attitude but not so pronounced is classified under
Frankl ?
I B. II C. III D. IV
The amount of pulp tissue to be removed in formocresol pulpotomy
procedure
half of the coronal pulp
coronal pulp tissue down to the cervical constriction of
each root
canal
C. infected portion of coronal pulp tissue
D. total removal of pulp tissue including radicular
pulp

Which of the following dental sequelae is likely in a child


with a history of generalized growth failure (failure to
thrive) in the first six months of life?
Retrusive maxilla
Enamel hypoplasia
Retrusive mandible
Small permanent teeth
Dentinogenesis imperfecta

In festooning and trimming a stainless steel crown, greater


length is necessary in the region of the mesiofacial bulge in
a primary:
first molar
second molar
maxillary canine
maxillary lateral incisor

Using a topical fluoride rinse before acid etch direct bonding


of orthodontic brackets is contraindicated because fluoride:
decreases the solubility of enamel
increases the pH of the etching agent
causes copious amounts of saliva
directly reacts chemically with the bonding agent.

If a 7-year old patient loses a primary mandibular canine about


the same time the adjacent lateral incisor is erupting or
shortly thereafter, the dentist should be alert to the possi-
bility of:
a tongue habit
a developing crossbite
an early eruption of the permanent canine
lingual collapse of mandibular anterior teeth

The average age at which calcification of crowns of permanent


central incisors is completed is:
birth
2-3 years of age
4-5 years of age
6-7 years of age

Amalgam is most often the restorative material of choice for


primary teeth. The most important modification in its use for
children is in:
cavity preparation
use of a rubber dam
use of a zinc-free alloy
condensation of the alloy
placement of matrix bands

The most frequent cause of fracture of a root tip during ex-


traction of a primary molar is:
ankylosis of the tooth
improper use of cowhorn forceps
presence of a supernumerary premolar
root resorption between the apex and the bifurcation
asymmetric root resorption in which only one root is com-
pletely resorbed.

The most common cause of sinus tracts in gingival tissues of


children is:
pericementitis
periapical cyst
acute periapical abscess
chronic periapical abscess

A light bluish, dome-shaped lesion on the inside lip of a 2-year


old child is most likely a:
mucocele
melanoma
hematoma
hemangioma
sucking callous

Primary second molars usually erupt during ages:


8-14 months
14-20 months
20-30 months
30-36 months
36-48 months

A child in long-term remission of acute leukemia has dental


problems characterized by unusual susceptibility to:
dental caries
oral infection
periodontal bone loss
development of jaw deformities
all of the above.

Supervision of a child’s development of occlusion is most crit-


ical at ages:
3-6 years
7-10 years
11-14 years
14-17 years

A 4-year old child has a traumatized central incisor with a


Class III (Ellis) fracture. The injury occurred about one
month ago, and examination indicates that the pulp is ne-
crotic. There are no other pathologic findings. Treatment of
choice is:
watchful observation
extraction and use of a space maintainer
pulpectomy and root canal filling using gutta-percha points
and cement
endodontic treatment and root canal filling with a resorba-
ble paste.

A child with congenital heart disease requires special treat-


ment planning for dental care because of potential problems
with:
bleeding
local infection
systemic infection
enamel hypoplasia
A. 1 , 2 & 3 B. 1 & 3 C. 1 only D. 3 only E.
1,2,3 & 4

Which of the ff. approaches is best for a child suffering from


celebral palsy
HOM exercise
towel method
use of sedation
psychological approaches

Type of fear which is produced by direct physical stimulation of


sense organ
associative fear
objective fear
subjective fear
acquired fear

Agent used to reduce pain without affecting consciousness


hypnotic drug
analgesic
barbiturate
general anesthesia

The drug that modifies fear and anxiety


A. flanax B. ethamizid C. celestamine
D. seconal

A general characteristic of a 12 year old child


mother as the center of his world
exhibits tantrums
separation anxiety
rejects parental authority

This is a psychological approach to manage a child


nitrous oxide inhalation
giving diazepam
mutisensory techniques
HOM

This is a contraindication of HOM


spoiled child
very young patient
defiant child
uncooperative behavior but capable of understanding

Induce formation of apical closure of young permanent molar us-


ing CAOH
apexogenesis C. apexification
B. apeximation D. apex formation

The growth movement of the mandible is complimented by the


growth of the
maxilla, which is:
down and forward C. downward and backward
B. forward only, D. upward and backward

Direction of displacement of the mandible in an individual with


developing Class II malocclusion:
down and forward, C. down and backward,
B. forward only, D. forward and upward,
E. upward and backward.

Child refuses to accept treatment or open the mount minimally:


A. Frankl 1 B. Frankl 2 C. Frankl 3 D. Frankl 4

Definitely negative behavior.


A. Frankl 1 B. Frankl 2 C. Frankl 3 D. Frankl 4

Child has good rapport with the dentist.


A. Frankl 1 B. Frankl 2 C. Frankl 3
D. Frankl 4.

The treatment option contraindicated in patients who are not


able to breathe nasally.
no treatment C. nitrous oxide & oxygen inhalation
B. conscious sedation D. genera anesthesia

The treatment option for patients who have sustained extensive


orofacial & or dental trauma
no treatment C. nitrous oxide & oxygen inhalation
B. conscious sedation D. general anesthesia

In infant oral care, clean mouth with gauze after feedings and
at bed time done during:
A. 0-6 mons B. 6-12 mons C. 12-24 mons. D. 24-
36 mons

Space differential between combine width of CDE and 345 is__


A. positive B . negative C. zero D. 2.2
mm E. 2.4 mm

Which of the developmental space will cause a decrease in arch


perimeter when pre-empted?
primate space C. interocclusal space
B, interdental space D. leeway space

This is an increase in size by expansion from within


sutural apposition C. interstitial growth
maturation D. development

This is growth movement which mean movement of whole bone as a


unit
A. drift B. displacement C. deposition D. reloca-
tion E. remodeling

Mandible comes from ? branchial arch


first B. second C. third D. fourth

Maxilla is formed from the maxillary processes and ?


medial nasal process C. globular process
B. lateral processes D. tuberculum impar
In young child, paradoxical excitement occurs most frequently
following premedication with:
a narcotic
a barbiturate
nitrous oxide
an amphetamine

The maximum amount of anesthesia that can be given to a 36-pound


child is:
4 carpules
3 carpules
2 carpules
1 carpule

Dental Fluorosis is most likely to occur on a:


two-year-old child ingesting 1 ppm of fluoride
17-year-old patient ingesting 1 ppm of fluoride
on a 3-year-old child ingesting 0.50 ppm of fluoride
D. none of the above.

Pulpotomy with formocresol of primary tooth:


result in lower success rate compared to calcium hydroxide,
result in higher success rate compared to calcium hydroxide,
should never be used,
induces formation of a dentin bridge at site of amputation,
none of the above.

A 4-year-old child visited a dentist for the first time and received
prophylaxis
quite well. Before he left, the dentist was glad that he
gave the child a toy.
This kind of gesture is called:
classical conditioning
positive reinforcement,
bribery,
public relations
none of the above.

If a primary tooth has been extracted before succedaneous tooth has


begun eruptive movement, its eruption will be __
hastened B. delayed C. not affected

ORTHODONTICS AND PEDIATRIC DENTRISTRY


Shade AE if the answer is not in the choices given.
Fractured maxillary anterior teeth generally occur most often in
children with which of the following dental conditions?
Class I malocclusion D. Class II, Division 1 malocclusion
Class II, Division 2 malocclusion E. Class III malocclusion
Marked incisor crowding

When a simple tipping force is applied to the crown of a single-


rooted tooth, the center of rotation is usually located:
at the apex D. at the cervical line
5mm beyond the apex E. one-third the root length from the apex
two-third the root length from the apex.

Which of the following is the most common orofacial malfor-


mation that produces malocclusion?
Cleft palate C. Ectodermal dysplasia
Pierre Robin syndrome D. Osteogenesis imperfect E. Cleidocranial dysos-
tosis

The cranial vault increases rapidly in size the first few years
postnatally and completes approximately 90% of its growth by 6
years of age. This growth is typical of which of the following
types of tissues?
Neural B. dental C. genital D. lymphoid E. general (somatic)

On the first dental visit, the basic fear of a child under the
age of 2 is related to:
fear of an injection
anxiety over being separated from a parent
anxiety regarding the instruments used to perform dental treatment
not understanding the reason for dental treatment.

The maximum amount of anesthesia that can be given to a 36-pound


child is:
4 carpules B. 3 carpules C. 2 carpules D. 1 carpule

Dental Fluorosis is most likely to occur on a:


two-year-old child ingesting 1 ppm of fluoride
17-year-old patient ingesting 1 ppm of fluoride
on a 3-year-old child ingesting 0.50 ppm of fluoride
D. none of the above.

Pulpotomy with formocresol of primary tooth:


result in lower success rate compared to calcium hydroxide,
result in higher success rate compared to calcium hydroxide,
should never be used,
induces formation of a dentin bridge at site of amputation,
none of the above.

A 4-year-old child visited a dentist for the first time and


received prophylaxis
quite well. Before he left, the dentist was glad
that he gave the child a toy.
This kind of gesture is called:
A. classical conditioning C. positive reinforcement,
B. bribery, D. public relations E.
none of the above.

If a primary tooth has been extracted before succedaneous tooth


has begun eruptive movement, its eruption will be __
hastened B. delayed C. not affected

The most common cause of sinus tracts in gingival tissues of


children is:
Pericementitis C. periapical cyst
acute periapical abscess D. chronic periapical abscess

The most frequent cause of fracture of a root tip during extrac-


tion of a primary molar is:
ankylosis of the tooth
improper use of cowhorn forceps
presence of a supernumerary premolar
root resorption between the apex and the bifurcation
asymmetric root resorption in which only one root is completely resorbed.

A light bluish, dome-shaped lesion on the inside lip of a 2-year


old child is most likely a:
Mucocele B. melanoma C. hematoma D. hemangioma E. sucking callous

Primary second molars usually erupt during ages:


8-14 months C. 14-20 months
20-30 months D. 30-36 months E. 36-48 months

A child in long-term remission of acute leukemia has dental


problems characterized by unusual susceptibility to:
dental caries C. oral infection
periodontal bone loss D. development of jaw deformities E. all of the
above.

Agent used to reduce pain without affecting consciousness


hypnotic drug B. analgesic C. barbiturate D. gen-
eral anesthesia

The drug that modifies fear and anxiety


A. flanax B. ethamizid C. celestamine
D. seconal

A general characteristic of a 12 year old child


mother as the center of his world C. exhibits tantrums
separation anxiety D. rejects parental authority

This is a psychological approach to manage a child


nitrous oxide inhalation C. giving diazepam
mutisensory techniques D. HOM

This is a contraindication of HOM


spoiled child C. very young patient
defiant child D. uncooperative behavior but capable of under-
standing

Child has good rapport with the dentist.


A. Frankl 1 B. Frankl 2 C. Frankl 3
D. Frankl 4.

The treatment option contraindicated in patients who are not


able to breathe nasally.
no treatment C. nitrous oxide & oxygen inhalation
B. conscious sedation D. genera anesthesia

The treatment option for patients who have sustained extensive


orofacial & or dental trauma
no treatment C. nitrous oxide & oxygen inhalation
B. conscious sedation D. general anesthesia

In infant oral care, clean mouth with gauze after feedings and
at bed time done during:
A. 0-6 mons B. 6-12 mons C. 12-24 mons.
D. 24-36 mons

Which of the developmental space will cause a decrease in arch


perimeter when pre-empted?
primate space C. interocclusal space
B, interdental space D. leeway space

This is an increase in size by expansion from within


sutural apposition C. interstitial growth
maturation D. development

This is growth movement which mean movement of whole bone as a


unit
A. drift B. displacement C. deposition D.
relocation E. remodeling

The normal downward and forward direction of facial growth re-


sults from
upward and backward growth of maxillary sutures and the
mandibular condyle
vertical eruption and mesial drift of the dentitions
interstitial growth in the maxilla and the mandible
epithelial induction at growth centers
A. 1 & 2 B. 1 & 3 C. 1 & 4 D. 2 & 3 E. 2 &
4

An 8-year-old girl has 2mm diastema between permanent maxillary


central incisors. Permanent maxillary lateral incisors are in
position. The diastema is probably the result of:
thumb-sucking
an abnormal frenum attachment
a normal developmental process
A. 1 & 2 B. 1 & 3 C. 2 & 3 D. 2 only E. 3 only

When comparing cementum to bone in their responses to orthodon-


tic forces, cementum resorbs:
more readily C. not at all
B. less readily D. under lighter loads E. by
indirect (undermining) resoprtion

The tooth in the mandibular arch most likely to be malposed in


cases of arch space discrepancy is the:
first molar B. second molar C. first premalor D. second pre-
molar

Generalized osteoclastic activity along the walls of the alveo-


lar socket is the bone response to:
depressing force C. extrusion
B. elongating force D. rotating force E.
both B & D

The major source of anchorage in a maxillary Hawley appliance


is the :
molar clasp C. acrylic portion
B. labial bow D. interproximal clasp E.
bracket of the band

A narrow maxillary arch with respect to midsagittal plane is


said to be in:
Protraction C. contraction
B. retraction D. distraction E. both C & D

A distal shoe space maintainer is indicated when a primary:


incisor is avulsed
first molar is prematurely lost
second molar is lost after eruption of a permanent first molar
second molar is lost before eruption of a permanent first molar

Complete unilateral lip-jaw palate cleft.


A. Class I B. Class II C. Class III
D. Class IV

Space differential between combine width of CDE and 345 is__


A. positive B . negative C. zero D. 2.2
mm E. 2.4 mm

The least desirable method used in child management is:


tell-show do technique, C. voice control,
gift before treatment, D. hand-over-mouth technique.

In festooning and trimming a stainless steel crown, greater


length is necessary in the region of the mesiofacial bulge in
a primary:
first molar B. second molar C. maxillary canine D. maxillary lateral in-
cisor

Which of the following conditions is usually present in a Class


II, Division 2 malocclussion?
Open bite C. Mesiocclusion of permanent first
molars
Steep mandibular plane D. Lingual inclination of maxillary
central incisors

Severely crowded mandibular incisors usually result from:


mesial migration of teeth C. premature loss of primary teeth
presence of supernumerary teeth D. tooth size-arch length discrepancy

Excessive orthodontic force used to move a tooth may:


cause hyalinization
cause undermining resorption
crush the periodontal ligament
A. 1 & 2 B. 2 & 3 C. 1 & 3 D. 1, 2 & 3 E. 3 only

How will extraction of a primary maxillary central incisor in a


5-year old child with incisal spacingaffect the size of the
intercanine space?
The intercanine space will increase in size.
The intercanine space will decrease in size.
No change will occur in the size of the intercanine space.

Orthodontic correction of which of the following is most easily


retained?
Diastema C. Expansion
B. Rotation D. Posterior crossbite E. Anterior
crossbite

A bimaxillary protrusion occurs when:


large horizontal overlap with biteral crossbite exits
the maxillary dentition is forward with respect to basal bone
both dental arches are forward with respect to basal bone
D. none of the choices.

The following are the conditions found on the area of pressure


in the presence of heavy forces:
area of hyalinization
occlusion of blood vessels
stretched periodontal fibers
undermining resoprtion
A. 1, 2& 3 B. 1, 2 & 4 C. 2 , 3 & 4 D. 3 only
E. 1 & 4 only

Anodontia, diagnosed in a 5-year-old child, primarily affects


the growth of the:
Midface C. mandible
B. maxilla D. alveolar bone E. maxilla
& mandible

Which of the following are likely to occur during orthodontic


therapy?
gingival irritation
cementum resorption
increased mobility of teeth
demineralization of enamel adjacent to appliances in pa-
tients with poor oral hygiene
1, 3 & 4 B. 1 & 4 only C. 2 & 4 only D. 1, 2, 3 & 4 E. 3 & 4 only

Slow progress in molar uprighting in an adult patient is usally


due to:
overextended bands D. an overcontoured spring
lack of anchorage control E. the occlusion not being relieved
the stabilizing wire not being passive

The “V” principle of growth is best illustrated by the:


nasal spetum C. mandibular symphysis
B. mandibular ramus D. spheno-occipital synchondro-
sis

Anterior crossbite in the primary dentition usually indicates a


developing:
Class I malocclusion C. Class II malocclusion
Class III malocclusion D. Class IV malocclusion E. none of the above

A mixed dentition analysis determines:


intercanine width D. skeletal growth pattern
discrepancies in jaw size E. size of permanent incisors
space available versus space required

A primary maxillary anterior tooth in a 4-year-old child was


traumatically intruded into the tissues so that only half the
tooth is visible. The most appropriate treatment is to:
extract the tooth.
perform a pulpotomy
administer no treatment
place orthodontic bands on adjacent teeth and draw the tooth down with elastics

A child who is reluctant to accept dental treatment ,exhibit


negative attitude but not so pronounced is classified under
Frankl ?
I B. II C. III D. IV

The amount of pulp tissue to be removed in formocresol pulpotomy


procedure
half of the coronal pulp
coronal pulp tissue down to the cervical constriction of each root canal
C. infected portion of coronal pulp tissue
D. total removal of pulp tissue including radicular
pulp
Which of the following dental sequelae is likely in a child
with a history of generalized growth failure (failure to
thrive) in the first six months of life?
Retrusive maxilla C. Enamel hypoplasia
Retrusive mandible D. Small permanent teeth E. Dentinogenesis im-
perfecta

Which of the following orthodontic movements of teeth are most


difficult to accomplish?
Tipping
Rotation
Intrusion
Extrusion
Translation
A. 1 & 2 B. 1 & 3 C. 1 & 5 D. 3 & 4 E. 3 & 5

Interstitial growth is observed at which of the following


sites?
Spheno-occipital syndrondrosis
Maxillary tuberosity
Mandibular condyle
Zygomaticomaxillary suture
Apex of an erupting premolar
A. 1 & 2 B. 1 & 3 C. 1 & 4 D. 2 & 4 E. 2 & 5

When force is applied, bone reflexes that produces deformation


of crystalline materials and allows flow of electric current
necessary for tooth movement:
deposition C. hyalinization
B. piezoelectricity D. undermining resoprtion

Cephalometrics is useful in assessing which of the following


relationships?
tooth-to-tooth
bone-to-bone
tooth-to-bone
A. 1 & 2 B. 1 & 3 C. 2 & 3 D. 1,2 & 3 E. 3 only

Which of the following is the most essential factor related to


correction of an anterior crossbite?
age of the patient
depth of the corssbite
shape of the tooth involved
space available mesiodistally

Crowding that occurs with mandibular incisors after age 18 is


most often the result of:
orthodontic relapse C. periodontal disease
impacted thid molars D. a physiologic maturational change
The undesirable side-effect most commonly associated with use of
a finger spring to tip the crown of a tooth is:
Pain D. gingival irritation
tendency for the tooth to intrude E. severe mobility of the tooth
tendency for the root apex to move in the direction opposite from the crown.

After the age 6, the greatest increase in size of the mandible


occurs:
at the symphysis C. along the lower border
between the canines D. distal to the first molars

A distal shoe space maintainer is indicated when a primary:


canine is in crossbite
first molar is prematurely lost
second molar is lost after eruption of a permanent first molar
second molar is lost prior to eruption of a permanent first molar

If the norm for the cephalometric angle SNA is 82°, a patient’s


reading of 90° for SNA most likely indicates:
maxillary protrusion C. the patient’s ethnic background
protrusive maxillary incisors D. dysplasia of the anterior cranial base.

Ideally, a malooclusion should be treated between the ages of:


5 and 8 years D. 8 and 10 years
10 and 12 years E. 12 and 14 years
None of the above. The age at which a malocclusion is treated should depend
on the problem involved.

Frankfort-horizontal is a reference plane constructed by joining


which of the following landmarks?
Porion and sella C. Porion and nasion
Porion and orbitale D. Nasion and Sella

Which of the following is not a space maintainer?


Lingual arch C. Nance holding arch
Class III restoration D. Stainless steel crown E. Palatal ex-
pansion appliance

Space closure is least likely to occur after loss of which of


the following teeth?
Primary mandibular canines D. Primary mandibular second mo-
lars
Primary maxillary first molars E. Primary maxillary central incisors
Permanent maxillary central incisors

Radiographs of a 5-year old patient show permanent maxillary


first molars inclined mesially with resulting resoprtion of
the distal portions of the roots of primary second molars. The
condition described is:
Ankylosis C. ectopic eruption
premature eruption D. internal resorption E. intrafollicular
resoprtion

A child with Down’s syndrome is characterized as being:


Affectionate C. fearful of quick movements
capable of learning dental procedures D. both A & B E. all of
these

A 3 ½ year old child has an acute fever, diarrhea, oral vesicu-


lar lesions and gingival tenderness. The most likely diagnosis
is:
Thrush C. drug allergy
aphthous ulcerations D. acute herpetic stomatitis E. necrotizing
ulcerative gingivitis

Transillumination of soft tissues is useful in detecting which


of the following problems in a child?
Koplick’s spot C. Sialolithiasis
Aortic stenosis D. Sickle cell disease E. abnormal frenum attach-
ment

Which of the following is least likely to result from persis-


tent long-term thumb-sucking?
a deep overbite D. protrusion of maxillary incisor
constriction of maxillary arch E. rotation of maxillary lateral incisor
lingual inclination of mandibular incisors

A 9-year-old patient has a slightly convex profile and a sus-


pected tooth mass-arch length (circumference) discrepancy. Be-
fore instituting space management procedures, the dentist
should:
complete a space analysis C. complete a cephalometric analysis
seek orthodontic consultation D. all of the above E. Either A or C

Using a topical fluoride rinse before acid etch direct bonding


of orthodontic brackets is contraindicated because fluoride:
decreases the solubility of enamel C. increases the pH of the etching agent
causes copious amounts of saliva D. directly reacts chemically with the bond-
ing agent.

If a 7-year old patient loses a primary mandibular canine about


the same time the adjacent lateral incisor is erupting or
shortly thereafter, the dentist should be alert to the possi-
bility of:
a tongue habit C. a developing crossbite
an early eruption of the permanent canine D. Lingual collapse of mandibular
anterior teeth
The average age at which calcification of crowns of permanent
central incisors is completed is:
Birth B. 2-3 years of age C. 4-5 years of age D. 6-7 years of age

Amalgam is most often the restorative material of choice for


primary teeth. The most important modification in its use for
children is in:
cavity preparation C. use of a rubber dam
use of a zinc-free alloy D. condensation of the alloy E. placement of matrix bands

Supervision of a child’s development of occlusion is most crit-


ical at ages:
3-6 years B. 7-10 years C. 11-14 years D. 14-17 years

A 4-year old child has a traumatized central incisor with a


Class III (Ellis) fracture. The injury occurred about one
month ago, and examination indicates that the pulp is ne-
crotic. There are no other pathologic findings. Treatment of
choice is:
watchful observation
extraction and use of a space maintainer
pulpectomy and root canal filling using gutta-percha points and cement
endodontic treatment and root canal filling with a resorbable paste.

A child with congenital heart disease requires special treat-


ment planning for dental care because of potential problems
with:
bleeding
local infection
systemic infection
enamel hypoplasia
A. 1 , 2 & 3 B. 1 & 3 C. 1 only D. 3 only E.
1,2,3 & 4

With a flush terminal plane, permanent first molars will:


initially be Class II C. immediately assume a normal relationship
initially be Class III D. erupt immediately into an end-to-end rela-
tionship

Arch shape & symmetry are best evaluated from the:


study cast C. frontal photograph
B. panoramic x-ray D. lateral cephalogram E. in-
traoral photograph

Bodily force that moves the central incisor mesially produces:


stretching of the periodontal fiber on the distal side
compression of the periodontal fiber in the distal side
osteoblastic activity on the mesial side
osteoclastic activity on the distal side

Which of the ff. approaches is best for a child suffering from


celebral palsy
HOM exercise C. towel method
use of sedation D. psychological approaches

Type of fear which is produced by direct physical stimulation of


sense organ
associative fear B. objective fear C. subjective fear D. acquired fear

Induce formation of apical closure of young permanent molar us-


ing CAOH
apexogenesis B. apexification C. apeximation D. apex formation

Gonion, menton and pogonion are cephalometric landmarks located


on the
midline B. mandible C. bony chin D. skeletal profile

A major disadvantage of treatment using cervical headgear is:


impaction of maxillary canines D. extrusion of maxillary incisors
extrusion of maxillary molars E. potential deformity of the neck
psychologic trauma due to appearance

Asymmetrical anterior open bite with normal posterior occlu-


sion is characteristic of:
thumb-sucking C. mouth breathing
abnormal swallowing habits B. both A & B E. both B & C

The growth movement of the mandible is complimented by the


growth of the
maxilla, which is:
down and forward C. downward and backward
B. forward only, D. upward and backward

Direction of displacement of the mandible in an individual with


developing Class II malocclusion:
down and forward, C. down and backward,
B. forward only, D. forward and upward,
E. upward and backward.

Child refuses to accept treatment or open the mount minimally:


A. Frankl 1 B. Frankl 2 C. Frankl 3
D. Frankl 4

Definitely negative behavior.


A. Frankl 1 B. Frankl 2 C. Frankl 3 D. Frankl 4

Mandible comes from ? branchial arch


first B. second C. third D. fourth

Maxilla is formed from the maxillary processes and ?


medial nasal process C. globular process
B. lateral processes D. tuberculum impar

Protrusiveness or retrusiveness of the chin point can be known by


analyzing the:
A. SNB B. FMLA C. IMPA D. FMA E. FH/NP

In young child, paradoxical excitement occurs most frequently


following premedication with:
a narcotic C. a barbiturate
nitrous oxide D. an amphetamine

ORTHODONTICS AND PEDIATRIC DENTISTRY

1. The principal growth site of the cranial base believed to be responsible for its anteropos-
terior growth is?
spheno-occipital C. spheno-ethmoidal
intersphenoidal D. intraoccipital

2. Which of the following is not a source of extra space for the resolution of permanent in-
cisor crowding in the lower arch?
increase in intercanine width
labial positioning of the permanent incisors
distal movement of the canines into the primate space
deposition of bone at the posterior border of the ramus

3. Which of the following factors will not increase maxillary width?


growth of the palate following “V” principle
deposition at the fronto-maxillary suture
deposition at the lateral walls
deposition at the median palatine sutre

4. The major mechanism for growth of the cranial case is the:


expansion of the cartilage cells
interstitial growth of bone
apposition of new bone at the synchondroses
apposition of new bone at the sutures.

5. The vertical lengthening of the maxillary complex is brought about by a composite of


factors, except:
deposition of bone on the oral side of the palate with compensating resopriton on
the entire oral side
deposition on the posterior facing cortical surface of the maxillary tuberosity
deposition on the various sutures where it contacts the other bones above it
6. The main growth site of the mandible which is responsible for its increase ion height?
Condyle C. symphysis
gonial angle D. posterior border of the ramus E. inferior border of the
body

7. The sequence of completion of facial growth by planes of space is?


depth, width, height C. height, depth, width
width, depth, height D. width, height, depth E. depth, height, width

8. The theory which states that the growth of the craniofacial bones is caused by the soft
tissues adjacent to them.
Functional matrix C. Sutural Dominance theory
Cartilaginous growth theory D. Limborgh’s theory

9. The “V” principle of growth is best illustrated by the?


nasal septum
posterior border of the mandibular ramus
mandibular symphysis
intersphenoidal synchondrosis

10. Space for eruption of permanent mandibular molars is created by:


apposition at the anterior border of the ramus
apposition at the alveolar process
resorption at the anterior border of the ramus
resorption at the posterior border of the ramus

11. Displacement of the mandible due to its growth at the condyle and posterior border of
the ramus is?
A. primary B. secondary C. tertiary D. cortical drift

12. Chin cup is used to intercept a?


Class 1 type 3 C. Class II
Class III D. Developing Class III

13. A direct growth movement that is produced by deposition on one side of the orbital
plate with resorption on the opposite side is?
Displacement C. translation
cortical drift D. remodeling

14.. The following are distinctive structural features related to cartilage of the cranial base,
except:
pressure- tolerant C. grows interstitially and appositionally
matrix is non-vascular D. grows appositionally

15. What is the chief factor in the formation of the alveolar process?
normal process of growth
eruption of teeth
lengthening of the condyle
overall growth of the maxilla and mandible

16. Which of the following is not a characteristic of a mature swallow?


active contractions of the muscles of the lips
tongue tip is placed against alveolar process behind the upper incisors
teeth are together during swallowing
both A & B
both A & C

17. These are six soft spots present between the bones of the skull roof.
Sutures B. synchondroses C. fontanelles D. carti-
lages

18. Class I permanent molar relationship can be achieved through the following, except
Late mesial shift after the loss of second primary molar
Greater forward growth of the mandible than the maxilla
Combination of both
None of the above.

19. An inherent disposition of most teeth to drift mesially even before they are in occlusion:
mesial drifting tendency
anterior component of force D. both A & B
physiologic movement of teeth E. both B & C

20. Spaces between the primary anteriors:


interdental space
primate space D. both A & B
Nance leeway space E. both B & C

21. Primate space is found between the:


maxillary canine and first molar
mandibular primary canine and lateral incisor
maxillary canine and lateral incisors
none of the above

22. At what stage in Nolla’s classification does a permanent tooth start to erupt?
Stage 7 C. Stage 6
Stage 5 D. Stage 8 E. Stage 4

23. From the flush terminal plane relationship of molars in the primary dentition, the per-
manent first molar relationship in the permanent dentition can become the following in the
transition period, except:
Class I C. Class III
Class II D. end to end / cusp to cusp E. none of the choices

24. Which of the following are normal signs of primary dentition?


ovoid arch form
deep overjet and overbite
straight terminal plane
primate spaces
A. I, II, III & IV B. II, III & IV C. I, III & IV D. I, II & IV

25. It is the difference in size between the primary teeth and their permanent successor in
the posterior segment:
posterior liability C. Posterior size discrepancy
Nance Leeway space D. Late mesial shift

26. Mouth breather who seldom approximate their lips do muscle exercises. This can be
varied
by. A. playing wind instrument B. blowing exercise C. A & B D. none

27. Child is 10years old. Tooth no. 55 has been exfoliated but tooth no. 65 is still very in-
tact.
What must be done?
A. none, wait for 65 to exfoliate C. do percussion test before extracting 65
B. Extract 65 at once D. take x-ray to evaluate

28. When is the proper time to remove a space maintainer?


crown successor visible clinically and it is in stage 8 radiographically
B. bone barrier has resorbed
C. crown of successor is fully erupted
D. age 12.

29. Two or more tooth moving opposite directions and pitted against each other is equal
and
opposite. The anchorage is.
A. simple B. stationary C. reciprocal D. none.

30. The following are caused by thumbsucking habit Except.


A. anterior open bite B. protrusion of upper incisor C. expansion of upper
arch

31. Mesial surface of E can disked in the case of 3 cannot erupt because of insufficient
space,
This is.
A. contingency of extraction B. occlusal equilibrium C. space regaining D.
none.

32. In a condition characterized by partial or complete absence of clavicle.


A. torticollis B. cerebral palsy C. cleidocranial dysostosis

33. A condition characterized by foreshortening of sternocleidomastoid muscle.


A. torticollis B. cerebral palsy C. cleidocranial dysostosis

34. Type of force recommended for removable orthodontic appliance.


A. continuous B. functional C. dissipating D. intermittent

35. Type of force which can be seen in fixed appliance is.


A. continuous B. functional C. dissipating D. intermittent

36. The ages of bone development for girls is.


A. 7,9,15,18 B. 8,10,12,17 C. 6,7,12,13 D. 7,8,11,12

37. It can detect thyroid disturbances which has an oral manifestation.


A. hand & wrist x-ray B. electromyographic exam C. BMR D. biostatic

38. Osteoclastic activity occurs on the.


A. tension side B. pressure side C. stress D. compression

39. Bone deposition occurs on the. A. tension side B. pressure side C. stress D. com-
pression

40. The tooth is push towards the socket and pulled away from occlusal plane.
A. elongating force B. rotation C. depressing D. heavy force

41. Neutroclusion with labioversion of max centrals & buccoversion of mand. 1st premo-
lar.
A. Class 1 type 2 & 3 B. Class 1 type 2 & 4 C. Class 1 type 2 & 5

42. A supraverted mand. Premolar with respect to Frankfurt horizontal plane is said it be
in.
A. contraction B. abstraction C. Attraction D. distraction

43.. The resorption occurs from behind an immediate site of pressure, there is accumula-
tion of
fluid and blood vessels are occluded.
direct resorption C. undetermining resorption
B. frontal osteoclastic attack D. all of the Above.

44. It refers to perfect arrangement of teeth when jaw’s are closed & condyies are at rest
in the
glenoid fossa.
A. normal occlusion B. ideal occlusion C. malocclusion

45. The incidence of malocclusion is very high in.


homogeneous population B. Heterogeneous population C. Eskimos D. none of the
above

46. The predominant type of malocclusion seen in mixed dentition is.


A. Crowding B. anterior open bite C. Class II Div. 1

47. A brachycephalic individual is associated with___ arch form:


A. square, B. ovoid, C. long and narrow, D. none of the above.

48. Increase in maxillary arch perimeter can be due to the following, except:
labial tipping or eruption of permanent incisors,
diverging increase in the height of alveolar bone,
interproximal wear out of tooth surfaces.

49. Direction of displacement of the mandible in an individual with developing Class


II malocclusion:
down and forward C. down and backward
forward only, D. forward and upward, E. upward and backward.

50. The growth movement of the mandible is complimented by the growth of the
maxilla, which is:
down and forward, C. down and backward,
forward only D. upward and backward.
A type of conditioning where in a specific response is immediately rewarded
Classical conditioning C. operator conditioning
Operant conditioning D. conditioning E. classic conditioning
A type of preventive dentistry where the main concern is to slow down the process of a
disease:
A. Primary prevention C. tertiary prevention
B. Secondary prevention D. All of the Above

Characteristic behavior that is based on belief:


A. Phenomelogical C. Behavioral
B. Neurobiological D. Psychoanalytical E. none

The patients history of childhood diseases are under


A. familial history C. social history
B. medical history D. dental history

The history on the behavior of the child toward dentistry:


A. familial history C. medical history
B. Dental history D. history of present illness

Method of early examination where the dentist and parent are seated face to face
A. Knee position C. leg position
B. cradling position D. Knee to knee position E. all

In pedodontic triangle the apex of the triangle is:


A. Child B. Dentist C. Parent D. Assistant

Parent who are mistrusting are:


A. Overprotective Parent C. Neglectful Parent
B. Manipulative Parent D. Hostile parent

Parent who are demanding and can extend to directing the course of the treatment is a
:
A. Overprotective Parent C. Neglectful Parent
B. Manipulative Parent D. Hostile parent

A parent who postponed the dental treatment of his/her child is


A. Overprotective Parent C. Neglectful Parent
B. Manipulative Parent D. Hostile parent

Panoramic Radiographic Film:


A. 5 “ x 7” B. 2”x 3” C. 5” x 12” D.. 8” x 10”

Radiograph examination for a three years old patient with no apparent abnormalities and
open contact:
A. 2 posterior bitewing C. 4 film series
B. 8 film series D. 12 film Series E. None

Radiographic examination needed for a 6-7 years old with no apparent abnormalities:
A. 2 posterior bitewing C. 4 film series
B. 8 film series D. 12 film Series E. None
Frankl behavior rating characterized by refusal of treatment and extreme negativism:
A. rating 1 B. rating 2 C. rating 3 D. rating 4

A period that child undergoes the phase of separation anxiety:


A. Toddler B. Pre school C. school age D. Adolescent

Stage where one exhibit absolute confidence that they are right about everything, and
their judgement are infallible:
A. Toddler B. Pre school C. school age D. Adolescent

In Nitous oxygen sedation, once a petient takes a distant gaze the concentration of the
nitous oxide- oxygen is reduces to:
A. 30-70 B. 40-60 C. 20-80 D. 10-90 E. none

Device that assess the arterial hemoglobin oxygen saturation and pulse rate:
A. automated vital sign monitor C. pulse oximeter
B. pretracheal stethoscope D. none

Quadrant used in the gluteal region when intramuscular sedation is used:


A. upper inner quadrant C. upper outer quadrant
B. Lower inner quadrant D. Lower outer quadrant

The most unstable form of fluoride used for topical application:


Acidulated Phosphate Fluoride C. Stannous Fluoride
Sodium Fluoride D. None

Retention of PFS is through:


A. Light curing C. Drilling dentin
B. Acid etching D. Application of fluoride

A sudden firm command use to get the child’s attention:


A. Voice control C. Multisensory communication
B. Aversive conditioning D. HOME

Procedure which slowly develops behavior by reinforcing successive approximation of


the desired behavior until desired behavior develop:
A. Pre appointment behavior modification C. behavior shaping
B. Behavior management D. Retraining

Parents should be allowed inside a treatment room when:


A. infant to 41 months C. handicapped patient
B. 7-8 Years old D. A & C E. None

A technique of diverting the attention of the patient from what may be perceived as an
umpleasant procedure:
A. tell show do C. Voice control
B. Positive reinforcement D. Distraction E Non verbal commu-
nication

Plastics applied to the occlusal surface of posterior teeth by simple acid etch method to
prevent dental caries & early carious lesion from developing further
A. Composite C. Fluoride
B. Preventive Restorative Resin D. Glass Ionomer E. None

Medicament used in type A PRR:


A. Glass ionomer C. Bonding agent
B. Composite D. Pit and fissure sealant E. all of these

Medicament used for Type C PRR:


A. Glass ionomer C. Bonding agent
B. Composite D. Pit and fissure sealant E. all of these

Advantage of Rubber dam utilization:


A. Control saliva C. Aids management
B. Provides Protection D. all of these E. None

Distance of punched holes on the rubber dam for primary teeth:


A. 1.0mm C. 2.0mm
B. 1.5 mm D. 2.5 mm E. 3.0mm
Gingival cavosurface beveling is not indicated in Class II cavity preparation for deciduous
teeth, because the direction of the enamel rods are toward:
A. Cervical B. Occlusal C. gingival D. Apical E. all of these

Etching time for deciduous dentition is ----- compared to permanent dentition:


A. shorter B. longer C. the same D. none

The least amount of tooth surface reduction in stainless steel crown preparation is:
A. Occlusal B. lingual C. Proximal D. Buccal E. none

Ellis classification of tooth fracture when the coronal pulp is exposed:


A. Class I B. Class II C. Class III D. Class IV E. Class V

Ellis classification of tooth fracture when a tooth is lost as a result of trauma


A. Class I B. Class II C. Class III D. Class IV E. Class V

Ellis classification of a non vital traumatized tooth:


A. Class I B. Class II C. Class III D. Class IV E. Class V

Treatment of choice for an immature non vital permanent tooth to stimulate root closure:
A. Indirect pulp capping C. Pulpotomy
B. Pulpectomy D. Apexogenesis E. Apexification

Use of formocresol:
A. to stop bleeding C. Stimulate secondary dentin
B. render area of fixation D. Dissolve the pulp tissue E. all of these

Materials that is used to induced apexification:


A. calcium hydroxide C. Formocresol
B. Zinc oxide eugenol D. IRM E. Gutta percha

Treatment of choice for a pulp exposed primary molar except:


A. Direct pulp capping C. pulpotomy
B. Pulpectomy D. all of these E. none of the above
Superficial wound due to scrapping of the mucosa:
A. laceration B. contusion C. abrasion D. Attrition E. Ero-
sion

Hypoplastic enamel is a result in the disturbance at:


A. initiation stage C. Appostion stage
B. Morphodifferentation D. Histodifferentiation E. None

Macrodontia incisor crown with two root canals:


A. Flexion B. Gemination C. Fusion D. Supernumerary root E.
none

Presence of enamel at the furcation of the root:


A. enameloma C. Enamelin
B. ameloblast D. Epstein Pearl E. all of the above

Radiograph picture of a caries is generally--- than the actual caries:


A. smaller B. larger C. the same D. deeper E. B&D

96. Rain coat:


A cotton B. rubber dam C. tooth paint D. tooth camera E. pointer

97. Anesthesia:
A. rubber dam B. tooth paint C. Sleeping juice D. tooth camera E. pointer

98 Explorer:
A. Pointer B. toothbrush C. Tooth paint D. Injection E. none

99. Pit and Fissure sealant:


A. Pointer B. toothbrush C. Tooth paint D. Injection E. none

100 Excavator:
A. Pointer B. toothbrush C. Tooth paint D. Injection E. none

ORTHODONTICS AND PEDIATRIC DENTISTRY


Shade AE if the answer is not found in the given choices.

A supra erupted mand. canine with respect to Frankfurt Horizontal plane


is said to be in?
A. attraction B. Abstraction C. Protraction D. Re-
traction E. Contraction.

A supra erupted max. canine with respect to Frankfurt Horizontal Plane


is said to be in?
A. attraction B. Abstraction C. Protraction D. Retraction
E. Contraction.

A linguoverted max. premolar with respect to Midsaggital plane is said


to be in?
A. attraction B. Abstraction C. Protraction D. Re-
traction E. Contraction.

A linguoverted max. incisor with respect to Orbital Plane is said to be


in?
A. attraction B. Abstraction C. Protraction D. Re-
traction E. Contraction.

Neutroclusion with maxillary anteriors are lingual in relation to the


lower anteriors and there is mesial drifting of molars.
A. Class 1 type 3 and 5 B. Class 1 type 3 and 4 C.
Class 1 type 5 and 4.

The following are vertical planes, except:


Axis of 1 B. v-axis C. facial plane D. palatal

Bone ossification can be detected with the use of:


A, hand and wrist x-ray B. BMR C. electromyographic exam
D. biostatic

The least desirable method used in child management is:


A. tell-show do technique, C. gift before treatment,
B. hand-over-mouth technique. D. voice control

Spoiled children are with parents who are:


A, dominating, B. rejecting, C overindulging,

A mechanical aid for keeping the mouth open and operates on a reverse
scissors action: A.finger guard C. padded and wrapped
tongue blade
B. Molt mouth prop D. rubber bite block E.
Mc Kesson bite block

Child’s arms and legs can be immobilized with:


A. forearm-body support B. plastic bowl C. Posey straps
D. head positioner

To avoid vomiting and complications during treatment with sedation, no


milk or solid food should be taken___ before the scheduled procedure.
A. 4 hrs. B. 6 hrs. C. 8 hrs. D. after midnight

For sedation via intramuscular route, the ___ of the gluteal region is
the safest. A. upper, inner B. up-
per outer C. lower inner D. lower outer

Stabilization period for teeth with fractured roots:


A. 2-3 months B. 2-3 days C. 2-3 weeks
D. 7-14 days

The correct angulation of the inclined plane in relation to the tooth


in crossbite is: A. . 15º B. 25º C. 35º D. 45º E. 55º

The following are sequelae of untreated crossbite, except:


A. faceting on the labial surface of the tooth in crossbite
B. faceting of the labial surface of the tooth opposing the inlocked
tooth
C. abrasion
D. periodontal involvement.
The appliance to be used in correcting the anterior crossbite is deter-
mined by the:
A. amount of overbite B. age of the patient C. coopera-
tion of the patient D. all of these

Inclined planes should not be left in the mouth for more than 2 months
to prevent creation of:
A. anterior open bite B. posterior open bite C. anterior cross
bite D. posterior cross bite.

The best appliance for 7 years old child with Class 1 Type 3 (inlocked
maxillary central incisor) is A. cross bite elastics C. myo-
functional appliance
B. band and crib D. mandibular acrylic inclined plane.

Appliance for correcting Class 1 deep bite in a growing child.


A. tongue depressor B. band and crib C. inclined plane
D. maxillary bite planes.

The overbite reduction achieved by the bite plane is due largely to:
A. intrusion of posterior teeth C. overeruption of pos-
terior teeth
B. mesial drifting of posterior teeth D. buccal and lingual
movement of posterior teeth.

In adult patient with excessive overbite the lower facial height in


relation to the upper and lower middle thirds is:
A. normal B. markedly short C. markedly long.

In deep bite the postural vertical dimension is/has:


A. in harmony with occlusal vertical dimension
B. not in harmony with occlusal vertical dimension
C. no relations with occlusal vertical dimension

Pulpotomy with formocresol of primary tooth:


A. result in lower success rate compared to calcium hydroxide,
B. result in higher success rate compared to calcium hydroxide,
C. should never be used,
D. induces formation of a dentin bridge at site of amputation,

The prognosis of a cervical third root fracture:


A is not favorable, B. is favorable, C. depends on
whether tooth is discolored

The most important predisposing factor to injury of the anterior teeth


is:
A. rotated anterior teeth, B. protruding anterior teeth
C. crossbite of anterior teeth,
The ultimate objective of pulp capping is:
A. to preserve the pulp and odontoblast so that secondary dentin
will be deposited
B. to prevent bacterial contamination, C. to pre-
vent pain,
The purpose of pulp treatment and root canal filling is to:
A. remove pulpal tissue,
B. prolong the usefulness of a tooth to function in mastication,
C. enlarge the root canal,

Stage of Nolla’s clacification wherein tooth begins eruptive movement:


2 B. 5 C. 6 D. 7 E. 10.

Which of the facial radiographs is best used in assessing whether the


patient is gummy or not? A. frontal view with lips repose B. lat-
eral view C. smiling photograph.

Lip profile is influenced more by:


A. lip size B. lip tonicity C. lip shape D. deposition
of upper incisors

The profile of the patient with protruded mandible and retruded maxilla
is:
A. straight B. convex C. concave.

Which of the following cannot be assessed from cephalometric radio-


graphs?
A. mandibular retrusion C. direction of mandibular growth
B. incisor inclination D. adequacy of dental arch pe-
rimeter.

Treatment for crown fracture involving the pulp of a primary incisor:


A. DPC B. formocresol pulpotomy C. pulpectomy D. extraction

The dental procedure that produces the greatest negative response in


children:
A. extraction B. injection C. cavity preparation
D. all of these

In inferior nerve block for a child patient, the injection must be


made__ than for an adult patient. A. slightly higher B. in
level C. slightly lower and more posteriorly.

The maximum number of cartridges of 2% lidocaine with 1:100,000 epi-


nephrine for a 40 pound child patient is:
A. 3 B. 5 C. 7 D. 9 E. 10.

In inferior alveolar nerve blocked, the barrel of the syringe should be


directed on the plane:
A. between C & D B. between D & E C. between E
& 6 D. on top of D.

Indicated restoration on a primary anterior tooth with small mesial and


distal caries and a cervical caries but without pulp involvement:
A. stainless steel crown, B. composite resin C.
strip-off crown,

In pulpectomy of primary teeth, filling of root canal should be:


A. 0.5 - 1 mm above the radiographic apex,
B. 2 - 3 mm above the radiographic apex, C. 4 - 5 mm above the
radiographic apex,

The type of fear that is based on the feelings and attitude that have
been suggested to the child by others without having had the expe-
rience:
A. objective fear B. subjective fear C. BOTH

On the first dental visit, the basic fear of the children below 2
years old is concerned with.
A. injections C.. anxiety & being
separated from parents
B. instruments used in dental treatment D. not understanding
the reason for treatment.

Giving gifts to children.


A, should promised to the child for every appointment to ensure co-
operation.
B, should be considered as a token for friendship
C, both A&B
D, Both B&C.

The arrangement of 2 forces of equal magnitude and opposite but noncol-


linear lines of action:
A. center of rotation B. couple C. moment D.
center of resistance

When one side of the arch is intact and there are several primary teeth
missing on the other side, use:
A. a Nance lingual arch C. a distal shoe
B. a transpalatal arch D. a partial denture space maintainer E.a
soldered fixed lingual arch

Choice of nursing nipple to prevent oral habit formation :


A. Evenflo B. Mimiflo C. Nuk Sauger nipple D. non-
physiologic nipple

Mesial surface of E can be disked in case the 3 cannot erupt because of


insufficient space. This is:
A. contingency of extraction B. occlusal equilibration C. space
regaining D. observation

In managing abnormal oral habits, therapy must be:


A. mechanical C. of physical control
B. of conditioning responses D. a means of punishment

Sudden and firm commands used to get the child’s attention:


A. HOME B. Tell, show and do C. voice control D.
Hypnodontics

Uses the principles of learning theory:


A. behavior modification B. behavior management C.
behavior shaping

Poor personal experiences in the dental office:


A. overprotective parents C. hostile parents
B. neglectful parents D. manipulative parents.
Reluctant to accept treatment:
A. definitely negative B. negative C.positive
D. definitely positive.

They have minimal apprehensions and are reasonably relaxed:


A. enthusiastic B. cooperative C. timid D. defiant.

In treating a pediatric patient, one must establish good communication


with the
A. child only, since he is the one to be treated
B. parents only, because they are the ones who will pay the bill
C. the caretaker or the yaya who takes care of the child
D. both parent and child

The use of nitrous oxide is classified as a


A. physical restraint B. pharmacologic approach C. psy-
chological approach
Guiding of behavior through eye contact, posture and facial expres-
sion.
A. distraction C. positive reinforcement
B. non-verbal communication D. HOM exercise
E. TSD

Partial or complete immobilization of the patient to protect him from


injury while providing dental care.
A. conscious sedation C. positive reinforcement
B. HOM exercise D. physical restraint E. nitrous oxide
and oxygen inhalation

The following are factors that will compensate incisor liability, ex-
cept:
A. intercanine width growth C. labial posi-
tioning of permanent incisors
C. upright position of primary incisors D. favorable tooth
size ratio

Crowding of permanent posterior teeth may be due to:


A. mesial drifting of permanent first molar C. crowding of an-
terior teeth
B. labioversion of anterior teeth D. spacing of ante-
rior teeth

This will decrease lower arch perimeter during transitional period:


A. late mesial shift of first permanent molar
B. distal tipping of lower cuspid C. labial position
of permanent incisors

As arch perimeter increase, arch length?


A. increases B. decreases C. no change.

This is measured from the distal of second primary molar to distal of


second primary molar on the other side following the contour of the
arch.
A. arch width B. intercanine width C. arch perimeter
D. arch length
Which of the developmental space will cause a decrease in arch perimeter
when pre-empted?
A. primate space B. interdental space C. leeway space D.
inter-occlusal space

Upper arch width increases significantly more than that of lower arch
due to?
A. diverging alveolar growth C. vertical alveolar growth
B. labial positioning of permanent incisors D. distal tipping
of cuspid

Point of injection for mandibular blocking in pediatric patient is ___


the occlusal plane.
A. above, B. below or at the level, C. higher than,

An incorrigible 4-year-old child who keeps on kicking and throwing


objects in the clinic can be managed by:
A. tell-show-do technique, C. physical restraint
and conscious sedation
B. general anesthesia, D. all of these

If a child complains of pain on a direct pulp capped tooth, the first


treatment option should be:
A. indirect pulp capping, B. pulpotomy, C. pulpec-
tomy, D. extraction.

The following irrigating solution can be used as irrigant in pulpec-


tomy procedure except:
A. normal saline solution, B. sodium hypochlorite and water
solution,
B. distilled water, D. anesthetic solution,

The first dental appointment of a child patient should be:


A. before 1 year old, B. 2 1/2 to 3 years old, C. 6 years
old

Least reduction in tooth preparation for posterior SSC:


A. buccal B. lingual C. occlusal D. proximal surfa-
ces.

Occlusal reduction for SSc :


A. 0.25-0.5mm B. 1-1.5mm C. 2-3mm D. 5mm.

Grinding of the sharp edges is sufficient.


A. lateral luxation B. dentin fracture C. enamel fracture
D. extrusion.

Restore with bonding agent and composite


A. extrusion C. dentin fracture
B. intrusion D. enamel fracture E. enamel-dentin
fracture.

In acute ingestion of fluoride, the following can be given to the pa-


tient to counteract its effect, except:
A. 2 tsp of ipecac syrup B. milk C. milk of magnesia
D. alum
Gingival margin of a tooth that will receive a stainless steel crown
should have a ___finish line:
A. shoulder B. feather edge C. BOTH

Chewable fluoride tablets:


A. systemic B. local C. BOTH

.Mandibular teeth are best anesthetized with:


A. infraorbital injection C. mental injec-
tion
B. an inferior alveolar and long buccal injection C. both B &
C

Pulpotomy with calcium hydroxide in the primary teeth:


A. may form calcific barrier on the pulp stump
B. result in acute inflammation of pulp
C. result in necrosis of pulp

Lack of anatomic detail of dental restoration produces:


A. increased arch length B. elongation of adjacent
teeth
B. elongation of opposing teeth D. A and B

Child is 11 years old. Tooth # 55 has been exfoliated but tooth # 65 is


still very intact. What must be done?
A. none, wait for 65 to exfoliate C. do percussion test
before extracting 65
B. extract 65 at once D. retain 65 and place a band and
loop for 55

Two or more teeth moving in opposite directions and pitted against each
other by the appliance. Usually, the resistance to each other is
equal and opposite. The anchorage is:
A. simple B. stationary C. reciprocal D. extraoral

The following are sources of extraoral anchorage except:


A. cervical B. occipital C.cranial D. fa-
cial E. mucosa

The order from greatest to least change of the dimensions of the cranium:
A. height, depth, width C. depth, width, height
B. depth, height , width D. width, height, depth

At age 4-5, what normal sign of primary dentition augers well for the
erupting permanent incisors in terms of space availability:
A. Class I cuspid relationship C. growth spaces, inter-
dental spaces
B. upright vertical incisor relationship D. flush terminal
plane

The basic form of the arch is determined:


A. by environmental factors B. by muscles
B. in intrauterine life D. two of the choices E. all
of these
The ffg are true regarding the deciduous maxillary central incisor,
except:
A. MD diameter of crown greater than its cervicoincisal length
B. developmental lines are usually not evident
C. well-developed marginal ridges on the lingual
D. root shape is conical

To avoid ingestion of lethal dose of fluoride, it is safe to dispense


up to __ of fluoride.
A. 120mg B. 300 mg C. 500 mg. D. 700 mg.

Medium-size punch hole is used for:


A. primary molars C. permanent molars
B. maxillary permanent incisors D. lower permanent incisors.

Effect of abnormal eruptive path:


A. space loss C. delayed eruption E.
deflection of eruption
B. increase in arch length D. elongation of adjacent
teeth

Proximal caries, if not restored, will: ... the arch length


A. increase B. decrease C. not affect

Lack of anatomic detail of dental restoration produces:


A. increased arch length C. elongation of adjacent
teeth
B. elongation of opposing teeth D. A and B

Trident factor of oral habit which is now considered as the most impt.
contributory factor to the dev’t of malocclusion:
A. duration B. frequency C. intensity

The ff. are true regarding thumb sucking EXCEPT:


A. it is a learned habit, not a symptom of a deeper emotional dis-
turbance
B. for the first 3 years, damage to occlusion is confined to the
anterior segment and is usually temporary
C. normal from 1-1 1/2 then disappears at age 2 with proper attention
to nursing
D. habit beyond age 4 can be controlled by badgering attempts of
parents to break the habit

Growth of maxilla follows the__ growth curve.


A. general / bodily B. lymphoid C. neural D. genital

At age 5-10, the mandible is__ completed.


A. 45% B. 40% C. 65% D. 96%

Lack of anatomic detail of dental restoration produces:


A. increased arch length C. elongation of adjacent
teeth
B. elongation of opposing teeth D. A and B

At age 4-5, what normal sign of primary dentition augers well for the
erupting permanent incisors in terms of space availability:
A. Class 1 cuspid relationship C. growth spaces, interdental
spaces
B. upright vertical incisor relationship D. flush terminal plane.

The basic form of the arch is determined:


A. by environmental factors B. by muscles C. in intrauter-
ine life D. two of the choices

This aims to recognize and eliminate potential irregularities and mal-


positions in the developing craniofacial complex:
A. Preventive Orthodontics C. Interceptive Ortho-
dontics
B. Limited Corrective Orthodontics D. Extensive Corrective
Orthodontics

The action taken to preserve the integrity of what appears to be normal


occlusion at a specific time:
A. Preventive Orthodontics C. Interceptive Orthodontics
B. Limited Corrective Orthodontics D. Extensive Corrective
Orthodontics

ORTHODONTICS AND PEDIATRIC DENTISTRY

Which of the developmental space will cause a decrease in arch perim-


eter when pre-empted?
primate space B. interocclusal space C. interdental
space D. leeway space

This is an increase in size by expansion from within


sutural apposition C. interstitial growth
maturation D. development

This is growth movement which mean movement of whole bone as a unit


A. drift B. displacement C. deposition D. relocation
E. remodeling

Mandible comes from ? branchial arch


first B. second C. third D. fourth

Maxilla is formed from the maxillary processes and ?


medial nasal process C. globular process
B. lateral processes D. tuberculum impar

In young child, paradoxical excitement occurs most frequently follow-


ing premedication with:
a narcotic B. a barbiturate C. nitrous oxide D. an
amphetamine

The maximum amount of anesthesia that can be given to a 36-pound


child is:
4 carpules B. 3 carpules C. 2 carpules D.
1 carpule

Dental Fluorosis is most likely to occur on a:


two-year-old child ingesting 1 ppm of fluoride
17-year-old patient ingesting 1 ppm of fluoride
on a 3-year-old child ingesting 0.50 ppm of fluoride
D. none of the above.

Pulpotomy with formocresol of primary tooth:


result in lower success rate compared to calcium hydroxide,
result in higher success rate compared to calcium hydroxide,
should never be used,
induces formation of a dentin bridge at site of amputation,
none of the above.

A 4-year-old child visited a dentist for the first time and received
prophylaxis
quite well. Before he left, the dentist was glad that he
gave the child a toy.
This kind of gesture is called:
classical conditioning C. positive reinforcement,
bribery, B. public relations E. none
of the above.

If a primary tooth has been extracted before succedaneous tooth has


begun eruptive movement, its eruption will be __
hastened B. delayed C. not affected

Which of the following conditions is usually present in a Class II,


Division 2 malocclussion?
Open bite
Steep mandibular plane
Mesiocclusion of permanent first molars
Lingual inclination of maxillary central incisors

Excessive orthodontic force used to move a tooth may:


cause hyalinization
cause undermining resorption
crush the periodontal ligament
A. 1 & 2 B. 2 & 3 C. 1 & 3 D. 1, 2 & 3 E. 3 only

Severely crowded mandibular incisors usually result from:


mesial migration of teeth
premature loss of primary teeth
presence of supernumerary teeth
D. tooth size-arch length discrepancy

How will extraction of a primary maxillary central incisor in a 5-year


old child with incisal spacingaffect the size of the intercanine
space?
The intercanine space will increase in size.
The intercanine space will decrease in size.
No change will occur in the size of the intercanine space.

The major criterion to differentiate between a true Class III and a


pseudo-Class III malocclusion is:
degree of anterior crossbite
presence of a bilateral crossbite
C. existence of a forward shift of the mandible during closure
D. occlusal relationship between maxillary and mandibular first
molars

Orthodontic correction of which of the following is most easily re-


tained?
Diastema C. Expansion
B. Rotation D. Posterior crossbite E. Anterior
crossbite

A bimaxillary protrusion occurs when:


large horizontal overlap with biteral crossbite exits
the maxillary dentition is forward with respect to basal bone
both dental arches are forward with respect to basal bone
D. none of the choices.

When a simple tipping force is applied to the crown of a single-rooted


tooth, the center of rotation is usually located:
at the apex D. at the cervical line
5mm beyond the apex E. one-third the root length from the
apex
two-third the root length from the apex.

Which of the following is the most common orofacial malformation that


produces malocclusion?
Cleft palate C. Ectodermal dysplasia
Pierre Robin syndrome D. Osteogenesis imperfecta E. Cleido-
cranial dysostosis

The cranial vault increases rapidly in size the first few years post-
natally and completes approximately 90% of its growth by 6 years of
age. This growth is typical of which of the following types of tis-
sues?
Neural B. dental C. genital D. lymphoid E. general
(somatic)
The following are the conditions found on the area of pressure in the
presence of heavy forces:
area of hyalinization
occlusion of blood vessels
stretched periodontal fibers
undermining resoprtion
A. 1, 2& 3 B. 1, 2 & 4 C. 2 , 3 & 4 D. 3 only E.
1 & 4 only

Generalized osteoclastic activity along the walls of the alveolar


socket is the bone response to:
depressing force C. extrusion
B. elongating force D. rotating force E. both B &
D

A 4-year old child has a traumatized central incisor with a Class III
(Ellis) fracture. The injury occurred about one month ago, and ex-
amination indicates that the pulp is necrotic. There are no other
pathologic findings. Treatment of choice is:
watchful observation
extraction and use of a space maintainer
pulpectomy and root canal filling using gutta-percha points and
cement
endodontic treatment and root canal filling with a resorbable
paste.

Supervision of a child’s development of occlusion is most critical at


ages:
3-6 years B. 7-10 years C. 11-14 years E. 14-
17 years

A child with congenital heart disease requires special treatment plan-


ning for dental care because of potential problems with:
bleeding
local infection
systemic infection
enamel hypoplasia
A. 1 , 2 & 3 B. 1 & 3 C. 1 only D. 3 only E.
1,2,3 & 4

In festooning and trimming a stainless steel crown, greater length


is necessary in the region of the mesiofacial bulge in a primary:
first molar B. second molar C. maxillary canine D. maxil-
lary lateral incisor

Using a topical fluoride rinse before acid etch direct bonding of or-
thodontic brackets is contraindicated because fluoride:
decreases the solubility of enamel
increases the pH of the etching agent
causes copious amounts of saliva
directly reacts chemically with the bonding agent.

If a 7-year old patient loses a primary mandibular canine about the


same time the adjacent lateral incisor is erupting or shortly
thereafter, the dentist should be alert to the possibility of:
a tongue habit
a developing crossbite
an early eruption of the permanent canine
lingual collapse of mandibular anterior teeth

Which of the ff. approaches is best for a child suffering from cele-
bral palsy
HOM exercise B. towel method C. use of sedation D. psy-
chological approaches

Type of fear which is produced by direct physical stimulation of sense


organ
associative fear B. objective fear C. subjective fear D.
acquired fear

Agent used to reduce pain without affecting consciousness


hypnotic drug B. analgesic C. barbiturate D. gen-
eral anesthesia

The drug that modifies fear and anxiety


A. flanax B. ethamizid C. celestamine D.
seconal

A general characteristic of a 12 year old child


mother as the center of his world C. exhibits tantrums
separation anxiety D. rejects parental authority

This is a psychological approach to manage a child


nitrous oxide inhalation C. giving diazepam
mutisensory techniques D. HOM

The major source of anchorage in a maxillary Hawley appliance is the :


molar clasp C. acrylic portion
B. labial bow D. interproximal clasp E. bracket of the
band

A narrow maxillary arch with respect to midsagittal plane is said to


be in:
Protraction C. contraction
B. retraction D. distraction E. both C & D

Complete unilateral lip-jaw palate cleft.


A. Class I B. Class II C. Class III D.
Class IV

Anodontia, diagnosed in a 5-year-old child, primarily affects the


growth of the:
Midface C. mandible
B. maxilla D. alveolar bone E. maxilla & man-
dible

Which of the following are likely to occur during orthodontic therapy?


gingival irritation
cementum resorption
increased mobility of teeth
demineralization of enamel adjacent to appliances in patients
with poor oral hygiene
1, 3 & 4 C. 2 & 4 only E. 3 & 4 only
B. 1 & 4 only D. 1, 2, 3 & 4

A distal shoe space maintainer is indicated when a primary:


incisor is avulsed
first molar is prematurely lost
second molar is lost after eruption of a permanent first molar
second molar is lost before eruption of a permanent first molar

The normal downward and forward direction of facial growth results


from
upward and backward growth of maxillary sutures and the man-
dibular condyle
vertical eruption and mesial drift of the dentitions
interstitial growth in the maxilla and the mandible
epithelial induction at growth centers
A. 1 & 2 B. 1 & 3 C. 1 & 4 D. 2 & 3 E. 2 & 4

An 8-year-old girl has 2mm diastema between permanent maxillary cen-


tral incisors. Permanent maxillary lateral incisors are in posi-
tion. The diastema is probably the result of:
thumb-sucking
an abnormal frenum attachment
a normal developmental process
A. 1 & 2 B. 1 & 3 C. 2 & 3 D. 2 only E. 3 only

When comparing cementum to bone in their responses to orthodontic


forces, cementum resorbs:
more readily C. not at all
B. less readily D. under lighter loads E. by
indirect (undermining) resoprtion

The tooth in the mandibular arch most likely to be malposed in cases


of arch space discrepancy is the:
first molar B.. first premalor C. second molar D. second
premolar

Slow progress in molar uprighting in an adult patient is usally due


to:
overextended bands D. an overcontoured spring
lack of anchorage control E. the occlusion not
being relieved
the stabilizing wire not being passive

A 9-year-old patient has a slightly convex profile and a suspected


tooth mass-arch length (circumference) discrepancy. Before insti-
tuting space management procedures, the dentist should:
complete a space analysis C. complete a cephalometric
analysis
seek orthodontic consultation D. all of the above E. Ei-
ther A or C above

A primary maxillary anterior tooth in a 4-year-old child was traumat-


ically intruded into the tissues so that only half the tooth is
visible. The most appropriate treatment is to:
extract the tooth.
perform a pulpotomy
administer no treatment
place orthodontic bands on adjacent teeth and draw the tooth
down with elastics

Which of the following orthodontic movements of teeth are most diffi-


cult to accomplish?
Tipping
Rotation
Intrusion
Extrusion
Translation
A. 1 & 2 B. 1 & 3 C. 1 & 5 D. 3 & 4 E. 3 & 5

Interstitial growth is observed at which of the following sites?


Spheno-occipital syndrondrosis
Maxillary tuberosity
Mandibular condyle
Zygomaticomaxillary suture
Apex of an erupting premolar
A. 1 & 2 B. 1 & 3 C. 1 & 4 D. 2 & 4 E. 2 & 5

When force is applied, bone reflexes that produces deformation of


crystalline materials and allows flow of electric current necessary
for tooth movement:
deposition C. hyalinization
B. piezoelectricity D. undermining resoprtion

Cephalometrics is useful in assessing which of the following rela-


tionships?
1. tooth-to-tooth
2. bone-to-bone
3. tooth-to-bone
A. 1 & 2 B. 1 & 3 C. 2 & 3 D. 1,2 & 3 E. 3 only

Which of the following is the most essential factor related to cor-


rection of an anterior crossbite?
age of the patient C. depth of the corssbite
shape of the tooth involved D. space available mesi-
odistally

Frankfort-horizontal is a reference plane constructed by joining which


of the following landmarks?
Porion and sella C. Porion and nasion
Porion and orbitale D. Nasion and Sella

Which of the following is not a space maintainer?


Lingual arch D. Nance holding arch
Class III restoration E. Stainless steel crown
Palatal expansion appliance

Space closure is least likely to occur after loss of which of the


following teeth?
Primary mandibular canines D. Primary mandibular second
molars
Primary maxillary first molars E. Primary maxillary
central incisors
Permanent maxillary central incisors

Crowding that occurs with mandibular incisors after age 18 is most of-
ten the result of:
orthodontic relapse C. eriodontal disease
impacted thid molars D. a physiologic maturational
change

The undesirable side-effect most commonly associated with use of a


finger spring to tip the crown of a tooth is:
Pain D. gingival irritation
tendency for the tooth to intrude E. severe mobility of the
tooth
tendency for the root apex to move in the direction opposite
from the crown.

After the age 6, the greatest increase in size of the mandible oc-
curs:
at the symphysis C. along the lower border
between the canines D. distal to the first molars

A distal shoe space maintainer is indicated when a primary:


canine is in crossbite
first molar is prematurely lost
second molar is lost after eruption of a permanent first molar
second molar is lost prior to eruption of a permanent first mo-
lar

The “V” principle of growth is best illustrated by the:


nasal spetum C. mandibular symphysis
B. mandibular ramus D. spheno-occipital synchondrosis

Anterior crossbite in the primary dentition usually indicates a de-


veloping:
Class I malocclusion C. Class II malocclusion
Class III malocclusion D. Class IV maloccusion E. none
of the above

A mixed dentition analysis determines:


intercanine width D. skeletal growth pattern
discrepancies in jaw size E. size of permanent incisors
space available versus space required

Gonion, menton and pogonion are cephalometric landmarks located on the


midline B. bony chin C. mandible D. skeletal
profile

A major disadvantage of treatment using cervical headgear is:


impaction of maxillary canines D. extrusion of maxil-
lary incisors
extrusion of maxillary molars E. potential deformity of the
neck
psychologic trauma due to appearance

Asymmetrical anterior open bite with normal posterior occlusion is


characteristic of:
thumb-sucking C. abnormal swallowing habits
mouth breathing D. both A & B E. both B &
C

If the norm for the cephalometric angle SNA is 82°, a patient’s read-
ing of 90° for SNA most likely indicates:
maxillary protrusion C. the patient’s ethnic back-
ground
protrusive maxillary incisors D. dysplasia of the anterior
cranial base.

Ideally, a malooclusion should be treated between the ages of:


5 and 8 years D. 8 and 10 years
10 and 12 years E. 12 and 14 years
None of the above. The age at which a malocclusion is treated
should depend on the problem involved.

Radiographs of a 5-year old patient show permanent maxillary first


molars inclined mesially with resulting resoprtion of the distal
portions of the roots of primary second molars. The condition de-
scribed is:
ankylosis C. ectopic eruption
premature eruption D. internal resorption E. in-
trafollicular resoprtion

Fractured maxillary anterior teeth generally occur most often in


children with which of the following dental conditions?
Class I malocclusion D. Class II, Division 1
malocclusion
Class II, Division 2 malocclusion E. Class III malocclu-
sion
Marked incisor crowding

On the first dental visit, the basic fear of a child under the age of
2 is related to:
fear of an injection
anxiety over being separated from a parent
anxiety regarding the instruments used to perform dental treat-
ment
not understanding the reason for dental treatment.

A child with Down’s syndrome is characterized as being:


affectionate C. fearful of quick movements
capable of learning dental procedures D. both A & B E.
all of these

A 3 ½ year old child has an acute fever, diarrhea, oral vesicular le-
sions and gingival tenderness. The most likely diagnosis is:
thrush D. drug allergy
aphthous ulcerations E. . acute herpetic stoma-
titis
C. necrotizing ulcerative gingivitis

Transillumination of soft tissues is useful in detecting which of the


following problems in a child?
Koplick’s spot C. Sialolithiasis
Aortic stenosis D. Sickle cell disease E. ab-
normal frenum attachment

Which of the following is least likely to result from persistent


long-term thumb-sucking?
a deep overbite D. protrusion of maxillary incisor
constriction of maxillary arch E. rotation of maxil-
lary lateral incisor
lingual inclination of mandibular incisors

The least desirable method used in child management is:


tell-show do technique, C. voice control,
gift before treatment, D. hand-over-mouth technique.

A child who is reluctant to accept dental treatment ,exhibit negative


attitude but not so pronounced is classified under Frankl ?
I B. II C. III D. IV

The amount of pulp tissue to be removed in formocresol pulpotomy pro-


cedure
half of the coronal pulp
coronal pulp tissue down to the cervical constriction of each
root canal
C. infected portion of coronal pulp tissue
D. total removal of pulp tissue including radicular pulp

Which of the following dental sequelae is likely in a child with a


history of generalized growth failure (failure to thrive) in the
first six months of life?
Retrusive maxilla C. Enamel hypoplasia
Retrusive mandible D. Small permanent teeth E. Dentino-
genesis imperfecta

The average age at which calcification of crowns of permanent central


incisors is completed is:
birth B. 2-3 years of age C. 4-5 years of age D. 6-7
years of age

Amalgam is most often the restorative material of choice for primary


teeth. The most important modification in its use for children is
in:
cavity preparation D. use of a rubber dam
use of a zinc-free alloy E. condensation of the alloy
placement of matrix bands

The most frequent cause of fracture of a root tip during extraction


of a primary molar is:
ankylosis of the tooth
improper use of cowhorn forceps
presence of a supernumerary premolar
root resorption between the apex and the bifurcation
asymmetric root resorption in which only one root is completely
resorbed.

The most common cause of sinus tracts in gingival tissues of children


is:
pericementitis C. periapical cyst
acute periapical abscess D. chronic periapical abscess

A light bluish, dome-shaped lesion on the inside lip of a 2-year old


child is most likely a:
Mucocele B. melanoma C. hematoma D. hemangioma E.
sucking callous

Primary second molars usually erupt during ages:


8-14 months C. 14-20 months
20-30 months D. 30-36 months E. 36-48 months

A child in long-term remission of acute leukemia has dental problems


characterized by unusual susceptibility to:
dental caries C. oral infection
periodontal bone loss D. development of jaw deformities
E. all of the above.

This is a contraindication of HOM


spoiled child C. very young patient
defiant child D. uncooperative behavior but capable of
understanding

Induce formation of apical closure of young permanent molar using


CAOH
apexogenesis C. apexification
B. apeximation D. apex formation

The growth movement of the mandible is complimented by the growth of


the
maxilla, which is:
down and forward C. downward and backward
B. forward only, D. upward and backward

Direction of displacement of the mandible in an individual with de-


veloping Class II malocclusion:
down and forward, C. down and backward,
B. forward only, D. forward and upward, E. up-
ward and backward.

Child refuses to accept treatment or open the mount minimally:


A. Frankl 1 B. Frankl 2 C. Frankl 3 D. Frankl 4

Definitely negative behavior.


A. Frankl 1 B. Frankl 2 C. Frankl 3 D. Frankl 4

Child has good rapport with the dentist.


A. Frankl 1 B. Frankl 2 C. Frankl 3
D. Frankl 4.
The treatment option contraindicated in patients who are not able to
breathe nasally.
no treatment C. nitrous oxide & oxygen inhala-
tion
B. conscious sedation D. genera anesthesia

The treatment option for patients who have sustained extensive orofa-
cial & or dental trauma
no treatment C. nitrous oxide & oxygen inhalation
B. conscious sedation D. general anesthesia

With a flush terminal plane, permanent first molars will:


initially be Class II
initially be Class III
immediately assume a normal relationship
erupt immediately into an end-to-end relationship

Arch shape & symmetry are best evaluated from the:


study cast C. frontal photograph
B. panoramic x-ray D. lateral cephalogram E. intraoral pho-
tograph

Bodily force that moves the central incisor mesially produces:


stretching of the periodontal fiber on the distal side
compression of the periodontal fiber in the distal side
osteoblastic activity on the mesial side
osteoclastic activity on the distal side

In infant oral care, clean mouth with gauze after feedings and at bed
time done during:
A. 0-6 mons B. 6-12 mons C. 12-24 mons. D. 24-
36 mons

Space differential between combine width of CDE and 345 is__


A. positive B . negative C. zero D. 2.2 mm
E. 2.4 mm
ORTHODONTICS AND PEDIATRIC DENTISTRY

1. Gonion, menton and pogonion are cepahlometric landmarks located on the :


A. midline B. mandible C. bony chin D. skeletal profile

2. Complete unilateral lip-jaw palate cleft.


A. Class I B. Class II C. Class III D. Class IV

3. The best alternative diagnostic aid in the absence of cephalometric head plate:
A. study cast C. facial photograph
B. radiograph D. both A & C E. none of these

4. A narrow maxillary arch with respect to midsagittal plane is said to be in:


A. protraction B. retraction C. contraction D. distraction

5. The downward and forward direction of facial growth results from:


A. upward and backward growth of the maxillary sutures and the mandibular condyle
B. vertical eruption and mesial drift of the dentitions
C. interstitial growth in the maxilla and the mandible
D. epithelial induction at the growth centers

6. The condyle of the mandible grows by:


A. membrane bone growth C. appositional bone growth
B. instertitial bone growth D. proliferation of cartilage E. none of these

7. Spinning bead retainer is the appliance of choice for:


A. night grinding C. thumb sucking
B. tongue thrusting D. mouth breathing E. none of these

8. Asymmetrical anterior open bite with normal posterior occlusion is characteristic of:
A. thumb-sucking C. abnormal swallowing habits
B. mouth breathing D. all of these E. none of these

9. An adjunctive diagnostic tool for treatment planning orthodontics that gives information on possible
growth pattern and can give an accurate bone age picture of the patient:
A. hand & wrist C. panoramic
B. cephalometrics D. tomogram E. periapical radiograph

10. Frankfort-horizontal is a reference plane constructed by joining which of the following landmarks?
A. porion and sella C. porion and orbitale
B. porion and nasion D. nasion and sella E. none of these

11. The distal step in the primary dentition will result in:
A. end to end molar relationship C. Class I molar relationship
B. Class III molar relationship D. Class II molar relationship

12. The space created upon exfoliation of primary molars which is conducive to proper relationship
of the permanent 1st molars is:
A. negative freeway space C. negative leeway space
B. positive freeway space D. positive leeway space

13. This is a process of reshaping and resizing as a consequence of progressive continuous relo-
cation:
A. drift C. remodeling
B. displacement D. translation E. none of these.
14. A 5-year old boy who lives in an area where fluoride content of the drinking water averages
0.75ppm should be supplemented with how much fluoride per day?
A. 0 mg. B. 0.25 mg C. 0.50 mg. D. 1.00mg.

15. The following is the difference in size between the primary and their permanent successor in the
posterior segment.
A. posterior liability C. posterior size discrepancy
B. nance’s leeway space D. late mesial shift E. none of these.

16. Precursor of the mandible:


A. tuberculum impar C. condyle
B. meckel’s cartilage D. copula E. none of these

17. Deep mentolabial sulcus:


A. hypertonic upper lip C. hypertonic lower lip
B. hypotonic upper lip D. hypotonic lower lip

18. Active mentalis muscle:


A. hyperactive upper lip C. hyperactive lower lip
B. hypoactive lower lip D. hypoactive upper lip

19. A desirable zone of flame in soldering metals is:


A. reducing B. oxidizing C. combustion D. air blast zone

20. Which of the following factors will not increase maxillary width?
A. growth of the palate following “V” principle
B. deposition at the fronto-maxillary suture
C. deposition at the lateral walls
D. deposition at the median palatine sutre

21. The treatment option contraindicated in patients who are not able to breathe nasally.
A. no treatment C. nitrous oxide & oxygen inhalation
B. conscious sedation D. genera anesthesia

22. The major mechanism for growth of the cranial case is the:
A. expansion of the cartilage cells
B. interstitial growth of bone
C. apposition of new bone at the synchondroses
D. apposition of new bone at the sutures.

23. . The main growth site of the mandible which is responsible for its increase ion height?
A. Condyle C. symphysis
B. gonial angle D. posterior border of the ramus E. inferior border of the body

24. The sequence of completion of facial growth by planes of space is?


A. depth, width, height C. height, depth, width
B. width, depth, height D. width, height, depth E. depth, height, width

25. The theory which states that the growth of the craniofacial bones is caused by the soft tissues
adjacent to them.
A. Functional matrix C. Sutural Dominance theory
B. Cartilaginous growth theory D. Limborgh’s theory

26. The “V” principle of growth is best illustrated by the?


A. nasal septum
B. posterior border of the mandibular ramus
C. mandibular symphysis
D. intersphenoidal synchondrosis

27. Chin cup is used to intercept a?


A. Class 1 type 3 C. Class II
B. Class III D. Developing Class III

28. A direct growth movement that is produced by deposition on one side of the orbital plate with
resorption on the opposite side is?
A. Displacement C. translation
B. cortical drift D. remodeling

29. The following are distinctive structural features related to cartilage of the cranial base, except:
A. pressure- tolerant C. grows interstitially and appositionally
B. matrix is non-vascular D. grows appositionally

30. What is the chief factor in the formation of the alveolar process?
A. normal process of growth
B. eruption of teeth
C. lengthening of the condyle
D. overall growth of the maxilla and mandible

31. Which of the following is not a characteristic of a mature swallow?


A. active contractions of the muscles of the lips
B. tongue tip is placed against alveolar process behind the upper incisors
C. teeth are together during swallowing
D. both A & B
E. both A & C

32. These are six soft spots present between the bones of the skull roof.
A. Sutures B. synchondroses C. fontanelles D. cartilages

33. Spaces between the primary anteriors:


A. interdental space
B. primate space D. both A & B
C. Nance leeway space E. both B & C

34. Primate space is found between the:


A. maxillary canine and first molar
B. mandibular primary canine and lateral incisor
C. maxillary canine and lateral incisors
D. none of the above

35. At what stage in Nolla’s classification does a permanent tooth start to erupt?
A. Stage 7 C. Stage 6
B. Stage 5 D. Stage 8 E. Stage 4

36. From the flush terminal plane relationship of molars in the primary dentition, the permanent first
molar relationship in the permanent dentition can become the following in the transition period, ex-
cept:
A. Class I C. Class III
B. Class II D. end to end / cusp to cusp E. none of the choices

37. Two or more tooth moving opposite directions and pitted against each other is equal and
opposite. The anchorage is.
A. simple B. stationary C. reciprocal D. none.
38. The following are caused by thumbsucking habit Except.
A. anterior open bite B. protrusion of upper incisor C. expansion of upper arch

39. In a condition characterized by partial or complete absence of clavicle.


A. torticollis B. cerebral palsy C. cleidocranial dysostosis

40. A condition characterized by foreshortening of sternocleidomastoid muscle.


A. torticollis B. cerebral palsy C. cleidocranial dysostosis

41. Type of force recommended for removable orthodontic appliance.


A. continuous B. functional C. dissipating D. intermittent

42. Type of force which can be seen in fixed appliance is.


A. continuous B. functional C. dissipating D. intermittent

43. Osteoclastic activity occurs on the.


A. tension side B. pressure side C. stress D. compression

44. Bone deposition occurs on the.


A. tension side B. pressure side C. stress D. compression

45. The tooth is push towards the socket and pulled away from occlusal plane.
A. elongating force B. rotation C. depressing D. heavy force

46. Neutroclusion with labioversion of max centrals & buccoversion of mand. 1st premolar.
A. Class 1 type 2 & 3 B. Class 1 type 2 & 4 C. Class 1 type 2 & 5

47. A supraverted mand. Premolar with respect to Frankfurt horizontal plane is said it be in.
A. contraction B. abstraction C. Attraction D. distraction

48. The resorption occurs from behind an immediate site of pressure, there is accumulation of
fluid and blood vessels are occluded.
A. direct resorption C. undetermining resorption
B. frontal osteoclastic attack D. all of the Above.

49. It refers to perfect arrangement of teeth when jaw’s are closed & condyies are at rest in the
glenoid fossa.
A. normal occlusion B. ideal occlusion C. malocclusion

50. The incidence of malocclusion is very high in.


A. homogeneous population B. Heterogeneous population
C. Eskimos D. none of the above

51. The predominant type of malocclusion seen in mixed dentition is.


A. Crowding B. anterior open bite C. Class II Div. 1

52. A brachycephalic individual is associated with___ arch form:


A. square B. ovoid C. long and narrow D. none of the above.

53. Method of early examination where the dentist and parent are seated face to face
A. Knee position C. leg position
B. cradling position D. Knee to knee position E. all

54. In pedodontic triangle the apex of the triangle is:


A. Child B. Dentist C. Parent D. Assistant

55. Parent who are mistrusting are:


A. Overprotective Parent C. Neglectful Parent
B. Manipulative Parent D. Hostile parent

56. Parent who are demanding and can extend to directing the course of the treatment is a :
A. Overprotective Parent C. Neglectful Parent
B. Manipulative Parent D. Hostile parent

57. A parent who postponed the dental treatment of his/her child is


A. Overprotective Parent C. Neglectful Parent
B. Manipulative Parent D. Hostile parent

58. Panoramic Radiographic Film:


A. 5 “ x 7” B. 2”x 3” C. 5” x 12” D.. 8” x 10”

59. Radiograph examination for a three years old patient with no apparent abnormalities and open
contact:
A. 2 posterior bitewing C. 4 film series
B. 8 film series D. 12 film Series E. None

60. Radiographic examination needed for a 6-7 years old with no apparent abnormalities:
A. 2 posterior bitewing C. 4 film series
B. 8 film series D. 12 film Series E. None

61. Frankl behavior rating characterized by refusal of treatment and extreme negativism:
A. rating 1 B. rating 2 C. rating 3 D. rating 4

62. A period that child undergoes the phase of separation anxiety:


A. Toddler B. Pre school C. school age D. Adolescent

63. Stage where one exhibit absolute confidence that they are right about everything, and their
judgement are infallible:
A. Toddler B. Pre school C. school age D. Adolescent

64. The most unstable form of fluoride used for topical application:
Acidulated Phosphate Fluoride C. Stannous Fluoride
Sodium Fluoride D. None

65. Retention of PFS is through:


A. Light curing C. Drilling dentin
B. Acid etching D. Application of fluoride

66. A sudden firm command use to get the child’s attention:


A. Voice control C. Multisensory communication
B. Aversive conditioning D. HOME

67. Procedure which slowly develops behavior by reinforcing successive approximation of the de-
sired behavior until desired behavior develop:
A. Pre appointment behavior modification C. behavior shaping
B. Behavior management D. Retraining

68. Parents should be allowed inside a treatment room when:


A. infant to 41 months C. handicapped patient
B. 7-8 Years old D. A & C E. None

69. Medicament used in type A PRR:


A. Glass ionomer C. Bonding agent
B. Composite D. Pit and fissure sealant E. all of these
70. Medicament used for Type C PRR:
A. Glass ionomer C. Bonding agent
B. Composite D. Pit and fissure sealant E. all of these

71. Advantage of Rubber dam utilization:


A. Control saliva C. Aids management
B. Provides Protection D. all of these E. None

72. Distance of punched holes on the rubber dam for primary teeth:
A. 1.0mm C. 2.0mm
B. 1.5 mm D. 2.5 mm E. 3.0mm

73. Gingival cavosurface beveling is not indicated in Class II cavity preparation for deciduous teeth,
because the direction of the enamel rods are toward:
A. Cervical B. Occlusal C. gingival D. Apical E. all of these

74. Ellis classification of a non vital traumatized tooth:


A. Class I B. Class II C. Class III D. Class IV E. Class V

75. Treatment of choice for an immature non vital permanent tooth to stimulate root closure:
A. Indirect pulp capping C. Pulpotomy
B. Pulpectomy D. Apexogenesis E. Apexification

76. Use of formocresol:


A. to stop bleeding C. Stimulate secondary dentin
B. render area of fixation D. Dissolve the pulp tissue E. all of these

77. Materials that is used to induced apexification:


A. calcium hydroxide C. Formocresol
B. Zinc oxide eugenol D. IRM E. Gutta percha

78. Treatment of choice for a pulp exposed primary molar except:


A. Direct pulp capping C. pulpotomy
B. Pulpectomy D. all of these E. none of the above

79. The average SNB angle for Filipino children with Class 1 occlusion is:
A. 82.0º B. . 82.5º C. 84.0º D. 84.5º E. 103º

80. The ANB angle of a patient with severe Class 2 skeletal malocclusion would most likely be: A.
–4 B. 0 C. 2 D. 8.

81. If the mandibular and Frankfurt horizontal plane converge and meet at a point close behind the
face, the direction of growth potential is :
A. balanced vertically and horizontally B. more vertical
C. more horizontal D. none.

82, Protrusiveness or retrusiveness of the chin point can be known by analyzing the:
A. SNB B. FMLA C. IMPA D. FMA E. FH/NP.

83. Physiologic tooth movements include the following, except:


A. eruption B. functional movement C. gaining of space with an appliance
D. mesial drifting in the absence of adjacent teeth E. none.

84. Force level decline abruptly to zero.


A. coil spring B. fix braces C. removable appliance D. arch wire.
85. Unilateral distocclusion with linguoversion of the maxillary central incisors.
A. Class 2 Div 1 B. Class 2 Div. 2 subd. C. Class 2 Div. 1 subd.

86. MB cusp of maxillar 6 occludes with the MB groove of mand 6 with crowding of the anterior teeth,
:
A. Class II type 1 B. Class I type 1 C. Class I type 3

87. MB cusp of max. 6 occludes with the MB groove of mand 6 with buccoversion of 2nd premolar:
A. Class II type 3 B. Class I type 3 C. Class I type 5.

88. Requires retentive groove in tooth preparation:


A. anterior SSC B. SOC C. polycarbonate crown

89. Retention is achieved through acid etching the tooth preparation:


A. anterior SSC B. SOC C. polycarbonate crown

90. Deciduous incisor is subjected to pulpectomy, final restoration is:


A. anterior SSC B. SOC C. polycarbonate crown.

91. A supra erupted mand. canine with respect to Frankfurt Horizontal plane is said to be in?
A. attraction B. Abstraction C. Protraction
D. Retraction E. Contraction.

92. A supra erupted max. canine with respect to Frankfurt Horizontal Plane is said to be in?
A. attraction B. Abstraction C. Protraction
D. Retraction E. Contraction.

93. When one side of the arch is intact and there are several primary teeth missing on the other side,
use:
A. a Nance lingual arch C. a distal shoe
B. a transpalatal arch D. a partial denture space maintainer
E .a soldered fixed lingual arch

94. Child has good rapport with the dentist.


A. Frankl 1 B. Frankl 2 C. Frankl 3 D. Frankl 4.

95. The order from greatest to least change of the dimensions of the cranium:
A. height, depth, width C. depth, width, height
B. depth, height , width D. width, height, depth

96. At age 4-5, what normal sign of primary dentition augers well for the erupting permanent incisors
in terms of space availability:
A. Class I cuspid relationship C. growth spaces, interdental spaces
B. upright vertical incisor relationship D. flush terminal plane

97. The basic form of the arch is determined:


A. by environmental factors B. by muscles
B. in intrauterine life D. two of the choices E. all of these

98. The drug that modifies fear and anxiety


A. flanax B.ethamizid C. celestamine D. seconal

99. This is a psychological approach to manage a child


A. nitrous oxide inhalation C. giving diazepam
B. mutisensory techniques D. HOM
100. This is a contraindication of HOM
A. spoiled child C. very young patient
B. defiant child D. uncooperative behavior but capable of understanding

ORTHODONTICS AND PEDIATRIC DENTRISTRY

1. It is considered to be the best space maintainer


A. crown and crib
B. a well-restored deciduous teeth
C. fixed space maintainer
D. functional space maintainer

2. Chin cap is used to redirect the growth of the


A. maxilla B. cranial base
C. cranial case D. mandible

3. Cases of maxillary protrusion in a growing child can be treated with


A. chin cap B. fixed braces only
C. headgear D. face mask

4. Deep bite cases in young children can be treated with


A. fixed braces B. bite plane
C. extraoral force appliance D. space maintainer

5. Fluoride reduces the incidence of carious lesions on the


A. pit and fissure B. occlusal surfaces
C. smooth surfaces D. lingual surfaces of teeth

6. Serial extraction is a part of:


A. preventive orthodontics B. interceptive orthodontics
C. limited corrective D. extensive corrective

7. Serial extraction is done if the space deficiency to align the is


A. 3 mm B. 6 mm C. 5 mm D. 8 mm

8. Its is a way of predicting the future alignment of erupting permanent teeth


A. mixed dentition analysis B. Nolla’s calcification
C. size of the teeth D. none of the above
9. Increase in breadth means increase in
A. depth B. vertical deimension
C. length D. width
E. height

10. Facial structures follow the


A. bodily growth curve B. neural growth curve
C. a and b D. none of the above

11. It is the synchondrosis whose activity stops after age of 25.


A. intersphenoidal
B. intraoccipital
C. spheno-occipital
D. sphenoethmoidal

12. It is the growth center of the mandible which is responsible for the major increase in length
A. alveolar process
B. condyle
C. lingual tuberosity
D. Ramus of the mandible

13. The cranial base lengthens because of the growth on the


A. interparietal suture B. coronal suture
C. parieto-temporal suture D. lambdoidal suture

14. The V principle of growth is found in the following structures of the skull except
A. mandible B. palate
C. orbits of the eyes D. alveolar process

15. It is the growth center that is largely dependent on the presence of teeth
A. nasal septum B. alveolar process
C. condyle D. max. and mand. Basal bones

16. The relationship of the upper and lower gum pads is such that
A. the upper is within the lower B. the upper is posterior to the lower
C. the lower is anterior to the upper D. it presents a convex profile

17. A child in the “ugly duckling” stage is characterized by


A. a flat nasal bone
B. absence of maxillary incisors
C. diastema between 2 central incisors
D. presence of only one maxillary central incisor

18. Late mesial shift is possible because of


A. primate spaces
B. terminal plane
C. space differential bet. Deciduous and permanent teeth in the posterior segment
D. none of the above

19. The space that allows an increase in the mandibular intercanine width is
A. leeway pace B. interdental space
C. primate space D. anterior intermaxillary space

20. It is the arch dimension that is occupied by all the succedaneous teeth
A. arch length B. arch width
C. arch perimeter D. intercanine length
21. What are the growth dimensions in the maxilla that are sex-linked?
A. width and depth B. height and width
C. height and depth D. width

22. From birth to 5 years, the dominance of growth is on the


A. nasomaxillary structures B. cranial structures
C. mandible D. muscular components of the body

23. Early loss of upper D will end up to what type of terminal plane?
A. mesial step B. flush terminal plane
C. distal step D. a ny of the above

24. This is growth movement which mean movement of whole bone as a unit
A. drift B. displacement C. deposition D. relocation E. remodeling

25. Mandible comes from ? branchial arch


A. first B. second C. third D. fourth

26. Maxilla is formed from the maxillary processes and ?


A. medial nasal process C. globular process
B. lateral processes D. tuberculum impar

27. Dental Fluorosis is most likely to occur on a:


A. two-year-old child ingesting 1 ppm of fluoride
B. 17-year-old patient ingesting 1 ppm of fluoride
C. on a 3-year-old child ingesting 0.50 ppm of fluoride
D. none of the above.

28. Pulpotomy with formocresol of primary tooth:


A. result in lower success rate compared to calcium hydroxide,
B. result in higher success rate compared to calcium hydroxide,
C. should never be used,
D. induces formation of a dentin bridge at site of amputation,
E. none of the above.

29. A 4-year-old child visited a dentist for the first time and received prophylaxis quite well. Be-
fore he left, the dentist was glad that he gave the child a toy. This kind of gesture is called:
A. classical conditioning C. positive reinforcement,
B. bribery, B. public relations E. none of the above.

30. If a primary tooth has been extracted before succedaneous tooth has begun eruptive move-
ment, its eruption will be __
A. hastened B. delayed C. not affected

31. Excessive orthodontic force used to move a tooth may:


1. cause hyalinization
2. cause undermining resorption
3. crush the periodontal ligament
A. 1 & 2 B. 2 & 3 C. 1 & 3 D. 1, 2 & 3 E. 3 only

32. Severely crowded mandibular incisors usually result from:


A. mesial migration of teeth
B. premature loss of primary teeth
presence of supernumerary teeth
tooth size-arch length discrepancy
33. How will extraction of a primary maxillary central incisor in a 5-year old child with incisal
spacingaffect the size of the intercanine space?
A. The intercanine space will increase in size.
B. The intercanine space will decrease in size.
C. No change will occur in the size of the intercanine space.

34. The major criterion to differentiate between a true Class III and a pseudo-Class III malocclusion
is:
A. degree of anterior crossbite
B. presence of a bilateral crossbite
C. existence of a forward shift of the mandible during closure
D. occlusal relationship between maxillary and mandibular first molars

35. Orthodontic correction of which of the following is most easily retained?


A. Diastema C. Expansion
B. Rotation D. Posterior crossbite E. Anterior crossbite

36. A bimaxillary protrusion occurs when:


A. large horizontal overlap with biteral crossbite exits
B. the maxillary dentition is forward with respect to basal bone
C. both dental arches are forward with respect to basal bone
D. none of the choices.

37. When a simple tipping force is applied to the crown of a single-rooted tooth, the center of rota-
tion is usually located:
A. at the apex D. at the cervical line
B. 5mm beyond the apex E. one-third the root length from the apex
C. two-third the root length from the apex.

38. Which of the following is the most common orofacial malformation that produces malocclusion?
A. Cleft palate C. Ectodermal dysplasia
B. Pierre Robin syndrome D. Osteogenesis imperfecta E. Cleidocranial dysostosis

39. A 4-year old child has a traumatized central incisor with a Class III (Ellis) fracture. The injury
occurred about one month ago, and examination indicates that the pulp is necrotic. There are no
other pathologic findings. Treatment of choice is:
A. watchful observation
B. extraction and use of a space maintainer
C. pulpectomy and root canal filling using gutta-percha points and cement
D. endodontic treatment and root canal filling with a resorbable paste.

40. Supervision of a child’s development of occlusion is most critical at ages:


A. 3-6 years B. 7-10 years C. 11-14 years E. 14-17 years

41. A child with congenital heart disease requires special treatment planning for dental care be-
cause of potential problems with:
A. bleeding
B. local infection
C. systemic infection
D. enamel hypoplasia
A. 1 , 2 & 3 B. 1 & 3 C. 1 only D. 3 only E. 1,2,3 & 4

42.In festooning and trimming a stainless steel crown, greater length is necessary in the region of
the mesiofacial bulge in a primary:
A. first molar B. second molar C. maxillary canine D. maxillary lateral incisor
43. Using a topical fluoride rinse before acid etch direct bonding of orthodontic brackets is contrain-
dicated because fluoride:
A. decreases the solubility of enamel
B. increases the pH of the etching agent
C. causes copious amounts of saliva
D. directly reacts chemically with the bonding agent.

44. If a 7-year old patient loses a primary mandibular canine about the same time the adjacent lat-
eral incisor is erupting or shortly thereafter, the dentist should be alert to the possibility of:
A. a tongue habit
B. a developing crossbite
C. an early eruption of the permanent canine
D. lingual collapse of mandibular anterior teeth

45. Which of the ff. approaches is best for a child suffering from celebral palsy
A. HOM exercise B. towel method C. use of sedation D. psychological Approaches

46. Type of fear which is produced by direct physical stimulation of sense organ
A. associative fear B. objective fear C. subjective fear D. acquired fear

47. A 4-year old child has a traumatized central incisor with a Class III (Ellis) fracture. The injury
occurred about one month ago, and examination indicates that the pulp is necrotic. There are no
other pathologic findings. Treatment of choice is:
A. watchful observation
B. extraction and use of a space maintainer
C. pulpectomy and root canal filling using gutta-percha points and cement
D. endodontic treatment and root canal filling with a resorbable paste.

48. Supervision of a child’s development of occlusion is most critical at ages:


A. 3-6 years B. 7-10 years C. 11-14 years E. 14-17 years

49. Which of the following are likely to occur during orthodontic therapy?
1. gingival irritation
2. cementum resorption
3. increased mobility of teeth
4. demineralization of enamel adjacent to appliances in patients with poor oral hygiene
A. 1, 3 & 4 C. 2 & 4 only E. 3 & 4 only
B. 1 & 4 only D. 1, 2, 3 & 4

50. A distal shoe space maintainer is indicated when a primary:


A. incisor is avulsed
B. first molar is prematurely lost
C. second molar is lost after eruption of a permanent first molar
D. second molar is lost before eruption of a permanent first molar

51. The normal downward and forward direction of facial growth results from
1. upward and backward growth of maxillary sutures and the mandibular condyle
2. vertical eruption and mesial drift of the dentitions
3. interstitial growth in the maxilla and the mandible
4. epithelial induction at growth centers
A. 1 & 2 B. 1 & 3 C. 1 & 4 D. 2 & 3 E. 2 & 4

52. An 8-year-old girl has 2mm diastema between permanent maxillary central incisors.
Permanent maxillary lateral incisors are in position. The diastema is probably the result of:
1. thumb-sucking
2. an abnormal frenum attachment
3. a normal developmental process
A. 1 & 2 B. 1 & 3 C. 2 & 3 D. 2 only E. 3 only

53. When comparing cementum to bone in their responses to orthodontic forces, cementum re-
sorbs:
A. more readily C. not at all
B. less readily D. under lighter loads E. by indirect (undermining)

54. The tooth in the mandibular arch most likely to be malposed in cases of arch space discrep-
ancy is the:
A. first molar B.. first premalor C. second molar D. second premolar

55. Asymmetrical anterior open bite with normal posterior occlusion is characteristic of:
A. thumb-sucking C. abnormal swallowing habits
B. mouth breathing D. both A & B E. both B & C

56. If the norm for the cephalometric angle SNA is 82°, a patient’s reading of 90° for SNA most
likely indicates:
A. maxillary protrusion C. the patient’s ethnic background
B. protrusive maxillary incisors D. dysplasia of the anterior cranial base.

57. A child with Down’s syndrome is characterized as being:


A. affectionate C. fearful of quick movements
B. capable of learning dental procedures D. both A & B E. all of these

58. A 3 ½ year old child has an acute fever, diarrhea, oral vesicular lesions and gingival tender-
ness. The most likely diagnosis is:
A. thrush D. drug allergy
B. aphthous ulcerations E. . acute herpetic stomatitis
C. necrotizing ulcerative gingivitis

59. Transillumination of soft tissues is useful in detecting which of the following problems in a
child?
A. Koplick’s spot C. Sialolithiasis
B. Aortic stenosis D. Sickle cell disease E. abnormal frenum attachment

60. Which of the following is least likely to result from persistent long-term thumb-sucking?
A. a deep overbite D. protrusion of maxillary incisor
B. constriction of maxillary arch E. rotation of maxillary lateral incisor
C. lingual inclination of mandibular incisors

61. The least desirable method used in child management is:


A. tell-show do technique, C. voice control,
B. gift before treatment, D. hand-over-mouth technique.

62. Induce formation of apical closure of young permanent molar using CAOH
A. apexogenesis C. apexification
B. apeximation D. apex formation

63. Neurocranium follows what Scammon’s curve?


A. lymphoid B. neural C. general D. bodily E. genital type

64. Method of early examination where the dentist and parent are seated face to face
A. Knee position B. leg position C. cradling position
D. Knee to knee position E. all

65. In pedodontic triangle the apex of the triangle is:


A. Child B. Dentist C. Parent D. Assistant
66. Device that assess the arterial hemoglobin oxygen saturation and pulse rate:
A. automated vital sign monitor B. pulse oximeter C.
pretracheal stethoscope D. none

69.Quadrant used in the gluteal region when intramuscular sedation is used:


A, upper inner quadrant B. upper outer quadrant C.
Lower inner quadrant D. Lower outer quadrant

70. Arch shape & symmetry are best evaluated from the:
A. study cast C. frontal photograph
B. panoramic x-ray D. lateral cephalogram E. intraoral photograph

71. Bodily force that moves the central incisor mesially produces:
A. stretching of the periodontal fiber on the distal side
B. compression of the periodontal fiber in the distal side
C. osteoblastic activity on the mesial side
D. osteoclastic activity on the distal side

72. Frankfort-horizontal is a reference plane constructed by joining which of the following land-
marks?
A. Porion and sella C. Porion and nasion
B. Porion and orbitale D. Nasion and Sella

73. Which of the following is not a space maintainer?


A. Lingual arch D. Nance holding arch
B. Class III restoration E. Stainless steel crown
C. Palatal expansion appliance

74.Space closure is least likely to occur after loss of which of the following teeth?
A. Primary mandibular canines D. Primary mandibular second molars
B. Primary maxillary first molars E. Primary maxillary central incisors
C. Permanent maxillary central incisors

75. Crowding that occurs with mandibular incisors after age 18 is most often the result of:
A. orthodontic relapse C. Periodontal disease
B. impacted thid molars D. a physiologic maturational change

76. The undesirable side-effect most commonly associated with use of a finger spring to tip the
crown of a tooth is:
A. Pain D. gingival irritation
B. tendency for the tooth to intrude E. severe mobility of the tooth
C. tendency for the root apex to move in the direction opposite from the crown.

77. After the age 6, the greatest increase in size of the mandible occurs:
A. at the symphysis C. along the lower border
B. between the canines D. distal to the first molars

78. Child refuses to accept treatment or open the mount minimally:


A. Frankl 1 B. Frankl 2 C. Frankl 3 D. Frankl 4

79. Definitely negative behavior.


A. Frankl 1 B. Frankl 2 C. Frankl 3 D. Frankl 4

80. Child has good rapport with the dentist.


A. Frankl 1 B. Frankl 2 C. Frankl 3 D. Frankl 4.
81. The treatment option contraindicated in patients who are not able to breathe nasally.
A. no treatment C. nitrous oxide & oxygen inhalation
B. conscious sedation D. genera anesthesia

82. The treatment option for patients who have sustained extensive orofacial & or dental Trauma
A. no treatment C. nitrous oxide & oxygen inhalation
B. conscious sedation D. general anesthesia

83. With a flush terminal plane, permanent first molars will:


A. initially be Class II
B. initially be Class III
C. immediately assume a normal relationship
D. erupt immediately into an end-to-end relationship

84. In infant oral care, clean mouth with gauze after feedings and at bed time done during:
A. 0-6 mons B. 6-12 mons C. 12-24 mons. D. 24-36 mons

85. Space differential between combine width of CDE and 345 is__
A. positive B . negative C. zero D. 2.2 mm E. 2.4 mm

86. A supra erupted mand. canine with respect to Frankfurt Horizontal plane is said to be in?
A. attraction B. Abstraction C. Protraction
D. Retraction E. Contraction.

87. A supra erupted max. canine with respect to Frankfurt Horizontal Plane is said to be in?
A. attraction B. Abstraction C. Protraction
D. Retraction E. Contraction

88. Child’s arms and legs can be immobilized with:


A. forearm-body support B. plastic bowl C. Posey straps D. head positioner

89. To avoid vomiting and complications during treatment with sedation, no milk or solid food
should be taken___ before the scheduled procedure.
A. 4 hrs. B. 6 hrs. C. 8 hrs. D. after midnight

90. For sedation via intramuscular route, the ___ of the gluteal region is the safest.
A. upper, inner B. upper outer C. lower inner D. lower outer

91. Stabilization period for teeth with fractured roots:


A. 2-3 months B. 2-3 days C. 2-3 weeks D. 7-14 days

92. The correct angulation of the inclined plane in relation to the tooth in crossbite is: A. . 15º B.
25º C. 35º D. 45º E. 55º

93. The following are sequelae of untreated crossbite, except:


A. faceting on the labial surface of the tooth in crossbite
B. faceting of the labial surface of the tooth opposing the inlocked tooth
C. abrasion
D. periodontal involvement.

94. Choice of nursing nipple to prevent oral habit formation :


A. Evenflo B. MimifloC. NukSauger nipple D. nonphysiologic nipple

95. Mesial surface of E can be disked in case the 3 cannot erupt because of insufficient space.
This is:
A. contingency of extraction B.occlusal equilibration
C. space regaining D. observation
96. In managing abnormal oral habits, therapy must be:
A. mechanical C.of physical control
B. of conditioning responses D. a means of punishment

97. Occlusal reduction for SSc :


A. 0.25-0.5mm B. 1-1.5mm C. 2-3mm D. 5mm.

98. Grinding of the sharp edges is sufficient.


A. lateral luxation B. dentin fracture C. enamel fracture D. extrusion.

99. Restore with bonding agent and composite


A. extrusion C. dentin fracture
B. intrusion D. enamel fracture E. enamel-dentin fracture.

100. In acute ingestion of fluoride, the following can be given to the patient to counteract its effect,
except:
A. 2 tsp of ipecac syrup B. milk C. milk of magnesia D. alum

ORTHODONTICS AND PEDIATRIC DENTISTRY

1. Which of the following is the most common orofacial malformation that produces malocclusion?
A. Cleft palate C. Ectodermal dysplasia
B. Pierre Robin syndrome D. Osteogenesis imperfecta
E.Cleidocranial dysostosis

2. The cranial vault increases rapidly in size the first few years postnatally and completes approxi-
mately 90% of its growth by 6 years of age. This growth is typical of which of the following types of
tissues?
A.Neural B. dental C. genital D. lymphoid E. general (somatic)
3. The following are the conditions found on the area of pressure in the presence of heavy forces:
area of hyalinization
occlusion of blood vessels
stretched periodontal fibers
undermining resoprtion
A. 1, 2& 3 B. 1, 2 & 4 C. 2 , 3 & 4 D. 3 only E. 1 & 4 only

4. Generalized osteoclastic activity along the walls of the alveolar socket is the bone response to:
A. depressing force C. extrusion
B. elongating force D. rotating force E. both B & D

5. A 4-year old child has a traumatized central incisor with a Class III (Ellis) fracture. The injury
occurred about one month ago, and examination indicates that the pulp is necrotic. There are no
other pathologic findings. Treatment of choice is:
A. watchful observation
B. extraction and use of a space maintainer
C. pulpectomy and root canal filling using gutta-percha points and cement
D. endodontic treatment and root canal filling with a resorbable paste.

6. Supervision of a child’s development of occlusion is most critical at ages:


A. 3-6 years B. 7-10 years C. 11-14 years E. 14-17 years

7. A child with congenital heart disease requires special treatment planning for dental care because
of potential problems with:
1 .bleeding
2. local infection
3. systemic infection
4. enamel hypoplasia
A. 1 , 2 & 3 B. 1 & 3 C. 1 only D. 3 only E. 1,2,3 & 4

8. In festooning and trimming a stainless steel crown, greater length is necessary in the region of the
mesiofacial bulge in a primary:
A. first molar B. second molar C. maxillary canine D. maxillary lateral incisor

9. Using a topical fluoride rinse before acid etch direct bonding of orthodontic brackets is contraindi-
cated because fluoride:
A. decreases the solubility of enamel
B. increases the pH of the etching agent
C .causes copious amounts of saliva
D. directly reacts chemically with the bonding agent.

10. If a 7-year old patient loses a primary mandibular canine about the same time the adjacent lateral
incisor is erupting or shortly thereafter, the dentist should be alert to the possibility of:
A. a tongue habit
B. a developing crossbite
C. an early eruption of the permanent canine
D. lingual collapse of mandibular anterior teeth

11. Which of the ff. approaches is best for a child suffering from celebral palsy
A. HOM exercise B. towel method C. use of sedation D. psychological approaches

12. Type of fear which is produced by direct physical stimulation of sense organ
A. associative fear B. objective fear C. subjective fear D. acquired fear

13. Agent used to reduce pain without affecting consciousness


A. hypnotic drug B. analgesic C. barbiturate D. general anesthesia
14. The drug that modifies fear and anxiety
A. flanax B. ethamizid C. celestamine D. seconal

15. A general characteristic of a 12 year old child


A. mother as the center of his world C. exhibits tantrums
B. separation anxiety D. rejects parental authority

16. This is a psychological approach to manage a child


A. nitrous oxide inhalation C. giving diazepam
B. mutisensory techniques D. HOM

17. The major source of anchorage in a maxillary Hawley appliance is the :


A. molar clasp C. acrylic portion
B. labial bow D. interproximal clasp E. bracket of the band

18. A narrow maxillary arch with respect to midsagittal plane is said to be in:
A. Protraction C. contraction
B. retraction D. distraction E. both C & D

19. Complete unilateral lip-jaw palate cleft.


A. Class I B. Class II C. Class III D. Class IV

20. Anodontia, diagnosed in a 5-year-old child, primarily affects the growth of the:
A. Midface C. mandible
B. maxilla D. alveolar bone E. maxilla & mandible

21. Which of the following are likely to occur during orthodontic therapy?
1. Gingival irritation
2. cementum resorption
3. increased mobility of teeth
4. demineralization of enamel adjacent to appliances in patients with poor oral hygiene
A. 1, 3 & 4 C. 2 & 4 only E. 3 & 4 only
B. 1 & 4 only D. 1, 2, 3 & 4

22. A distal shoe space maintainer is indicated when a primary:


A. incisor is avulsed
B. first molar is prematurely lost
C. second molar is lost after eruption of a permanent first molar
D. second molar is lost before eruption of a permanent first molar

23. The normal downward and forward direction of facial growth results from
1. upward and backward growth of maxillary sutures and the mandibular condyle
2. vertical eruption and mesial drift of the dentitions
3. interstitial growth in the maxilla and the mandible
4. epithelial induction at growth centers
A. 1 & 2 B. 1 & 3 C. 1 & 4 D. 2 & 3 E. 2 & 4

24. An 8-year-old girl has 2mm diastema between permanent maxillary central incisors. Permanent
maxillary lateral incisors are in position. The diastema is probably the result of:
1. thumb-sucking
2. an abnormal frenum attachment
3. a normal developmental process
A. 1 & 2 B. 1 & 3 C. 2 & 3 D. 2 only E. 3 only

25. When comparing cementum to bone in their responses to orthodontic forces, cementum resorbs:
A. more readily C. not at all
B. less readily D. under lighter loads E. by indirect (undermining) resoprtion
26. The tooth in the mandibular arch most likely to be malposed in cases of arch space discrepancy
is the:
A. first molar C.. first premalor
B. second molar D. second premolar

27. Slow progress in molar uprighting in an adult patient is usally due to:
A. overextended bands D. an overcontoured spring
B. lack of anchorage control E. the occlusion not being relieved
C. the stabilizing wire not being passive

28. A 9-year-old patient has a slightly convex profile and a suspected tooth mass-arch length (cir-
cumference) discrepancy. Before instituting space management procedures, the dentist should:
A. complete a space analysis C. complete a cephalometric analysis
B. seek orthodontic consultation D. all of the above E. Either A or C above

29. A primary maxillary anterior tooth in a 4-year-old child was traumatically intruded into the tissues
so that only half the tooth is visible. The most appropriate treatment is to:
A. extract the tooth.
B. perform a pulpotomy
C. administer no treatment
D. place orthodontic bands on adjacent teeth and draw the tooth down with elastics

30. Which of the following orthodontic movements of teeth are most difficult to accomplish?
1. Tipping
2. Rotation
3. Intrusion
4. Extrusion
5. Translation
A. 1 & 2 B. 1 & 3 C. 1 & 5 D. 3 & 4 E. 3 & 5

31. Interstitial growth is observed at which of the following sites?


1. Spheno-occipital syndrondrosis
2. Maxillary tuberosity
3. Mandibular condyle
4. Zygomaticomaxillary suture
5. Apex of an erupting premolar
A. 1 & 2 B. 1 & 3 C. 1 & 4 D. 2 & 4 E. 2 & 5

32. Which of the developmental space will cause a decrease in arch perimeter when pre-empted?
A. primate space B. interocclusal space
C. interdental space D. leeway space

33. This is an increase in size by expansion from within


A.sutural apposition C. interstitial growth
B. maturation D. development

34.This is growth movement which mean movement of whole bone as a unit


A. drift B. displacement
C. deposition D. relocation E. remodeling

35. Mandible comes from ? branchial arch


A. first B. second C. third D. fourth

36. Maxilla is formed from the maxillary processes and ?


A. medial nasal process C. globular process
B. lateral processes D. tuberculum impar
37. In young child, paradoxical excitement occurs most frequently following premedication with:
A. a narcotic B. a barbiturate C. nitrous oxide D. an amphetamine

38. The maximum amount of anesthesia that can be given to a 36-pound child is:
A. 4 carpules B. 3 carpules C. 2 carpules D. 1 carpule

39. Dental Fluorosis is most likely to occur on a:


A. two-year-old child ingesting 1 ppm of fluoride
B.17-year-old patient ingesting 1 ppm of fluoride
C. on a 3-year-old child ingesting 0.50 ppm of fluoride
D. none of the above.

40. Pulpotomy with formocresol of primary tooth:


A. result in lower success rate compared to calcium hydroxide,
B. result in higher success rate compared to calcium hydroxide,
C. should never be used,
D. induces formation of a dentin bridge at site of amputation,
E. none of the above.

41. If a primary tooth has been extracted before succedaneous tooth has begun eruptive move-
ment, its eruption will be __
A. hastened B. delayed C. not affected

42. Which of the following conditions is usually present in a Class II, Division 2 malocclussion?
Open bite
Steep mandibular plane
Mesiocclusion of permanent first molars
D. Lingual inclination of maxillary central incisors

43. Excessive orthodontic force used to move a tooth may:


cause hyalinization
cause undermining resorption
crush the periodontal ligament
A. 1 & 2 B. 2 & 3 C. 1 & 3 D. 1, 2 & 3 E. 3 only

44. Severely crowded mandibular incisors usually result from:


A. mesial migration of teeth
B. premature loss of primary teeth
C. presence of supernumerary teeth
D. tooth size-arch length discrepancy

45. How will extraction of a primary maxillary central incisor in a 5-year old child with incisal spac-
ing affect the size of the intercanine space?
A. The intercanine space will increase in size.
B. The intercanine space will decrease in size.
C. No change will occur in the size of the intercanine space.

The major criterion to differentiate between a true Class III and a pseudo-Class III malocclusion is:
A. degree of anterior crossbite
B. presence of a bilateral crossbite
C. existence of a forward shift of the mandible during closure
D. occlusal relationship between maxillary and mandibular first molars

47. Orthodontic correction of which of the following is most easily retained?


A. Diastema C. Expansion
B. Rotation D. Posterior crossbite E. Anterior crossbite
48. A bimaxillary protrusion occurs when:
A. large horizontal overlap with biteral crossbite exits
B. the maxillary dentition is forward with respect to basal bone
C. both dental arches are forward with respect to basal bone
D. none of the choices.

49. When a simple tipping force is applied to the crown of a single-rooted tooth, the center of rota-
tion is usually located:
A. at the apex D. at the cervical line
B. 5mm beyond the apex E. one-third the root length from the apex
C two-third the root length from the apex.

50. Bite plane therapy for deep bite, active Hawley’s plate are examples of:
A. Preventive Orthodontics C. Limited Corrective Orthodontics
B. Interceptive Orthodontics D. Extensive Corrective Orthodontics

51. The normal sequential events from fertilization to death:


A. growth C. differentiation E. translocation
B. development D. none of the choices

52.. ______will give rise to tissues that will become the mandible
A. Reichert’s cartilage C. Meckel’s cartilage
B. 3rd branchial arch D. 1st branchial arch E. none of the choices

53. Emphasized how cartilage of nasal septum during growth paced the growth of maxilla
A. Genetic Theory C. Moss’ Hypothesis
B. Sicher’s Hypothesis D. Scott’s Hypothesis E. Petrovic’s Hypothesis

54. Skeletal tissues grow in response to soft tissue growth


A. Genetic Theory C. Moss’ Hypothesis
B. Sicher’s Hypothesis D. Scott’s Hypothesis E. Petrovic’s Hypothesis

55. Cranial vault grows primarily by cartilage growth at:


A. synchondrosis B. sutures C. fontanelles D. disappears at birth

56. Cranial base grows primarily by__ growth at the synchondrosis:


A. intramembranous C. endochondral
B. combination of intramembranous and endochondral

57. Over 90% of brain growth is achieved by age: A. 5 B. 6 C. 7 D. 8 E. 15

58. Neurocranium follows what Scammon’s curve?


A. lymphoid B. neural C. general D. bodily E. genital type

59. Growth of brain case is primarily by proliferation and ossification of:


A. synchondrosis B. sutural connective tissue
C. fontanelles D. none of the choices

60. Nasomaxillary complex is hafted to the cranium by the following sutures except:
A. frontomaxillary C. zygomaticotemporal E. zygomaticomaxillary
B. pterygopalatine D. frontotemporal

61. As the maxilla decends, there is __ on the orbital floor, __ on the nasal floor and __ on the infe-
rior palatal surface:
A. + + - B. - + - C. + - + D. + + + E. - - +
62. The increase in length of the maxilla is due to:
A. apposition on the posterior border of the maxillary tuberosity
B. progressive movement of the entire zygomatic process in posterior direction
D. A&B
C. growth in the median suture E. none of the choices

63. Mechanism for maxillary growth:


A. sutural connective tissue proliferation C. surface apposition / resorption
B. ossification D. translation E. all of the
choices

64. The mechanism for bone growth of the nasomaxillary complex is:
A. intramembranous B. endochondral C. combination

65. The sutures of the nasomaxillary complex are oblique and more or less parallel with each other
thus, growth in these areas would serve to displace the maxilla:
A. downward C. forward and upward
B. downward and forward D. upward and backward

66. Continued apposition of alveolar bone on the free borders of the alveolar process as the teeth
erupt increases the __ of the maxilla.
A. height B. width C. length

67. Increase in the width of the maxilla involves palatal growth following the expanding V principle.
The buccal segments move:
A. downward and forward B. posterior and superior
C. upward and posterior D. upward

68. The maxilla Is displaced downward and forward by growth in __ parts of the bone:
A. anterior and inferior C. posterior and superior
B. anterior and superior D. posterior and anterior

69. What characteristic growth pattern is shown in the vertical section through the coronoid pro-
cess?
A. (+) on the lingual, (-) on the buccal
B.. (-) on the lingual, (+) on the buccal surface C.(+)
on the lingual, (-) on half of the buccal surface
D. (-) on the lingual, (-) on the buccal surface

70. Normal terminal plane relationship:


A. mesial step B. distal step C. flush/straight

71. The most common sequence of eruption for the upper arch:
A. 6124537 B. 6123457 C. 6142537 D. 6125437

72. The stage at which a tooth begins its eruptive movement:


A. stage 0 B. stage 2 C. stage 3 D. stage 6 E. stage 10

73.Periapical lesions, pulpitis and pulpotomy of a primary molar will___ the eruption of successor
premolar.
A. hasten B. delay C. have no effect on

74. If a primary tooth is extracted prior to the onset of permanent eruptive movement (prior to root
formation), the permanent tooth is likely to be:
A. delayed B. accelerated C. no effect on eruption

75. Which one is an incorrect description of Nolla’s stages of calcification?


A. Stage 4, 2/3 of crown completed C. Stage 7, 1/3 of root completed
B. Stage 6, crown almost completed D. Stage 9, root almost complete, open apex

76. Space differential in the anterior segment ( BA/AB ) - 21/12 gives:


A. excess space B. no space differential C. negative space

77. The proper crown angulation of a permanent incisor is one where the gingival portion of the
long axis of the crown should be __ to the incisal portion.
A. mesial B. distal C. incisal D. cervical

78. Rotated teeth occupy less space within the arch: A. true B. false

79. Plane of occlusion for permanent teeth: A. flat B. exaggerated

80. Posterior crown inclination from canine to molar:


A. lingual crown inclination B. buccal crown inclination C. upright

81. Size of teeth correlates well with stature. Size of teeth is sex-linked.
A. 1st statement is true, 2nd is false C. both are true
B. 1st statement is false, 2nd is true D. both are false

82. The most prevalent malocclusion in the primary dentition:


A. Class II Div. 1 C. Class I type 5 E. Class III
B. anterior openbite D. Class II types 1

83. A dolichofacial face would likely to have ___ arch form.


A. round C. average
B. broad and square D. long and tapering

84. A face that is broad and short is:


A. dolichofacial B. brachyfacial C. mesofacial D. normofacial

85. The most common malocclusion in the mixed dentition period is:
A. Class II Div. 1 B. anterior open bite
C. posterior open bite D. crowding E. Class I type 2

86. A type of conditioning where in a specific response is immediately rewarded


Classical conditioning C. operator conditioning
Operant conditioning D. conditioning E. classic conditioning

87. The patients history of childhood diseases are under


A. familial history B. social history C. medical history D. dental history

88. The history on the behavior of the child toward dentistry:


A. familial history B. medical history
C. Dental history D. history of present illness

89. Method of early examination where the dentist and parent are seated face to face
A. Knee position B. leg position
C. cradling position D. Knee to knee position E. all

90. In pedodontic triangle the apex of the triangle is:


A. Child B. Dentist C. Parent D. Assistant

91. Parent who are mistrusting are:


Overprotective Parent C. Neglectful Parent
Manipulative Parent D. Hostile parent
92. Parent who are demanding and can extend to directing the course of the treatment is a :
A. Overprotective Parent B. Neglectful Parent
C. Manipulative Parent D. Hostile parent

93. Etching time for deciduous dentition is ----- compared to permanent dentition:
A. shorter B. the same C. longer D. none

94. Panoramic Radiographic Film:


A. 5 “ x 7” B. 5” x 12” C. 2”x 3” D. 8” x 10”

95. Frankl behavior rating characterized by refusal of treatment and extreme negativism:
A. rating 1 B. rating 2 C. rating 3 D. rating 4

96. A period that child undergoes the phase of separation anxiety:


A. Toddler B. school age C. Pre school D. Adolescent

97. In Nitous oxygen sedation, once a petient takes a distant gaze the concentration of the nitous
oxide- oxygen is reduces to:
A. 30-70 B. 20-80 C. 40-60 D. 10-90 E. none

98. Device that assess the arterial hemoglobin oxygen saturation and pulse rate:
A. automated vital sign monitor C. pulse oximeter
B. pretracheal stethoscope D. none

99. Quadrant used in the gluteal region when intramuscular sedation is used:
A.upper inner quadrant B. upper outer quadrant
C. Lower inner quadrant D. Lower outer quadrant

100. The most unstable form of fluoride used for topical application:
A. Acidulated Phosphate Fluoride B. Stannous Fluoride
C. Sodium Fluoride . D. None

You might also like